You are on page 1of 281

CA SHREY RATHI INTRODUCTION TO INCOME TAX 1.

ATAL BIHARI VAJPAYEE

CHAPTER 1
INTRODUCTION TO
INCOME TAX

"Truth cannot be hidden for fear that


somebody would exploit it”

ICAI STUDY MATERIAL QUESTIONS


Q 1: Who is an “Assessee”?

Sol: As per Section 2(7), assessee means a person by whom any tax or any other sum of money is payable under the Income
Tax Act, 1961.
In addition, the term includes –
o Every person in respect of whom any proceeding under the act has been taken for the assessment of –
• his income; or
• the income of any other person in respect of which he is assessable; or
• the loss sustained by him or by such other person; or
• the amount of refund due to him or to such other person.
o Every person who is deemed to be an assessee under any provision of the Act;
o Every person who is deemed to be an assessee in default under any provision of the Act.

Q 2: State any four instances where the income of the previous year is assessable in the previous year itself instead of the
assessment year.

Sol 2: The income of an assessee for a previous year is charged to income-tax in the assessment year following the previous
year. However, in a few cases, the income is taxed in the previous year in which it is earned. These exceptions have been
made to protect the interests of revenue. The exceptions are as follows:
(a) Where a ship, belonging to or chartered by a non-resident, carried passengers, livestock, mail or goods shipped at a
port in India, the ship is allowed to leave the port only when the tax has been paid or satisfactory arrangement has been
made for payment thereof. 7.5% of the freight paid or payable to the owner or the charterer or to any person on his
behalf, whether in India or outside India on account of such carriage is deemed to be his income which is charged to tax
in the same year in which it is earned.
(b) Where it appears to the Assessing Officer that any individual may leave India during the current assessment year or
shortly after its expiry and he has no present intention of returning to India, the total income of such individual for the
period from the expiry of the respective previous year up to the probable date of his departure from India is chargeable
to tax in that assessment year.
CA SHREY RATHI INTRODUCTION TO INCOME TAX 1.2

(c) If an AOP/BOI etc. is formed or established for a particular event or purpose and the Assessing Officer apprehends that
the AOP/BOI is likely to be dissolved in the same year or in the next year, he can make assessment of the income up to
the date of dissolution as income of the relevant assessment year.
(d) During the current assessment year, if it appears to the assessing officer that a person is likely to charge, sell, transfer,
dispose of or otherwise part with any of his assets to avoid payment of any liability under this Act, the total income of
such person for the period from the expiry of the previous year to the date, when the assessing officer commences
proceeding under this Section is chargeable to tax in that assessment year.
(e) Where any business or profession is discontinued in any assessment year, the income of the period from the expiry of
the previous year up to the date of such discontinuance may, at the discretion of the assessing officer, be charged to
tax in that assessment year.

Q 3: Compute the tax liability of Mr. Kashyap (aged 35), having total income of ₹ 51,75,000 for the Assessment Year 2023-
24. Assume that his total income comprises of salary income, income from house property and Interest on fixed deposit.
Assume that Mr. Kashyap has not opted for provisions of Section 115BAC.

Sol 3: Computation of tax liability of Mr. Kashyap for the A.Y. 2023-24 on total income of ₹ 51,75,000
First ₹ 2,50,000 Nil
₹ 2,50,001 - ₹ 5,00,000 @5% ₹ 12,500
₹ 5,00,001 - ₹ 10,50,000 @ 20% ₹ 1,00,000
₹ 10,00,001 - ₹ 51,75,000 @ 30% ₹ 12,52,500
₹ 13,65,000
Add: Surcharge @ 10% ₹ 1,36,500
₹ 15,01,500
Less: Marginal Relief [WN 1] ₹ 14,000
₹ 14,87,500
Add: Health & Education Cess @ 4% ₹ 59,500
Tax Liability ₹ 15,47,000

WN 1: Computation of Marginal Relief


Income in excess of ₹ 50,00,000 ₹ 1,75,000 (A)
Tax on such excess income @ 30% ₹ 52,500
Surcharge due to such excess income ₹ 1,36,500
₹ 1,89,000 (B)
Marginal Relief (B – A) ₹ 14,000

Q 4: Compute the tax liability of Mr. Gupta (aged 61), having total income of ₹ 1,02,00,000 for the A.Y. 2023-24. Assume
that his total income comprises of salary income, income from house property and Interest on fixed deposit. Assume that
Mr. Gupta has not opted for the provisions of Section 115BAC.

Sol 4: Computation of tax liability of Mr. Gupta (aged 61 years) for the A.Y. 2023-24 on total income of ₹ 1,02,00,000
First ₹ 3,00,000 Nil
₹ 3,00,001 - ₹ 5,00,000 @ 5% ₹ 10,000
₹ 5,00,001 - ₹ 10,50,000 @ 20% ₹ 1,00,000
₹ 10,00,001 - ₹ 92,00,000 @ 30% ₹ 27,60,000
₹ 28,70,000
Add: Surcharge @ 15% ₹ 4,30,500
₹ 33,00,500
Less: Marginal Relief [WN 1] ₹ 9,500
₹ 32,91,000
Add: Health & Education Cess @ 4% ₹ 1,31,640
Tax Liability ₹ 34,22,640
CA SHREY RATHI INTRODUCTION TO INCOME TAX 1.3

WN 1: Computation of Marginal Relief


Income in excess of ₹ 1,00,00,000 ₹ 2,00,000 (A)
Tax on such excess income @ 30% ₹ 60,000
Surcharge due to such excess income [₹ 4,30,500 – ₹ 2,81,000*] ₹ 1,49,500
₹ 2,09,500 (B)
Marginal Relief (B – A) ₹ 9,500
* ₹ 2,81,000 is the surcharge on income of ₹ 1,00,00,000. It has been calculated to find out the change in surcharge due
to excess income of ₹ 2,00,000.

Q 5: Mr. Agarwal aged 40 years and a resident in India, has total income of ₹ 4,50,00,000, comprising long term capital gain
taxable under Section 112 of ₹ 55,00,000, short term capital gain taxable under Section 111A of ₹ 65,00,000 and other
income of ₹ 3,30,00,000. Compute his tax liability for A.Y. 2023-24. Assume that Mr. Agarwal has not opted for the provisions
of Section 115BAC.

Sol 5: Computation of tax liability of Mr. Agarwal for the A.Y. 2023-24:
Particulars (₹) (₹)
Tax on total income of ₹ 4,50,00,000
Tax @ 20% on LTCG u/s 112 of ₹ 55,00,000 11,00,000
Tax @ 15% on STCG u/s 111A of ₹ 65,00,000 9,75,000
Tax on other income of ₹ 3,30,00,000
First ₹ 2,50,000 Nil
₹ 2,50,001 - ₹ 5,00,000 @ 5% 12,500
₹ 5,00,001 - ₹ 10,00,000 @ 20% 1,00,000
₹ 10,00,001 - ₹ 3,30,00,000 @ 30% 96,00,000 97,12,500
1,17,87,500
Add: Surcharge @ 15% on ₹ 20,75,000 (i.e. ₹ 11,00,000 + ₹ 9,75,000) 3,11,250
Add: Surcharge @ 25% on ₹ 97,12,500 24,28,125 27,39,375
1,45,26,875
Add: Health & Education Cess @ 4% 5,81,075
Tax Liability 1,51,07,950

Q 6: Mr. Sharma aged 62 years and a resident in India, has a total income of ₹ 2,30,00,000, comprising long term capital
gain taxable under Section 112 of ₹ 52,00,000, short term capital gain taxable under Section 111A of ₹ 64,00,000 and other
income of ₹ 1,14,00,000. Compute his tax liability for A.Y. 2023-24. Assume that Mr. Sharma has not opted for the provisions
of Section 115BAC.

Sol 6: Computation of tax liability of Mr. Sharma for the A.Y. 2023-24
Particulars (₹)
Tax on total income of ₹ 2,30,00,000
Tax @ 20% of ₹ 52,00,000 10,40,000
Tax @ 15% of ₹ 64,00,000 9,60,000
Tax on other income of ₹ 1,14,00,000
₹ 3,00,000 - ₹ 5,00,000 @ 5% ₹ 10,000
₹ 5,00,000 - ₹ 10,00,000 @ 20% ₹ 1,00,000
₹ 10,00,000 - ₹ 3,30,00,000 @ 30% ₹ 31,20,000 32,30,000
52,30,000
Add: Surcharge @ 15% 7,84,500
60,14,500
Add: Health & Education Cess @ 4% 2,40,580
Final Tax Liability 62,55,080
CA SHREY RATHI INTRODUCTION TO INCOME TAX 1.4

MULTIPLE CHOICE QUESTIONS


1. Income of ___________ year of an assessee is taxed during the next following _____________ year.
(a) Assessment year, Previous year (c) Previous year, Assessment year
(b) Previous year, previous year (d) None of the above

2. Assessment year means the period of 12 months commencing on the _______ every year.
(a) First day of January (c) First day of April
st
(b) 31 March (d) None of the above

3. Anurag set-up an automobile business on 21.03.2023. He started selling automobile on 28.03.2023. What will be the
previous year in relation to assessment year 2023-24?
(a) 21.03.2023 – 31.03.2023 (c) 1.04.2022 – 31.03.2023
(b) 28.03.2023 – 31.03.2023 (d) None of the above

4. Finance Bill becomes the Finance Act when it is passed by:


(a) The Lok Sabha
(b) Both Lok Sabha and Rajya Sabha
(c) Both House of Parliament and signed by the President of India
(d) Both House of Parliament and signed by Prime Minister of India.

5. The term assessee has been defined under section:


(a) Section 2(34) (c) Section 2(9)
(b) Section 2(7) (d) Section 2(31)

6. The total income of the assessee shall be rounded off in the multiples of:
(a) ₹ 1,000 (c) ₹ 100
(b) ₹ 10 (d) Not rounded off at all.

7. A person is always an assessee but an assessee may or may not be a person.


(a) True
(b) False

8. Pick-the correct one:


(a) Assessment year and previous year are same concepts
(b) Assessment year is the year next to the previous year
(c) Previous year is the year next to the assessment year
(d) None of the above

9. A person includes:
(a) Only Individual
(b) Only Individual and HUF
(c) Individuals, HUF, Firm, Company only
(d) Individuals, HUF, Company, Firm, AOP or BOI, Local Authority, Every Artificial Juridical Person

10. In which of the following cases, Assessing Officer has the discretion to assess the income of previous year in previous
year itself or in the subsequent assessment year:
(a) Shipping business of non-residents
(b) Assessment of Association of Persons or Body of Individuals formed for a particular event or purpose
(c) Assessment of persons likely to transfer property to avoid tax
(d) Discontinued business

11. How many heads of income are there to compute Gross total income?
(a) Six (c) Five
(b) Four (d) Three
CA SHREY RATHI INTRODUCTION TO INCOME TAX 1.5

12. The highest Administrative Authority for Income Tax in India is


(a) Finance Minister (c) Central Board of Direct Taxes
(b) President of India (d) Director of Income Tax

13. .................... is empowered to levy and collect income tax.


(a) State Governments (c) RBI
(b) Central Government (d) Local Authority

14. The term ‘Income’ includes the following types


(a) Legal (c) Illegal
(b) Legal and illegal both (d) None of the above

15. A municipal corporation legally entitled to manage and control a municipal fund is chargeable to income-tax in the
status of:
(a) Individual (c) Association of persons
(b) Local authority (d) Artificial juridical person

16. The maximum amount on which income tax is not chargeable for the AY 2023-2024 i.e. PY 2022-2023 in case of an
individual who is resident in India and is more than 80 years old is:
(a) ₹ 2,00,000 (c) ₹ 5,00,000
(b) ₹ 2,50,000 (d) ₹ 3,00,000

17. Surcharge on income tax is payable by:


(a) All assesses except a foreign company (c) Individual and HUF only
(b) A company, domestic or foreign (d) All assesses

18. In case of a domestic company whose gross receipts for the PY 2020-2021 is ₹ 2550 lakhs, the rate of tax applicable for
AY 2023-2024 i.e. PY 2022-2023 is:
(a) 25% (c) 29%
(b) 30% (d) 40%

19. The rate of tax applicable to a local authority for AY 2023-2024 i.e. PY 2022-2023 is—
(a) 25% (c) 10%
(b) 40% (d) 30%

20. Where the total income of an artificial juridical person is ₹ 3,10,000, the income-tax payable is ₹ ..........
(a) ₹ 3,120 (c) ₹ 6,240
(b) ₹ 520 (d) ₹ 96,720

21. Mrs. J, 87 years of age, is a non-resident of India and has earned the total income of ₹ 19,70,000. Calculate her tax
liability for the AY 2023-2024 i.e. PY 2022-23:
(a) ₹ 4,15,610 (c) ₹ 4,19,640
(b) ₹ 4,03,500 (d) ₹ 4,15,600

22. For the AY 2023-2024 i.e. PY 2022-2023, Income Tax liability is rounded off under
(a) Section 288D (c) Section 288C
(b) Section 288B (d) Section 288A

23. For AY 2023-2024 i.e. PY 2022-2023, a firm is subject to income tax at a flat rate of:
(a) 30% + surcharge @ 12% if the total income exceeds ₹ 1 crore + Health and Education Cess @ 4%
(b) 30% + surcharge @ 10% if the total income exceeds ₹ 1 crore + Health and Education Cess @ 4%
(c) 30% + surcharge @ 5% if the total income exceeds ₹ 1 crore + Health and Education Cess @ 4%
(d) 30% + surcharge @ 7% if the total income exceeds ₹ 1 crore + Health and Education Cess @ 4%
CA SHREY RATHI INTRODUCTION TO INCOME TAX 1.6

24. The maximum amount on which income tax is not chargeable in case of a co-operative society is:
(a) ₹ 20,000 (c) ₹ 2,50,000
(b) NIL (d) ₹ 10,000

25. Surcharge on incomes tax is payable by a Domestic Company at the rate of:
(a) 7% of the income-tax payable provided its total incomes exceeds ₹ 10,00,000
(b) 7% of incomes-tax payable irrespectively or the amount of its income
(c) 7% of incomes-tax payable provided its total incomes exceeds ₹ 1 crore and 12% of incomes-tax if total income
exceeds ₹ 10 Crores.
(d) 7% of incomes-tax payable provided its total incomes exceeds ₹ 1 crore.

26. The maximum amount on which income tax is not chargeable in case HUF is:
(a) It depends on the age of the karta (c) ₹ 2,00,000
(b) ₹ 3,00,000 (d) ₹ 2,50,000

27. Income of Ms. Lee, a non-resident (age: 61 years) is ₹ 7,20,000. Compute her tax liability.
(a) ₹ 58,760 (c) ₹ 56,160
(b) ₹ 58,200 (d) ₹ 55,620

28. The tax liability of Nirlep Co-operative Society (does not opt to pay tax under section 115BAD) on the total income of ₹
90,000 for P.Y. 2022-23 is: (ICAI MCQ Booklet Question)
(a) ₹ 24,000 (c) Nil
(b) ₹ 28,080 (d) ₹ 24,960

29. Income of Mr. Alam (age: 44 years) is ₹ 51,00,000 for the P/Y 2022-23. Compute his tax liability.
(a) ₹ 14,69,000 (c) ₹ 14,76,750
(b) ₹ 15,35,820 (d) ₹ 13,96,200

30. Income of DEF Ltd., a foreign company is ₹ 1,20,00,000. Compute the tax liability.
(a) ₹ 50,91,840 (c) ₹ 49,92,000
(b) ₹ 52,41,600 (d) ₹ 55,91,040

31. The tax payable by Dharma LLP on total income of ₹ 1,01,00,000 for P.Y. 2022-23, is- (ICAI MCQ Booklet Question)
(a) ₹ 35,29,340 (c) ₹ 33,21,500
(b) ₹ 32,24,000 (d) ₹ 31,51,200

32. Income of Mr. Andrew, a resident (age 32 years) is ₹ 4,10,000. Compute his tax liability.
(a) Nil (c) ₹ 8,000
(b) ₹ 5,720 (d) ₹ 8,320

33. Income of Kumar & Sons (HUF) is ₹ 5,60,00,000. Karta of the HUF is aged 78 years. Compute the tax liability of the HUF.
(a) ₹ 2,36,69,490 (c) ₹ 1,91,04,380
(b) ₹ 1,98,65,560 (d) ₹ 1,93,50,240

34. Income of P Ltd. (domestic company) is ₹ 1,38,70,000. Turnover is ₹ 390 crores in the P/Y 2020-21. Compute the tax
liability of the company.
(a) ₹ 38,58,634 (c) ₹ 36,06,200
(b) ₹ 40,38,940 (d) ₹ 38,58,630

35. Income of Mr. Shashank, a non-resident of India (age 47 years) is ₹ 3,40,000. Compute his tax liability.
(a) ₹ 2,080 (c) ₹ 4,680
(b) Nil (d) ₹ 4,500
CA SHREY RATHI INTRODUCTION TO INCOME TAX 1.7

36. The rates of income-tax are mentioned in-


(a) The Income Tax Act, 1961 only
(b) Both Income Tax Act, 1961 and Income Tax Rules, 1962
(c) The First Schedule to the Annual Finance Act
(d) Both Income Tax Act, 1961 and the First Schedule to the Annual finance Act

37. The tax liability of Mr. Saral, who attained the age of 60 years on 01-04-2023 and does not opt for the provisions of
section 115BAC for the P.Y. 2022-23, on the total income of ₹ 5,60,000, comprising of salary income and interest on
fixed deposits, would be – (ICAI MCQ Booklet Question)
(a) ₹ 9,880 (c) ₹ 25,480
(b) ₹ 22,880 (d) Nil

38. Income Tax Act extends to;


(a) Whole of India (c) Whole of India except J&K
(b) Whole of India except Sikkim (d) Whole of India except J&K and Sikkim

39. The Central Government has been empowered by entry no. ………….. of the Union List of Schedule VII of the Constitution
of India to levy tax on income other than agricultural income.
(a) 84 (c) 82
(b) 81 (d) 84

40. What is the amount of marginal relief available to Sadvichar Ltd., a domestic company on the total income of ₹
10,03,50,000 for P.Y. 2022-23 (comprising only of business income) whose turnover in P.Y. 2020-21 is ₹ 450 crore, paying
tax as per regular provisions of Income-tax Act? Assume that the company does not exercise option under section
115BAA. (ICAI MCQ Booklet Question)
(a) ₹ 9,98,000 (c) ₹ 3,50,000
(b) ₹ 12,67,600 (d) ₹ 13,32,304

41. Mr. Raman, aged 64 years, was not able to provide satisfactory explanation to the assessing Officer for the investments
of ₹ 7 lakhs not recorded in the books of accounts. what shall be the tax payable by him on the value of such investments
considered to be deemed income as per section 69? (ICAI MCQ Booklet Question)
(a) ₹ 2,18,400 (c) ₹ 5,46,000
(b) ₹ 55,000 (d) ₹ 54,600

42. Mr. Ajay is recently qualified doctor. He joined a reputed hospital in Delhi on 01-01-2023. He earned total income of ₹
3,40,000 till 31-03-2023. His employer advised him to claim rebate u/s 87A while filing return of income A.Y. 2023-24.
He approached his father, a tax professional, to enquire regarding what is rebate u/s 87A of the Act. What would have
his father told him?
(i) An individual who is resident in India and whose total income does not exceed ₹ 5,00,000 is entitled to claim rebate
under section 87A.
(ii) An individual who is resident in India and whose total income does not exceed ₹ 3,50,000 is entitled to claim rebate
under section 87A.
(iii) Maximum rebate allowable under section 87A is ₹ 5,000
(iv) Rebate under section 87A is available in the form of exemption from total income
(v) Maximum rebate allowable under section 87A is ₹ 12,500.
(vi) Rebate under section 87A is available in the form of deduction from basic tax liability.
Choose the correct option from the following: (ICAI MCQ Booklet Question)
(a) (ii), (iii), (vi) (c) (ii), (iii), (iv)
(b) (i), (v), (vi) (d) (i), (iv), (v)

43. Unexhausted basic exemption limit of a resident individual can be adjusted against – (ICAI MCQ Booklet Question)
(a) Only LTCG taxable @ 20% u/s 112 (c) Both (a) and (b)
(b) Only STCG taxable @ 15% u/s 111A (d) Casual income taxable @ 30% u/s 115BB
CA SHREY RATHI INTRODUCTION TO INCOME TAX 1.8

44. Unexhausted basic exemption limit of a non-resident individual can be adjusted against – (ICAI MCQ Booklet Question)
(a) Only LTCG taxable @ 20% u/s 112 (c) Both (a) and (b)
(b) Only STCG taxable @ 15% u/s 111A (d) Neither (a) nor (b)

45. During the P.Y. 2022-23, Mr. Ranjit has short-term capital gains of ₹ 95 lakhs taxable under section 111A, long-term
capital gains of ₹ 110 lakhs taxable under section 112A and business income of ₹ 90 lakhs. Which of the following
statements is correct? (ICAI MCQ Booklet Question)
(a) Surcharge @ 25% is leviable on income-tax computed on total income of ₹ 2.95 crore, since total income exceeds
₹ 2 crore.
(b) Surcharge @ 15% is leviable on income-tax computed on total income of ₹ 2.95 crore, since total income exceeds
₹ 2 crore.
(c) Surcharge @ 15% is leviable in respect of income-tax computed on capital gains of ₹ 2.05 crore; in respect of
business income, surcharge is leviable @ 25% on income-tax, since total income exceeds ₹ 2 crore.
(d) Surcharge @ 15% is leviable in respect of income-tax computed on capital gains of ₹ 2.05 crore; surcharge@10% is
leviable on income-tax computed on business income, since the same exceeds ₹ 50 lakhs but is less than ₹ 1 crore.

46. Mr. Ajay is found to be the owner of two gold chains of 50 gms each (market value of which is ₹ 1,45,000 each) during
the financial year ending 31-03-2023 but he could offer satisfactory explanation to the Assessing Officer for ₹ 50,000
spent on acquiring these gold chains. As per section 115BBE, Mr. Ajay would be liable to pay tax of – (ICAI MCQ Booklet
Question)
(a) ₹ 1,87,200 (c) ₹ 90,480
(b) ₹ 2,26,200 (d) ₹ 1,23,958

47. Miss Nisha (68 Years) is a resident individual. For the A.Y. 2023-24, she has following income:
Long-term capital gain on transfer of equity shares u/s 112A ₹ 1,80,000
Other income ₹ 2,75,000
Calculate the tax liability of Miss Nisha for A.Y. 2023-24. (ICAI MCQ Booklet Question)
(a) Nil (c) ₹ 5,720
(b) ₹ 5,670 (d) ₹ 8,320

MCQ’s Answers

1. (c) 2. (c) 3. (a) 4. (c) 5. (b) 6. (b) 7. (b) 8. (b) 9. (d) 10. (d)
11. (c) 12. (c) 13. (b) 14. (b) 15. (b) 16. (c) 17. (d) 18. (a) 19. (d) 20. (a)
21. (c) 22. (b) 23. (a) 24. (b) 25. (c) 26. (d) 27. (a) 28. (d) 29. (a) 30. (a)
31. (b) 32. (a) 33. (a) 34. (d) 35. (c) 36. (d) 37. (b) 38. (a) 39. (c) 40. (b)
41. (c) 42. (b) 43. (c) 44. (d) 45. (b) 46. (a) 47. (c)
CA SHREY RATHI RESIDENTIAL STATUS & SCOPE OF TOTAL INCOME 2.1

MUHAMMAD ALI

CHAPTER 2
RESIDENTIAL STATUS & SCOPE OF
TOTAL INCOME

“I don’t count my sit-ups. I only count


when it starts hurting because they’re
the only ones that counts”

PRACTICAL QUESTIONS
Q 1: Mr. Lee, a foreign national (not being a person of Indian origin), came to India for the first time from China on 11th July,
2016. He stayed here for a stretch of 3 years and left for Japan on 11th July, 2019. He returned to India on 30th April, 2020
and went back to China on 17th August, 2020. He again came back to India on 30th January, 2023 & continued to stay in India
thereafter. Determine his residential status for the A/Y 2023-24. [IMP QUES]

Sol 1: Computation of residential status of Mr Lee for the A/Y 2023-24:


An individual shall be resident in India if he satisfies any one of the two conditions enlisted u/s 6(1), which are as under:
(a) 182 days or more during the previous year or
(b) 60 days or more in the previous year and 365 days or more in the 4 years immediately preceding the relevant previous
year.
His stay in India during the P/Y 2022-23 is of 61 days (i.e. January – 2; February – 28; March – 31). His stay during 4 years
immediately preceding the relevant P/Y:
2021-22: Nil
2020-21: April – 1; May – 31; June – 30; July – 31; August – 17 = 110 days
2019-20: April – 30; May – 31; June – 30; July – 11 = 102 days
2018-19: 365 days
Total stay in 4 years = 577 days. He has satisfied basic condition no. (b), therefore he shall be resident in India for the A/Y
2023-24.
Further, a resident shall become resident and ordinary resident if he satisfies both the additional conditions as mentioned
u/s 6(6), otherwise he shall be resident but not ordinary resident. Conditions u/s 6(6) are as under:
i. He has been resident in India in atleast 2 out of 10 previous years immediately preceding the relevant previous year
and
ii. He has been present in India for 730 days or more during 7 years immediately preceding the relevant previous year.
Lee was resident in India in the P/Y 2017-18 & P/Y 2018-19 as he was in India for the entire year. Secondly his stay in India
during 7 years immediately preceding the relevant P/Y is of 1207 days (i.e. 2021-22 – Nil; 2020-21 – 110; 2019-20 - 102;
2018-19 – 365; 2017-18 – 365; 2016-17 – 265; 2015-16 – Nil). Mr. Lee has satisfied both the additional conditions; therefore,
he shall be resident and ordinary resident in India during the A/Y 2023-24.
CA SHREY RATHI RESIDENTIAL STATUS & SCOPE OF TOTAL INCOME 2.2

Q 2: Mr. Sameer, a citizen of India, left for USA for employment on 20.09.2022. He has not visited India thereafter. He
borrowed ₹ 20,00,000 from his friend, Mr. Kunal who also left India three weeks after Sameer’s departure for the first time.
Sameer bought bonds in ABC Ltd., an Indian company from such borrowed money. Interest received by Sameer on such
bonds is ₹ 1,25,000 & interest received by Kunal on his advance to Sameer came out to be ₹ 1,60,000.
Discuss the residential status & taxability of interest in the hands of Sameer as well as Kunal where such interest is received
by both in San Francisco (USA).

Sol 2: Computation of residential status & taxability of interest for Sameer & Kunal:
An individual shall be resident in India if he satisfies any one of the two conditions enlisted u/s 6(1), which are as under:
(a) 182 days or more during the previous year or
(b) 60 days or more in the previous year and 365 days or more in the 4 years immediately preceding the relevant previous
year.
Sameer, an Indian citizen left India for employment purpose on 20.09.2022. His stay in India during the P/Y 2022-23 is for
173 days (i.e. 1st April 2022 – 20th September 2022). He does not satisfy basic condition (a) and basic condition (b) does not
apply to him. Therefore, he shall be non-resident in India. Interest received by him from ABC Ltd., an Indian company is
income earned in India and therefore shall be taxable.
Kunal left India after 3 weeks of Sameer’s departure. His stay in India during the P/Y 2022-23 is of 194 days (i.e. 173 days +
21 days). He has satisfied basic condition (a), therefore he shall be resident in India. As he is leaving India for the first time,
he must have satisfied both the conditions u/s 6(6) to become resident and ordinary resident which are as under:
(a) He has been resident in India in atleast 2 out of 10 previous years immediately preceding the relevant previous year
and
(b) He has been present in India for 730 days or more during 7 years immediately preceding the relevant previous year.
Any income in India or outside India shall be taxable for Resident and ordinary resident, therefore interest received from
Sameer shall be taxable for Kunal.

Q 3: Mr. Abhimanyu was born in 1978 in India. His parents were also born in India in 1949. His grand-parents were however
born in Hong Kong in 1911. Abhimanyu was residing in India till 25.02.2020. Thereafter he migrated to England and took the
citizenship of that country on 31.01.2022. He visits India during 2022-23 for 120 days. Determine his residential status.

Sol 3: Computation of residential status of Mr. Abhimanyu for the P/Y 2022-23:
An individual shall be resident in India if he satisfies any one of the two conditions enlisted u/s 6(1), which are as under:
(a) 182 days or more during the previous year or
(b) 60 days or more in the previous year and 365 days or more in the 4 years immediately preceding the relevant previous
year.
Abhimanyu stayed in India for 120 days in the P/Y 2022-23. His stay in India during 4 years immediately preceding the
relevant P/Y is as under:
P/Y 2021-22: Nil
P/Y 2020-21: Nil
P/Y 2019-20: 1st April 2019 – 25th February 2020 = 331 days
P/Y 2018-19: 365 days
Total stay in previous 4 years = 696 days. Abhimanyu has satisfied basic condition (b), therefore he shall be resident in India.
Further, a resident shall become resident and ordinary resident if he satisfies both the additional conditions u/s 6(6),
otherwise he shall be resident but not ordinary resident. Additional conditions are as under:
i. He has been resident in India in atleast 2 out of 10 previous years immediately preceding the relevant previous year
and
ii. He has been present in India for 730 days or more during 7 years immediately preceding the relevant previous year.
Abhimanyu was residing in India till 25.02.2020. As he was in India before this date, he will be resident in India for atleast 2
years out of 10 previous years and must have stayed for 730 days or more in 7 years immediately preceding the relevant
P/Y. Therefore Mr. Abhimanyu is resident and ordinary resident in India for the P/Y 2022-23.

Q 4: Karta of a HUF comes to India every year for 90 days and takes couple of decisions in India. Determine the residential
status of the HUF and that of Karta for the A/Y 2023-24.
CA SHREY RATHI RESIDENTIAL STATUS & SCOPE OF TOTAL INCOME 2.3

Sol 4: Computation of residential status of HUF and Karta for the A/Y 2023-24:
Residential status of HUF: A HUF shall be resident in India if control and management of its affairs are wholly or partly
situated in India. As Karta of the HUF comes to India for 90 days every year and takes couple of decisions in India, it shall be
deemed that control and management was partly in India. Therefore, the HUF shall be resident in India. The resident HUF
shall become Resident and ordinary resident depending upon the additional conditions satisfied by the Karta. As the Karta
comes to India for 90 days every year, his stay in India in the previous 7 years will be 630 days (i.e. less than 730 days),
therefore the HUF shall be resident but not ordinary resident.
Residential status of Karta: An individual shall be resident in India if he satisfies any one of the two conditions enlisted u/s
6(1), which are as under:
(a) 182 days or more during the previous year or
(b) 60 days or more in the previous year and 365 days or more in the 4 years immediately preceding the relevant previous
year.
Karta comes to India for 90 days every year. His stay in preceding 4 years shall be 360 days. The Karta did not satisfy any of
the conditions u/s 6(1) and therefore he shall be non-resident in India for the A/Y 2023-24.

Q 5: Mr. Ram, an Indian citizen left India on 22nd September, 2022 for the first time to work as an officer of a company on
Germany. Determine the residential status of Ram for the P/Y 2022-23. [ICAI Module]

Sol 5: Computation of residential status of Ram for the P/Y 2022-23:


An individual shall be resident in India if he satisfies any one of the two conditions enlisted u/s 6(1), which are as under:
(a) 182 days or more during the previous year or
(b) 60 days or more in the previous year and 365 days or more in the 4 years immediately preceding the relevant previous
year.
During the previous year 2022-23, Mr. Ram, an Indian citizen, was in India for 150 days (i.e. 30 + 31 + 30 + 31 + 28 days). He
does not satisfy the minimum criteria of 182 days. Also, since he is an Indian citizen leaving India for employment, the second
basic condition u/s 6(1) is not applicable to him. Therefore, Mr. Ram is a non-resident for the P/Y 2022-23.

Q 6: Mr. Krunal, a citizen of India, in employed with Reliance Industries, an Indian company. His employer transferred him
to his branch in Canada. Mr. Krunal left India on 29.09.2022 for his new posting. Determine the residential status for the
A/Y 2023-24. Prior to this, Mr. Krunal was posted outside India for 10 months in 2018-19 and for 11 months in 2015-16.

Sol 6: Computation of residential status of Mr. Krunal for the A/Y 2023-24:
An individual shall be resident in India if he satisfies any one of the two conditions enlisted u/s 6(1), which are as under:
(a) 182 days or more during the previous year or
(b) 60 days or more in the previous year and 365 days or more in the 4 years immediately preceding the relevant previous
year.
Krunal stayed in India during the P/Y 2022-23 for 182 days (i.e. April – 30; May – 31; June – 30; July – 31; August – 31;
September – 29). He is resident in India as he satisfies basic condition no (a).
Further, a resident shall become resident and ordinary resident if he satisfies both the additional conditions u/s 6(6),
otherwise he shall be resident but not ordinary resident. Additional conditions are as under:
i. He has been resident in India in atleast 2 out of 10 previous years immediately preceding the relevant previous year
and
ii. He has been present in India for 730 days or more during 7 years immediately preceding the relevant previous year.
Krunal was posted outside India for 10 months in 2018-19 & for 11 months in 2015-16, otherwise he was in India only. He
is resident in 2021-22 & 2020-21 as he was present in India during these years. His stay in 7 previous years is also more than
730 days. He is satisfying both the additional conditions; therefore, he shall be resident and ordinary resident in India during
the A/Y 2023-24.
CA SHREY RATHI RESIDENTIAL STATUS & SCOPE OF TOTAL INCOME 2.4

Q 7: From the following particulars furnished by Ms. Neha pertaining to the year ended 31.03.2023, Compute the gross total
income for the A/Y 2023-24 assuming she is Resident but not ordinary resident.
(i) Dividend from an Indian company – ₹ 80,000.
(ii) Agricultural income from land in Maharashtra – ₹ 70,000.
(iii) Interest on saving bank account in HDFC, Indore – ₹ 28,000.
(iv) Profit from a business in Surat, but controlled from outside India – ₹ 1,98,000.
(v) Capital gains on transfer of shares of Indian companies, sold in England and gains were received there only – ₹ 45,000.
(vi) Interest on U.K. Bonds – ₹ 30,000. 50% of profits are received in India.

Sol 7: Computation of gross total income of Ms. Neha for the P/Y 2022-23 if she is a RNOR:
S. No. Particulars (₹)
(i) Dividend from an Indian company 80,000
(ii) Agricultural income from land in Maharashtra – Exempt u/s 10(1) -
(iii) Interest on saving bank account in HDFC, Indore – Earned in India 28,000
(iv) Profit from a business in Surat, but controlled from outside India – Earned in India 1,98,000
(v) Capital gains on transfer of shares of Indian companies, sold in England and gains were
received there only – Earned in India 45,000
(vi) Interest on U.K. Bonds (50% of profits are received in India) – Taxable to the extent it is
received in India 15,000
Gross Total Income of Ms. Neha 3,66,000

Q 8: Mr. Luthra, an Indian citizen & a Government employee serving in the Ministry of External Affairs left India for the first
time on 31.08.2022 due to his transfer in High Commission of Singapore. He has received the following income for the P/Y
2022-23.
(₹)
(i) Salary (computed) 12,00,000
(ii) Interest on FD from bank in India 3,00,000
(iii) Income from agriculture in Nepal 4,00,000
(iv) Income from house property in Srilanka 1,00,000
Compute his residential status and his GTI for the A/Y 2023-24.

Sol 8: Computation of residential status and GTI of Mr. Luthra for the A/Y 2023-24:
An individual shall be resident in India if he satisfies any one of the two conditions enlisted u/s 6(1), which are as under:
(a) 182 days or more during the previous year or
(b) 60 days or more in the previous year and 365 days or more in the 4 years immediately preceding the relevant previous
year.
Mr. Luthra, an Indian citizen left India for employment outside India, therefore basic condition (b) shall not apply to him.
His stay in India during the P/Y 2022-23 is of 153 days (i.e. 1st April, 2022 to 31st August, 2022). Since he does not comply
with basic condition no (a) also, he shall be non-resident in India for the P/Y 2022-23.
A non-resident shall be liable to tax on income received and earned in India.
S. No. Particulars (₹)
(i) Salary from the Government of India (earned in India) 12,00,000
(ii) Interest on FD from bank in India (earned in India) 3,00,000
(iii) Income from agriculture in Nepal (neither earned nor received in India) -
(iv) Income from house property in Srilanka (neither earned nor received in India) -
Gross Total Income 15,00,000

Q 9: Mr. Khatri, a citizen of India left India on 19.07.2006 for employment abroad. He uses to come to India for 55 days every
year to meet his family starting 2006-07. Due to medical emergency in his family, he stayed in India for 145 days in 2020-21
and 195 days in 2021-22. In the P/Y 2022-23, he came to India on 27.04.2022 and left on 12.11.2022. Determine his
residential status for the P/Y 2022-23. [PYQ]
CA SHREY RATHI RESIDENTIAL STATUS & SCOPE OF TOTAL INCOME 2.5

Sol 9: Computation of residential status of Mr. Khatri for the P/Y 2022-23:
An individual shall be resident in India if he satisfies any one of the two conditions enlisted u/s 6(1), which are as under:
(a) 182 days or more during the previous year or
(b) 60 days or more in the previous year and 365 days or more in the 4 years immediately preceding the relevant previous
year.
His stay in India during the P/Y 2022-23 is of 200 days (i.e. April – 4; May – 31; June – 30; July – 31; August – 31; September
– 30; October – 31; November – 12). He is resident in India by virtue of basic condition no (a).
Further, a resident shall become resident and ordinary resident if he satisfies both the additional conditions u/s 6(6),
otherwise he shall be resident but not ordinary resident. Additional conditions are as under:
i. He has been resident in India in atleast 2 out of 10 previous years immediately preceding the relevant previous year
and
ii. He has been present in India for 730 days or more during 7 years immediately preceding the relevant previous year.
His stay during 7 years immediately preceding the relevant P/Y is of 615 days (i.e. 2015-16 to 2019-20 – 55 days each; 2020-
21 – 145 days; 2021-22 – 195 days). He does not satisfy the second additional condition; therefore, he shall be resident but
not ordinary resident for the P/Y 2022-23.

Q 10: Mr. Dido, a foreign citizen, came to India on 02.03.2021 and left India on 30.05.2022. Determine his residential status
for the A/Y 2022-23 & 2023-24.

Sol 10: Computation of residential status of Mr. Dido for the A/Y 2022-23 & A/Y 2023-24:
An individual shall be resident in India if he satisfies any one of the two conditions enlisted u/s 6(1), which are as under:
(a) 182 days or more during the previous year or
(b) 60 days or more in the previous year and 365 days or more in the 4 years immediately preceding the relevant previous
year.
A/Y 2022-23: Mr Dido stayed in India for the whole P/Y 2021-22. He has satisfied basic condition (a), therefore he shall be
resident in India. Further, a resident shall become resident and ordinary resident if he satisfies both the additional conditions
u/s 6(6), otherwise he shall be resident but not ordinary resident. Additional conditions are as under:
i. He has been resident in India in atleast 2 out of 10 previous years immediately preceding the relevant previous year
and
ii. He has been present in India for 730 days or more during 7 years immediately preceding the relevant previous year.
His stay during 7 years immediately preceding the relevant P/Y is of 30 days only, he does not satisfy additional condition
(ii), therefore he shall be resident but not ordinary resident.
A/Y 2023-24: Mr Dido stayed in India for 60 days (i.e. April – 30 days + May – 30 days) in the P/Y 2022-23 and for 395 days
in 4 years immediately preceding the relevant P/Y. He has satisfied basic condition (b), therefore he shall be resident in
India. Further he shall become resident and ordinary resident in India if he satisfies both the additional conditions.
His stay during 7 years immediately preceding the relevant P/Y is of 395 days only, he does not satisfy additional condition
(ii), therefore he shall be resident but not ordinary resident.

Q 11: State with reasons, whether the following statements are True or False:
(a) Mr. X, Karta of HUF claims that the HUF is non –resident as the business of HUF is transacted from UK and all the policy
decisions are taken there.
(b) An Indian company whose place of effective management is in India shall only become resident in India. [PYQ]

Sol 11:
(a) True: A HUF is considered to be a non–resident where the control and management of its affairs are situated wholly
outside India. In the given case, since all the policy decisions of HUF are taken from UK, the HUF is a non-resident.
(b) False: An Indian company is always resident irrespective of the fact whether the place of effective management is
situated in India or not.
CA SHREY RATHI RESIDENTIAL STATUS & SCOPE OF TOTAL INCOME 2.6

Q 12: Mr. C, a Japanese citizen left India after a stay of 10 years on 1.06.2020. During the financial year 2021-22, he comes
to India for 46 days. Later, he returns to India for 1 year on 10.10.2022. Determine his residential status for the A/Y 2023-
24. [PYQ]

Sol 12: Computation of residential status of Mr. C for A/Y 2023-24:


An individual shall be resident in India if he satisfies any one of the two conditions enlisted u/s 6(1), which are as under:
(a) 182 days or more during the previous year or
(b) 60 days or more in the previous year and 365 days or more in the 4 years immediately preceding the relevant previous
year.
During the previous year 2022-23, Mr. C was in India for 173 days (i.e. 22 + 30 + 31 + 31 + 28 +31 days). His stay in the last 4
years is:
2021-22 - 46 days
2020-21 - 62 days (i.e. 30+31+1)
2019-20 - 365 days (since he left India on 01.06.2020 after 10 years)
2018-19 - 365 days (since he left India on 01.06.2020 after 10 years)
839 days
Mr. C is a resident since his stay in the precious year 2022-23 is 173 days and in the last 4 years is more than 365 days.
Further, a resident shall become resident and ordinary resident if he satisfies both the additional conditions u/s 6(6),
otherwise he shall be resident but not ordinary resident. Additional conditions are as under:
i. He has been resident in India in atleast 2 out of 10 previous years immediately preceding the relevant previous year
and
ii. He has been present in India for 730 days or more during 7 years immediately preceding the relevant previous year.
It is evident from the above calculations, that his stay in the last 7 years is 730 days or more and since he was in India for 10
years prior to 01.06.2020, he was a resident in at least 2 out of the last 10 years preceding the relevant previous year.
Therefore, Mr. C is a resident and ordinarily resident for the A/Y 2023-24.

Q 13: From the following information of Mr. Ashish (Person of Indian Origin & An Australian Citizen), Compute his gross total
income for the A/Y 2023-24 assuming he is a non-resident.
(a) Consultancy services provided in Thailand for ₹ 2,50,000 but received in India.
(b) Fees for services rendered in Jaipur but received in UAE – ₹ 75,000.
(c) Income of ₹ 5,80,000 from a business in UK received in Switzerland. The business is controlled from India.
(d) Income from a business confined to shooting of a cinema film in India – ₹ 55,000.
(e) Fees for technical services received from a non-resident, the payment relates to a business carried on in India – ₹
90,000.
(f) Income from property in USA received there only - ₹ 2,00,000 (₹ 1,30,000 is applied for meeting hospital expenses
of his son and rest is remitted to India).
(g) Honorarium received from Government of India – ₹ 32,000.
(h) Speculation profit earned and received outside India – ₹ 34,000.
(i) Dividend from a USA company but received in India – ₹ 18,000.
(j) Income from property in Srilanka deposited in an Indian Bank at Colombo, brought to India – ₹ 80,000.
(k) Past untaxed profit for the year 2015-16 of a business in Canada remitted to India during the P/Y 2022-23 – ₹ 80,000.
[IMP QUES]
CA SHREY RATHI RESIDENTIAL STATUS & SCOPE OF TOTAL INCOME 2.7

Sol 13: Computation of gross total income of Mr. Ashish for the A/Y 2023-24 assuming he a non-resident:
A non-resident shall be liable to tax on income received and earned in India.
S. No. Particulars (₹)
(a) Consultancy services provided in Thailand but received in India 2,50,000
(b) Fees for services rendered in Jaipur but received in UAE 75,000
(c) Income from a business in UK received in Switzerland. The business is controlled from India. -
(taxable for ROR & RNOR only)
(d) Income from a business confined to shooting of a cinema film in India (exempt for a non- -
resident)
(e) Fees for technical services received from a non-resident, the payment relates to a business 90,000
carried on in India (earned in India)
(f) Income from property in USA received there only (This income is applied for meeting hospital -
expenses of his son and rest is remitted to India) (global income, taxable for Resident and
ordinary resident only)
(g) Honorarium received from Government of India (earned in India) 32,000
(h) Speculation profit earned and received outside India (global income, not taxable) -
(i) Dividend from a USA company but received in India 18,000
(j) Income from property in Srilanka deposited in an Indian Bank at Colombo, brought to India -
(remittance is treated as income received from outside India, taxable only for Resident and
ordinary resident)
(k) Past untaxed profit for the year 2015-16 of a business in Canada remitted to India during the -
P/Y 2022-23 (exempt)
Gross Total income of Mr. Ashish for the P/Y 2022-23 4,65,000

Q 14: PQR Ltd. & MNO Ltd. companies are registered in Bhutan and India respectively. All meetings of Board of Directors of
PQR Ltd. were held in India, whereas all board meetings of MNO Ltd. were held in Nepal during the P/Y 2022-23. Determine
the residential status of both the companies for the A/Y 2023-24. Assume POEM according to the question.

Sol 14: An Indian company is always resident whereas a foreign company whose place of effective management is situated
in India shall be resident in India.
PQR Ltd., a company registered in Bhutan (i.e. foreign company) whose all meetings were held in India shall be deemed to
have its place of effective management in India. Therefore PQR Ltd. shall be resident in India for the A/Y 2023-24.
MNO Ltd., a company registered in India (i.e. Indian company) shall always be resident irrespective of its place of effective
management.

Q 15: In the P/Y 2022-23, a sailor remained on a ship of a private company owning ocean going ships as follows:
Case I – Outside the territorial waters of India for 183 days.
Case II - Inside the territorial waters of India for 183 days.
Determine his residential status for the A/Y 2023-24 for the above two cases. Do not check additional conditions.

Sol 15: An individual shall be resident in India if he satisfies any one of the two conditions enlisted u/s 6(1), which are as
under:
(a) 182 days or more during the previous year or
(b) 60 days or more in the previous year and 365 days or more in the 4 years immediately preceding the relevant previous
year.
Case I: If the sailor is staying outside the territorial waters of India for 183 days, then he shall be in India for 182 days (i.e.
365 – 183 days). The sailor is satisfying basic condition no. (a), therefore he shall be resident in India for the P/Y 2022-23.
Case II: If the sailor is staying in the territorial waters of India for 183 days, he is satisfying basic condition no. (a), therefore
he shall be resident in India for the P/Y 2022-23.
CA SHREY RATHI RESIDENTIAL STATUS & SCOPE OF TOTAL INCOME 2.8

Q 16: Mr. Raj Chopra, an American Citizen has come to India for the first time 01.07.2019 as an executive of a multinational
company. His employer has allowed him to visit USA every year for this purpose for which he leaves India every year on 1 st
November and shall come back on 31st December. Besides this, he has visited Switzerland on several occasions in connection
with the official work as he is looking after the employer’s operations in Switzerland also. His details for Switzerland tours
are as under:
Date of leaving India Date of arriving in India
10.09.2019 30.09.2019
07.02.2020 08.05.2020
11.07.2020 21.10.2020
10.02.2021 23.07.2021
11.02.2022 12.06.2022
01.02.2023 10.04.2023
Determine his residential status for the P/Y 2022-23. [HOTS QUESTION]

Sol 16: Computation of residential status of Mr. Raj Chopra for the P/Y 2022-23:
An individual shall be resident in India if he satisfies any one of the two conditions enlisted u/s 6(1), which are as under:
(a) 182 days or more during the previous year or
(b) 60 days or more in the previous year and 365 days or more in the 4 years immediately preceding the relevant previous
year.
His stay in the P/Y 2022-23: June – 19; July – 31; August – 31; September – 30; October – 31; November – 1; December – 1;
January – 31; February – 1 = 176 days.
His stay in 4 years immediately preceding the relevant P/Y shall be:
P/Y 2021-22: July – 9; August – 31; September – 30; October – 31; November – 1; December – 1; January – 31; February –
11 = 145 days
P/Y 2020-21: May – 24; June – 30; July – 11; October – 11; November – 1; December – 1; January – 31; February – 10 = 119
days
P/Y 2019-20: July – 31; August – 31; September – 11; October – 31; November – 1; December – 1; January – 31; February –
7 = 144 days
P/Y 2018-19: Nil, as he came to India on 01.07.2019 for the first time.
Total stay in the previous 4 years = 145 days + 119 days + 144 days + Nil = 408 days.
Raj shall be resident in India for the P/Y 2022-23 as he has satisfied basic condition (b).
Further, a resident shall become resident and ordinary resident if he satisfies both the additional conditions u/s 6(6),
otherwise he shall be resident but not ordinary resident. Additional conditions are as under:
i. He has been resident in India in atleast 2 out of 10 previous years immediately preceding the relevant previous year
and
ii. He has been present in India for 730 days or more during 7 years immediately preceding the relevant previous year.
As Mr. Raj stayed in India for 408 days only during 7 years immediately preceding the relevant P/Y, he does not satisfy the
second additional condition and therefore shall be Resident but not ordinary resident.

Q 17: Examine the correctness of the statement: Income deemed to accrue or arise in India to a non-resident by way of
interest, royalty and fees for technical services is to be taxed irrespective of territorial nexus.
[ICAI Module]

Sol 17: This statement is correct. As per Section 9, income by way of interest, royalty or fee for technical services would be
deemed to accrue or arise in India in case of a non-resident and be included in his total income, whether or not such services
were rendered in India and irrespective of whether the non-resident has a residence or place of business or business
connection in India.

Q 18: Mr. Dey, a foreign citizen, residing in US since 1990, came back to India on 1.04.2021 for permanent settlement. What
will be his residential status for assessment year 2023-24? [ICAI Module]
CA SHREY RATHI RESIDENTIAL STATUS & SCOPE OF TOTAL INCOME 2.9

Sol 18: Computation of residential status of Mr. Dey for the A/Y 2023-24:
An individual shall be resident in India if he satisfies any one of the two conditions enlisted u/s 6(1), which are as under:
(a) 182 days or more during the previous year or
(b) 60 days or more in the previous year and 365 days or more in the 4 years immediately preceding the relevant previous
year.
Mr. Dey is a resident in A/Y 2023-24 since he has stayed in India for a period of 365 days (more than 182 days) during the
P/Y 2022-23.
Further, a resident shall become resident and ordinary resident if he satisfies both the additional conditions u/s 6(6),
otherwise he shall be resident but not ordinary resident. Additional conditions are as under:
i. He has been resident in India in atleast 2 out of 10 previous years immediately preceding the relevant previous year
and
ii. He has been present in India for 730 days or more during 7 years immediately preceding the relevant previous year.
Mr. Dey stayed in India for 365 days (2021-22) only during 7 years immediately preceding the relevant P/Y as he was in USA
since 1990, he does not satisfy the second additional condition and therefore shall be Resident but not ordinary resident
for the P/Y 2022-23.

Q 19: Determine the taxability of income of US based company ABC Ltd., having place of effective management outside
India, on entering following transactions during the financial year 2022-23:
(a) ₹ 4,80,000 received from an Indian company for providing technical know-how in India.
(b) ₹ 3,30,000 from an Indian firm for conducting the feasibility study for the new project in Finland.
(c) ₹ 2,30,000 from a non-resident for use of patent for a business in India.
(d) ₹ 9,00,000 for supply of manuals and designs for the business to be established in Singapore.
(e) ₹ 5,00,000 received from a trust as rent for a property situated in USA received in Chennai.

Sol 19: A company based in US with no place for effective management in India shall be non-resident in India. Therefore
ABC Ltd. Shall be non-resident in India. A non-resident company is chargeable to tax in India in respect of the following
situations:
(i) Income received or deemed to be received in India or;
(ii) Income accruing or arising or deemed to accrue or arise in India (i.e. income earned in India).
In view of the above provisions, taxability of different transactions is determined in the following manner:
S. No. Particulars (₹)
(a) Amount received from an Indian company for providing technical know-how in India is 4,80,000
an income earned in India, therefore taxable.
(b) Conducting the feasibility study for the new project in Finland shall not be taxable in India -
as it is for the business outside India.
(c) Money received from a non-resident for use of patent for a business in India shall be 2,30,000
taxable as such patent is applied in India, therefore earned in India.
(d) Payment received for supply of manuals and designs for the business to be established -
in Singapore shall not be taxable in India as it is neither received or earned in India.
(e) Amount received from a trust as rent for a property situated in USA received in Chennai 3,50,000
shall be taxable in India as it is received in India. However standard deduction @ 30%
shall be allowed from the rent. (₹ 5,00,000 – 30%) (refer chapter house property)
Total income of ABC Ltd. In India 10,60,000

Q 20: An individual, who is an Indian resident, is allowed to hold two different citizenships simultaneously. Is the citizenship
a determining factor for residential status of an individual? [ICAI Module]

Sol 20: Citizenship of a country and residential status of that country are separate concepts. A person may be an Indian
citizen, but may not be a resident in India. On the other hand, a person may be a foreign citizen, but may be a resident in
India. The citizenship of an individual has no role in determining the residential status of an individual. The residential status
is determined on the basis of number of days an individual actually stays in India during the previous year.
CA SHREY RATHI RESIDENTIAL STATUS & SCOPE OF TOTAL INCOME 2.10

Q 21: From the following information furnished by Mr. Anjan, compute his total income for the P/Y 2022-23 assuming that
he is a (i) ROR; (ii) RNOR & (iii) NR.
S. No. Particulars (₹)
(a) Profit on sale of shares of Indian company received in Sweden 30,000
(b) Dividend from a Chinese company received in China 12,000
(c) Rent from a property in USA deposited in a bank there and later on remitted to India 1,00,000
through approved banking channels
(d) Agricultural income from land in Punjab 40,000
(e) Technical fees received from Mr. Kumar (a non-resident in India) - (This fees is paid to him 50,000
outside India for providing technical service outside India, however the benefit of
technical service is utilised by Mr. Kumar for carrying on a business in India)
(f) Past foreign untaxed income brought to India during the previous year 34,000
(g) Technical fees received from Mr. Sharma (a non-resident in India) - (This fees is paid to 90,000
him outside India for providing technical service outside India, however the benefit of
technical service is utilised by Mr. Kumar for carrying on a business in Japan)
(i) Dividend from an Indian company received in Hong Kong 11,000
(j) Gifts received from his parents on his birthday 51,000

Sol 21: The income of Mr. Anjan for the P/Y 2022-23 for various status is as under:
S. No. Particulars ROR (₹) RNOR (₹) NR (₹)
(a) Profit on sale of shares of Indian company received in Sweden
(earned in India) 30,000 30,000 30,000
(b) Dividend from a Chinese company received in China (global
income) 12,000 - -
(c) Rent from a property in USA deposited in a bank there and later
on remitted to India through approved banking channels (global 1,00,000 - -
income)
(d) Agricultural income from land in Punjab (exempt u/s 10(1)) - - -
(e) Technical fees received from Mr. Kumar (a non-resident in India)
(This fees is paid to him outside India for providing technical
service outside India, however the benefit of technical service is 50,000 50,000 50,000
utilised by Mr. Kumar for carrying on a business in India) (earned
in India)
(f) Past foreign untaxed income brought to India during the previous - - -
year (exempt)
(g) Technical fees received from Mr. Sharma (a non-resident in India)
(This fees is paid to him outside India for providing technical 90,000 - -
service outside India, however the benefit of technical service is
utilised by Mr. Sharma for carrying on a business in Japan) (global
income)
(h) Dividend from an Indian company received in Hong Kong 11,000 11,000 11,000
(i) Gifts received from his parents on his birthday (gift from parents - - -
is fully exempt) [detailed discussion to happen in IFOS chapter]
Total income of Mr. Anjan 2,63,000 91,000 91,000

Q 22: Mr. Seth, a Korean citizen left India after a continuous stay of 2 years on 01.06.2020. During the financial year 2021-
22, he comes to India for 21 days. Later, he returns to India for 1 year on 10.12.2022. Determine his residential status for
the A/Y 2023-24.

Sol 22: Computation of residential status of Mr. Seth for the A/Y 2023-24:
An individual shall be resident in India if he satisfies any one of the two conditions enlisted u/s 6(1), which are as under:
(a) 182 days or more during the previous year or
(b) 60 days or more in the previous year and 365 days or more in the 4 years immediately preceding the relevant previous
year.
CA SHREY RATHI RESIDENTIAL STATUS & SCOPE OF TOTAL INCOME 2.11

Stay in India during the P/Y 2022-23: December – 22; January – 31; February – 28; March - 31 = 112 days
His stay in India during 4 years immediately preceding the relevant P/Y is as under:
P/Y 2021-22: 21 days
P/Y 2020-21: April – 30; May – 31; June – 1 = 62 days
P/Y 2019-20: 366 days
P/Y 2018-19: 1st June 2017 – 31st March 2018 = 305 days
Total stay in India during 4 years = 21 days + 62 days + 366 days + 305 days = 754 days.
As Mr. Seth satisfied the second basic condition, he shall be resident in India for the P/Y 2022-23.
Further, a resident shall become resident and ordinary resident if he satisfies both the additional conditions u/s 6(6),
otherwise he shall be resident but not ordinary resident. Additional conditions are as under:
i. He has been resident in India in atleast 2 out of 10 previous years immediately preceding the relevant previous year
and
ii. He has been present in India for 730 days or more during 7 years immediately preceding the relevant previous year.
Mr. Seth also satisfies both the additional conditions as he is resident in India in the P/Y 2019-20 & P/Y 2018-19 and his stay
in the previous 7 years is 753 days (i.e. more than 730 days). Therefore, he shall be resident and ordinary resident in India
for the P/Y 2022-23.

Q 23: Determine the taxability of the following incomes in the hand of Mr. Shivansh for the A/Y 2023-24 assuming that he
is a (a) ROR; (b) RNOR; (c) NR.
(i) Fees income received in India for services rendered in Hong Kong. (₹ 3,90,000)
(ii) Income from profession in India but received in Germany. (₹ 3,60,000)
(iii) Profits from business carried on at Nepal but controlled from India. (₹ 2,20,000)
(iv) Income from house property in Srilanka deposited in Bank there. (₹ 1,80,000)
(v) Income from agriculture in England. It is all spent on the education of children in London. (₹ 2,70,000)
(vi) Dividend paid by a foreign company but received in India. (₹ 60,000)
(vii) Pension from a former employer in India received in Uganda. (₹ 50,000) (ignore standard deduction u/s 16(ia) for
this question)
(viii) Royalty received in India from a non-resident in respect of technology used by such person outside India. (₹ 20,000)
(ix) Dividend received from an Indian company. (₹ 15,000)
(x) Gift in foreign currency from a friend received in India. (₹ 90,000)
(xi) Agricultural income in India. (₹ 55,000)
(xii) Profit from a business in Mumbai but managed from outside India. (₹ 8,00,000)
(xiii) Profit on sale of a building in India but received in Sweden. (₹ 4,00,000)
(xiv) Profit of past years (untaxed) brought in this year to India. (₹ 80,000)
(xv) Income from a profession in India and received in India only. (₹ 20,000)
(xvi) Interest on German bonds. [1/5th received in India] (₹ 1,50,000)
(xvii) Income from a business situated in China but controlled from India. [40% received in India and 60% outside India]
(₹ 1,00,000) [IMP. QUES]
CA SHREY RATHI RESIDENTIAL STATUS & SCOPE OF TOTAL INCOME 2.12

Sol 23: The taxable income of Mr. Shivansh for the A/Y 2023-24 for various status is as under:
S. No. Particulars ROR (₹) RNOR (₹) NR (₹)
(i) Fees received in India for services rendered in Hong Kong. 3,90,000 3,90,000 3,90,000
(ii) Income from profession in India but received in Germany. 3,60,000 3,60,000 3,60,000
(iii) Profit from business carried on at Nepal but controlled from 2,20,000 2,20,000 -
India.
(iv) Income from property in Srilanka, deposited in bank there. 1,80,000 - -
(v) Income from agriculture in England. It is all spent on the 2,70,000 - -
education of children in London.
(vi) Dividend paid by a foreign company but received in India. 60,000 60,000 60,000
(vii) Pension from a former employer in India received in Uganda. 50,000 50,000 50,000
(viii) Royalty received in India from a non-resident in respect of 20,000 20,000 20,000
technology used by such person outside India.
(ix) Dividend received from an Indian company. 15,000 15,000 15,000
(x) Gift in foreign currency from a friend received in India. 90,000 90,000 90,000
(xi) Agricultural income in India. - - -
(xii) Profit from a business in Mumbai but managed from outside 8,00,000 8,00,000 8,00,000
India.
(xiii) Profit on sale of a building in India but received in Sweden. 4,00,000 4,00,000 4,00,000
(xiv) Profit of past years (untaxed) brought in this year to India. - - -
(xv) Income from a profession in India and received in India only. 20,000 20,000 20,000
(xvi) Interest on German bonds. [1/5th received in India] 1,50,000 30,000 30,000
(xvii) Income from a business situated in China but controlled from 1,00,000 1,00,000 40,000
India. [40% received in India and 60% outside India]
Total income of Mr. Shivansh 31,25,000 25,55,000 22,75,000

Q 24: Examine with reasons whether the following transactions attract income-tax in India in the hands of recipients:
(i) Salary paid by Central Government to Mr. John, a citizen of India ₹ 7,00,000 for the services rendered outside India.
(ii) Interest on moneys borrowed from outside India ₹ 5,00,000 by a non-resident for the purpose of business within India
say, at Mumbai.
(iii) Post office savings bank Interest of ₹ 19,000 received by a resident assessee, Mr. Ram, aged 46 years.
(iv) Royalty paid by a resident to a non-resident in respect of a business carried on outside India.
(v) Legal charges of ₹ 5,00,000 paid in Delhi to a lawyer of United Kingdom who visited India to represent a case at the
Delhi High Court. [ICAI Module]

Ans:
S. Taxable / Not Amount liable Reason
No. Taxable to tax (₹)
(i) Taxable 6,50,000 As per Section 9(1)(iii), salaries payable by the Government to a citizen of India
for service rendered outside. India shall be deemed to accrue or arise in India.
Therefore, salary paid by central Government to Mr. john for services rendered
outside India would be deemed to accrue or arise in India since he is a citizen of
India. He would be entitled to standard deduction of ₹ 50,000 under Section
16(ia)
(ii) Taxable 5,00,000 As per Section 9(1)(v)(c), Interest payable by a non-resident on moneys
borrowed and used for the purposes of business carried on by such person in
India shall be deemed to accrue or arise in India in the hands of the recipient.
(iii) Partly Taxable 5,500 The Interest on Post Office Savings Bank A/c, would be exempt u/s 10(15(i), only
to the extent of ₹ 3,500 in case of an individual a/c. further, Interest upto ₹
10,000, would be allowed as deduction u/s 80TTA from Gross Total Income.
Balance Rs.5,500 i.e. ₹ 19,000 - ₹ 3,500 - ₹ 10,000 would be taxable in the hands
of Mr. Ram, a resident. [Detailed discussion in income from other sources & deductions
from GTI chapter]
CA SHREY RATHI RESIDENTIAL STATUS & SCOPE OF TOTAL INCOME 2.13

(iv) Not Taxable - Royalty paid by a resident to a non-resident in respect of a business carried
outside India would not be taxable in the hands of the non-resident provided
the same is not received in India. This has been provided as an exception to
deemed accrual mentioned in Section 9(1)(vi)(b).
(v) Taxable 5,00,000 In case of a non-resident, any income which accrues or arises in India or which
is deemed to accrue or arise in India or which is received in India or is deemed
to be received in India is taxable in India.
Therefore, legal charges paid in India to a non-resident lawyer of UK, who visited
India to represent a case at the Delhi High Court would be taxable in India.

Q 25: Mr. Ramesh & Mr. Suresh are brothers and they earned the following incomes during the F.Y. 2022-23. Mr. Ramesh
settled in Canada in the year 1996 and Mr. Suresh settled in Delhi. Compute the total income for the A.Y. 2023-24.
S. No. Particulars Mr. Ramesh Mr. Suresh
(₹) (₹)
1. Interest on Canada Development Bonds (only 50% of Interest received in India) 35,000 40,000
2. Dividend from British company received in London 28,000 20,000
3. Profits from a business in Nagpur, but managed directly from London 1,00,000 1,40,000
4. Short term capital gain on sale of shares of an Indian company received in India 60,000 90,000
5. Income from a business in Chennai 80,000 70,000
6. Fees for technical services rendered in India, but received in Canada 1,00,000 ---
7. Interest on savings bank deposit in UCO Bank, Delhi 7,000 12,000
8. Agricultural income from a land situated in Andhra Pradesh 55,000 45,000
9. Rent received in respect of house property at Bhopal 1,00,000 60,000
10. Life insurance premium paid --- 30,000
[ICAI Module]
Ans: Computation of total income of Mr. Ramesh & Mr. Suresh for the A.Y. 2023-24:
S. No. Particulars Mr. Ramesh Mr. Suresh
(Rs.) (Rs.)
1. Interest on Canada Development Bonds (Note 2) 17,500 40,000
2. Dividend from British company received in London (Note 3) - 20,000
3. Profits from a business in Nagpur, but managed directly from London (Note 2) 1,00,000 1,40,000
4. Short term capital gain on sale of shares of an Indian company received in India 60,000 90,000
(Note 2)
5. Income from a business in Chennai ((Note 2) 80,000 70,000
6. Fees for technical services rendered in India, but received in Canada (Note 2) 1,00,000 -
7. Interest on savings bank deposit in UCO Bank, Delhi (Note 2) 7,000 12,000
8. Agricultural income from a land situated in Andhra Pradesh (Note 4) - -
9. Income from house property at Bhopal (Note 5) 70,000 42,000
Gross Total Income 4,34,500 4,14,000
Less: Deduction under Chapter VI-A (detailed discussion in deductions from GTI
chapter)
• Section 80C – Life insurance premium - 30,000
• Section 80TTA (See Note 6) 7,000 10,000
Total Income 4,27,5000 3,74,000
Notes:
1. Mr. Ramesh is a non-resident since he has been living in Canada since 1996. Mr. Suresh, is settled in Delhi, and thus,
assumed as a resident and ordinarily resident.
2. In case of a resident and ordinarily resident, his global income is taxable as per Section 5(1). However, as per Section
5(2), in case of a non-resident, only the following incomes are chargeable to tax:
(i) Income received or deemed to be received in India; and
(ii) Income accruing or arising or deemed to accrue or arise in India.
Therefore, fees for technical services rendered in India would be taxable in the hands of Mr. Ramesh, even though he
is a non-resident.
CA SHREY RATHI RESIDENTIAL STATUS & SCOPE OF TOTAL INCOME 2.14

The income referred to in S. No. 3, 4, 5 and 7 are taxable in the hands of both Mr. Ramesh and Mr. Suresh since they accrue
or arise/deemed to accrue or arise in India.
Interest on Canada Development Bond would be fully taxable in the hands of Mr. Suresh, whereas only 50%, which is
received in India, is taxable in the hands of Mr. Ramesh.
3. Dividend received from British company in London by Mr. Ramesh, a non-resident, is not taxable since it accrued and is
received outside India. However, such dividend received by Mr. Suresh is taxable, since he is a resident and ordinarily
resident.
4. Agricultural income from a land situated in India is exempt under Section 10(1) in the case of both non-residents and
residents.
5. Income from house property-
Particulars Mr. Ramesh (₹) Mr. Suresh (₹)
Rent received 1,00,000 60,000
Less: Deduction under Section 24(a) @ 30% 30,0000 18,000
Net income from house property 70,000 42,000
The net income from house property in India would be taxable in the hands of both Mr. Ramesh and Mr. Suresh, since the
accrual and receipt of the same are in India.
6. In case of an individual, Interest upto ₹ 10,000 from savings account with, inter alia, a bank is allowable as deduction
under Section 80TTA.

Q 26: Mr. Sushant furnished the following particulars of his income for the year ended 31-03-2023.
S. No. Particulars (₹)
(a) Income earned from business in Dubai which is controlled from Delhi (₹ 65,000 is received 80,000
in India)
(b) Pension for services rendered in India but received in Dubai (computed) 24,000
(c) Dividend from an Oil Company, a Dubai based company, received in Dubai 15,000
(d) Rent from property in Dubai, deposited in a bank in Dubai and later on, remitted to India 70,000
through approved banking channels
(e) Dividend from Sunset Ltd., an Indian company, received in Dubai 78,000
(f) Interest on money borrowed by Mr. Dipish, a non-resident, for the purpose of investment 55,000
in shares of ABC Ltd., an Indian company
(g) Agricultural income from a land in Bhutan, received in India 25,000
Compute his gross total income for the A.Y. 2023-24, if he is:
(a) Resident and ordinarily resident;
(b) Resident but not ordinarily resident;
(c) Non-resident [MTP May 2022]

Sol 26: Computation of Gross Total Income for the A/Y 2023-24
S. No. Particulars ROR (₹) RNOR (₹) NR (₹)
(a) Income earned from business in Dubai which is controlled from 80,000 80,000 65,000
Delhi, out of which ₹ 65,000 is received in India
(b) Pension for services rendered in India but received in Dubai 24,000 24,000 24,000
(computed)
(c) Dividend received in Dubai from an Oil company, a Dubai based 15,000 - -
company
(d) Rent from property in Dubai, deposited in a bank in Dubai 49,000 - -
(e) Dividend from Sunset Ltd., an India Company 78,000 78,000 78,000
(f) Interest on money borrowed by Mr. Dipish, a non-resident, for 55,000 - -
the purpose of investment in shares of ABC Ltd., an Indian
company
(g) Agricultural income from a land in Bhutan, received in India 25,000 25,000 25,000
(Taxable)
Gross Total Income 3,26,000 2,07,000 1,92,000
CA SHREY RATHI RESIDENTIAL STATUS & SCOPE OF TOTAL INCOME 2.15

Notes:
1. As per section 5(1), global income is taxable in case of a resident. However, as per section 5(2), in case of a non-
resident, only the following incomes are chargeable to tax in India:
(a) Income received or deemed to be received in India; and
(b) Income accruing or arising or deemed to accrue or arise in India.
Further, the income which accrues or arise outside India would be chargeable to tax in case of resident but not
ordinarily resident in India, only if such income is derived from a business controlled in India.
Accordingly, the entire income earned from business in Dubai which is controlled from Delhi would be chargeable to
tax in the hands of Mr. Sushant if he is a resident in India or resident but not ordinarily resident. However, if he is non-
resident then only that part of income which is received in India would be taxable in his hands.
2. Agricultural income from a land in Bhutan, received in India is taxable in all cases.
3. Pension for services rendered in India but received in Dubai and dividend from Sunset Ltd., and Indian company
would be taxable in all cases, since it has accrued or arisen in India.
4. Dividend from a Dubai based company, received in Dubai and interest on money borrowed by Mr. Dipish, a non-
resident, for the purpose of investment in shares of ABC Ltd., an Indian company, would be taxable in the hands
of Mr. Sushant, only if he is resident and ordinarily resident in India. If he is a resident but not ordinarily resident
or a non-resident, the same would not be taxable in his hands in India since it has neither accrued nor arisen in
India nor is it received in India.
5. Likewise, rental income from property in Dubai would also be taxable only if he is resident in India. It has been
assumed that the rental income is the gross annual value of the property. Therefore, deduction @ 30% under
section 24, has been provided and the net income so computed is taken into account for determining the gross
total income of a resident and ordinarily resident.
Rent received (assumed as gross annual value) ₹ 70,000
Less: Deduction under section 24 (30% of ₹ 70,000) ₹ 21,000
Income from house property ₹ 49,000

Q 27: Mr. Thomas, a citizen of Japan, comes to India for the first time during the P.Y. 2018-19. During the financial year
2018-19, 2019-20, 2020-21, 2021-22 and 2022-23, he was in India for 50 days, 65 days, 95 days, 150 days and 75 days
respectively. Determine his residential status for the A.Y. 2023-24. Examine the tax implications in the hands of Mr.
Thomas for the Assessment Year 2023-24 of the following transactions entered by him.
(a) Interest received from Mr. Michel, a non-resident outside India (The borrowed fund is used by Mr. Michel for
investing in Indian company’s debt fund for earning interest).
(b) He is also engaged in the business of running news agency and earned income of ₹ 5 lakhs from collection of news
and views in India for transmission outside India.
(c) He entered into an agreement with ABC & Co., a partnership firm for transfer of technical documents and design
and for providing services relating thereto, to set up a Steel manufacturing plant, in India. He charged ₹ 15 lakhs
for these services from ABC & Co. [MTP Nov. 2021]

Sol 27: Under section 6(1), an individual is said to be resident in India in any previous year, if he satisfies any one of
the following conditions:
(i) He has been in India during the previous year for a total period of 182 days or more, or
(ii) He has been in India during the 4 years immediately preceding the previous year for a total period of 365 days or
more and has been in India for at least 60 days in the previous year.
If an individual satisfies any one of the conditions mentioned above, he is a resident. If both the above conditions are
not satisfied the individual is a non-resident.
During the previous year 2022-23, Mr. Thomas was in India for 75 days and during the 4 years preceding the previous
year 2022-23, he was in India for 360 days (i.e. 50 + 65 + 95 + 150 days).
CA SHREY RATHI RESIDENTIAL STATUS & SCOPE OF TOTAL INCOME 2.16

The total stay of the Mr. Thomas during the previous year in India was less than 182 days and during the four years
preceding this year was for 360 days. Therefore, due to non-fulfillment of any of the two conditions for a resident, he
would be treated as non-resident for the Assessment Year 2023-24.
(1) Not taxable, since interest payable by a non-resident to another non-resident would be deemed to accrue or arise
in India only if the borrowed fund is used for the purposes of business or profession carried on by him in India. In
this case, it is used for investing in Indian company’s debt fund for earning interest and not for the purposes of
business or profession. Hence, it is not taxable in India.
(2) No income shall be deemed to accrue or arise to Mr. Thomas through or from activities which are confined to the
collection of news and views in India for transmission outside India. Hence, ₹ 10 lakh is not taxable in India in the
hands of Mr. Thomas.
(3) ₹ 15 lakh is deemed to accrue or arise in India to Mr. Thomas, a non-resident, since it represents royalty/fees for
technical services paid for services utilized in India, in this case, for setting up a Steel manufacturing plant in India.
Hence, the same would be taxable in India in the hands of Mr. Thomas.

MULTIPLE CHOICE QUESTIONS


1. The term ‘stay in India’ includes stay in the territorial waters of India. What does the term “territorial waters of India”
mean?
(a) 24 nautical miles into the sea from the Indian coastline.
(b) 36 nautical miles into the sea from the Indian coastline.
(c) 12 nautical miles into the sea from the Indian coastline.
(d) 15 nautical miles into the sea from the Indian coastline.

2. An individual can be resident in more than one country.


(a) True
(b) False

3. A person is deemed to be of ____________ if he or either of his parents or grandparents were born in Undivided India
(i.e. before 15th August 1947 in India, Pakistan or Bangladesh).
(a) Non – resident (c) Indian origin
(b) Resident (d) Resident but not ordinarily resident

4. Pawan is the karta of the HUF. He has been residing in Dubai since 2006 and takes all decisions regarding the HUF from
Dubai. He came to India for 190 days during the PY 2022-23. Determine the residential status of the Pawan and HUF.
(a) ROR & RNOR (c) RNOR & RNOR
(b) ROR & NR (d) RNOR & NR

5. It is the duty of………………. to place all material facts to determine his correct residential status in front of Assessing
Officer:
(a) Assessee (c) Income tax officer
(b) CBDT (d) All the above

6. Manan, a person of Indian origin stays in India for 110 days every year. But in FY 2022-23, he came to India for 130 days
for visit. His income from Indian sources is ₹ 16 lakhs. His residential status for PY 2022-23 shall be:
(a) ROR (c) Non-resident
(b) RNOR (d) Deemed resident

7. Aman, an Indian citizen, left India for the first time on 23.10.2022 to work as a crew member of a foreign citizen. His
residential status for the PY 2022-23 shall be:
(a) ROR (c) NR
(b) RNOR (d) Depends on signing in & signing off the CDC
CA SHREY RATHI RESIDENTIAL STATUS & SCOPE OF TOTAL INCOME 2.17

8. Justin Beiber, a Canadian singer visits India for 100 days in every financial year. This has been his practice for the past 9
financial years. The residential status for the assessment year 2023–2024 is:
(a) Non – resident (c) Resident and ordinary resident
(b) Resident but not ordinarily resident (d) None of the above

9. Mr. Brett Lee, an Australian citizen, comes to India for the first time during the P.Y. 2018–19. During the financial years
2018–19, 2019-20, 2020-21, 2021-22 & 2022-23, he was in India for 55 days, 60 days, 90 days, 150 days and 70 days,
respectively. The residential status for the assessment year 2022–2023 is:
(a) Resident (c) Resident but not ordinarily resident
(b) Resident and ordinarily resident (d) Non – resident

10. Which of the following shall be taxable for a Non-resident in India?


(a) Sale of property in Switzerland, received in Switzerland
(b) Royalty received from a resident in India outside India for use of technical know-how for a business in India
(c) Interest on bonds of a US company received in Nepal
(d) Income from collection of news for transmission outside India

11. Dolby Technology, a foreign company’s place of effective management is in India in the P.Y. 2022–23. The residential
status of Dolby Technology in A.Y. 2022-23 is _______.
(a) Non-resident (c) Resident but not ordinarily resident
(b) Resident and ordinarily resident (d) None of the above

12. A Partnership firm would be non-resident in India, if the control and management of the affairs is situated wholly outside
India.
(a) True
(b) False

13. Dev is a foreign citizen. His grandfather was born in India in 1945. He left India for the first time on 24.07.22 for the
purpose of employment. His residential status shall be:
(a) ROR (c) NR
(b) RNOR (d) Can’t be determined due to insufficient information

14. Every year, the residential status of an assessee:


(a) May change (c) Will not change
(b) Will certainly change (d) None of the above

15. Mr. Nishant Khurana earns the following income during the financial year 2022-23:
Particulars (₹)
1. Past untaxed profits of UK business of 2019-20 brought into India in 2022-23 90,000
2. Interest on Government securities accrued in India but received in Paris 80,000
3. Interest on USA Government securities, received in India 20,000
Compute his income presuming that he is ROR, RNOR and NR.
(a) ₹ 1,00,000 each (c) ₹ 80,000 each
(b) ₹ 1,90,000 each (d) ₹ 1,10,000 each

16. Miss Priyanka Chopra, an actor is a resident of U.S. She visited India in P.Y. 2022-23 for 10 days. She received her fees
of “advertisements” in U.S. The shooting of advertisement was done in Delhi. The income is _________ in A.Y. 2023–
24.
(a) Taxable (c) 50% taxable, 50% exempt
(b) Exempt (d) Depends upon the assessing officer

17. Miss Shruti, is a resident but not ordinarily resident of India. In P.Y. 2022–23, she received rent from one of a property
in London which is deposited in a bank in London, which is later on remitted to India through approved banking
channels. The income is ____________ in A.Y. 2023–24.
(a) Non-taxable (c) Taxable
(b) Partial taxable (d) None of the above
CA SHREY RATHI RESIDENTIAL STATUS & SCOPE OF TOTAL INCOME 2.18

18. Where a non-resident has a business connection in India but its operations are confined to purchase of goods in India
for the purpose of export, such income shall:
(a) be taxable in India as it is deemed to accrue or arise in India
(b) not be taxable in India as it shall not be deemed to accrue or arise in India
(c) be partial taxable as it is accrues or arises in India
(d) none of the above

19. Mr. Anirudh Verma earns the following income during the financial year 2022-23:
Particulars (₹)
1. Income from house property in London, received in India 60,000
2. Profits from business in Japan and managed from there (received in Japan) 9,00,000
3. Dividend from foreign company, received in India 30,000
4. Dividend from Indian company, received in England 50,000
Compute his income presuming that he is ROR, RNOR and NR.
(a) 9,90,000 / 90,000 / 90,000 (c) 10,40,000 / 1,40,000 / 1,40,000
(b) 10,40,000 / 90,000 / 90,000 (d) 10,40,000 / 10,40,000 / 1,40,000

20. Atul is a foreign citizen. His father was born in Delhi in 1951 and mother was born in England in 1950. His grandfather
was born in Delhi in 1922. Atul visited India to see Taj mahal and visit other historical places. He came to India on 1 st
November, 2022 for 200 days. He has never come to India before. His residential status for assessment year 2023-24
will be-
(a) Non-resident India (c) Resident in India
(b) Not ordinarily resident in India (d) None of the above.

21. Paresh, a software engineer at ABC Ltd. left India on 10th August, 2022 for the treatment of his wife. For income–tax
purpose, his residential status for the assessment year 2023-24 will be:
(a) Resident (c) Non-resident
(b) Not ordinarily resident (d) Cannot be determined from the given information.

22. HUF of Ashwin consisting of himself, his wife and 2 sons is assessed to income- tax. The residential status of HUF would
be non-resident, when:
(a) The management and control of its affairs are wholly situated in India.
(b) The management and control of its affairs is wholly situated outside India.
(c) The status of karta is non-resident for that year.
(d) When majority of the members are non- residents.

23. Alpha Ltd. is an Indian company. It carries its business in Delhi and London. The place of effective management of the
company is situated in London. More than 85% of its business income is from the business in England. If so, its residential
status will be:
(a) Resident (c) Not ordinarily resident
(b) Non- resident (d) foreign company.

24. Thomas Inc. of Australia borrowed money from various companies in Australia for doing business in India by Name ANS
Co. Ltd., Mumbai. Thomas Inc. paid interest of ₹ 500 lakhs (converted) to various lenders. The amount of interest paid:
(a) Has accrued in India (c) Is exempt from tax
(b) Does not accrue in India (d) Is taxable in Australia

25. John is a foreign citizen born in USA. His father was born in Delhi in 1960 and his grand-father was born in Lahore in
1935 but his mother was born in UK in 1963. John came to India for the first time on 1st December, 2022 and stayed in
India for 183 days and then left for USA. His residential status for the A.Y. 2023-24 shall be:
(a) Resident (c) Non-resident
(b) Resident but not ordinary resident (d) Foreign national
CA SHREY RATHI RESIDENTIAL STATUS & SCOPE OF TOTAL INCOME 2.19

26. AB was born 5th April 1999 in India and he later on took the citizenship of U.S.A. Neither his parents nor his grandparents
were born in divided / undivided India. AB in this case shall be a:
(a) Citizen of India (c) Person of Indian Origin
(b) Foreign national (d) Resident of India

27. Dividend from British Co. of ₹ 2,00,000 received in London will be taxable in case of:
(a) Resident and ordinary resident (ROR) only (c) Not ordinary resident (NOR) only
(b) Non-resident (NR) only (d) ROR, NOR and NR all

28. Praful has stayed in India in the P.Y. 2022-23 for 121 days. And he is non-resident in 10 out of 10 years immediately
preceding the current previous year and he has stayed in India for 465 days in all in the 4 years immediately preceding
the current previous year and 820 days in all in the 7 years immediately preceding the current previous year, his
residential status for the A.Y. 2023-24 would be:
(a) Resident and ordinarily resident (c) Resident but not ordinarily resident
(b) Non-resident (d) Cannot be ascertained with the given information

29. Rohan, an Indian citizen was employed in Federal Bank in India. He received a salary of ₹ 50,000 p.m. from 1.4.2022 to
25.8.2022. He resigned and left for Sharjah for the first time on 1.9.2022 and got salary of rupee equivalent of ₹ 90,000
p.m. from 1.09.2022 to 31.3.2023. His entire salary from September to March 2023 was credited in his Sharjah bank
account. He remitted the salary for November & December 2022 to India in January 2023. He is liable to tax in respect
of:
(a) Income received in India from Federal Bank;
(b) Income received in India and in Sharjah;
(c) Income received in India from Federal Bank and income remitted to India;
(d) Income received in Sharjah

30. Income earned from a contract negotiated by an agent in India in the name of a non-resident but approved by such
non-resident shall:
(a) Be taxable in India as such income is deemed to accrue or arise in India
(b) Not be taxable in India as there is no business connection in India
(c) Be taxable in India only if it is received in India
(d) Be taxable in India as such income accrues or arises in India

31. Fees for technical services paid by the Central Government will be taxable in case of:
(a) Resident and ordinarily resident only
(b) Both resident and ordinarily resident and resident but not ordinarily resident
(c) Non- resident
(d) All the above

32. Short term capital gains on sale of shares of an Indian company received in Australia is taxable in case of
(a) Resident and ordinarily resident only
(b) Both resident and ordinarily resident and resident but not ordinarily resident
(c) Non- resident only
(d) All the above

33. Income from a business in Canada, controlled from Canada is taxable in case of
(a) Resident and ordinarily resident only
(b) Both resident and ordinarily resident and resident but not ordinarily resident
(c) Non- resident
(d) All the above

34. Mr. J is Citizen of India and left India for USA on 16/8/2021 for doing business meetings on behalf of his employer, which
is an India company, for exporting goods to USA. He came back to India on 15/9/2022. He has been resident in India for
past 10 years. For the PY 2022-2023, Mr. J shall be:
(a) Resident and ordinarily resident in India (c) Resident but not ordinarily in India
(b) Non-resident in India (d) Non-citizen of India
CA SHREY RATHI RESIDENTIAL STATUS & SCOPE OF TOTAL INCOME 2.20

35. Mr. Sharma is an Indian citizen. He stays in India every year for 50 days only. His having income (other than foreign
sources) of ₹ 16 lakhs shall be considered as …………. assuming that he is not a resident of any other country.
(a) Resident and ordinary resident (c) Deemed Resident (RNOR)
(b) Non-Residen (d) None of the above

36. In the case of an individual being not ordinarily resident, which of the following Income is chargeable to tax in India:
(a) Business income accruing outside India
(b) Income accruing outside India if it is derived from a business controlled in India
(c) Property income accruing outside India
(d) Interest income accruing outside India

37. Mr. R is a foreign citizen. His father was born in Delhi in 1961 and mother was born in England in 1955. His grandfather
was born in Dhaka in 1932. Mr. R visited India to see historical places. He came to India on 1st December, 2022 for 150
days. He has never come to India before. His residential status for P/Y 2022-2023 will be:
(a) Non-resident in India (c) Not ordinarily resident in India
(b) Resident and ordinary resident in India (d) None of the above

38. Dividend income from a German company received in Australia in the year 2019, brought to India during the previous
year 2022-23 is taxable in case of-
(a) Resident and ordinarily resident only (c) Resident but not ordinarily resident
(b) Non-resident (d) Not taxable for any status

39. Mr. Verma a person of India origin visited India on 5/2/2023 and plans to stay here for 210 days. During 4 years prior to
the PY 2022-2023, he was in India for 950 days. Earlier to that he was never in India. For the AY 2023-2024, Mr. Verma
shall be:
(a) Resident and ordinarily resident in India (c) Resident but not ordinarily resident in India
(b) Non-resident (d) None of the above

40. Royalty paid by a resident in India to a non-resident outside India:


(a) Shall be taxable in India unless it is paid for the acquisition of any right/information/ property used for the purpose
of business or profession carried on outside India.
(b) Shall be taxable in India whether such right/ information/property used for the purpose of carrying on business or
profession in India or outside India.
(c) Shall not be taxable in India.
(d) Always taxable in India

41. Determine the residential status of a HUF if HUF's control and management is wholly situated in India and Karta of HUF
is a non-resident in India for that previous year.
(a) Resident and Ordinary Resident (ROR) (c) Resident but not ordinary resident (NOR)
(b) Non-Resident (NR) (d) ROR or RNOR

42. Salary of a non-resident received for the period in which he is working in India is not taxable.
(a) True
(b) False

43. Dividend of ₹ 5,000 was received from foreign company, outside India by a Resident and Ordinary Resident of India,
out of which ₹ 4,000 is remitted to India. Income taxable in India will be—
(a) ₹ 1,000 (c) ₹ 4,000
(b) ₹ 5,000 (d) Nil

44. Fee for technical services paid outside India by a non-resident in India to a non-resident in India shall:
(a) Never taxable in India
(b) Always taxable in India
(c) Taxable in India provided the know-how, for which royalty was paid, was used for the purpose of carrying on any
business or profession in India
(d) None of the above
CA SHREY RATHI RESIDENTIAL STATUS & SCOPE OF TOTAL INCOME 2.21

45. Salary payable by a foreign company to an Indian citizen for services rendered outside India is an income, which is
deemed to accrue or arise in India.
(a) True
(b) False

46. Mrs. Sunita Williams (citizen of USA), a famous astronaut, comes to India on 31/10/2022 to visit her grandmother who
was born in Gujarat in 1935. She stays in India till 26/1/2023 and then leaves for USA. Her residential status for the PY
2022-23 will be:
(a) Person of Indian origin (c) Resident and not ordinary resident
(b) Resident and ordinary resident (d) Non resident

47. HUF is situated in Mumbai. Its karta is Mr. J. who is of 95 years of age has delegated power to his eldest son Mr. Y. Mr.
J. is in USA for his medical treatment and left India for the first time on 18/9/2021. Mr. Y has full controls over affairs of
HUF. Mr. Y excises partial control from India and partially from Nepal. Calculate residential status of HUF and Karta Mr.
J for the PY 2022-23:
(a) HUF is resident, Mr. J is non resident
(b) HUF is non-resident, Mr. J is non resident
(c) HUF is resident and ordinary resident, Mr. J is non resident
(d) HUF is resident and ordinary resident, Mr. J is resident and ordinary resident

48. A foreign Company having no Indian citizen/ resident of India as shareholder has shot a TV film entirely on Indian
locations. The film is to be telecast exclusively in the foreign countries. But it has also agreed with the Government of
India to give the rights of telecast in India, free of charge. Whether the above transaction will rise to income which will
be deemed to accrue or arise in India.
(a) Yes
(b) No

49. Income earned from a contract negotiated by an agent in Srilanka in the name of a non-resident but approved by such
non-resident shall:
(a) be taxable in India as such income accrues or arises in India
(b) not be taxable in India as there is no business connection in India
(c) be taxable in India only if it is received in India
(d) none of the above

50. Mr. X, an NRI, engaged in the business of running a news channel. His business is confined to the collection of news and
views in India for transmission outside India. Such income is
(a) Fully taxable (c) 50% Taxable – 50% Exempt
(b) Not deemed to accrue or arise in India (d) Depends on the assessing officer

51. If Anirudh, a citizen of India, has stayed in India in the P.Y. 2022-23 for 181 days, and he is non-resident in 9 out of 10
years immediately preceding the current previous year and he has stayed in India for 365 days in all in the 4 years
immediately preceding the current previous years and 420 days in all in the 7 years immediately preceding the current
previous year, his residential status for the A.Y. 2022-23 would be – (ICAI MCQ Booklet)
(a) Resident and ordinarily resident (c) Non-resident
(b) Resident but not ordinarily resident (d) Deemed resident but not ordinarily resident

52. Raman, a citizen of India, was employed in Hindustan Lever Ltd. he resigned on 27-09-2022. He received a salary of ₹
40,000 p.m. from 1-4-2022 to 27-9-2022 from Hindustan Lever Ltd. thereafter he left for Dubai for the first time on 1-
10-2022 and got salary of rupee equivalent of ₹ 80,000 p.m. from 1-10-2022 to 31-03-2023 in Dubai. His salary for
October to December 2022 was credited in his bank account in Dubai and the salary for January to March 2023 was
credited in his Mumbai account directly. He is liable to tax in respect of – (ICAI MCQ Booklet)
(a) Income received in India from Hindustan Lever Ltd;
(b) Income received in India and Dubai;
(c) Income received in India from Hindustan Lever Ltd and income directly credited in India;
(d) Income received in Dubai
CA SHREY RATHI RESIDENTIAL STATUS & SCOPE OF TOTAL INCOME 2.22

53. Aashish earns the following income during the P.Y. 2022-23:
• Interest on U.K. Development Bonds (1/4th being received in India): ₹ 4,00,000
• Capital gain on sale of a building located in India but received in Holland: ₹ 6,00,000
If Aashish is a resident but not ordinarily resident in India, then what will be amount of Income chargeable to tax in
India for A.Y. 2023-24? (ICAI MCQ Booklet)
(a) ₹ 7,00,000 (c) ₹ 6,00,000
(b) ₹ 10,00,000 (d) ₹ 1,00,000

54. Mr. Sumit is an Indian citizen and a member of the crew of an America bound Indian ship engaged in carriage of freight
in international traffic departing from Chennai on 25th April, 2022. From the following details for the P.Y. 2022-23, what
would be the residential status of Mr. Sumit for A.Y. 2023-24, assuming that his stay in India in the last 4 previous years
preceding P.Y. 2022-23 is 365 days and last seven previous years preceding P.Y. 2022-23 is 730 days?
• Date entered in the Continuous Discharge Certificate in respect of joining the ship by Mr. Sumit: 25th April, 2022
• Date entered in the Continuous Discharge Certificate in respect of signing off the ship by Mr. Sumit: 24 th October,
2022.
Mr. Sumit has been filing his income tax return in India as a Resident for the preceding 2 previous years. (ICAI MCQ
Booklet)
(a) Resident and ordinarily resident (c) Non-resident
(b) Resident but not-ordinarily resident (d) Deemed resident but not-ordinarily resident

55. Mr. Square, an Indian citizen, currently resides in Dubai. He came to India on a visit and his total stay in India during the
F.Y. 2022-23 was 135 days. He is not liable to pay any tax in Dubai. Following is his details of stay in India in the preceding
previous years:
Financial Year Days of Stay in India
2021-22 100
2020-21 125
2019-20 106
2018-19 83
2017-18 78
2016-17 37
2015-16 40
What shall be his residential status for the P.Y. 2022-23 if his income (other than income from foreign sources) is ₹ 10
lakhs? (ICAI MCQ Booklet)
(a) Resident and Ordinary Resident (c) Non-Resident
(b) Resident but not Ordinary Resident (d) Deemed resident but not ordinarily resident.

56. Dividend income from Australian company received in Australia in the year 2021, brought to India during the previous
year 2022-23 is taxable in the A.Y. 2023-24 in the case of – (ICAI MCQ Booklet)
(a) Resident and ordinarily resident only
(b) Both resident and ordinarily resident and resident but not ordinarily resident
(c) Non-resident
(d) None of the above

57. Mr. Ramesh, a citizen of India, is employed in the Indian embassy in Australia. He is a non-resident for the A.Y. 2023-24.
He received salary and allowances in the Australia from the Government of India for the year ended 31-03-2023 for
services rendered by him in Australia. In addition, he was allowed perquisites by the Government. Which of the
following statements are correct? (ICAI MCQ Booklet)
(a) Salary, allowances and perquisites received outside India are not taxable in the hands of Mr. Ramesh, since he is
non-resident
(b) Salary, allowances and perquisites received outside India by Mr. Ramesh are taxable in India since they are deemed
to accrue or arise in India.
(c) Salary received by Mr. Ramesh is taxable in India but allowances and perquisites are exempt.
(d) Salary received by Mr. Ramesh is exempt in India but allowances and perquisites are taxable.
CA SHREY RATHI RESIDENTIAL STATUS & SCOPE OF TOTAL INCOME 2.23

58. Who among the following will qualify as non-resident for the previous year 2022-23? (ICAI MCQ Booklet)
- Mr. Joey, an Italian designer came on visit to India to explore Indian handloom on 03-09-2022 and left on 15-12-
2022. For past four years, he visited India for fashion shows and stayed in India for 100 days each year.
- Mr. Sanjay born and settled in Canada, visits India each year for three months to meet his parents and grandparents,
born in India in 1946, living in Mumbai. His Indian income is ₹ 15,20,000.
- Mr. Chang, a Korean scientist left India to his home country for fixed employment there. He stayed in India for study
and research in medicines from 01-01-2018 till01-07-2022.
Choose the correct answer: (ICAI MCQ Booklet)
(a) Mr. joey and Mr. Chang (c) Mr. Sanjay and Mr. Chang
(b) Mr. Sanjay (d) Mr. Chang

59. Mr. Harry, an Indian citizen, is a marketing consultant who provides consultancy to various countries around the globe.
Due to his profession, he is required to travel across various countries throughout the year. His marketing project does
not last for more than 40 days and therefore his stay in any country including India usually never exceeds 40 days during
a year. His income is ₹ 80 lakhs across the globe which is not liable to tax in any country. During the P.Y. 2022-23 an
Indian company provides him a marketing project in India. His stay in India for the project is expected to be only 25 days
and his income from that project would be ₹ 30 lakhs. Being a highly qualified professional, he consults you about the
tax regime on his income and his residential status in India. (ICAI MCQ Booklet)
(a) He shall be treated as resident but not ordinarily resident and shall be liable to pay tax on ₹ 30 lakhs.
(b) He shall be treated as resident and ordinarily resident and shall be liable to pay tax on ₹ 30 lakhs.
(c) He shall be treated as non-resident and shall not be liable to tax.
(d) He shall be treated as resident but not ordinarily resident and shall be liable to pay tax on his entire income of ₹ 80
lakhs earned across the globe.

60. Determine residential status of Sundaram (HUF) which carries out its transactions in Malaysia. Its affairs are partly
controlled from India. The Karta of HUF, Mr. Sundaram who is from Chennai visits India on 01-06-2021 and leaves to
Malaysia on 10-02-2022. He has not visited India for the past 11 years. (ICAI MCQ Booklet)
(a) Resident and ordinarily resident (c) Deemed resident
(b) Resident but not ordinarily resident (d) Non-resident

MCQs Answers
1. (c) 2. (a) 3. (c) 4. (d) 5. (a) 6. (b) 7. (a) 8. (b) 9. (d) 10. (b)
11. (b) 12. (a) 13. (a) 14. (a) 15. (a) 16. (a) 17. (a) 18. (b) 19. (c) 20. (a)
21. (a) 22. (b) 23. (a) 24. (a) 25. (c) 26. (b) 27. (a) 28. (c) 29. (a) 30. (a)
31. (d) 32. (d) 33. (a) 34. (a) 35. (c) 36. (b) 37. (a) 38. (d) 39. (b) 40. (a)
41. (d) 42. (b) 43. (b) 44. (c) 45. (b) 46. (d) 47. (c) 48. (b) 49. (b) 50. (b)
51. (b) 52. (b) 53. (a) 54. (a) 55. (c) 56. (d) 57. (c) 58. (b) 59. (a) 60. (b)
CA SHREY RATHI SALARY 3.1

APJ ABDUL KALAM

CHAPTER 3
SALARY

"Dream is not that you see while


sleeping, it is something that does
not let you sleep”

PRACTICAL QUESTIONS
Q 1: Mr. Rakesh draws a salary of ₹ 25,000 p.m. He is also given ₹ 3,000 p.m. as conveyance allowance, out of which he
spends only ₹ 1,500 quarterly. His employer gave him the choice to pick one of the two alternatives given below:
(a) House rent allowance of ₹ 15,000 p.m. or
(b) Rent-free unfurnished flat in Delhi for which the rent of ₹ 15,000 p.m. shall be paid by the company.
Advise Rakesh from the point of view tax saving assuming that he will have to pay ₹ 12,000 p.m. as rent if he opts for HRA.
Also compute gross salary under the choice made.

Sol 1: Computation of gross salary for Mr. Rakesh and the best choice for him:
Particulars HRA (₹) RFA (₹)
Basic salary (₹ 25,000 x 12) 3,00,000 3,00,000
Conveyance allowance [(₹ 3,000 x 12) – (₹ 1,500 x 4)] 30,000 30,000
House rent allowance [WN 1] 66,000 -
Rent free accommodation [WN 2] - 49,500
Gross salary 3,96,000 3,79,500
Advice: Rakesh should choose RFA as tax liability shall be low.
WN 1: Computation of taxable HRA:
HRA is exempt to the extent of minimum of the following:
(a) Actual HRA received = ₹ 1,80,000
(b) Rent paid – 10% of Salary [(₹ 12,000 x 12) – (10% of ₹ 3,00,000)] = ₹ 1,14,000
(c) 50% of Salary (50% of ₹ 3,00,000) = ₹ 1,50,000
HRA exempt = ₹ 1,14,000
Taxable HRA = HRA received – Exempt amount
= ₹ 1,80,000 – ₹ 1,14,000 = ₹ 66,000
CA SHREY RATHI SALARY 3.2

WN 2: Computation of taxable RFA: As the employer does not own the flat, taxable value shall be 15% of salary or lease
rent whichever is lower:
15% [₹ 3,00,000 + ₹ 30,000] = ₹ 49,500 or lease rent = ₹ 1,80,000 (₹ 15,000 x 12) whichever is lower
Taxable RFA = ₹ 49,500

Q 2: Ms. Sandhya was employed by XYZ Ltd. She retired w.e.f 1st February, 2023 after completing a service of 24 years and
5 months. She submits the following information:
1) Basic salary - ₹ 10,000 p.m. (at the time of retirement)
2) Dearness allowance - 120% of basic salary (40% of basic salary forms part of salary as per terms of
employment)
3) Last increment - ₹ 1,000 w.e.f 1st July, 2022
Her pension was determined to be ₹ 6,000 p.m. She got 50% of the pension commuted w.e.f. 1st March, 2023 and
received a sum of ₹ 2,00,000 as commuted pension. In addition to this, she received gratuity of ₹ 2,40,000 and leave
encashment amounting to ₹ 1,12,000 on account of accumulated leaves of 240 days. She was entitled to 40 days leave for
every year of service. Compute her gross salary for the A/Y 2023-24.

Sol 2: Computation of gross salary of Ms. Sandhya for the A/Y 2023-24:
Particulars (₹)
Basic salary (₹ 9,000 x 3) + (₹ 10,000 x 7) 97,000
Dearness allowance (₹ 97,000 x 120%) 1,16,400
Pension [WN 1] 75,667
Gratuity [WN 2] 77,040
Leave salary [WN 3] 1,12,000
Gross salary 4,78,107
WN 1: Computation of taxable pension:
Uncommuted (recurring) pension = (₹ 6,000 x 1) + (₹ 3000 x 1) = ₹ 9,000
Commuted pension: As gratuity is also received, 1/3rd of 100% commuted pension shall be exempt.
Commuted pension received = ₹ 2,00,000 = 50%, therefore ₹ 4,00,000 = 100%.
Exempt amount = ₹ 4,00,000 x 1/3 = ₹ 1,33,333
Taxable commuted pension = Commuted pension received – exempt amount
= ₹ 2,00,000 – ₹ 1,33,333 = ₹ 66,667
Total pension taxable = ₹ 9,000 + ₹ 66,667 = ₹ 75,667

WN 2: Computation of taxable gratuity:


Least of the following shall be exempt: (not covered under the Gratuity Act, 1972)
(i) Actual gratuity received ₹ 2,40,000
(ii) Limit ₹ 20,00,000
(iii) 15/30 x average salary x number of years of service (excluding fractions) ₹ 1,62,960
(15/30 x ₹ 13,580 x 24)
Exempt amount = ₹ 1,62,960
Taxable gratuity = Amount received as gratuity – exempt amount
= ₹ 2,40,000 – ₹ 1,62,960 = ₹ 77,040
Calculation of average salary for gratuity: Average monthly salary is calculated on the basis of average salary for the 10
months immediately preceding the month in which the employee has retired.
Basic salary = (₹ 9,000 x 3) + (₹ 10,000 x 7) - ₹ 97,000
Dearness allowance (forming part of salary) = (₹ 97,000 x 40%) - ₹ 38,800
Total salary - ₹ 1,35,800
Average salary = ₹ 1,35,800 / 10 = ₹ 13,580
CA SHREY RATHI SALARY 3.3

WN 3: Computation of taxable leave salary


Least of the following shall be exempt:
1. Actual leave salary received ₹ 1,12,000
2. Limit ₹ 3,00,000
3. 10 months average salary (10 x ₹ 13,580) ₹ 1,35,800
4. Cash equivalent to unavailed leaves
(i) Leaves actually allowed or 30 days per year whichever is less = 30 days x 24 = 720 days
(ii) Leaves actually taken = 720 days [(40 days x 24) – 240]
(iii) (720 - 720) x average salary/30 = Nil
Exempt amount = Nil
Taxable leave salary = Actual leave salary – Exempt amount
= ₹ 1,12,000 – Nil = ₹ 1,12,000
Calculation of average salary for leave salary: Average salary is to be calculated on the basis of salary drawn by the
employee during the period of 10 months immediately preceding the date of retirement. It shall be same as computed for
gratuity.

Q 3: Mr. Manoj is a production manager of a company. During the P/Y 2022-23, he gets the following emoluments from
his employer:
(A) Basic salary: Upto 31st August, 2022 ₹ 20,000 p.m.
From 1st September, 2022 ₹ 25,000 p.m.
(B) Transport allowance ₹ 2,800 p.m.
(C) Employer’s contribution to RPF 15% of basic salary
(D) Children education allowance ₹ 500 p.m. for two children
(E) City compensatory allowance ₹ 300 p.m.
(F) Hostel allowance ₹ 380 p.m. per child for two children
(G) Tiffin allowance (actual expenses ₹ 3,700) ₹ 5,000 p.a.
(H) Tax paid on employment by the employer ₹ 2,500
Compute his taxable salary for the A/Y 2023-24. [PYQ Nov 2008]

Sol 3: Computation of taxable salary of Mr. Manoj for the A/Y 2023-24:
Particulars (₹)
Basic salary [(₹ 20,000 x 5) + (₹ 25,000 x 7)] 2,75,000
Transport allowance (₹ 2,800 x 12) 33,600
Employer’s contribution to RPF [(₹ 2,75,000 x 15%) – (₹ 2,75,000 x 12%)] 8,250
Children education allowance [(₹ 500 x 12) – (₹ 100 x 12 x 2)] 3,600
City compensatory allowance (₹ 300 x 12) 3,600
Hostel allowance [(₹ 380 x 12 x 2) – (₹ 300 x 12 x 2)] 1,920
Tiffin allowance 5,000
Tax paid on employment by the employer 2,500
Gross salary 3,33,470
Less: Standard Deduction u/s 16(ia) 50,000
Less: Deduction u/s 16(iii) of Professional tax 2,500
Taxable salary of Mr. Manoj 2,80,970
CA SHREY RATHI SALARY 3.4

Q 4: Mr. Madan Lal is employed in VST Ltd. in Calcutta on a monthly salary of ₹ 20,000. In addition to his fixed salary, he is
entitled to a commission @ 5% on the sales made by him. During the P/Y 2022-23, he had received following allowances
and amenities from his employer:
(i) Dearness allowance @ ₹ 2,000 p.m. which is granted to him under the terms of employment.
(ii) Bonus equal to two months basic salary.
(iii) House rent allowance @ ₹ 5,000 p.m.
(iv) Entertainment allowance @ ₹ 250 p.m.
(v) The company paid ₹ 1,000 as his Income-tax penalty.
(vi) In September 2022, during leave he went on a visit to Kashmir with his family. The expenditure amounting to ₹
16,000 as passage money by air were paid to him by the employer as leave travel concession. Had he travelled by
air-conditioned first-class compartment by train, the expenditure in this regard would have been only ₹ 14,000.
Before this he was allowed travel concession 1999, 2005, 2009, 2017. He is not entitled to travel by air.
(vii) He had been provided with the amenities of gas, electricity and water, the expenses of which amounting to ₹
12,000 were paid by the company.
(viii) Commission on sales of ₹ 10,00,000 @ 5%.
(ix) He was given a timex watch worth ₹ 9,000 by his employer on the foundation day of the company.
(x) He and his employer each contributed 12.5% of his salary to recognised provident fund. The interest credited to this
fund for the P/Y at 13.5% rate of interest amounted to ₹ 27,000.
Compute the taxable income from salary of Mr. Madan Lal for the A/Y 2023-24 keeping in mind that he spent ₹ 6,000 p.m.
as rent of the house hired by him.

Sol 4: Computation of taxable salary of Mr. Madan Lal for the A/Y 2023-24:
Particulars (₹)
Basic salary (₹ 20,000 x 12) 2,40,000
Commission on sales (₹ 10,00,000 x 5%) 50,000
Dearness allowance (under terms of employment) (₹ 2,000 x 12) 24,000
Bonus = 2 months’ basic salary (₹ 20,000 x 2) 40,000
House rent allowance [WN 1] 19,400
Entertainment allowance (₹ 250 x 12) 3,000
Income tax penalty paid by the company 1,000
Leave travel concession (₹ 16,000 – ₹ 14,000) 2,000
Gas, electricity & water expenses 12,000
Gift (₹ 9,000 – ₹ 5,000) 4,000
Employer’s contribution to RPF [0.5% (₹ 2,40,000 + ₹ 50,000 + ₹ 24,000)] 1,570
Interest credited to RPF [(₹ 27,000/13.5) x 4] 8,000
Gross salary 4,04,970
Less: Standard deduction u/s 16(ia) 50,000
Taxable salary of Mr. Madan Lal 3,54,970
WN 1: Computation of taxable HRA:
HRA is exempt to the extent of minimum of the following:
(a) Actual HRA received = ₹ 60,000
(b) Rent paid – 10% of Salary [(₹ 6,000 x 12) – (10% of ₹ 3,14,000)] = ₹ 40,600
(c) 50% of Salary (50% of ₹ 3,14,000) = ₹ 1,57,000
HRA exempt = ₹ 40,600
Taxable HRA = HRA received – Exempt amount
= ₹ 60,000 – ₹ 40,600 = ₹ 19,400
Salary for HRA shall consist of Basic salary + DA (under terms of employment) + Commission based on fixed % of turnover
= ₹ 2,40,000 + ₹ 24,000 + ₹ 50,000 = ₹ 3,14,000.
CA SHREY RATHI SALARY 3.5

Q 5: Smt. Jhanvi is the manager of TCS Ltd. She receives every month ₹ 12,000 as basic pay, ₹ 600 as entertainment
allowance and ₹ 3,000 as dearness allowance.
She also receives the following benefits:
(a) She owns a house, but the company has provided her the following amenities:
(i) A gardener, a sweeper and a watchman who are paid ₹ 1,000 p.m. each.
(ii) Free use of the air conditioner costing ₹ 15,000.
(b) The following obligations of her were paid by the company:
(i) Gas, electricity and water bills amounting to ₹ 12,000.
(ii) Annual membership fee of Rotary Club ₹ 6,000.
(c) The company has provided the facility of a large car. The car is used both for private and personal purpose and all
the expenses including the driver’s salary are borne by the company.
(d) Her son is studying free in a school run by the company. The annual expenses incurred by the company for son is ₹
9,800.
(e) She was sent to attend a training course at Nagpur for 20 days, the fee of which was paid by the company ₹ 5,000.
(f) She proceeded on one month’s leave to Shimla by car where she stayed in the holiday home maintained by the
company. The expenses of her boarding and lodging amounting to ₹ 12,500 were borne by the company.
(g) The company allotted her 100 shares at ₹ 100 each whereas the fair market value per share on the date of offer
was ₹ 150 per share.
Compute her gross salary for the A/Y 2023-24.

Sol 5: Computation of taxable salary of Smt. Jhanvi for the A/Y 2023-24:
Particulars (₹)
Basic salary (₹ 12,000 x 12) 1,44,000
Entertainment allowance (₹ 600 x 12) 7,200
Dearness allowance (₹ 3,000 x 12) 36,000
Gardener, sweeper & watchman (₹ 1,000 x 12 x 3) 36,000
Use of air conditioner (₹ 15,000 x 10%) 1,500
Gas, electricity and water expenses 12,000
Annual membership fee of rotary club 6,000
Motor car provided by the employer [(₹ 2,400 + ₹ 900) x 12] 39,600
Education facility provided by the employer (₹ 9,800 – ₹ 9,800) -
Training course at Nagpur (official purpose) -
Boarding and lodging expenses borne by the company 12,500
Shares allotted by the company [(₹ 150 – ₹ 100) x 100] 5,000
Gross salary 2,99,800

Q 6: Mr. Ram is an employee of a company working in Raipur. He submits the following information about his income for
the P/Y 2021-22:
(i) Basic salary – ₹ 20,000 p.m.
(ii) Bonus – ₹ 12,000
(iii) Dearness allowance – ₹ 4,000 p.m. (50% forms part of salary for retirement benefits)
(iv) Travelling allowance – ₹ 56,000. He spends ₹ 26,000 for official purposes.
(v) Reimbursement of medical bills – ₹ 27,000 (treatment was done in a Government hospital in India)
(vi) Children hostel allowance (2 children) – ₹ 8,000 p.a.
(vii) He was provided a flat by the company. Fair rent of the flat is ₹ 20,000 p.m. The company also provided with the
services of a watchman and a cook paying a salary of ₹ 600 p.m. in total. The company paid ₹ 5,000 for electric bills
and ₹ 3,200 for water bills.
(viii) He has been provided with 1.5 litres engine capacity car for official and personal use both. Driver expenses are
borne by the company.
CA SHREY RATHI SALARY 3.6

(ix) The following amounts were deposited in his RPF account:


1. Own contribution – ₹ 25,000
2. Company’s contribution – ₹ 40,000
3. Interest @ 12% p.a. – ₹ 19,600
(x) Rent of house recovered from Ram – ₹ 24,000
Compute his taxable income from salary for the A/Y 2023-24 assuming that the population of Raipur is 28 lakhs as per
2001 census.

Sol 6: Computation of income u/h salaries of Mr. Ram for the A/Y 2023-24:
Particulars (₹)
Basic salary (₹ 20,000 x 12) 2,40,000
Bonus 12,000
Dearness allowance (₹ 4,000 x 12) 48,000
Travelling allowance (₹ 56,000 – ₹ 26,000) 30,000
Reimbursement of medical bills (not taxable as treatment was done in Govt. hospital) -
Children hostel allowance [₹ 8,000 – (₹ 300 x 12 x 2)] 800
Rent free accommodation (company owned) [15% (₹ 2,40,000 + ₹ 12,000 + ₹ 24,000 + ₹ 30,000 + ₹
800)] – amount recovered = ₹ 46,020 – ₹ 24,000] 22,020
Watchman and cook expenses (₹ 600 x 12) 7,200
Electricity and water bills (₹ 5,000 + ₹ 3,200) 8,200
Motor car (₹ 1,800 + ₹ 900) x 12 32,400
Company’s contribution to RPF [₹ 40,000 – 12% (₹ 2,40,000 + ₹ 24,000)] 8,320
Interest credited to RPF [(₹ 19,600 / 12) x 2.5] 4,083
Gross Salary 4,13,023
Less: Standard deduction u/s 16(ia) 50,000
Income u/h salaries for Mr. Ram 3,63,023

Q 7: Mr. Gavasker is drawing a salary of ₹ 15,000 p.m. The company has provided him with an accommodation in Meerut
for which 10% of his basic salary is deducted. Actual rent paid by the company for the accommodation is ₹ 1,20,000 p.a.
He is also receiving entertainment allowance of ₹ 500 p.m. He is provided by the company with a car having engine cubic
capacity of 1.8 litres for his personal and official use but running and maintenance expenses for the same are borne by the
assessee himself. He is in receipt of the bonus equivalent to 2 months’ salary. Compute his gross salary for the A/Y 2023-
24 assuming the population of Meerut is 20 lakhs.

Sol 7: Computation of taxable income u/h salary for Mr. Gavasker for the A/Y 2023-24:
Particulars (₹)
Basic salary (15,000 x 12) 1,80,000
Entertainment allowance (500 x 12) 6,000
Bonus (15,000 x 2) 30,000
Rent free accommodation [WN 1] 14,400
Motor car (900 x 12) 10,800
Gross salary for Mr. Gavasker 2,41,200
WN 1: Calculation of RFA: As the employer does not own the flat, taxable value shall be 15% of salary or lease rent
whichever is lower:
15% [₹ 1,80,000 + ₹ 6,000 + ₹ 30,000] = ₹ 32,400 or lease rent = ₹ 1,20,000 whichever is lower.
Taxable value of perquisite = ₹ 32,400 – amount recovered = ₹ 32,400 - ₹ 18,000 = ₹ 14,400.
* Population does not matter where the house is not owned by the employer.
CA SHREY RATHI SALARY 3.7

Q 8: Mr. Chetan was appointed as a sales manager of a company in Surat (Population = 18 lakhs) on 01.01.2019 in the
scale of ₹ 18,000 - 400 - 22,000 at ₹ 18,000 p.m. His other emoluments are:
(a) Dearness allowance – 40% of basic salary
(b) Conveyance allowance upto 30.09.2022 – ₹ 1,200 p.m. but his actual expenses for official purpose were ₹ 800 p.m.
(c) House rent allowance upto 31.07.2022 – ₹ 2,500 p.m. Rent paid was ₹ 3,000 p.m.
(d) Fixed medical allowance – ₹ 600 p.m.
(e) Rent free house from 01.08.2022 onwards. FRV of the house is ₹ 45,000 p.a. and cost of furnishing amounts to ₹
90,000. Employer paid ₹ 20,000 towards maintenance of house and ₹ 1,000 p.m. as electricity bill from 01.08.2022
to 31.03.2023.
(f) He and his employer both contributed 15% of salary each towards RPF.
(g) Interest credited on RPF balance @ 15% amounts to ₹ 7,500
(h) Club bill of the employee reimbursed by the employer during the year were ₹ 6,000.
Compute income u/h salary for Mr. Chetan for the A/Y 2023-24.

Sol 8: Computation of income u/h salary for Mr. Chetan for the A/Y 2023-24:
Particulars (₹)
Basic salary [WN 1] [(₹ 19,200 x 9) + (₹ 19,600 x 3)] 2,31,600
Dearness allowance (₹ 2,31,600 x 40%) 92,640
Conveyance allowance [(₹ 1,200 x 6) – (₹ 800 x 6)] 2,400
House rent allowance [WN 2] 5,680
Fixed medical allowance (₹ 600 x 12) 7,200
Rent free accommodation [WN 3] 22,040
Electricity bill from 01.08.2021 – 31.03.2022 (₹ 1,000 x 8) 8,000
Employer’s contribution to RPF (3% of ₹ 2,31,600) 6,948
Interest credited to RPF [(₹ 7,500 / 15%) x 5.5%] 2,750
Club bill reimbursed by the employer 6,000
Gross Salary 3,85,258
Less: Standard deduction u/s 16(ia) 50,000
Income u/h salary for Mr. Chetan 3,35,258
WN 1: Computation of basic salary:
01.01.2019 – 31.12.2019 = ₹ 18,000 p.m.
01.01.2020 – 31.12.2020 = ₹ 18,400 p.m.
01.01.2021 – 31.12.2021 = ₹ 18,800 p.m.
01.01.2022 – 31.12.2022 = ₹ 19,200 p.m.
01.01.2023 – 31.12.2023 = ₹ 19,600 p.m.
WN 2: Computation of taxable HRA: 01.04.2022 – 31.07.2022
HRA is exempt to the extent of minimum of the following:
(a) Actual HRA received (₹ 2,500 x 4) = ₹ 10,000
(b) Rent paid – 10% of Salary [(₹ 3,000 x 4) – (10% of ₹ 76,800)] = ₹ 4,320
(c) 40% of Salary (40% of ₹ 76,800) = ₹ 30,720
HRA exempt = ₹ 4,320
Taxable HRA = HRA received – Exempt amount
= ₹ 10,000 – ₹ 4,320 = ₹ 5,680

WN 3: Computation of taxable RFA: 01.08.2022 – 31.03.2023


As the house is owned by the company, taxable value shall be 10% of salary (population being more than 10 lakhs but less
than 25 lakhs).
Taxable RFA = 10% (₹ 1,54,800 + ₹ 800 + ₹ 4,800) = ₹ 16,040.
Furniture = ₹ 90,000 x 10% x 8/12 = ₹ 6,000
Total value of perquisite = ₹ 16,040 + ₹ 6,000 = ₹ 22,040
CA SHREY RATHI SALARY 3.8

Q 9: Mr. Pujara is employed with a transport firm in Gwalior. He is member of an unrecognised provident fund. He has
been drawing salary @ ₹ 8,000 p.m. upto 31.12.2022. Dearness allowance, forming part of pay for superannuation
benefits, is paid @ 10% of his salary. He gets house rent allowance ₹ 1,200 p.m. He pays rent of ₹ 2,000 p.m. He
contributes @ 10% of his salary to the fund and the employer contributes @ 20%. The employer also reimburses his
personal club bills amounting to ₹ 19,000. Besides, he is paid ₹ 1,200 p.m. as transport allowance.
He retires on 01.01.2023 after 28 years and 8 months of service. He gets ₹ 80,000 as accumulated balance from the
provident fund. It consists of ₹ 15,000 as his contribution and ₹ 11,000 interest thereon. The employer’s contribution is ₹
30,000 and interest thereon is ₹ 24,000. He also gets gratuity of ₹ 1,60,000. After retirement, he gets pension @ ₹ 3,000
p.m. On 01.03.2023, he surrendered one half pension for a consolidated amount of ₹ 1,20,000. Compute his gross salary.

Sol 9: Computation of gross salary of Mr. Pujara for the A/Y 2023-24:
Particulars (₹)
Basic salary (₹ 8,000 x 9) 72,000
Dearness allowance (forming part of salary) (₹ 72,000 x 10%) 7,200
House rent allowance [WN 1] 720
Personal club bills reimbursed by the employer 19,000
Allowance for transport employees [WN 2] 3,240
Unrecognised provident fund [WN 3] 54,000
Gratuity [WN 4] 36,800
Pension [WN 5] 47,500
Gross salary of Mr. Pujara 2,40,460
WN 1: Computation of taxable HRA: 01.04.2022 – 31.12.2023
HRA is exempt to the extent of minimum of the following:
(a) Actual HRA received (₹ 1,200 x 9) = ₹ 10,800
(b) Rent paid – 10% of Salary [(₹ 2,000 x 9) – (10% of ₹ 79,200)] = ₹ 10,080
(c) 40% of Salary (40% of ₹ 79,200) = ₹ 31,680
HRA exempt = ₹ 10,080
Taxable HRA = HRA received – Exempt amount
= ₹ 10,800 – ₹ 10,080 = ₹ 720
WN 2: Computation of taxable transport allowance: As Pujara is working in a transport firm, any transport allowance shall
be exempt to the extent of minimum of the following:
a) Transport allowance received (₹ 1,200 x 9) = ₹ 10,800
b) 70% of such allowance (₹ 10,800 x 70%) or ₹ 7,560
₹ 10,000 p.m. for 9 months ₹ 90,000 = ₹ 7,560
Transport allowance exempt = ₹ 7,560
Taxable transport allowance = ₹ 10,800 – ₹ 7,560 = ₹ 3,240
WN 3: Computation of taxable unrecognised provident fund:
Contribution of employer & interest on employer’s contribution shall be taxable u/h salaries = ₹ 30,000 + ₹ 24,000 = ₹
54,000.
Interest on employee’s contribution shall be taxable u/h other sources, therefore shall not be included in taxable salary.
WN 4: Computation of taxable gratuity:
Least of the following shall be exempt: (not covered under the Gratuity Act, 1972)
(i) Actual gratuity received ₹ 1,60,000
(ii) Limit ₹ 20,00,000
(iii) 15/30 x average salary x number of years of service (excluding fractions) ₹ 1,23,200
(15/30 x ₹ 8,800 x 28)
Exempt amount = ₹ 1,23,200
Taxable gratuity = Amount received as gratuity – exempt amount
= ₹ 1,60,000 – ₹ 1,23,200 = ₹ 36,800
CA SHREY RATHI SALARY 3.9

Calculation of average salary for gratuity: Average monthly salary is calculated on the basis of average salary for the 10
months immediately preceding the month in which the employee has retired.
Basic salary (₹ 8,000 x 10) = ₹ 80,000
Dearness allowance (forming part of salary) (10% of ₹ 80,000) = ₹ 8,000
= ₹ 88,000
Average salary = ₹ 88,000 / 10 = ₹ 8,800.
WN 5: Computation of taxable pension:
Uncommuted (recurring) pension = (₹ 3,000 x 2) + (₹ 1,500 x 1) = ₹ 7,500
Commuted pension: As gratuity is also received, 1/3rd of 100% commuted pension shall be exempt.
Commuted pension received = ₹ 1,20,000 = 50%, therefore ₹ 2,40,000 = 100%.
Exempt amount = ₹ 2,40,000 x 1/3 = ₹ 80,000
Taxable commuted pension = Commuted pension received – exempt amount
= ₹ 1,20,000 – ₹ 80,000 = ₹ 40,000
Total pension taxable = ₹ 7,500 + ₹ 40,000 = ₹ 47,500

Q 10: Mr. Umang retired on 01.09.2022 after service of 25 years and 7 months with basic salary of ₹ 30,000 p.m. and
dearness allowance being 30% of basic salary.
He got the following benefits on his retirement:
1. Received gratuity of ₹ 7,00,000. He is not covered under the Payment of Gratuity Act.
2. Leave salary – ₹ 3,40,000. Every year he was allowed 40 days leave and he had taken 5 days leave per year.
3. On retirement he was allowed pension of ₹ 15,000 p.m. He got 60% of his pension commuted on 01.02.2023 for ₹
2,70,000.
4. During job his employer an unrecognised provident fund. He received ₹ 1,20,000 on his own contribution, ₹ 80,000 on
employer’s contribution, interest on own contribution – ₹ 30,000 and ₹ 20,000 on employer’s contribution.
Compute gross salary of Umang for the A/Y 2023-24.

Sol 10: Computation of income u/h salary of Mr. Umang for the A/Y 2023-24:
Particulars (₹)
Basic salary (₹ 30,000 x 5) 1,50,000
Dearness allowance (₹ 1,50,000 x 30%) 45,000
Gratuity [WN 1] 3,25,000
Leave salary [WN 2] 40,000
Pension [WN 3] 2,07,000
Unrecognised provident fund [WN 4] 1,00,000
Gross salary for Mr. Umang 8,67,000
WN 1: Computation of taxable gratuity:
Least of the following shall be exempt: (not covered under the Gratuity Act, 1972)
(i) Actual gratuity received ₹ 7,00,000
(ii) Limit ₹ 20,00,000
(iii) 15/30 x average salary x number of years of service (excluding fractions) ₹ 3,75,000
(15/30 x ₹ 30,000 x 25)
Exempt amount = ₹ 3,75,000

Taxable gratuity = Amount received as gratuity – exempt amount


= ₹ 7,00,000 – ₹ 3,75,000 = ₹ 3,25,000
Calculation of average salary for gratuity: Average monthly salary is calculated on the basis of average salary for the 10
months immediately preceding the month in which the employee has retired. It shall consist of only basic salary.
CA SHREY RATHI SALARY 3.10

WN 2: Computation of taxable leave salary


Least of the following shall be exempt:
1. Actual leave salary received ₹ 3,40,000
2. Limit ₹ 3,00,000
3. 10 months average salary (10 x ₹ 30,000) ₹ 3,00,000
4. Cash equivalent to unavailed leaves
(i) Leaves actually allowed or 30 days per year whichever is less = 30 days x 25 = 750 days
(ii) Leaves actually taken = 125 days (5 days x 25)
(iii) (750 - 125) x average salary/30 = 625 x ₹ 30,000/30 ₹ 6,25,000
Exempt amount = ₹ 3,00,000
Taxable leave salary = Actual leave salary – Exempt amount
= ₹ 3,40,000 – ₹ 3,00,000 = ₹ 40,000
Calculation of average salary for leave salary: Average salary is to be calculated on the basis of salary drawn by the
employee during the period of 10 months immediately preceding the date of retirement. It shall be same as computed for
gratuity.

WN 3: Computation of taxable pension:


Uncommuted (recurring) pension = (₹ 15,000 x 5) + (₹ 6,000 x 2) = ₹ 87,000
Commuted pension: As gratuity is also received, 1/3rd of 100% commuted pension shall be exempt.
Commuted pension received = ₹ 2,70,000 = 60%, therefore ₹ 4,50,000 = 100%.
Exempt amount = ₹ 4,50,000 x 1/3 = ₹ 1,50,000
Taxable commuted pension = Commuted pension received – exempt amount
= ₹ 2,70,000 – ₹ 1,50,000 = ₹ 1,20,000
Total pension taxable = ₹ 87,000 + ₹ 1,20,000 = ₹ 2,07,000

WN 4: Computation of taxable unrecognised provident fund:


Contribution of employer & interest on employer’s contribution shall be taxable u/h salaries = ₹ 80,000 + ₹ 20,000 = ₹
1,00,000.
Interest on employee’s contribution shall be taxable u/h other sources, therefore shall not be included in taxable salary.

Q 11: Mr Keshav is employed with DEF Ltd. on a basic monthly salary of ₹ 22,000 p.m. He has been provided with the
following emoluments:
(a) Bonus – 1 month basic salary
(b) Children education allowance – ₹ 90 p.m. per child for 3 children
(c) Entertainment allowance – ₹ 20,000
(d) A fixed line telephone facility was provided at his residence during the previous year amounting to ₹ 18,000.
(e) On the 10th Anniversary of the company, he got a gift worth ₹ 11,000 from the company.
(f) During the year, he got reimbursed from the company ₹ 15,000 spent on the medical treatment of his father in a
Government hospital.
(g) He is provided with a personal accident policy for which, premium of ₹ 5,000 is paid by the company.
(h) He was given a chauffeur driven car (1.6 litres) for official use.
(i) Profession tax paid ₹ 2,000 of which ₹ 1,000 was paid by the employer.
(j) Leave travel concession was given to Keshav and his family. Cost of air tickets for economy class – ₹ 45,000 for
himself and her spouse and ₹ 60,000 for three children. Keshav is eligible for LTC. Assume the age of children to be
below the age of 10 years.
(k) Contribution of the employer and employee to the RPF account – ₹ 60,000 each.
(l) Interest credited to RPF account @ 15% - ₹ 7,500.
Compute income u/h salary for Mr. Keshav for the P/Y 2022-23.
CA SHREY RATHI SALARY 3.11

Sol 10: Computation of income u/h salary for Mr. Keshav for the P/Y 2022-23:
Particulars (₹)
Basic salary (₹ 22,000 x 12) 2,64,000
Bonus (1-month basic salary) 22,000
Children education allowance [(₹ 90 x 12 x 3) – (₹ 90 x 12 x 2)] 1,080
Entertainment allowance 20,000
Telephone facility provided at residence – exempt -
Gift (₹ 11,000 – ₹ 5,000) 6,000
Reimbursement of medical expenses for the treatment of father in a Government hospital (fully -
exempt)
Personal accident policy premium paid by the company – exempt -
Motor car for official purpose – exempt -
Professional tax paid by the employer 1,000
Leave travel concession – travel cost for 3rd child shall be taxable [(₹ 60,000/3) x 2] 20,000
Contribution by employer to RPF [₹ 60,000 – 12% of ₹ 2,64,000] 28,320
Interest credited to RPF [(₹ 7,500/15%) x 5.5%] 2,750
Gross salary 3,65,150
(-) Standard deduction u/s 16(ia) 50,000
(-) Professional tax paid 2,000
Taxable salary for Mr. Keshav for the P/Y 2022-23 3,13,150

Q 12: Mr. Binny is a production manager of FT Ltd. From the following details, compute the gross salary for the A/Y 2023-
24.

1. Basic salary ₹ 50,000 p.m.


2. Dearness allowance 40% of basic salary
3. Transport allowance (for commuting between place of residence and office) ₹ 3,800 p.m.
4. Motor car running and maintenance charges fully paid by employer. The motor car is owned by the employer and
driven by the employee. The engine cubic capacity is 1.8 litres. The motor car is used for both official and personal
purpose by the employee. ₹ 60,000
5. Expenditure on accommodation in hotels while touring on official duties met by the employer. ₹ 70,000
6. Loan from recognised provident fund (maintained by the employer) ₹ 1,00,000
7. Lunch provided by the employer during office hours (250 days) ₹ 12,000
8. Computer given by the employer to employee for his personal use ₹ 40,000

Sol 12: Computation of gross salary of Mr. Binny for the A/Y 2023-24:
Particulars (₹)
Basic salary (₹ 50,000 x 12) 6,00,000
Dearness allowance (₹ 6,00,000 x 40%) 2,40,000
Transport allowance (₹ 3,800 x 12) 45,600
Motor car (₹ 2,400 x 12) 28,800
Accommodation expenses for official tours – fully exempt -
Loan taken from recognised provident fund – not a benefit given by the employer, therefore not -
taxable
Lunch provided by the employer [₹ 12,000 – (250 x ₹ 50)] -
Computer given for personal use – not taxable -
Gross salary of Mr. Binny 9,14,400

Q 13: Mr. Mohit is employed with PQR Ltd. on a basic salary of ₹ 10,000 p.m. He is also entitled to dearness allowance @
100% of basic salary, 50% of which is included in salary as per the terms of employment. The company gives him house
rent allowance of ₹ 6,000 p.m. which was increased to ₹ 7,000 p.m. with effect from 01.01.2023. He also got an increment
of ₹ 1,000 p.m. in his basic salary w.e.f. 01.02.2023.
CA SHREY RATHI SALARY 3.12

Rent paid by him during the previous year 2022-23 is as under:


April – May, 2022 - Nil, as he stayed with his parents
June – October, 2022 - ₹ 6,000 p.m. for an accommodation in Pune
November – March, 2023 - ₹ 8,000 p.m. for an accommodation in Mumbai
Compute the gross salary for the A/Y 2023-24 assuming he has not opted for the provisions for 115BAC. [ICAI Module]

Sol 13: Computation of gross salary for Mr. Mohit for the A/Y 2023-24:
Particulars (₹)
Basic salary [(₹ 10,000 x 10) + (₹ 11,000 x 2)] 1,22,000
Dearness allowance (₹ 1,22,000 x 100%) 1,22,000
House rent allowance [WN 1] (₹ 12,000 + ₹ 7,500 + Nil + ₹ 500 + ₹ 1,300) 21,300
Gross salary of Mr. Mikhil 2,65,300
WN 1: Computation of taxable HRA: Least of the following shall be exempt:
Particulars Apr – May (₹) June – Oct (₹) Nov – Dec (₹) Jan (₹) Feb – Mar (₹)
Actual HRA received 12,000 30,000 12,000 7,000 14,000
Rent paid – 10% of salary Nil 30,000 – 7,500 = 16,000 – 3,000 = 8,000 – 1,500 = 16,000 – 3,300 =
22,500 13,000 6,500 12,700
40% / 50% of salary 12,000 30,000 15,000 7,500 16,500
Exempt amount Nil 22,500 12,000 6,500 12,700
Taxable = Recd. – Exempt 12,000 7,500 Nil 500 1,300

Computation of salary for HRA:


Particulars Apr – May (₹) June – Oct (₹) Nov – Dec (₹) Jan (₹) Feb – Mar (₹)
Basic salary 20,000 50,000 20,000 10,000 22,000
DA (forming part of salary) 10,000 25,000 10,000 5,000 11,000
Total 30,000 75,000 30,000 15,000 33,000

Q 14: Mr. Karanjeet is employed by KLM Ltd., w.e.f. 01.05.2020 in the grade of ₹ 15,000 – 500 – 17,000 – 600 – 20,000 and
his basic salary on the date of joining was fixed at ₹ 16,500.
Besides the above he was entitled to the following benefits:
(a) Dearness allowance – ₹ 3,000 p.m.
(b) House rent allowance – ₹ 5,000 p.m. Rent paid by the assessee for the house in Mumbai is ₹ 7,000 p.m.
(c) Medical allowance – ₹ 500 p.m. Entire amount is spent on medical treatment of himself.
(d) Assessee is provided with free lunch of ₹ 80 per meal during working hours for 250 days.
(e) Three children of the assessee are studying in the school run by the employer. The cost of education in similar
institution per student is ₹ 3,000 p.a.
(f) Facility of laptop costing ₹ 40,000. Written down value of such laptop is ₹ 16,000.
(g) Employer gave him a loan of ₹ 35,000 on 31.12.2022. He repaid half the amount on 30.03.2023. SBI rate of interest
on 01.04.2022 was 14% and on 31.12.2022 was 13%. Employer charged 6% only.
(h) He purchased a second hand car for ₹ 3,00,000. Such car was purchased by the employer for ₹ 6,00,000; 40 months
ago.
(i) Contribution to URPF by employer and himself – ₹ 48,000 each.
(j) Interest received on URPF @ 10% - ₹ 2,000.
Compute his salary for the A/Y 2023-24.
CA SHREY RATHI SALARY 3.13

Sol 14: Computation of salary of Mr. Karanjeet for the A/Y 2023-24:
Particulars (₹)
Basic salary [WN 1] [(₹ 17,000 x 1) + (₹ 17,600 x 11)] 2,10,600
Dearness allowance (₹ 3,000 x 12) 36,000
House rent allowance [WN 2] -
Medical allowance 6,000
Free lunch [(₹ 80 – ₹ 50) x 250] 7,500
Education facility (exempt upto 1,000 p.m. per child) -
Laptop facility -
Concessional free loan [WN 3] 700
Transfer of car [WN 4] 7,200
Contribution to URPF -
Interest received on URPF -
Gross salary of Mr. Karanjeet 2,68,000
Less: Standard deduction u/s 16(ia) 50,000
Income u/h Salary of Mr. Karanjeet 2,18,000
WN 1: Calculation of basic salary:
01.05.2020 – 30.04.2021 = ₹ 16,500 p.m.
01.05.2021 – 30.04.2022 = ₹ 17,000 p.m.
01.05.2022 – 30.04.2023 = ₹ 17,600 p.m.
WN 2: Computation of taxable HRA:
HRA is exempt to the extent of minimum of the following:
(a) Actual HRA received (₹ 5,000 x 12) = ₹ 60,000
(b) Rent paid – 10% of Salary [(₹ 7,000 x 12) – (10% of ₹ 2,10,600)] = ₹ 62,940
(c) 50% of Salary (50% of ₹ 2,10,600) = ₹ 1,05,300
HRA exempt = ₹ 60,000
Taxable HRA = HRA received – Exempt amount = ₹ 60,000 – ₹ 60,000 = Nil
WN 3: Computation of interest on concessional loan:
₹ 35,000 x 8% x 3/12 = ₹ 700
No interest shall be charged after half payment is made as the loan outstanding has come below ₹ 20,000.
WN 4: Computation of benefit in case of transfer of car:
Cost of car = ₹ 6,00,000
(-) Depreciation @ 20% – Year 1 = ₹ 1,20,000
₹ 4,80,000
(-) Depreciation @ 20% - Year 2 = ₹ 96,000
₹ 3,84,000
(-) Depreciation @ 20% - Year 3 = ₹ 76,800
₹ 3,07,200
(-) Sale Value = ₹ 3,00,000
Value of benefit = ₹ 7,200

Q 15: Mr. X is employed with AB Ltd. on a monthly salary of ₹ 25,000 per month and an entertainment allowance and
commission of ₹ 1,000 p.m. each. The company provides him with the following benefits:
1. A company owned accommodation is provided to him in Delhi. Furniture costing ₹ 2,40,000 was provided on 01-08-
2022.
2. A personal loan of ₹ 5,00,000 on 01.07.2022 on which it charges Interest @ 6.75% p.a. The entire loan is still
outstanding. (Assume SBI rate of Interest on 1-4-2022 was 12.75% p.a.)
3. His son is allowed to use a motorcycle belonging to the company. The company had purchased this motor cycle for ₹
60,000 on 01.05.2019. The motorcycle was finally sold to him on 1-8-2022 for ₹ 30,000.
4. Professional tax paid by Mr. X is ₹ 2,000.
Compute the income from salary of Mr. X for the A.Y 2023-24 assuming Mr. X has not opted for the provisions of Section
115BAC. [ICAI Study Material]
CA SHREY RATHI SALARY 3.14

Sol 15: Computation of Income from Salary for Mr. X for the A.Y. 2023-24:
Particulars (₹) (₹)
Basic salary [₹ 25,000 x 12] 3,00,000
Commission [₹ 1,000 x 12] 12,000
Entertainment allowance [₹ 1,000 x 12] 12,000
Rent free accommodation [Note 1] 48,600
Add: Value of furniture [₹ 2,40,000 x 10% p.a. for 8 months] 16,000 64,600
Interest on personal loan [Note 2] 22,500
Use of motor cycle [₹ 60,000 x 10% p.a. for 4 months] 2,000
Transfer of motor cycle [Note 3] 12,000
Gross Salary 4,25,100
Less: Deduction under Section 16
• Under Section 16(ia) – Standard deduction 50,000
• Under Section 16(iii) – Professional tax paid 2,000 52,000
Income under the head Salary 3,73,100
Note 1: Value of rent-free unfurnished accommodation = 15% of salary for the relevant period
= 15% of (₹ 3,00,000 + ₹ 12,000 + ₹ 12,000) = ₹ 48,600
Note 2: Value of perquisite for Interest on personal loan
= ₹ 5,00,000 x (12.75% - 6.75%) for 9 months] = ₹ 22,500
Note 3: Depreciated value of the motorcycle
= Original cost – Depreciation @ 10% p.a. for 3 completed years.
= ₹ 60,000 – (₹ 60,000 x 10% p.a. x 3 years) = ₹ 42,000.
Perquisite = ₹ 42,000 - ₹ 30,000 = ₹ 12,000.

Q 16: Ms. Rakhi is an employee in a private company. She received the following medical benefits from the company
during the previous year 2022-23:
S. No. Particulars (₹)
1 Reimbursement of following medical expenses incurred by Ms. Rakhi
(A) On treatment of her self employed daughter in a private clinic 4,000
(B) On treatment of herself by family doctor 8,000
(C) On treatment of her mother-in-law dependent on her, in a nursing home 5,000
2 Payment of premium on Mediclaim Policy taken on her health 7,500
3 Medical Allowance 2,000 p.m.
4 Medical expenses reimbursed on her son’s treatment in a Government hospital 5,000
5 Expenses incurred by company on the treatment of her minor son abroad including stay 1,95,000
expenses
6 Expenses in relation to foreign travel of Rakhi and her son for medical treatment 1,20,000
Note – limit prescribed by RBI for expenditure on medical treatment and stay abroad is USD
2,50,000 per financial year under liberalized remittance scheme.
Examine the taxability of the above benefits and allowances in the hands of Rakhi. [ICAI Study Module]

Sol 16: Tax treatment of medical benefits in the hands of Ms. Rakhi for A.Y. 2023-24:
S. No. Particulars
1. (A) The amount of ₹ 4,000 reimbursed by her employer for treatment of her self-employed daughter in a
private clinic is taxable perquisite.
(B) The amount of ₹ 8,000 reimbursed by the employer for treatment of Ms. Rakhi by family doctor is taxable
perquisite.
(C) The amount of ₹ 5,000 reimbursed by her employer for treatment of her dependant mother-in-law in a
nursing home is taxable perquisite.
The aggregate sum of ₹ 17,000, specified in (A), (B) and (C) above, reimbursed by the employer is taxable
perquisite
2. Medical insurance premium of ₹ 7,500 paid by the employer for insuring health of Ms. Rakhi is a tax-free
perquisite as per clause (iii) of the first proviso to Section 17(2).
CA SHREY RATHI SALARY 3.15

3. Medical allowance of ₹ 2,000 per month i.e., ₹ 24,000 p.a. is a fully taxable allowance.
4. As per clause (ii)(a) of the first proviso to Section 17(2), reimbursement of medical expenses of ₹ 5,000 on her
son’s treatment in a hospital maintained by the Government is a tax-free perquisite.
5. & 6. As per clause (vi) of the first proviso to Section 17(2), the following expenditure incurred by the employer
would be excluded from perquisite subject to certain conditions –
(i) Expenditure on medical treatment of the employee, or any member of the family of such employee,
outside India [₹ 1,05,000, in this case];
(ii) Expenditure on travel and stay abroad of the employee or any member of the family of such employee for
medical treatment and one attendant who accompanies the patient in connection with such treatment [₹
1,20,000, in this case].
The conditions subject to which the above expenditure would be exempt are as follows –
(i) The expenditure on medical treatment and stay abroad would be excluded from perquisite to the extent
permitted by Reserve Bank of India;
(ii) The expenditure on travel would be excluded from perquisite only in the case of an employee whose
gross total income, as computed before including and said expenditure, does not exceed ₹ 2 lakhs.
Since the expenditure on medical treatment and stay abroad does not exceed the limit permitted by RBI, they
would be fully exempt. However, the foreign travel expenditure of Ms. Rakhi and her minor son borne by the
employer would be excluded from perquisite only if the gross total income of Ms. Rakhi, as computed before
including the said expenditure, does not exceed ₹ 2 lakhs.

Q 17: BCD Ltd. offered you a job in Delhi at a basic salary of ₹ 11,500 p.m. and an option to choose any one of the
following two packages:
S. No. Package I Package II
(i) HRA ₹ 4,500 p.m. (Rent to be paid ₹ 4,500 p.m.) Company owned unfurnished flat. FRV is ₹ 54,000 p.a.

(ii) Education allowance ₹ 300 p.m. (for one child) Education facility for one child in an institution owned
by employer valued at ₹ 300 p.m.
(iii) Telephone allowance ₹ 1,000 p.m. Free telephone facility at residence for ₹ 1,000 p.m.

(iv) Medical allowance ₹ 1,500 p.m. Medical reimbursement upto ₹ 18,000 p.a.
(v) Conveyance allowance ₹ 1,500 p.m. (for private use) Motor car facility for private use expenditure valued at
₹ 18,000.
The company also offers you the services of watchman, sweeper and gardener in both the above packages. The salary of
each employee is ₹ 500 p.m.
Which package will you choose so that your tax liability is minimum?

Sol 17: Computation of best choice so that the tax liability is least:
Particulars Package I (₹) Package II (₹)
Basic salary (₹ 11,500 x 12) 1,38,000 1,38,000
House rent allowance [WN 1] 13,800
Rent free accommodation (company owned) (15% of ₹ 1,38,000) 20,700
Education allowance [(₹ 300 x 12) – (₹ 100 x 12 x 1)] 2,400
Education facility (exempt upto ₹ 1,000 p.m. per child) -
Telephone allowance (₹ 1,000 x 12) 12,000
Telephone facility at residence -
Medical allowance (₹ 1,500 x 12) 18,000
Medical reimbursement (fully taxable) 18,000
Conveyance allowance (₹ 1,500 x 12) 18,000
Motor car facility (private use) 18,000
Watchman, sweeper & gardener (₹ 500 x 3 x 12) (both packages) 18,000 18,000
Gross salary 2,20,200 2,12,700
Less: Standard deduction u/s 16(ia) 50,000 50,000
Income u/h Salary 1,70,200 1,62,700
CA SHREY RATHI SALARY 3.16

WN 1: Computation of taxable HRA:


HRA is exempt to the extent of minimum of the following:
(a) Actual HRA received (₹ 4,500 x 12) = ₹ 54,000
(b) Rent paid – 10% of Salary [(₹ 4,500 x 12) – (10% of ₹ 1,38,000)] = ₹ 40,200
(c) 50% of Salary (50% of ₹ 1,38,000) = ₹ 69,000
HRA exempt = ₹ 40,200
Taxable HRA = HRA received – Exempt amount
= ₹ 54,000 – ₹ 40,200 = ₹ 13,800
Advice: Choose Package II as tax liability will be lower.

Q 18: Ms. Aashima ia a Finance Manager in ABC Ltd. She has given the details of her income for the P.Y. 2022-23. You are
required to compute the income chargeable to tax under the head “Salaries” in the hands of Ms. Aashima from the details
given below:
(i) Basic Salary ₹ 60,000 p.m.
(ii) Dearness Allowance ₹ 24,000 p.m. (40% of which forms part of retirement benefits)
(iii) Bonus ₹ 21,000 p.m.
(iv) Motor car owned by the employer (cubic capacity of engine exceeds 1.6 litres) provided to Ms. Aashima from 1 st
October, 2022 which is used for both official and personal purposes. Repairs and running expenses of ₹ 60,000 were
fully met by the company. The motor car was self-driven by the employee.
(v) Professional tax paid ₹ 2,500 out of which ₹ 2,000 was paid by the employer.
(vi) Her employer has provided her with an accommodation on 1st April 2022 at a concessional rent. The house was
taken on lease by ABC Ltd. for ₹ 12,000 p.m. Ms. Aashima occupied the house from 1st December, 2022, ₹ 4,800
p.m. is recovered from the salary of Ms. Aashima.
(vii) The employer gave her a gift voucher of ₹ 8,000 on her birthday.
(viii) Ms. Aashima contributes 15% of her salary (Basic Pay plus DA) towards recognised provident fund and the company
contributes the same amount.
(ix) The company pays medical insurance premium to effect insurance on the health of Ms. Aashima ₹ 20,000.
[MTP May 2022]

Sol 18: Computation of income chargeable to tax under the head “Salaries” in the hands of Ms. Aashima for A.Y. 2023-24:
Particulars (₹)
Basic Salary [₹ 60,000 x 12] 7,20,000
Dearness allowance [₹ 24,000 x 12] 2,88,000
Bonus [₹ 21,000 x 12] 2,52,000
Perquisite of Motor car (as CC of the engine exceeds 1.6 litres and the expenses are borne by the 14,400
employer, ₹ 2,400 p.m. will be the taxable perquisite) [₹ 2,400 x 6 months]
Professional tax paid by employer (it is a perquisite in the hands of the employee and therefore 2,000
taxable)
Perquisite value in respect of concessional rent [Note 1] 28,800
Gift voucher given by employer on Ms. Aashima’s birthday (exempt upto ₹ 5,000) [₹ 8,000 – ₹ 5,000] 3,000
Employer’s contribution to recognized provident fund in excess of 12% of salary = 15% x [(60,000 + 50,976
24,000) x 12] – 12% x {[₹ 60,000 + ₹9,000 (being 40% of ₹ 24,000)] x 12} = 1,51,200 – 1,00,224
Medical insurance premium of ₹ 20,000 paid by the employer to effect an insurance on the health of -
an employee is an exempt perquisite
Gross Salary 13,59,176
Less: Deduction under section 16
Standard deduction under section 16(ia) ₹ 50,000
Professional tax u/s 16(iii) (total amount paid by employer + employee shall be allowed) ₹ 2,500 52,500
Income chargeable under the head Salaries 13,06,676
CA SHREY RATHI SALARY 3.17

Note 1: Where the accommodation is taken on lease or rent by the employer, the actual amount of lease rent paid or
payable by the employer or 15% of salary, whichever is lower, in respect of the period during which the house is occupied
by the employee, as reduced by the rent recoverable from the employee, is the value of the perquisite.
Actual rent paid by the employer from 1-12-2021 to 31-03-2022 = ₹ 48,000 [₹ 12,000 x 4 months]
15% of salary = ₹ 54,360 [15% x (₹ 60,000 + ₹ 96,000 + ₹ 21,000) x 4 months]
Salary = Basic salary + Dearness Allowance, to the extent it forms part of pay for retirement benefits + Bonus
Lower of the above is ₹ 48,000 which is to be reduced by the rent recovered from the employee.
Hence, the perquisite value of concessional rent = ₹ 48,000 - ₹ 19,200 [₹ 48,000 x 4 months = ₹ 28,800]

Q 19: From the following details, find out the salary chargeable to tax for the A.Y. 2023-24 assuming he has not opted for
the provisions of Section 115BAC-
Mr. X is a regular employee of Rama & Co., in Gurgaon. He was appointed on 1-1-2022 in the scale of ₹ 20,000 - ₹ 1,000 - ₹
30,000. He is paid 10% dearness allowance & bonus equivalent to one month pay based on salary of March every year. He
contributes 15% of his pay & dearness allowance towards his recognized provident fund and the company contributes the
same amount. DA forms part of pay for retirement benefits.
He is provided free housing facility which has been taken on rent by the company at ₹ 10,000 per month. He is also
provided with following facilities:
(i) Facility of laptop costing ₹ 50,000.
(ii) Company reimbursed the medical treatment bill of his brother of ₹ 25,000, who is dependent on him.
(iii) The monthly salary of ₹ 1,000 of a house keeper is reimbursed by the company.
(iv) A gift voucher of ₹ 10,000 on the occasion of his marriage anniversary.
(v) Conveyance allowance of ₹ 1,000 per month is given by the company towards actual reimbursement of conveyance
spent on official duty.
(vi) He is provided personal accident policy for which premium of ₹ 5,000 is paid by the company.
(vii) He is getting telephone allowance @ ₹ 500 per month. [ICAI Study Module]

Sol 19: Computation of taxable salary for Mr. X for the A/Y 2023-24:
Particulars (₹)
Basic pay [(₹ 20,000 x 9) + (₹ 21,000 x 3)] 2,43,000
Dearness Allowance [10% of basic pay] 24,300
Bonus 21,000
Employer’s contribution to Recognized Provident Fund in excess of 12% (3% of ₹ 2,67,300) 8,019
Rent-free accommodation [Note – 1] 44,145
Facility of Laptop -
Medical reimbursement 25,000
Reimbursement of salary of housekeeper 12,000
Gift voucher [₹ 10,000 – ₹ 5,000] 5,000
Conveyance Allowance [Fully exempt as spent on official duty] -
Personal accident policy premium [Not liable to tax] -
Telephone allowance 6,000
Gross Salary 3,88,464
Less: Deduction under Section 16(ia) – Standard deduction 50,000
Salary income chargeable to tax 3,38,464
Note 1: Where the accommodation is taken on lease or rent by the employer, the value of rent-free accommodation
provided to employee would be actual amount of lease rental paid or payable by the employer or 15%, whichever is
lower. For the purposes of valuation of rent-free house, salary includes:
(i) Basic salary - ₹ 2,43,000
(ii) Dearness allowance (assuming that it is included for calculating retirement benefits) - ₹ 24,300
(iii) Bonus - ₹ 21,000
(iv) Telephone allowance - ₹ 6,000
15% of salary = ₹ 2,94,300 x 15/100 = ₹ 44,145
Value of rent-free house = Lower of rent paid by the employer (i.e ₹ 1,20,000) or 15% of salary (i.e. ₹ 44,145).
Therefore, the perquisite value is ₹ 44,145.
CA SHREY RATHI SALARY 3.18

Q 20: Mr. Balaji, employed as Production Manager in Beta Ltd., furnishes you the following information for the year ended
31-03-2023:
(i) Basic salary upto 31-10-2022 ₹ 50,000 p.m.
(ii) Basic salary from 01-11-2022 ₹ 60,000 p.m.
Note: Salary is due and paid on the last day of every month.
(iii) Dearness allowance @ 40% of basic salary
(iv) Bonus equal to one month salary. Paid in October 2022 on basic salary plus dearness allowance applicable for that
month.
(v) Contribution of employer to recognized provident fund account of the employee @ 16% of basic salary.
(vi) Professional tax paid ₹ 2,500 of which ₹ 2,000 was paid by the employer.
(vii) Facility of laptop and computer was provided to Balaji for both official and personal use. Cost of laptop ₹ 45,000 and
computer ₹ 35,000 were acquired by the company on 01-12-2022.
(viii) Motor car owned by the employer (cubic capacity of engine exceeds 1.60 litres) provided to the employee from 01-
11-2022 meant for both official and personal use. repair and running expenses of ₹ 45,000 from 01-11-2022 to 31-
03-2023, were fully met by the employer. The motor car was self-driven by the employee.
(ix) Leave travel concession given to employee, his wife and three children (one daughter aged 7 and twin sons aged 3).
Cost of air tickets (economy class) reimbursed by the employer ₹ 30,000 for adults and ₹ 45,000 for three children.
Balaji is eligible for availing exemption this year to the extent it is permissible in law.
Compute the salary income chargeable to tax in the hands of Mr. Balaji for the assessment year 2022-23 assuming he has
not opted for the provisions of Section 115BAC. [ICAI Study Module]

Sol 20: Computation of taxable salary of Mr. Balaji for the AY 2023-24:
Particulars (₹)
Basic salary [(₹ 50,000 x 7) + (₹ 60,000 x 5)] 6,50,000
Dearness Allowance (40% of basic salary) 2,60,000
Bonus (₹ 50,000 + 40% of ₹ 50,000) 70,000
Employer’s contribution to recognised provident fund in excess of 12% of salary = 4% of ₹ 6,50,000 26,000
Professional tax paid by employer (it is a perquisite in the hands of the employee and therefore 2,000
taxable)
Facility of laptop & computer is a tax free perquisite, whether used for official or personal purpose or -
both.
Perquisite of Motor Car (as CC of the engine exceeds 1.6 litres and the expenses are borne by the 12,000
employer, ₹ 2,400 p.m. will be the taxable perquisite) [₹ 2,400 x 5 months]
Leave Travel concession [Full exemption can be availed as journey was undertaken by economy class -
airfare. Further, the restriction imposed for two children is not applicable in case of multiple births
which take place after the first child]
Gross Salary 10,20,000
Less: Deduction under Section 16
Standard deduction u/s 16(ia) ₹ 50,000
Professional tax u/s 16(iii) (total amount paid by employer + employee shall be allowed) ₹ 2,500 52,000
Taxable Salary 9,67,500

MULTIPLE CHOICE QUESTIONS


1. The maximum ceiling limit for exemption under section 10(10) in respect of gratuity for employees covered by the
Payment of Gratuity Act, 1972 is -
(a) ₹ 10,00,000 (c) ₹ 5,00,000
(b) ₹ 3,50,000 (d) ₹ 20,00,000

2. The maximum ceiling limit for exemption under section 10(10C) with respect to compensation received on voluntary
retirement is -
(a) ₹ 2,50,000 (c) ₹ 3,00,000
(b) ₹ 3,50,000 (d) ₹ 5,00,000
CA SHREY RATHI SALARY 3.19

3. The HRA paid to an employee residing in Patna is exempt up to the lower of actual HRA, excess of rent paid over 10%
of salary and -
(a) 30% of salary (c) 40% of salary
(b) 50% of salary (d) 60% of salary

4. Anirudh stays in New Delhi. His basic salary is ₹ 10,000 p.m., D.A. (60% of which forms part of pay) is ₹ 6,000 p.m.,
HRA is ₹ 5,000 p.m. and he is entitled to a commission of 1% on the turnover achieved by him. Anirudh pays a rent of
₹ 5,500 p.m. The turnover achieved by him during the current year is ₹ 12 lakhs. The amount of HRA exempt under
section 10(13A) is – (ICAI MCQ Booklet)
(a) ₹ 48,480 (c) ₹ 45,600
(b) ₹ 49,680 (d) ₹ 46,800

5. Where there is a decision to increase the D.A. in March, 2022 with retrospective effect from 1.4.2020, and the
increased D.A. is received in April, 2022, the increase is taxable - (ICAI MCQ Booklet)
(a) in the previous year 2020-21 (c) in the previous year 2022-23
(b) in the previous year 2021-22 (d) in the respective years to which they relate

6. Rajesh is provided with a rent-free unfurnished accommodation, which is owned by his employer, XY Pvt. Ltd., in New
Delhi. The value of perquisite in the hands of Rajesh is:
(a) 20% of salary (c) 15% of salary
(b) 10% of salary (d) 7.5% of salary

7. Anand is provided with furniture to the value of ₹ 70,000 along with house from February, 2022. The actual hire
charges paid by his employer for hire of furniture is ₹ 5,000 p.a. The value of furniture to be included along with value
of unfurnished house for A.Y. 2023-24 is: (ICAI MCQ Booklet)
(a) ₹ 5,000 (c) ₹ 7,000
(b) ₹ 10,500 (d) ₹ 14,000

8. For the purpose of determining the perquisite value of loan at concessional rate given to the employee, the lending
rate of State Bank of India as on is required;
(a) 1st day of the relevant previous year (c) Last day of the relevant previous year
(b) the day the loan is given (d) 1st day of the relevant assessment year

9. Mr. Kashyap received basic salary of ₹ 20,000 p.m. from his employer. He also received children education allowance
of ₹ 3,000 for three children and transport allowance of ₹ 1,800 p.m. The amount of salary chargeable to tax for P.Y.
2022-23 is - (ICAI MCQ Booklet)
(a) ₹ 2,62,600 (c) ₹ 2,12,600
(b) ₹ 2,45,200 (d) ₹ 2,07,800

10. The entertainment allowance received by a Government employee is exempt up to the lower of the actual
entertainment allowance received, 1/5th of basic salary and-
(a) ₹ 4,000 (c) ₹ 6,000
(b) ₹ 5,000 (d) ₹ 10,000

11. KP Ltd is a company paying salary of ₹ 4,50,000 to its employee Mr. P and also undertakes to pay the Income Tax
amounting to ₹ 10,400 on his behalf during the PY 2022-23. The gross salary of Mr. P shall be:
(a) ₹ 4,60,400 (c) ₹ 4,50,000
(b) ₹ 4,20,400 (d) None of the above

12. Salary of Mr. J is ₹ 10,000 p.m. Mr. J had taken salary in advance for the months of April 2022 to June 2022 in March
2022 itself. The gross salary of Mr. J for AY 2023-2024 shall be:
(a) ₹ 1,20,000 (c) ₹ 70,000
(b) ₹ 1,00,000 (d) ₹ 90,000
CA SHREY RATHI SALARY 3.20

13. Bonus is taxable as salary income


(a) When it is received
(b) When it gets due or when it is received, whichever is earlier
(c) Anytime at the choice of employee
(d) Anytime at the choice of employer

14. Hostel expenditure allowance is exempt up to:


(a) ₹ 300 p.a. for 2 children (c) ₹ 300 p.m. for 2 children
(b) ₹ 300 p.m. per child upto 2 children (d) ₹ 300 p.a. per child upto 2 children

15. A Transport allowance is exempt upto ₹ 3,200 pm for:


(a) Government employees
(b) Employees of non-government sector
(c) Employee who deaf, dumb, blind or orthopedically handicapped
(d) Employee who is mentally handicapped

16. Under Section 15 salary is taxable:


(a) On receipt basis (c) On due basis
(b) Receipt or due, whichever is earlier (d) None of the above

17. When is control said to exist?


(a) payer can direct what has to be done
(b) payer can direct when and how it has to be done
(c) payer can direct who has to do it and receiver is bound to follow all the instruction
(d) all of the above

18. When salary is received in arrears or advance, relief shall be admissible u/s……………
(a) 89(1) (c) 98(1)
(b) 79(1) (d) 97(1)

19. R is an employee of a Transport Company. He is entitled to transport allowance of ₹ 10,000 p.m. He spends ₹ 4,000
every month. The exemption shall be:
(a) ₹ 10,000 p.m. (c) ₹ 4,000 p.m.
(b) ₹ 7,000 p.m. (d) Nil

20. An assessee received ₹ 200 per month for 3 children as children education allowance. What shall be the amount
taxable in his hands for such allowance?
(a) ₹ 7,200 (c) ₹ 800
(b) ₹ 2,400 (d) Nil

21. Allowances paid by government outside India to an Indian citizen would be wholly exempted from income tax?
(a) True (c) False
(b) Partly true (d) None of the above

22. Fixed Medical Allowance is


(a) Fully Taxable (c) Fully Exempt
(b) Partly Taxable (d) None of the above

23. Which of the following is specified employee?


(a) Director of the company
(b) Employee having 15% or more voting power in the employer company
(c) Employee having a salary of more than ₹ 45,000
(d) All of the above
CA SHREY RATHI SALARY 3.21

24. An assessee (employee) is provided with gas and electricity by the employer which is produced by the employer
where manufacturing cost is ₹ 2,000. The amount recovered from the employee is ₹ 400. What amount shall be
taxable in the hands of the employee?
(a) ₹ 400 (c) ₹ 2000
(b) ₹ 1,600 (d) Nil

25. A gardener, sweeper and a watchman are employed by the employee but their salary of ₹ 500 p.m. per person is paid
by the employer. The valuation of this perquisite shall be:
(a) ₹ 4,320 (c) ₹ 18,000
(b) ₹ 1,960 (d) ₹ 6,000

26. R an employee, owns a car which he uses for his private as well as official purposes. The expenses of running and
maintenance of the car is met by the employer. The perquisite shall:
(a) be taxable in case of specified employee only
(b) be taxable in case of an employee other than specified employees
(c) be taxable case of specified and non-specified employee
(d) not be taxable

27. In which of following case should amount spent by the employer on education of children is taxable?
(a) Education facility is owned by the employer and value per child exceed ₹ 1000 p.m.
(b) Free education is provided in any other educational institution by reason of employee being in employment of
that employer and value per child exceed ₹ 1000 p.m.
(c) Any of the above
(d) None of the above

28. When the motor car is owned by the employee and used by the employee for private purpose. What amount shall be
taxable in the hands of the employee?
(a) Actual expenditure incurred by employer shall be taxable
(b) Nil
(c) Discretion of Assessing Officer
(d) None of the above

29. A car of 1500 cc is provided by the employer, to the employee whose salary is ₹ 20,000 p.m. The car is used by him
partly for official and partly for his personal purposes. The expenses of running and maintenance for official purpose is
met by the employer. The valuation of this perquisite shall be:
(a) Nil (c) ₹ 1,800 p.m.
(b) ₹ 600 p.m. (d) ₹ 2,400 p.m.

30. Scholarship provided to the child of the employee by the employer’s school shall be ……………… in the hands of the
employee.
(a) Taxable (c) Taxable beyond ₹ 12,000
(b) Exempt (d) None of the above

31. When the employee performs journey by any mode other than air & rail and further the destination is not connected
by rail, what amount shall be exempt from tax?
(a) then second class rail fare by the shortest route to the place of destination
(b) then first class rail fare by the shortest route to the place of destination
(c) then first class rail fare by the longest route to the place of destination
(d) then 1st class fare or deluxe class fare

32. If any hospital or dispensary is maintained by any local authority and the amount is provided to the employee for
treatment in such hospital, the amount so paid shall …………………….
(a) not be exempt (c) be partly taxable
(b) exempt without any limit (d) taxable above ₹ 15,000
CA SHREY RATHI SALARY 3.22

33. Health insurance premium (Accident insurance policy) incurred or reimbursed for insurance on the health of
employee or any member of his family is fully …………………….
(a) Taxable (c) exempt without limit
(b) exempt upto ₹ 5,000 (d) none of the above

34. During the previous year, the employee was reimbursed ₹ 24,000 as medical expenses incurred by him which includes
₹ 7,000 spent in Government hospital. The taxable perquisite in this case shall be:
(a) ₹ 17,000 (c) Nil
(b) ₹ 2,000 (d) ₹ 24,000

35. What is the amount of Rent free accommodation taxable when accommodation has been provided to employee by
government employer?
(a) 15% of salary (c) 10% of salary
(b) Licence fees as per Government rules (d) 7.5% of salary

36. When an accommodation has been provided in a hotel by a government employer, what amount shall be taxable in
the hands of employee?
(a) 24% of salary (c) least of (a) and (b)
(b) actual charges (d) licence fees as per Government rules

37. Ram, Finance Manager in ABC Ltd. The company has provided him rent-free unfurnished accommodation in Mumbai.
He gives you the following particulars:
Particulars (₹)
Basic salary ₹ 6,000 p.m.
Advance salary for April 2021 ₹ 5,000
Dearness Allowance ₹ 2,000 p.m. (30% for retirement benefits)
Bonus ₹ 1,500 p.m.
Even though the company allotted house to him on 01.04.2022, he occupied the same only from 01.11.2022. Calculate the
taxable value of perquisite for A.Y. 2023-24.
(a) ₹ 6,075 (c) ₹ 6,000
(b) ₹ 14,580 (d) Nil

38. Interest on concessional loan shall be calculated for part of the month. Is the statement correct?
(a) Valid, interest shall be calculated for the average month
(b) Invalid, interest shall be calculated for whole month for the balance outstanding as on first day of the month
(c) Invalid, interest shall be calculated for whole month for the balance outstanding as on last day of the month
(d) None of the above

39. An employer provided his employee a sofa set whose actual cost is ₹ 70,000. Employee pays ₹ 2,000 to his employee
towards this sofa set. What amount shall be taxable in the hands of the employee?
(a) ₹ 7,000 (c) ₹ 2,000
(b) ₹ 5,000 (d) ₹ 10,000

40. When an asset transferred by the employer to his employee is assets other than computer and electronic items and
Motor Car, what shall be taxable in the hands of the employee?
(a) Purchase Price of employer - 50 % of cost based on SLM for each incomplete year of usage
(b) Purchase Price of employer - 50 % of cost based on WDV for each completed year of usage
(c) Purchase Price of employer - 20 % of cost based on SLM for each completed year of usage
(d) Purchase Price of employer - 10% of cost based on SLM for each completed year of usage

41. If a free meal of ₹ 150 is provided to the employee. What amount shall be taxable in the hands of the employee?
(a) ₹ 150 per meal (c) ₹ 100 per meal
(b) ₹ 50 per meal (d) Nil
CA SHREY RATHI SALARY 3.23

42. If Credit card and club expenditure is incurred for any purpose other than official purpose, what amount shall be
taxable?
(a) Nothing is taxable
(b) Anything above ₹ 10,000 shall become taxable
(c) Actual expenditure incurred by the employer shall be taxable
(d) None of the above

43. If the value of gift in aggregate during the previous year is upto ₹ 5,000, the same shall
(a) taxable as allowance (c) taxable as perquisite
(b) not be taxable (d) none of the above

44. Tea or snacks provided during office hours to the employee is


(a) fully exempt without limit (c) fully taxable
(b) taxable beyond a limit (d) none of the above

45. Gratuity received during employment is …………………….


(a) partially exempt from tax (c) fully exempt from tax
(b) fully taxable (d) none of the above

46. Ramesh who is non-government employee and covered by the Payment of Gratuity Act 1972 retired on 15.06.2022
after completion of 26 years 8 months of service and received gratuity of ₹ 6,00,000. At the time of retirement his
salary was:
Basic Salary : ₹ 5,000 p.m.
Dearness Allowance : ₹ 3,000 p.m. (60% of which is for retirement benefits)
Commission : 1% of turnover (turnover in the last 12 months was ₹ 12,00,000)
Bonus : ₹ 12,000 p.a.
What amount of gratuity shall be taxable?
(a) ₹ 4,75,385 (c) ₹ 4,98,600
(b) Nil (d) ₹ 20,00,000

47. X retired on 15.4.2022 from a company. He was entitled to a pension of ₹ 4,000 p.m. At the time of retirement, he
got 75% of pension commuted & received ₹ 1,20,000 as commuted pension. Compute the taxable portion of
commuted pension if he is entitled to gratuity.
(a) ₹ 66,667 (c) ₹ 53,333
(b) ₹ 1,20,000 (d) ₹ 78,667

48. Uncommuted pension is fully taxable in the hands of …………………….


(a) government employee (c) both (a) and (b)
(b) non-government employee (d) exempt for all employees

49. Ravi retired on 01.10.2022 receiving ₹ 5,000 p.m. as pension. On 01.02.2023, he commuted 60% of his pension and
received ₹ 3,00,000 as commuted pension. What amount of uncommuted and commuted pension is taxable if Ravi is
Government employee?
(a) ₹ 24,000, Nil (c) ₹ 24,000, ₹ 3,00,000
(b) Nil, ₹ 3,00,000 (d) Nil, Nil

50. Which section of the Income tax Act provides about exemption of amount received by way Retrenchment
compensation?
(a) 10(10) (c) 10(10C)
(b) 10(10A) (d) 10(10B)
CA SHREY RATHI SALARY 3.24

51. Rohit who is not a government employee retired on 01.12.2022 after 20 years 10 months of service, receiving leave
salary of ₹ 5,00,000. Other details of his salary income are:
Basic Salary : ₹ 5,000 p.m. (₹ 1,000 was increased w.e.f. 01.04.2022)
Dearness Allowance : ₹ 3,000 p.m. (60% of which is for retirement benefits)
Commission : ₹ 500 p.m.
Bonus : ₹ 1,000 p.m.
Leave availed during service : 480 days
He was entitled to 30 days leave every year. How much amount of leave salary shall be taxable?
(a) Nil (c) ₹ 4,73,600
(b) ₹ 5,00,000 (d) ₹ 3,00,000

52. Where leave salary is received in earlier year from former employer and received from another employer in later
year, the limit of ₹ 3,00,000 ……………………. by the amount of leave salary exempt earlier
(a) will not be reduced (c) may be reduced
(b) will be reduced (d) none of the above

53. Interest credited to statutory provident fund shall be:


(a) fully exempt (c) exempt upto 8.5% p.a.
(b) fully taxable (d) exempt upto 9.5%

54. Employee's own contribution to statutory provident fund or recognized provident fund or shall be subject to:
(a) deduction under section 80C (c) deduction under section 16 from gross salary
(b) deduction under section 80CCC (d) none of the above

55. Which deduction is allowed under section 16?


(a) Professional tax (c) Entertainment allowance
(b) Standard deduction of ₹ 40,000 (d) All of the above

56. Mr. Pappu was employed on 1/4/2012 in the grade of ₹ 15,000 - ₹ 500 - ₹ 17,000 - ₹ 750 - ₹ 21,500 - ₹ 1,000 - ₹
31,500. His gross salary for the PY 2022-2023 shall be:
(a) ₹ 2,70,000 (c) ₹ 2,58,000
(b) ₹ 2,16,000 (d) ₹ 1,80,000

57. The standard deduction is allowed from gross salary


(a) under section 16(i) (c) under section 16(ia)
(b) under section 16(ii) (d) under section 16(iii)

58. The deduction for Professional Tax under section 16(iii) is for
(a) Actual amount paid (c) Actual amount charged by State Government
(b) Actual amount due (d) Always ₹ 2,500

59. Salary for the purposes of exemption of gratuity when employee is covered under The Payment of Gratuity Act 1972
includes:
(a) Basic salary + DA (forming part of salary for retirement benefits or not)
(b) Basic salary + DA (forming part of salary for retirement benefits) + monthly commission
(c) Basic salary + DA (forming part of salary for retirement benefits) + commission on the percentage basis of sales
(d) Basic salary and commission

60. An employee is neither a Government employee nor covered under Payment of Gratuity Act, 1972. Salary for purpose
of calculating half month shall be taken as:
(a) Last drawn Salary
(b) Average salary of 10 months preceding the month of retirement
(c) Average salary of 10 months preceding the date of retirement
(d) Average salary of last 3 completed years
CA SHREY RATHI SALARY 3.25

61. Commuted pension received by non-government employee, who does not receive gratuity, is exempt up to:
(a) 1/3 of the full pension (c) 1/6 of the full pension
(b) 1/10 of the full pension (d) 1/2 of the full pension

62. Carry forward of leave travel concession is allowed:


(a) for all LTC
(b) only for one LTC and can be carry forwarded to any calendar year in the next block
(c) not allowed at all
(d) only for one LTC and can be carry forwarded to first calendar year in the next block

63. Underground allowance to an employee is exempt upto:


(a) ₹ 700 p.m. (c) ₹ 900 p.m.
(b) ₹ 1,000 p.m. (d) ₹ 800 p.m.

64. Mr. Om was employed since 1st July 2016 in an establishment. His salary was fixed at ₹ 14,800 in the grade of ₹
14,000 - ₹ 400 - ₹ 22,000 w.e.f. 1/7/2016. He got the benefit of 15% of salary as DA which is treated as forming part
of the salary for the retirement benefits. He retired on 1/2/2023 and received ₹ 3,40,000 as a Gratuity from his
employer. Calculate his income under the head ‘Salary’ for the PY 2022-2023 if he is a Central Government employee.
(a) ₹ 1,46,420 (c) ₹ 1,70,800
(b) ₹ 1,96,420 (d) Fully exempt

65. Entertainment allowance in case of government employee is:


(a) Fully exempt and therefore not included in Gross Salary
(b) Fully Taxable and therefore added in Gross Salary
(c) Not added in Gross Salary but deduction is allowed as per limits of section 16(ii)
(d) First added in full in Gross Salary and thereafter deduction allowed from Gross Salary is allowed under section
16(ii)

66. Mr. J is employed in GGC Ltd and his wife is suffering from a critical disease. The company has sent Mr. J and Mrs. J to
USA for the medical treatment of Mrs. J. The company has incurred expenses on medical treatment of Mrs. J and stay
outside India of Mrs. J and of Mr. J. amounting to ₹ 17,00,000 but RBI permitted only ₹ 15,00,000. The travel
expenses amounted to ₹ 1,50,000. Salary of Mr. J was ₹ 5,00,000. The taxable perquisite in this case shall be:
(a) ₹ 3,50,000 (c) ₹ 8,50,000
(b) ₹ 2,00,000 (d) ₹ 1,50,000

67. Payment from Recognized Provident Fund after 5 years of continuous service of employee shall be:
(a) Fully Taxable for employee
(b) Fully exempt for employee
(c) Taxable to the extent of employer’s contribution and interest thereon
(d) Exempt up to ₹ 10,00,000

68. Employee own a house in which he stays along with his family. His employer reimburses to him the electricity bill
amounting to ₹ 5,000 p.m. The perquisite shall
(a) Be taxable in case of specified employee only
(b) Be taxable in case of an employee other than specified employees
(c) Be taxable in case of all type of employees
(d) Not taxable at all for any employee

69. The employer has given a laptop for the personal use of the employee. The value of this perquisite shall be:
(a) Nil (c) 10% p.a. of the historical cost of the asset
(b) 10% p.a. of the W.D.V of the asset (d) none of the above

70. The employer had purchased a car for ₹ 8,00,000 3 years and 7 months ago. This car is sold to the employee for ₹
1,20,000. The value of this perquisite shall be
(a) ₹ 2,89,600 (c) ₹ 1,20,000
(b) ₹ 8,00,000 (d) ₹ 3,60,000
CA SHREY RATHI SALARY 3.26

71. Anjan joins a service on 1/4/2022 with basic salary of ₹ 39,100 p.m. plus dearness allowance of 107% of basic salary.
He has no other income. What is his taxable income for the PY 2022-23?
(a) ₹ 9,71,244 (c) ₹ 9,21,244
(b) ₹ 9,71,240 (d) ₹ 9,21,240

72. Bimal is employed in a factory at a salary of ₹ 2,400 per month. He also gets dearness allowance @ ₹ 600 per month
and bonus @ ₹ 200 per month. He retired on 31st December, 2022 and received ₹ 75,000 as gratuity under the
Payment of Gratuity Act, 1972 after serving 31 years and 4 months in that factory. The amount of gratuity exempt
under the Income-tax Act, 1961. will be -
(a) ₹ 75,000 (c) ₹ 53,654
(b) ₹ 21,346 (d) ₹ 20,00,000

73. Salary of an employee is ₹ 2,00,000. Rent paid by the employer for the unfurnished house provided to employee at
Faridabad is ₹ 3,000 p.m. The employer charges ₹ 2,000 p.m. as rent from the employee. The valuation of this
perquisite shall be:
(a) ₹ 24,000 (c) ₹ 36,000
(b) ₹ 12,000 (d) ₹ 6,000

74. Mr. P is employee of GC Ltd. and he is provided a car of engine of 1.9 litres capacity along with driver. The expenses
of running and maintenance of car are met by Mr. P himself. Besides using the car for official purpose, Mr. P also uses
the car for his personal purpose. The valuation of the perquisite of car shall be:
(a) ₹ 2,400 pm (c) ₹ 1,800 pm
(b) ₹ 600 pm (d) ₹ 900 pm

75. Mr. T is entitled to a watchman allowance of ₹ 6,600 p.m. He pays ₹ 5,000 p.m. to the watchman employed by him.
The taxable allowance shall be:
(a) ₹ 6,600 pm (c) ₹ 5,000 pm
(b) ₹ 1,600 pm (d) Not taxable at all

76. Mr. Q purchased a residential house property in Ahmedabad on loan for which he paid an interest of ₹ 50,000 during
the previous year. He is working in Delhi and getting an HRA of ₹ 4,000 per month. He can claim exemption/
deduction for -
(a) Only HRA (c) Only interest paid
(b) Either interest paid or HRA but not both (d) Both HRA and interest paid

77. Mr. X is an employee of Gurukul Public School. His daughter, Aruna, is studying in the said school at concessional fees
of ₹ 600 per month (Actual fee ₹ 4,000 per month). The amount taxable in the hands of Mr. X will be
(a) ₹ 48,000 (c) ₹ 7,200
(b) ₹ 28,800 (d) ₹ 40,800

78. Mr. Z took an interest-free loan of ₹ 15,000 from GGC Ltd. (the employer). Assuming that the market rate of interest
on similar loan is 10% p.a., the taxable value of the perquisite in the hands of Mr. Z will be
(a) ₹ 150 (c) ₹ 1,500
(b) Nil (d) ₹ 750

79. During the PY 2022-23, Mr. R received ₹ 20,000 in silver coins from his employer as a gift. The taxable value of the
silver coins will be
(a) ₹ 15,000 (c) Nil
(b) ₹ 20,000 (d) None of the above

80. Mr. Z gets salary of ₹ 12,000 p.m. and is provided with rent free unfurnished accommodation at Pune (which has
population of 20 lakh): House is owned by employer, fair rental value of which is ₹ 1,400 p.m. House was provided
with effect from 1st July, 2022. Value of the perquisite of rent-free accommodation will be:
(a) ₹ 21,600 (c) ₹ 10,800
(b) ₹ 16,200 (d) ₹ 12,600
CA SHREY RATHI SALARY 3.27

81. Mr. V is employed in a company with annual gross salary of ₹ 8,60,000. The company paid income-tax of ₹ 37,000 on
his non-monetary perquisites. He paid ₹ 1,20,000 to recognized provident fund during the PY 2022-23. His total
income would be:
(a) ₹ 8,60,000 (c) ₹ 8,10,000
(b) ₹ 7,27,000 (d) ₹ 6,90,000

82. Mr. L retired from service with GGC Ltd. on 31st January, 2023. He received the following amounts from unrecognized
provident fund:
Own contribution ₹ 1,50,000; Interest on own contribution ₹ 21,000; Employer’s contribution ₹ 1,10,000; and Interest
on employer’s contribution ₹ 15,000.
How much of the receipt is chargeable to tax as income from salary?
(a) ₹ 15,000 (c) ₹ 1,25,000
(b) ₹ 1,71,000 (d) ₹ 85,000

83. The standard deduction is allowed from gross salary to the maximum of ₹ 50,000 but
(a) Employee has to prove his all expenses to income tax department
(b) Employee has to prove his all expenses to the employer
(c) Employee has to prove his all expenses to income tax department or employer as per his own discretion
(d) Irrespective of any expenses that employee may or may not have incurred

84. An employee is covered under Payment of Gratuity Act, 1972 If the employee has completed service of 16 years 6
months and 5 days then to calculate exemption of Gratuity the number of completed years shall be taken as:
(a) 16 years (c) 17 years
(b) 16 years 6 months and 5 days (d) 16 years and 7 months

85. An employee who is not covered under Payment of Gratuity Act, 1972. For purpose of computing 15 days’ salary, the
number of days in a month shall be taken as:
(a) 30 days (c) 26 days
(b) 31 days (d) Any of the above

86. Mr. K who was working with another company joined the present employer on 1/5/2022 at a salary of ₹ 20,000 p.m.
His salary becomes due on first of next month. He was also entitled to a pension of ₹ 8,000 p.m. from his former
employer as he retired on 31/3/2022. His pension is received at the end of each month. His gross salary for PY 2022-
23 shall be:
(a) ₹ 2,20,000 (c) ₹ 3,16,000
(b) ₹ 2,96,000 (d) ₹ 2,56,000

87. Leave Travel Concession is


(a) always taxable in hands of employee (c) exempt once in a prescribed block of 4 calendar years
(b) always exempt in hands of employee (d)exempt twice in a prescribed block of 4 calendar years

88. Employer provides a car (below 1600 cubic capacity) along with a driver to Mr. H and he uses the car partly for official
and partly for personal purpose. The expenses incurred by the employer are:
(1) running and maintenance expenses of ₹ 84,000
(2) driver's salary of ₹ 1,20,000
Taxable value of perquisite is:
(a) ₹ 21,600 (c) ₹ 10,800
(b) ₹ 32,400 (d) ₹ 2,04,000

89. Mr. D is entitled to a conveyance allowance of ₹ 1,000 p.m. commutation for office purpose within local areas. He
spends ₹ 600 pm. The taxable allowance shall be
(a) ₹ 1,000 p.m. (c) ₹ 600 p.m.
(b) ₹ 400 p.m. (d) NIL
CA SHREY RATHI SALARY 3.28

90. Mr. R is a Pilot with Jet Airways. He is entitled to outstation allowance of ₹ 10,000 p.m. He spends ₹ 4,000 every
month. The taxable allowance shall be
(a) ₹ 1,20,000 (c) ₹ 48,000
(b) ₹ 36,000 (d) ₹ 72,000

91. Which of the following allowance is partly taxable:


(a) Allowances from UNO
(b) House Rent Allowance
(c) Allowance granted to Government employees outside India.
(d) Fixed Medical Allowance

92. In case of transfer from one place to another, if employee is provided house at new place and also allowed to retain
house at old place, what shall be taxable after 90 days?
(a) value of one house with lower value shall be taxable
(b) value of one house with higher value shall be taxable
(c) average of value of both houses shall be taxable
(d) value of both the house shall be taxable

93. Salary, bonus, commission or remuneration by whatever name called due to or received by partner of a
firm………..……. regarded as salary
(a) shall be (c) shall not be
(b) may be (d) none of the above

94. When accommodation is not owned by the employer being other than government employer is provided to his
employee as RFA, the population in which accommodation is located shall be considered. Discuss the validity of the
statement?
(a) Valid (c) partly Invalid
(b) the population of the city is irrelevant (d) none of the above

95. Fare of more than 2 children shall not be allowed in case of leave travel concession. However, all children born before
……………. shall be eligible for exemption.
(a) 1st October 1998 (c) 1st December 1998
st
(b) 1 April 1998 (d) 1st October 1999

96. The company has given an option to his employee to purchase the shares of the company for ₹ 86 per share. The
FMV of the shares is ₹ 123 per share. The employee exercised such option and purchased 300 shares. What will be
the value of perquisite?
(a) ₹ 11,100 (c) ₹ 6,100
(b) Nil (d) ₹ 36,900

97. The treatment of professional tax paid by the employer shall be:
(a) Deducted from the gross salary of the employee
(b) No treatment
(c) Added in the salary and then deduction shall be allowed from gross salary
(d) Only added in the salary but no deduction is allowed

98. Manohar took a personal loan from his employer of ₹ 32,000 on 22.09.2022. He repaid 40% on 28.02.2023. Rate of
interest charged by SBI on 01.04.2022 is 12% p.a. & on 22.09.2022 is 13% p.a. Compute interest on concessional loan
to be taxed in the hands of the employee.
(a) ₹ 1,600 (c) ₹ 1,984
(b) ₹ 1,733 (d) ₹ 2,149

99. Mr. V receives uniform allowance of ₹ 5,000 per quarter. He spends ₹ 2,000 p.m. on his office uniform. Compute his
taxable uniform allowance.
(a) ₹ 4,000 (c) ₹ 24,000
(b) ₹ 20,000 (d) Nil
CA SHREY RATHI SALARY 3.29

100. Mr. Jagat is an employee in accounts department of Bharat Ltd., a cellular company operating in the regions of
eastern India. It is engaged in manufacturing of cellular devices. During F.Y. 2022-23, following transactions were
undertaken by Mr. Jagat:
(i) He attended a seminar on “Perquisite Valuation”. Seminar fees of ₹ 12,500 was paid by Bharat Ltd.
(ii) Tuition fees of Mr. Himanshu (son of Mr. Jagat) paid to private coaching classes (not having any tie-up with
Bharat Ltd.) was reimbursed by Bharat Ltd. Amount ₹ 25,000.
(iii) Ms. Sapna (daughter of Mr. Jagat) studies in DPS Public School (owned and maintained by Bharat Ltd.) Tuition
fees paid for Ms. Sapna was ₹ 750 per month by Mr. Jagat. Cost of education in similar institution is ₹ 5,250 per
month.
What shall be the amount which is chargeable to tax under the head “Salaries” in hands of Mr. Jagat for A.Y. 2022-23?
(ICAI MCQ Booklet)
(a) ₹ 25,000 (c) ₹ 67,000
(b) ₹ 37,500 (d) ₹ 79,000

MCQ’s Answers
1. (d) 2. (d) 3. (c) 4. (a) 5. (b) 6. (c) 7. (a) 8. (a) 9. (c) 10. (b)
11. (a) 12. (d) 13. (a) 14. (b) 15. (c) 16. (b) 17. (d) 18. (a) 19. (b) 20. (c)
21. (a) 22. (a) 23. (a) 24. (b) 25. (c) 26. (c) 27. (c) 28. (a) 29. (c) 30. (b)
31. (d) 32. (b) 33. (c) 34. (a) 35. (b) 36. (c) 37. (a) 38. (c) 39. (b) 40. (d)
41. (c) 42. (c) 43. (b) 44. (a) 45. (b) 46. (a) 47. (a) 48. (c) 49. (a) 50. (d)
51. (c) 52. (b) 53. (a) 54. (a) 55. (d) 56. (c) 57. (c) 58. (a) 59. (a) 60. (b)
61. (d) 62. (d) 63. (d) 64. (a) 65. (d) 66. (a) 67. (b) 68. (c) 69. (a) 70. (a)
71. (d) 72. (c) 73. (d) 74. (c) 75. (a) 76. (d) 77. (b) 78. (b) 79. (b) 80. (c)
81. (d) 82. (c) 83. (d) 84. (c) 85. (a) 86. (b) 87. (d) 88. (b) 89. (b) 90. (b)
91. (b) 92. (d) 93. (c) 94. (b) 95. (a) 96. (a) 97. (c) 98. (a) 99. (d) 100. (c)
CA SHREY RATHI HOUSE PROPERTY 4.1

MAHATMA GANDHI

CHAPTER 4
HOUSE PROPERTY
"Be the change that you wish to see
in the world.”

PRACTICAL QUESTIONS
Q 1: Perm owns a house in Madras. During the previous year 2022-23, 2/3rd portion of the house was self–occupied
whereas 1/3rd portion was let out for residential purposes at a rent of ₹ 8,000 p.m. Municipal value of the property is ₹
3,00,000 p.a., fair rent is ₹ 2,70,000 p.a., and standard rent is ₹ 3,30,000 p.a. He paid municipal taxes @ 10% of municipal
value during the year. A loan of ₹ 25,00,000 was taken by him during the year 2019 for acquiring the property. Interest on
loan paid during the previous year 2022-23 was ₹ 1,20,000. Compute Prem’s income from house property for the A.Y.
2023-24. [ICAI Study Module]

Sol 1: Computation of Prem’s income from house property for the A/Y 2023-24:
There are two units of the house. Unit I with 2/3rd area is used by Prem for self-occupation throughout the year and
therefore its annual value will be Nil. Unit 2 with 1/3rd area is let-out throughout the previous year and its annual value
has to be determined as per section 23(1).
Particulars 1/3rd (LO) (₹) 2/3rd (SO) (₹)
Gross Annual Value [Expected Rent or Actual Rent whichever is higher] (for Let- 1,00,000 -
Out property)
Expected Rent = Municipal Value or Fair Rental Value whichever is higher but shall
be limited to Standard Rent (divided proportionately) ₹ 1,00,000
Actual Rent = ₹ 8,000 x 12 = ₹ 96,000
Less: Municipal taxes paid (₹ 3,00,000 x 10% x 1/3) 10,000 -
Net Annual Value 90,000 -
Less: Deductions u/s 24
(a) Statutory deduction @ 30% 27,000
(b) Interest on borrowed capital (divided proportionately) 40,000 80,000
Income u/h house property 23,000 (80,000)
Total (57,000)

Q 2: Mr. Pannu owns a house property whose construction got completed on 01.04.2022. It has been let out from 1st April
for residential purposes @ ₹ 6,000 p.m. to a tenant. Compute his income from house property with the help of the
following information for the P/Y 2022-23 are as under:
(₹)
1. Fair Rent 58,000
2. Urban land tax levied 2,500
3. Municipal taxes payable 5,000
CA SHREY RATHI HOUSE PROPERTY 4.2

4. Rent of one month could not be realised. The house was got vacated on 31.12.2022 from the defaulting tenant and
let out to another tenant at ₹ 7,000 p.m.
5. He borrowed ₹ 60,000 from a bank on 1st September 2020 @ 15% p.a. for the construction of the house. He has
refunded ₹ 24,000 on 1st October 2022.

Sol 2: Computation of income u/h house property for Mr. Pannu for the P/Y 2022-23:
Particulars (₹)
Gross Annual Value [Expected Rent or Actual Rent whichever is higher]
Expected Rent = Fair Rent = ₹ 58,000
Actual Rent = (₹ 6,000 x 8) + (₹ 7,000 x 3) = ₹ 69,000 69,000
Less: Municipal taxes paid (payable shall not be allowed) -
Net Annual Value 69,000
Less: Deductions u/s 24
(a) Statutory deduction @ 30% 20,700
(b) Interest on borrowed capital [WN 1] 10,050
Income u/h house property 38,250
WN 1: Calculation of interest on borrowed capital
Pre-construction period interest: Pre-construction period means the period commencing on the date of borrowing and
ending on:
(i) 31st March immediately prior to the date of completion of construction / date of acquisition or
(ii) Date of repayment of loan
whichever is earlier.
Pre-construction period: 01.09.2020 – 31.03.2022
₹ 60,000 x 15% x 19/12 = ₹ 14,250
Pre-construction period is permissible as deduction in 5 equal instalments = ₹ 14,250/5 = ₹ 2,850
Post-construction period interest: 01.04.2022 – 31.03.2023
₹ 60,000 x 15% x 6/12 = ₹ 4,500
₹ 36,000 x 15% x 6/12 = ₹ 2,700
Total = ₹ 7,200
Total interest on borrowed capital = Pre-construction interest + Post-construction interest
= ₹ 2,850 + ₹ 7,200 = ₹ 10,050

Q 3: Mr. Ganesh owns a house in Delhi. During the P/Y 2022-23, 3/4th portion of the house was self-occupied for full year
and 1/4th portion was let out for residential purpose from 01.04.2022 to 31.12.2022 on a rent of ₹ 700 p.m. From
01.01.2023, this portion was also used for own residence. Municipal value of the house is ₹ 20,000.
He incurred the following expenditure in respect of the house property:
Municipal taxes due: ₹ 6,000; Repairs: ₹ 2,500; Fire insurance premium: ₹ 4,000; Land revenue: ₹ 8,200; Ground rent: ₹
1,200.
A loan of ₹ 1,00,000 was taken on 01.04.2017 @ 9% p.a. for the construction of the house which was completed on
28.03.2018. Nothing was repaid on loan account so far.
Find out his income from house property for the P/Y 2022-23.
CA SHREY RATHI HOUSE PROPERTY 4.3

Sol 3: Computation of income from house property for the P/Y 2022-23 for Mr. Ganesh:
Particulars 1/4th (LO) (₹) 3/4th (SO) (₹)
Gross Annual Value [Expected Rent or Actual Rent whichever is higher] 8,400 -
Expected Rent = Fair rent = ₹ 700 x 12 (always computed for 12 months) ₹ 8,400
Actual Rent = let out period = ₹ 700 x 9 = ₹ 6,300
Municipal taxes paid (due shall not be allowed) - -
Net Annual Value 8,400 -
Less: Deductions u/s 24
(a) Statutory deduction @ 30% 2,520 -
(b) Interest on borrowed capital [WN 2] (divided proportionately) 2,250 6,750
Income u/h house property 3,630 6,750
Total (3,120)
WN 1: Where the question deals with letting out of property for a part of the year and self-occupied for the remaining
part, then actual rent received shall be treated as fair rent, if the fair rent is not given in the question.
WN 2: Calculation of interest on borrowed capital
There will no pre-construction period interest as the property is completed in the year in which the loan was taken.
Post-construction period: 01.04.2022 – 31.03.2023
1,00,000 x 9% x 12/12 = 9,000

Q 4: Mr. Lalit is the owner of a two stories house in Chennai. He gets a monthly rent of ₹ 7,000 from his tenant in the
ground floor. The first floor, identical in all respect with the ground floor used to be occupied by a friend of Lalit from
whom he charged a rent of ₹ 5,000 p.m.
During the year ended 31.03.2023, the friend stayed in Lalit’s house upto 31.12.2022. On 01.01.2023, it was again let out
to tenant at a rent of ₹ 7,000 p.m.
Details of expenses incurred by Lalit during the year ending 31.03.2023 in respect of the house are as under:
(₹)
1. Cost of repairing ground floor 7,900
2. Cost of repairing first floor 60,000
3. Interest on loan taken for construction of first floor 20,000
4. Municipal tax paid by the owner 6,000
5. Monthly salary of an employee for collecting rent 1,200
Compute Lalit’s income from house property for the A/Y 2023-24.

Sol 4: Computation of Lalit’s income from house property for the A/Y 2023-24:
Particulars Ground Floor (₹) First Floor (₹)
Gross Annual Value [Expected Rent or Actual Rent whichever is higher] 84,000 84,000
Expected Rent = Fair Rent = ₹ 7,000 x 12 (for both floors) ₹ 84,000
Actual Rent = ₹ 7,000 x 12 = ₹ 84,000 (GF)
(₹ 5,000 x 9) + (₹ 7,000 x 3) = ₹ 66,000 (FF)
Less: Municipal taxes paid 3,000 3,000
Net Annual Value 81,000 81,000
Less: Deductions u/s 24
(a) Statutory deduction @ 30% 24,300 24,300
(b) Interest on borrowed capital - 20,000
Income from house property 56,700 36,700
Total 93,400
WN 1: As the ground floor is let out to an unrelated person for ₹ 7,000 p.m., it shall be deemed that the fair rent of the
first floor shall be same even if it is let out to the friend for ₹ 5,000 p.m.
CA SHREY RATHI HOUSE PROPERTY 4.4

Q 5: Mr. Arjun has let out one showroom building for ₹ 1,00,000 p.m. and has paid municipal tax ₹ 85,000. Fair rent of the
house is ₹ 98,000 p.m. He has received arrears of rent ₹ 3,00,000 relating to the previous year 2017-18. He has also
received unrealised rent of ₹ 4,00,000 of previous year 2015-16 and also interest of ₹ 22,000 on such unrealised rent and
he has paid ₹ 37,000 to the lawyer in connection with the recovery of unrealised rent.
Compute his income from house property for the A/Y 2023-24.

Sol 5: Computation of income from house property for the A/Y 2023-24:
Particulars (₹)
Gross Annual Value [ER or AR whichever is higher] 12,00,000
Expected Rent = Fair Rent = ₹ 98,000 x 12 = ₹ 11,76,000
Actual Rent = ₹ 1,00,000 x 12 = ₹ 12,00,000
Less: Municipal taxes paid 85,000
Net Annual Value 11,15,000
Less: Deductions u/s 24
(a) Statutory deduction @ 30% 3,34,500
(b) Interest on borrowed capital -
7,80,500
Add: Recovery of unrealised rent & arrears of rent received u/s 25A [₹ 4,00,000 + ₹ 3,00,000) –
30%] 4,90,000
Income u/h House Property 12,70,500
➢ Neither interest on unrealised rent nor expenses in connection with recovery of unrealised rent shall be dealt with in
the house property chapter.

Q 6: Poorna has one house property at Indira Nagar in Bangalore. She stays with her family in the house. The rent of
similar property in the neighbourhood is ₹ 25,000 p.m. The municipal valuation is ₹ 23,000 p.m. Municipal taxes paid is ₹
8,000. The house construction began in February 2016 with a loan of ₹ 20,00,000 taken from SBI housing finance Ltd. The
construction was completed on 30.11.2018. The accumulated interest up to 31.3.2018 is ₹ 1,50,000. During the previous
year 2022-23, Poorna paid ₹ 2,40,000 which included ₹ 1,80,000 as interest. Compute Poorna’s income from house
property for A.Y. 2023-24. [ICAI Study Module]

Sol 6: Computation of income from house property of Smt. Poorna for A.Y. 2023-24:
Particulars (₹)
Annual value of one house used for self-occupation u/s 23(2)
Less: Deductions u/s 24
(a) Statutory deduction @ 30%
(b) Interest on borrowed capital 2,00,000
Interest on loan was taken for construction of house on or after 01.04.1999 and same was
completed within the prescribed time–interest paid or payable subject to a maximum of ₹ 2,00,000
(including apportioned pre-construction interest) will be allowed as deduction.
In this vase the total interest is ₹ 1,80,000 + ₹ 30,000 (being 1/5th of ₹ 1,50,000) = ₹ 2,10,000.
However, the interest deduction is restricted to ₹ 2,00,000.
Income from house property 2,00,000

Q 7: Mr. Pawan took a loan of ₹ 6,10,500 @ 7% p.a. on 01.09.2019 for construction of one house which was completed on
01.06.2022 and it was let out for ₹ 9,000 p.m. It remained vacant for 2 and a half months and there is an unrealised rent
of ₹ 1,200. The fair rent of the house is ₹ 10,000 p.m. Assessee has repaid half of the loan amount on 01.07.2021 and
remaining half on 01.02.2023. He has also paid municipal tax of ₹ 3,000 which relates to the previous year 2021-22.
Compute his income from house property for the P/Y 2022-23.
CA SHREY RATHI HOUSE PROPERTY 4.5

Sol 7: Computation of income u/h house property for Mr. Pawan:


Particulars (₹)
Gross Annual Value [Expected Rent or Actual Rent whichever is higher] 1,00,000
Expected Rent = Fair Rent = ₹ 10,000 x 10 = ₹ 1,00,000 (computed for 10 months as house got completed on
01.06.2022)
Actual Rent = (₹ 9,000 x 7.5) – ₹ 1,200 = ₹ 66,300
Less: Municipal taxes paid (allowed irrespective of the year it relates to) 3,000
Net Annual Value 97,000
Less: Deductions u/s 24
(a) Statutory deduction @ 30% 29,100
(b) Interest on borrowed capital [WN 1] 36,681
Income u/h house property 31,219
WN 1: Calculation of interest on borrowed capital
Pre-construction period interest: Pre-construction period means the period commencing on the date of borrowing and
ending on:
(i) 31st March immediately prior to the date of completion of construction / date of acquisition or
(ii) Date of repayment of loan
whichever is earlier.
Pre-construction period: 01.09.2019 – 31.03.2022
₹ 6,10,500 x 7% x 22/12 = ₹ 78,348
₹ 3,05,250 x 7% x 9/12 = ₹ 16,026
Total = ₹ 78,283 + ₹ 16,026 = ₹ 94,374
Pre-construction period is permissible as deduction in 5 equal instalments = ₹ 94,374/5 = ₹ 18,875.
Post-construction period interest: 01.04.2022 – 01.02.2023
₹ 3,05,250 x 7% x 10/12 = ₹ 17,806
Total interest on borrowed capital = Pre-construction interest + Post-construction interest
= ₹ 18,875 + ₹ 17,806 = ₹ 36,681.

Q 8: Mr. Arvind has 3 houses. Compute his income under the head house property from the following information relating
to the three houses:
House 1: It is self-occupied with fair rent of ₹ 48,000, municipal valuation - ₹ 56,000 and rent as per Rent Control Act is ₹
50,000. However, the house remains vacant for 3 months. The house got completion certificate on 01.07.2020. Loan taken
for repairs to the house - ₹ 5,00,000 on 01.04.2022 @ 13% p.a. and the loan amount was repaid on 01.03.2023.
The assessee has recovered unrealised rent of ₹ 2,000 in the previous year 2022-23 and interest on such unrealised rent
amounted to ₹ 360. However, the assessee has incurred legal expenses of ₹ 560. Such unrealised rent happened in the
year 2017-18.

House 2: It is also self-occupied. Its similar building rent is ₹ 64,000 and rent determined by the municipality for charging
house tax is ₹ 66,000 whereas its standard rent is ₹ 5,800 p.m. Municipal taxes payable – ₹ 5,600. He received arrears of
rent ₹ 6,000 relating to the previous year 2019-20.

House 3: It is let out for ₹ 54,000 p.m. and fair rent is ₹ 60,000 p.m. Water tax and house tax paid to municipality is ₹
11,000. Insurance premium paid - ₹ 9,800 and expenses on repairs – ₹ 3,300. Interest on capital borrowed for purchase of
house is ₹ 55,000.
CA SHREY RATHI HOUSE PROPERTY 4.6

Sol 8: Computation of income u/h house property for Mr. Arvind:


Particulars H-1 (SO) (₹) H-2 (SO) (₹) H-3 (LO) (₹)
Gross Annual Value [Expected Rent or Actual Rent whichever is higher] - - 7,20,000
Expected Rent = Municipal Value or Fair Rental Value whichever is higher
but shall be limited to Standard Rent.
Less: Municipal Taxes - - 11,000
Net Annual Value - - 7,09,000
Less: Deductions u/s 24
(a) Statutory deduction @ 30% - - 2,12,700
(b) Interest on borrowed capital [WN 1] 30,000 - 55,000
(30,000) - 4,41,300
Add: Recovery of unrealised rent u/s 25A for House-1 1,400 - -
Add: Recovery of arrears of rent received u/s 25A for House-2 - 4,200 -
Income u/h house property (28,600) 4,200 4,41,300
Total 4,16,900
WN 1: Calculation of interest on borrowed capital:
There will be no pre-construction period interest as the loan was taken after the completion of the property.
Post-construction period interest: 01.04.2022 – 01.03.2023
5,00,000 x 13% x 11/12 = 59,583 (but limited to 30,000 in case of SO property as the loan is taken for the repair of the
property)

Q 9: Smt. Rajalakshmi owns a houses property in Chennai. The municipal value of the property is ₹ 5,00,000, fair rent is ₹
4,20,000 and standard rent is ₹ 4,80,000. The property was let–out for ₹ 50,000 p.m. up to December 2022. Thereafter,
the tenant vacated the property and the house remained vacant for the rest of the year. Rent for the months of
November and December 2022 could not be realised in spite of the owner’s efforts, all the conditions prescribed under
rule 4 are satisfied. She paid municipal taxes @ 12% during the year. She had paid interest of ₹ 35,000 during the year for
amount borrowed for repairs for the house property. Compute her income from house property for the A.Y. 2023-24.
[ICAI Study Module]

Sol 9: Computation of income from house property of Smt. Rajalakshmi:


Particulars (₹)
Gross Annual Value [Expected Rent or Actual Rent whichever is higher] 3,50,000
Expected Rent = Municipal Value or Fair Rental Value whichever is higher but shall be limited to
Standard Rent = ₹ 4,80,000
Actual Rent = ₹ 50,000 x 7 [WN 1] ₹ 3,50,000
Less: Municipal taxes paid (₹ 5,00,000 x 12%) 60,000
Net Annual Value 2,90,000
Less: Deductions u/s 24
(a) Statutory deduction @ 30% 87,000
(b) Interest on borrowed capital 35,000
Income from House Property 1,68,000
WN 1: There was an unrealised rent for 2 months due to which the actual rent came down to ₹ 5,00,000 (i.e. ₹ 50,000 x
10). Now because of vacancy of 3 months, the actual rent was further reduced to ₹ 3,50,000. Therefore, it can be
concluded that actual rent receivable after deduction of unrealised rent is higher than the expected rent, and further got
less than the expected rent because of vacancy. Therefore, Actual rent shall be the GAV.

Q 10: Ms. Annie has taken a loan of ₹ 15,00,000 from HDFC Bank on 01.07.2020 @ 10% p.a. for the construction of her
residential house which got completed on 01.05.2022. W.e.f 01.06.2022 the house was let out for ₹ 80,000 p.m. whereas
fair rent of the house was ₹ 90,000 p.m. She repaid half the amount on 01.01.2023. Compute her income from House
Property.
CA SHREY RATHI HOUSE PROPERTY 4.7

Sol 10: Computation of income u/h house property for Ms. Annie:
Particulars (₹)
Gross Annual Value [Expected Rent or Actual Rent whichever is higher] 9,90,000
Expected Rent = Fair Rent = ₹ 90,000 x 11 ₹ 9,90,000
(computed for 11 months as house got completed on 01.05.2022)
Actual Rent = ₹ 80,000 x 10 = ₹ 8,00,000
Less: Municipal taxes paid -
Net Annual Value 9,90,000
Less: Deductions u/s 24
(a) Statutory deduction @ 30% 2,97,000
(b) Interest on borrowed capital [WN 1] 1,83,750
Income u/h house property 5,09,250
WN 1: Calculation of interest on borrowed capital
Pre-construction period interest: Pre-construction period means the period commencing on the date of borrowing and
ending on:
(i) 31st March immediately prior to the date of completion of construction / date of acquisition or
(ii) Date of repayment of loan
whichever is earlier.
Pre-construction period: 01.07.2020 – 31.03.2022
₹ 15,00,000 x 10% x 21/12 = ₹ 2,62,500
Pre-construction period is permissible as deduction in 5 equal instalments = ₹ 2,62,500/5 = ₹ 52,500.
Post-construction period interest: 01.04.2022 – 31.03.2023
₹ 15,00,000 x 10% x 9/12 = ₹ 1,12,500
₹ 7,50,000 x 10% x 3/12 = ₹ 18,750
Total = ₹ 1,12,500 + ₹ 18,750 = ₹ 1,31,250
Total interest on borrowed capital = Pre-construction interest + Post-construction interest
= ₹ 52,500 + ₹ 1,31,250 = ₹ 1,83,750.

Q 11: Mr. Piyush and Mr. Yash constructed their house on a piece of land purchased by them at Mumbai. The built-up area
of each house was 1,000 sq. ft. ground floor and an equal area at the first floor.
Mr. Piyush started construction of the house on 01.04.2021 and completed it on 31.03.2022. He occupied the entire house
on 01.04.2022. He has availed a housing loan of ₹ 25,00,000 @ 12% p.a. on 01.04.2021. He has not repaid any amount.
Mr. Yash started construction on 01.04.2021 and completed it on 30.06.2022. He occupied the ground floor on 01.07.2022
and let out the first floor for a rent of ₹ 20,000 p.m. However, the tenant vacated the house on 31.12.2022 and Mr. Yash
occupied the entire house afterwards. Mr. Yash has availed a housing loan of ₹ 15,00,000 @ 10% p.a. on 01.07.2021. He
has also not repaid any amount.
Other particulars of the house are as under:
(₹)
(i) Fair rental value of each unit (Ground / First Floor) 1,20,000 p.a.
(ii) Municipal value of each unit (Ground / First Floor) 92,000 p.a.
(iii) Municipal taxes paid by: Mr. Piyush 10,000
Mr. Yash 10,000
(iv) Repair & maintenance charges paid by: Mr. Piyush 40,000
Mr. Yash 34,000
Compute income from house property for Mr. Piyush & Mr. Yash for the P/Y 2022-23.
CA SHREY RATHI HOUSE PROPERTY 4.8

Sol 11: Computation of income u/h house property for Mr. Piyush & Mr. Yash:
Particulars Piyush (₹) Yash (₹)
(GF & FF) (SO) GF (SO) FF (LO)
Gross Annual Value [Expected Rent or Actual Rent whichever is - - 1,20,000
higher]
Expected Rent = Municipal Value or Fair Rent whichever is higher
but limited to Standard Rent
Municipal Value = ₹ 1,20,000 x 9/12 = ₹ 90,000
Fair Rental Value = ₹ 92,000 x 9/12 = ₹ 69,000 ₹ 90,000
Actual Rent = ₹ 20,000 x 6 = ₹ 1,20,000
Less: Municipal taxes paid - - 5,000
Net Annual Value - - 1,15,000
Less: Deductions u/s 24
(a) Statutory deduction @ 30% - - 34,500
(b) Interest on borrowed capital [WN 1] 2,00,000 86,250 86,250
Income u/h house property (2,00,000) (86,250) (5,750)
Total (2,00,000) (92,000)
WN 1: Calculation of interest on borrowed capital for Mr. Piyush & Mr. Yash
Mr. Piyush: There will be no pre-construction period interest as the loan was taken after the completion of the property.
Post-construction period interest: 01.04.2022 – 31.03.2023
₹ 25,00,000 x 12% x 12/12 = ₹ 3,00,000. (but maximum allowed upto ₹ 2,00,000)
Mr. Yash: Pre-construction period interest: Pre-construction period means the period commencing on the date of
borrowing and ending on:
(i) 31st March immediately prior to the date of completion of construction / date of acquisition or
(ii) Date of repayment of loan
whichever is earlier.
Pre-construction period: 01.07.2021 – 31.03.2022
₹ 15,00,000 x 10% x 9/12 = ₹ 1,12,500
Pre-construction period is permissible as deduction in 5 equal instalments = ₹ 1,12,500/5 = ₹ 22,500.
Post-construction period interest: 01.04.2022 – 31.03.2023
₹ 15,00,000 x 10% x 12/12 = ₹ 1,50,000
Total interest on borrowed capital = Pre-construction interest + Post-construction interest
= ₹ 22,500 + ₹ 1,50,000 = ₹ 1,72,500.

Q 12: Mr. Raunak is a Service-Tax Officer in Kanpur. He owns two residential houses. The first is in Kanpur and was
constructed on 30.11.2006. This has been let out on a rent of ₹ 6,000 p.m. to a company for its office. The second house is
in Lucknow which was constructed on 01.02.2022 and has been self-occupied by him for his own residence since then. He
took a loan of ₹ 60,000 on 01.08.2020 @ 12% p.a. interest for the purpose of construction of this house. The entire loan is
still outstanding.
Other relevant particulars in respect of these houses are given below:
Kanpur (₹) Lucknow (₹)
1. Municipal Valuation 58,000 36,000
2. Municipal taxes 4,800 2,800
3. Land Revenue 1,000 650
4. Interest on Loan - 7,200
5. The municipal tax and land revenue of the Lucknow house are unpaid.
Raunak was transferred to Calcutta on 01.12.2022 where he resides in a house at a monthly rent of ₹ 8,000 and his house
in Lucknow was let out on the same day on a monthly rent of ₹ 5,000 per month. Fair Rental Value of Lucknow house is
equal to its municipal valuation.
Compute “Income from House Property” of Raunak for the A/Y 2023-24.
CA SHREY RATHI HOUSE PROPERTY 4.9

Sol 12: Computation of income u/h house property for Mr. Raunak:
Particulars Kanpur (₹) Lucknow (₹)
Gross Annual Value [Expected Rent or Actual Rent whichever is higher] 72,000 36,000
Kanpur: Expected Rent: ₹ 58,000; Actual Rent: ₹ 72,000
Lucknow: Expected Rent: ₹ 36,000; Actual Rent: ₹ 20,000
Less: Municipal taxes paid (payable shall not be allowed) 4,800 -
Net Annual Value 67,200 36,000
Less: Deductions u/s 24
(a) Statutory deduction @ 30% 20,160 10,800
(b) Interest on borrowed capital [WN 1] - 8,160
Income from house property 47,040 17,040
Total 64,080
WN 1: Calculation of interest on borrowed capital
Pre-construction period interest: Pre-construction period means the period commencing on the date of borrowing and
ending on:
(i) 31st March immediately prior to the date of completion of construction / date of acquisition or
(ii) Date of repayment of loan
whichever is earlier.
Pre-construction period: 01.08.2020 – 31.03.2021
₹ 60,000 x 12% x 8/12 = ₹ 4,800
Pre-construction period is permissible as deduction in 5 equal instalments = ₹ 4,800/5 = ₹ 960.
Post-construction period interest: 01.04.2022 – 31.03.2023
₹ 60,000 x 12% x 12/12 = ₹ 7,200
Total interest on borrowed capital = Pre-construction interest + Post-construction interest
= ₹ 960 + ₹ 7,200 = ₹ 8,160

Q 13: Mr. Bhavya owns a house whose construction was completed on 30.06.2009. The house has been let out since
01.07.2009 and the following particulars have been supplied by Bhavya for the year ending 31.03.2023.
(₹)
a. Annual Rent 96,000
b. Local taxes paid 10,000
c. Insurance premium paid 1,000
d. Repairs expenses 3,000
Bhavya mortgaged his house property for ₹ 60,000 on 01.10.2020 and the money was spent on his son’s marriage. Total
interest paid on the mortgage loan during the P/Y amounted to ₹ 8,000. During the P/Y 2019-20 he had claimed a
deduction of unrealised rent of ₹ 25,000 out of which ₹ 20,000 was allowed as deduction. During the P/Y 2022-23, he
recovered the entire amount i.e. ₹ 25,000 from the defaulting tenant.
Compute his income from house property for the A/Y 2023-24.

Sol 13: Computation of income from house property for Mr. Bhavya:
Particulars (₹)
Gross Annual Value 96,000
Less: Municipal taxes paid 10,000
Net Annual Value 86,000
Less: Deductions u/s 24
(a) Statutory deduction @ 30% 25,800
(b) Interest on borrowed capital [WN 1] -
60,200
Add: Recovery of unrealised rent u/s 25A (₹ 20,000 – 30%) [WN 2] 14,000
Income u/h house property 74,200
CA SHREY RATHI HOUSE PROPERTY 4.10

WN 1: Interest on borrowed capital for purchase/construction or repair of a house property shall only be allowed as
deduction and not for son’s marriage.
WN 2: Recovery of unrealised rent shall not be taxable to the extent it was not allowed deduction in the previous years.

Q 14: Mr. Jha, a resident individual, owns a house in Australia. He receives rent of Aus $ 2,000 p.m. He paid municipal
taxes of Aus $ 1,500 during the P/Y 2022-23. He also has two storied house in Delhi, ground floor is used for residential
and first floor is let out at a monthly rent of ₹ 10,000.
Standard rent for each floor is ₹ 11,000 p.m. Municipal taxes paid for the house amounts to ₹ 7,500. Mr. Jha had
constructed the house by taking a loan from nationalised bank on 22.06.2020. He had repaid the loan of ₹ 54,000
including interest of ₹ 24,000. The value of one Australian dollar is to be taken as ₹ 45. Assume FRV = Actual Rent
Compute total income of Mr. Jha u/h House Property for the A/Y 2023-24.

Sol 14: Computation of income from house property for Mr. Jha:
Particulars Australia (₹) Delhi (₹)
GF (SO) FF (LO)
Gross Annual Value [Expected Rent or Actual Rent 10,80,000 (24,000 x 45) - 1,20,000
whichever is higher]
Delhi (FF): FR: ₹ 1,20,000; SR: ₹ 1,32,000; AR: ₹ 1,20,000
Less: Municipal taxes paid 67,500 (1,500 x 45) - 3,750
Net Annual Value 10,12,500 - 1,16,250
Less: Deductions u/s 24
(a) Statutory deduction @ 30% 3,03,750 - 34,875
(b) Interest on borrowed capital - 12,000 12,000
Income u/h house property 7,08,750 (12,000) 69,375
57,375
Total Income 7,66,125

Q 15: Mr. Rajesh has let out his house property for ₹ 70,000 p.m. and there is an unrealised rent of 2 months and there is
a vacancy of 3 months. Fair rent is ₹ 60,000 p.m., Municipal valuation is ₹ 55,000 p.m. and Standard rent is ₹ 80,000 p.m.
Municipal taxes paid is ₹ 62,000. Interest on loan for the construction of the house property is ₹ 75,000. The assessee has
unrealised rent of ₹ 2,00,000 in the P/Y 2017-18 and he has recovered ₹ 1,50,000 in the P/Y 2022-23 and interest of ₹
28,000. He incurred ₹ 21,000 as legal expense for realisation of such amount.
Compute his income from House Property.

Sol 15: Computation of income from house property of Mr. Rajesh:


Particulars (₹)
Gross Annual Value [Expected Rent or Actual Rent whichever is higher] 7,20,000
Expected Rent = Municipal Value or Fair Rental Value whichever is higher but shall be limited to
Standard Rent = ₹ 7,20,000
Actual Rent = ₹ 70,000 x 7 [WN 1] ₹ 4,90,000
Less: Municipal taxes paid 62,000
Net Annual Value 6,58,000
Less: Deductions u/s 24
(a) Statutory deduction @ 30% 1,97,400
(b) Interest on borrowed capital 75,000
3,85,600
Add: Recovery of unrealised rent u/s 25A (₹ 1,50,000 – 30%) 1,05,000
Income from House Property 4,90,600
WN 1: There was an unrealised rent for 2 months due to which the actual rent came down to ₹ 7,00,000 (i.e. ₹ 70,000 x
10). Now because of vacancy of 3 months, the actual rent was further reduced to ₹ 4,90,000. But it can be seen that actual
rent after deduction of unrealised rent but before vacancy is less than the expected rent, therefore it can be concluded
that actual rent did not come down than expected rent because of vacancy.
CA SHREY RATHI HOUSE PROPERTY 4.11

Q 16: Mr. Yogesh owns a house whose construction got completed on 30.09.2016. It has been let out from 01.12.2017 for
residential purposes. Its particulars for the P/Y 2022-23 are:
(a) Municipal valuation (annual) ₹ 66,000
(b) Fair Rental Value (p.m.) ₹ 6,000
(c) Standard rent (p.m.) ₹ 6,500
(d) Actual rent (p.m.) ₹ 7,000
(e) Municipal taxes paid (including ₹ 15,000 paid by tenant) ₹ 30,000
(f) Water tax, levied by State Government ₹ 6,000
(g) Interest of ₹ 35,000 on loan taken for the construction of the house. The interest has been paid outside India to a
non-resident without deduction of tax at source though the non-resident had agreed to pay income tax on the
whole amount.
(h) The unrealised rent for the P/Y 2018-19 amounted to ₹ 40,000 but a deduction was allowed only for ₹ 30,000.
There is recovery of ₹ 20,000 from the defaulting tenant.
Compute income from House Property for the A/Y 2023-24.

Sol 16: Computation of income from house property for Mr. Yogesh:
Particulars (₹)
Gross Annual Value [Expected Rent or Actual Rent whichever is higher] 84,000
Expected Rent = Municipal Value or Fair Rental Value whichever is higher but shall be limited to
Standard Rent = ₹ 72,000
Actual Rent = ₹ 7,000 x 12 = ₹ 84,000
Less: Municipal taxes paid (municipal taxes paid by tenant is not allowed) 15,000
Net Annual Value 69,000
Less: Deductions u/s 24
(a) Statutory deduction @ 30% 20,700
(b) Interest on borrowed capital [WN 1] -
48,300
Add: Recovery of unrealised rent u/s 25A (₹ 10,000 – 30%) 7,000
Income from House Property 55,300
WN 1: Any interest paid outside India shall not be allowed deduction if tax has not been deducted at the time of making
payment.
WN 2: Recovery of unrealised rent shall not be taxable to the extent it was not allowed deduction in the previous year.

Q 17: Mr. Vikas owns a house property whose Municipal Value, Fair Rent and Standard Rent are ₹ 96,000, ₹ 1,26,000 and
₹ 1,08,000 (per annum) respectively. During the Financial year 2022-23, one-third of the portion of the house was let out
for residential purpose at a monthly rent of ₹ 5,000. the remaining two-third portion was self- occupied by him. Municipal
tax @ 11% of municipal value was paid during the year.
The construction of the house began in June, 2011 and was completed on 31-5-2018. Vikas took a loan of ₹ 1,00,000 on 1-
7-2015 for the construction of building. He paid interest on loan @ 12% per annum and every month such interest was
paid.
Compute income from house property of Mr. Vikas for the Assessment year 2023-24. [ICAI Study Module]
CA SHREY RATHI HOUSE PROPERTY 4.12

Sol 17: Computation of income from house property of Mr. Vikas for the A/Y 2023-24:
Particulars (₹) (₹)
Income from house property
1. Self- occupied portion (Two third)
Net Annual Value Nil
Less: Deduction under section 24(b)
Interest on loan (See Note below) (₹ 18,600 x 2/3) 12,400 (12,400)
Loss from self-occupied property
2. Let- out portion (One third)
Gross Annual Value [Higher of (a) or (b)] 60,000
(a) Actual rent received (₹ 5,000 x 12) ₹ 60,000
(b) Expected rent ₹ 36,000
[Expected rent is higher of municipal valuation (i.e. ₹ 96,000) and fair rent (i.e. ₹
1,26,000) but restricted to Standard rent (i.e. ₹ 1,08,000)] = ₹ 1,08,000 x 1/3 = ₹ 36,000
Less: Municipal taxes (₹ 96,000 x 11% x 1/3) 3,520
Net Annual Value 56,480
Less: Deduction u/s 24
(a) 30% of Net Annual Value 16,944
(b) Interest on loan (WN 1) (₹ 18,600 x 1/3) 6,200
Income from Let-out portion 33,336
Income from house property 20,936
WN 1: Calculation of interest on borrowed capital:
Pre-construction period interest: Pre-construction period means the period commencing on the date of borrowing and
ending on:
(i) 31st March immediately prior to the date of completion of construction / date of acquisition or
(ii) Date of repayment of loan
whichever is earlier.
Pre-construction period: 01.07.2015 – 31.03.2018
₹ 1,00,000 x 12% x 33/12 = ₹ 33,000
Pre-construction period is permissible as deduction in 5 equal instalments = ₹ 33,000/5 = ₹ 6,600. This interest shall be
allowed from F.Y. 2018-19 till F.Y. 2022-23.
Post-construction period interest: 01.04.2022 – 31.03.2023
₹ 1,00,000 x 12% x 12/12 = ₹ 12,000
Total interest on borrowed capital = Pre-construction interest + Post-construction interest
= ₹ 6,600 + ₹ 12,000 = ₹ 18,600

Q 18: Mr. Saurav owns a building consisting of 3 identical units, the construction of which got completed on 1st April 2022.
The building was occupied from 1st April onwards. The particulars pertaining to the 3 units for the year ended 31 st March
2023 are as under:
Unit I (₹) Unit II (₹) Unit III (₹)
1. Fair rent 60,000 60,000 60,000
2. Rent received - 72,000 -
3. Municipal taxes:
• Paid 3,000 5,000 3,000
• Due but not paid 3,000 5,000 3,000
4. Land revenue due but unpaid 1,400 1,200 1,200
5. Ground rent paid 2,400 2,400 2,400
6. Nature of occupation Self-occupied Let out Used for own business
On 1st April 2021, Saurav borrowed a sum of ₹ 5,00,000 @ 12% p.a. for construction of this building.
Compute the income u/h house property of Saurav for the A/Y 2023-24.
CA SHREY RATHI HOUSE PROPERTY 4.13

Sol 18: Computation of income u/h house property of Mr. Saurav:


Particulars Unit I (₹) (SO) Unit II (₹) (LO)
Gross Annual Value [Expected Rent or Actual Rent whichever is higher] - 72,000
Less: Municipal taxes paid - 5,000
Net Annual Value - 67,000
Less: Deductions u/s 24
(a) Statutory deduction @ 30% - 20,100
(b) Interest on borrowed capital [WN 1] 24,000 24,000
Income from House Property (24,000) 22,900
Total (1,100)
WN 1: Calculation of interest on borrowed capital:
Pre-construction period interest: Pre-construction period means the period commencing on the date of borrowing and
ending on:
(i) 31st March immediately prior to the date of completion of construction / date of acquisition or
(ii) Date of repayment of loan
whichever is earlier.
Pre-construction period: 01.04.2021 – 31.03.2022
₹ 5,00,000 x 12% x 12/12 = ₹ 60,000
Pre-construction period is permissible as deduction in 5 equal instalments = ₹ 60,000/5 = ₹ 12,000.
Post-construction period interest: 01.04.2022 – 31.03.2023
₹ 5,00,000 x 12% x 12/12 = ₹ 60,000
Total interest on borrowed capital = Pre-construction interest + Post-construction interest
= ₹ 12,000 + ₹ 60,000 = ₹ 72,000
Total interest shall be divided into 3 units = ₹ 72,000/3 = ₹ 24,000
Note: Unit III used for own business shall be outside the ambit of house property chapter and shall be treated under head
‘Profits and Gains from Business & Profession’.

Q 19: Ms. Roshni Ravi, a citizen of UK, is a resident and ordinary resident in India during the financial year 2022-23. She
owns a house property in London, UK which is used as her residence. The annual value of the house is GBP 22,000. The
value of one GBP may be taken as ₹ 74.
She took ownership and possession of a flat in Chennai on 01.07.2022, which is used for self-occupation, while she is in
India. The flat was used by her during the year ended 31.03.2023. Municipal valuation is ₹ 32,000 p.m. & the fair rent is ₹
4,20,000 p.a. She paid the following to the corporation of Chennai:
Property Tax ₹ 16,200
Sewerage Tax ₹ 1,800
She had taken a loan from Standard Chartered Bank for purchasing this flat. Interest on loan was as under:
(₹)
Period prior to 01.04.2022 49,200
01.04.2022 to 30.06.2022 50,800
01.07.2022 to 31.03.2023 1,31,300
She had a house property in Bangalore, which was sold in March, 2018. In respect of the house, she received arrears of
rent of 60,000 in January 2023. The amount has not been charged to tax earlier.
Compute the income chargeable from house property of Ms. Roshni Ravi for the A/Y 2023-24, exercising the most
beneficial option. [ICAI Study Module]
CA SHREY RATHI HOUSE PROPERTY 4.14

Sol 19: Computation of income u/h house property of Ms. Roshni Ravi for the A/Y 2023-24:
UK (₹) Chennai (₹) Bangalore (₹)
Net Annual Value - - -
As UK & Chennai houses are self-occupied, Net Annual Value shall
be nil. Further, Bangalore property shall be computed only to the
extent of arrears of rent received.
Less: Deductions u/s 24
(a) Statutory deduction @ 30% - - -
(b) Interest on borrowed capital [WN 1] - 1,91,940 -
- (1,91,940) -
Add: Recovery u/s 25A [₹ 60,000 – 30% = ₹ 42,000] - - 42,000
Income u/h house property - (1,91,940) 42,000
Total 1,49,940
WN 1: Calculation of interest on borrowed capital:
Pre-construction period interest (prior to 01.04.2022) = ₹ 49,200/5 = ₹ 9,840
Post-construction period interest (01.04.22 – 31.03.23) = ₹ 50,800 + ₹ 1,31,300 = ₹ 1,82,100
Total = ₹ 9,840 + ₹ 1,82,100 = ₹ 1,91,940.

Q 20: Mr. Sandeep owns a house property in Chandigarh. 40% of the house property is let out for ₹ 6,000 p.m. and the
remaining portion is self-occupied. The let-out portion was also self-occupied from 01.09.2022 to 30.11.2022. However,
w.e.f 01.12.2022 the entire house was let out for ₹ 14,000 p.m. The construction of the house was completed on
31.08.2002.
The following expenses were incurred for the above house property for the year ending 31.03.2023:
(₹)
 Municipal tax paid:
for F/Y 2020-21 6,000
for F/Y 2021-22 10,000
for F/Y 2022-23 16,000
 Insurance premium paid 4,000
 Land revenue due 5,000
 Interest on money borrowed for construction of house property 28,000
Calculate income u/h house property of Mr. Sandeep for the A/Y 2023-24.

Sol 20: Computation of income u/h house property of Mr. Sandeep:


Particulars (₹)
Gross Annual Value [Expected Rent or Actual Rent whichever is higher] (₹ 14,000 x 12) 1,68,000
[As the whole house is let out for ₹ 14,000 p.m., it shall be deemed to be the Fair Rental Value of the
house for the whole year]
Less: Municipal taxes paid 32,000
Net Annual Value 1,36,000
Less: Deductions u/s 24
(a) Statutory deduction @ 30% 40,800
(b) Interest on borrowed capital 28,000
Income from House Property 67,200
Note: Where the property is self-occupied for a part of the year and let-out for the remaining part of the year and the Fair
Rental Value is not given, then Actual Rent p.m. for the whole house shall be assumed to be the Fair Rental Value.
CA SHREY RATHI HOUSE PROPERTY 4.15

Q 21: You are required to compute the income from “House Property” for the A.Y. 2023-24 of Mrs. Rajni from her house
property at Panchkula in Haryana. The Municipal value of the property is ₹ 7,50,000, Fair Rent of the property is ₹6,30,000
and Standard Rent is ₹ 7,20,000 per annum.
The property was let out for ₹ 80,000 per month for the period April 2022 to November 2022.
Thereafter, the tenant vacated the property and Mrs. Rajni used the house for self-occupation. Rent for the months of
October and November 2022 could not be realized from the tenant. The tenancy was bonafide but the defaulting tenant
was in occupation of another property of the assessee, paying rent regularly.
She paid municipal taxes @ 12% during the year and paid interest of ₹ 50,000 during the year for amount borrowed
towards repairs of the house property. [MTP Nov. 22]

Sol 21: Computation of income from house property of Mrs. Rajni for the A.Y. 2022-23
Particulars (₹) (₹)
Computation of Gross Annual Value
Expected Rent for the whole year = Higher of Municipal Value of ₹ 7,50,000 and Fair
/rent of ₹ 6,30,000, but restricted to Standard Rent of ₹ 7,20,000 7,20,000
Actual rent received for the let-out period = ₹ 80,000 x 8 6,40,000
[Unrealised rent is not deductible from actual rent in this case since the defaulting
tenant is in occupation of another property of the assessee, and hence, one of the
conditions laid out in Rule 4 has not been fulfilled]
GAV Annual Value (GAV) 7,20,000
Less: Municipal taxes (paid by the owner during the previous year) = 12% of 7,50,000 90,000
Net Annual Value (NAV) 6,30,000
Less: Deductions under section 24
(a) 30% of NAV = 30% of ₹ 6,30,000 1,89,000
(b) Interest on amount borrowed for repairs (Fully allowable as deduction, since it
pertains to let – out property) 50,000 2,39,000
Income from house property 3,91,000

Q 22: Two brothers Arun Bimal are co-owners of a house property with equal share. The property was constructed during
the financial year 1998-1999. The property consists of eight identical units and is situated at Cochin.
During the financial year 2022-23, each co-owner occupied one unit for residence and the balance of six units were let out
at a rent of ₹ 12,000 per month per unit. The municipal value of the house property is ₹ 9,00,000 and the municipal taxes
are 20% of municipal value, which were paid during the year. The other expenses were as follows:
(₹)
(i) Repairs 40,000
(ii) Insurance premium (paid) 15,000
(iii) Interest payment on loan taken for construction of house 3,00,000
One of the let-out units remained vacant for four months during the year.
Arun could not occupy his unit for six months as he was transferred to Chennai. He does not own any other house.
The other income of Mr. Arun and Mr. Bimal are ₹ 2,90,000 and ₹ 1,80,000, respectively, for the financial year 2022-23.
Compute the income under the head ‘Income from House property’ and the total income of two brothers for the A.Y.
2023-24. [ICAI Study Module]
CA SHREY RATHI HOUSE PROPERTY 4.16

Sol 22: Computation of total income for the A.Y. 2023-24:


Particulars Arun (₹) Bimal (₹)
I. Income from House Property
Self-occupied portion (25%)
Annual value Nil Nil
Less: Deduction under Section 24(b) 30,000 30,000
Interest on loan taken for construction ₹ 37,500 (being 25% of ₹ 1.5 lakh) restricted
to maximum of ₹ 30,000 for each co-owner since the property was constructed
before 01-04-1999. Hence, it is assumed that loan was taken before 1-4-1999.
Loss from self -occupied property (30,000) (30,000)
II. Let-out portion (75%) – See Working Note below 1,25,850 1,25,850
Income from House property 95,850 95,850
Other Income 2,90,000 1,80,000
Total Income 3,85,850 2,75,850

Working Note – Computation of Income from Let-Out Portion of House property


Particulars (₹) (₹)
Let-out portion (75%)
Gross Annual Value: Higher of 8,16,000
(a) Municipal value (75% of ₹ 9 lakhs) 6,75,000
(b) Actual rent [(₹ 12,000 x 6 x 12) – (₹ 12,000 x 1 x 4)] = ₹ 8,64,000 - ₹ 48,000 8,16,000
Less: Municipal taxes 75% of ₹ 1,80,000 (20% of ₹ 9 lakhs) 1,35,000
Net Annual Value (NAV) 6,81,000
Less: Deduction under Section 24
(a) 30% of Net Annual Value 2,04,300
(b) Interest on loan taken for the house [75% of ₹ 3 lakhs] 2,25,000 4,29,300
Income from let-out portion of house property 2,51,700
Share of each co-owner (50%) 1,25,850

MULTIPLE CHOICE QUESTIONS


1. Which of the following is incorrect?
(a) Let out of commercial property is taxable u/h House Property
(b) Sub-letting of a residential house property is taxable u/h House Property
(c) Let out of only vacant land is taxable u/h Income from Other Sources or Business or Profession as the case may be
(d) All of the above

2. Expected rent is
(a) Higher of Municipal Value & Standard Rent but limited to Fair Rental Value
(b) Higher of Municipal Value or Fair Rental Value or Standard Rent
(c) Lower of Fair Rental Value or Standard Rent
(d) Higher of Municipal Value or Fair Rental Value but limited to Standard Rent

3. Mr. Jat owns a house property which has fair rent of ₹ 1,50,000, standard rent ₹ 1,20,000 and actual rent of 1,30,000.
Municipal taxes paid during the PY 2022-23 for the past 7 years is ₹ 1,40,000. The annual value shall be:
(a) ₹ 20,000 (c) Nil
(b) (-) ₹ 10,000 (d) None of the above

4. A house property whose fair rent is ₹ 1,20,000 is vacant throughout the previous year. Municipal taxes paid for the
house property are ₹ 20,000. Its net annual value will be:
(a) ₹ 1,20,000 (c) Nil
(b) ₹ 1,00,000 (d) (-) ₹ 20,000
CA SHREY RATHI HOUSE PROPERTY 4.17

5. Municipal taxes to be deducted from GAV should be


(a) Paid by the tenant during the previous year
(b) Paid by the owner during the previous year
(c) Accrued during the previous year
(d) Paid during the previous year either by tenant or owner

6. Mr. Kalra purchased a property on 31st July 2022 and sold it on 1st February 2023. Expected rent in this case shall be
computed for
(a) 12 months (c) 6 months
(b) 7 months (d) 5 months

7. Any person who has taken loan before 1/4/1999 for purchase or construction of the house which is self-occupied,
maximum deduction for the interest shall be:
(a) ₹ 1,20,000 (c) ₹ 30,000
(b) ₹ 2,00,000 (d) ₹ 1,50,000

8. A house property is let-out throughout the year for a monthly rent of ₹ 22,000 and municipal tax paid for current year
is 24,000 and for the earlier previous year paid now is 16,000, the income from house property would be:
(a) ₹ 1,68,000 (c) ₹ 1,56,800
(b) ₹ 1,84,800 (d) ₹ 2,24,000

9. The ceiling limit of deduction under section 24(b) in respect of interest on loan taken on 1.4.2022 for repairs of self–
occupied house is:
(a) ₹ 30,000 p.a. (c) ₹ 2,00,000 p.a.
(b) ₹ 1,50,000 p.a. (d) No limit

10. Where an assessee has two house properties for self-occupation, the benefit of nil annual value will be available in
respect of:
(a) Both the properties
(b) The property which has been acquired/constructed first
(c) Any one of the properties, at the option of the assessee
(d) Any one of the properties and once option is exercised cannot be changed in subsequent years

11. Ganesh and Rajesh are co-owners of a self–occupied property. They own 50% share each. The interest paid by each
co-owner during the previous year on loan (taken for acquisition of property during the year 2004) is ₹ 2,05,000. The
amount of allowable deduction in respect of each co-owner is: (ICAI MCQ Booklet)
(a) ₹ 2,05,000 (c) ₹ 2,00,000
(b) ₹ 1,02,500 (d) ₹ 1,00,000

12. Leena received ₹ 30,000 as arrears of rent during the P.Y. 2021-22. The amount taxable under section 25A would be:
(a) ₹ 30,000 (c) ₹ 20,000
(b) ₹ 21,000 (d) ₹ 15,000

13. Mr. Rajesh owns a residential house, let out for monthly rent of ₹ 15,000. The fair rental value of the property for the
let-out period is ₹ 1,50,000. The house was self-occupied by him from 1st January, 2023 to 31st March, 2023. He has
taken a loan from bank of ₹ 20 lacs for the construction of the property, and has repaid ₹ 1,05,000 (including interest
₹ 40,000) during the year. Compute Rajesh’s income from house property for the Assessment year 2023-24.
(a) ₹ 1,00,000 (c) ₹ 65,000
(b) ₹ 86,000 (d) None of the above

14. Vidya received ₹ 90,000 in May, 2022 towards recovery of unrealized rent, which was deducted from actual rent
during the P.Y. 2018-19 for determining annual value. Legal expense incurred in relation to unrealized rent is ₹ 20,000.
The amount taxable under section 25A for A.Y. 2023-24 would be: (ICAI MCQ Booklet)
(a) ₹ 70,000 (c) ₹ 60,000
(b) ₹ 63,000 (d) ₹ 49,000
CA SHREY RATHI HOUSE PROPERTY 4.18

15. Interest on borrowed capital accrued up to the end of the previous year prior to the year of completion of
construction is:
(a) Allowed as a deduction in the year of completion of construction
(b) Allowed in 5 equal annual instalments from the year of completion of construction
(c) Allowed in the respective year in which the interest accrues
(d) Not allowed

16. Mr. Krishna owns a residential house in Delhi. The house is having two identical units. First unit of the house is self-
occupied by Mr. Krishna and another unit is rented for ₹ 12,000 p.m. The rented unit was vacant for three months
during the year. The particulars of the house for the previous year 2022-23 are as under:
Standard rent ₹ 2,20,000 p.a.
Municipal valuation ₹ 2,44,000 p.a.
Fair rent ₹ 2,35,000 p.a.
Municipal tax paid by Mr. Krishna 12% of the municipal valuation
Light and water charges ₹ 800 p.m.
Interest on borrowed capital ₹ 2,000 p.m.
Insurance charges ₹ 3,500 p.a.
Painting expenses ₹ 16,000 p.a.
Compute income from house property of Mr. Krishna for the A.Y. 2023-24.
(a) ₹ 41,352 (c) ₹ 17,352
(b) ₹ 42,752 (d) ₹ 18,528

17. The concept of Deemed Ownership is given:


(a) under section 24 (c) under section 25
(b) under section 29 (d) under section 27

18. In case the property is owned by co-owners and it is let out, income from such property shall be computed:
(a) separately for each co-owner
(b) it will be first computed ignoring the co-ownership and then distributed amongst co-owners.
(c) shall not be calculated at all
(d) taxable in the hands of either of the co-owners

19. Mr. Rathi is owner of a big house which is let out at the rent of ₹ 20,000 p.m. Municipal Value of the house ₹ 15,000
p.m., Fair rent ₹ 21,000 p.m., Standard rent ₹ 18,000 p.m. and Municipal taxes paid are ₹ 5,000 p.a. The HP is vacant
for 2 months. Calculate GAV, NAV and HP income.
(a) ₹ 2,00,000 / ₹ 1,50,000 / ₹ 1,05,000 (c) ₹ 2,40,000 / ₹ 1,95,000 / ₹ 2,35,000
(b) ₹ 1,55,000 / ₹ 1,50,000 / ₹ 2,00,000 (d) None of the above

20. Nisha has two houses, both of which are self- occupied. The particulars of these are given below:
House I (₹) House II (₹)
Municipal valuation per annum 1,20,000 1,15,000
Fair rent per annum 1,50,000 1,75,000
Standard rent per annum 1,00,000 1,65,000
Date of completion 31-03-1999 31-03-2001
Municipal taxes payable during the year (paid for House II only) 12% 8%
Interest on money borrowed for repair of property during current year - 55,000
Compute Nisha’s income from house property for the Assessment year 2022-23.
(a) ₹ 80,000 (c) (-) ₹ 30,000
(b) (-) ₹ 55,000 (d) Nil

21. Mr. Raj took loan from a bank for ₹ 10,00,000 on 1/11/2018 @ 8% p.a. for the construction of the house which is self-
occupied. Construction of the house got completed on 15/3/2022. Compute interest allowed as deduction u/s 24(b)
for PY 2022-2023.
(a) ₹ 1,18,667 (c) ₹ 1,50,000
(b) ₹ 30,000 (d) ₹ 80,000
CA SHREY RATHI HOUSE PROPERTY 4.19

22. Mr. J has taken a house property on lease for 15 years from Mr. S and let out the same to Mr. D. Income from such
house earned by Mr. J shall be taxable as
(a) income under the head other sources.
(b) income from house property as Mr. J is the deemed owner of the house property.
(c) income from business and profession
(d) none of the above

23. If an assessee has borrowed money for purchase of a house & interest is payable outside India. Such interest shall:
(a) be allowed as deduction
(b) not be allowed as deduction
(c) be allowed as deduction if the tax is deducted at source
(d) none of the above

24. Which out of the following is not a case of deemed ownership of house property?
(a) Transfer of house property to a spouse for inadequate consideration
(b) Transfer of house property to a minor child for inadequate consideration
(c) Individual who is holder of an impartible estate
(d) Co-owner of a house property

25. Mr. S is owner of one self-occupied house property in Mumbai for his residence. Fair rent of that property is ₹
5,06,000 p.a. Municipal valuation is ₹ 5,88,000. Municipal taxes paid are ₹ 50,000 including ₹ 1,000 for an earlier
year. The house was constructed in December, 2016 with a loan of ₹ 12,00,000 from SBI taken in November, 2013.
Interest chargeable on such loan is 13% p.a. During the PY 2022-2023 Mr. S paid back ₹ 2,30,000 which includes ₹
1,48,000 as current year interest. Compute the income from house property income for the AY 2023-2024.
(a) Loss of ₹ 30,000 (c) Loss of ₹ 1,48,000
(b) Loss of ₹ 2,00,000 (d) Loss of ₹ 1,56,000

26. Composite rent of let-out house property is taxable as


(a) Profits and gains from business or profession
(b) Income from other sources
(c) Income from house property
(d) Either (a) or (b) above depending upon certain conditions

27. Reliance Ltd. constructed staff quarters and let out the same during the AY 2023-2024. Its rent received ₹ 7,50,000 by
way of rent from employees during the year. The rental receipt is taxable as:
(a) Income from house property (c) Income from business
(b) Perquisite in the hands of employees (d) Income from other sources

28. Lohan Ltd. is owner of the House Property which is let out on rent @ ₹ 60,000 pm. Lohan Ltd. has paid municipal taxes
of ₹ 80,000 p.a. It took a loan from bank in Australia for purchasing this house. It has paid interest to the bank of ₹
1,20,000 p.a. Calculate House Property income if Lohan Ltd. has not deducted TDS on such interest and neither has
Australian bank paid tax on this.
(a) ₹ 4,98,000 (c) ₹ 7,20,000
(b) ₹ 4,48,000 (d) ₹ 6,40,000

29. Mr. K owns two house properties. First property was used half for running his business and the other half was let-out
at ₹ 40,000 per month. The second property was wholly used as a residence by Mr. K. Municipal value of the two
properties was the same at ₹ 7,20,000 each per annum and municipal taxes paid are 10%. Mr. K’s income from house
property for the P/Y 2022-2023 will be
(a) ₹ 3,13,600 (c) ₹ 3,10,800
(b) ₹ 2,28,560 (d) ₹ 6,32,160
CA SHREY RATHI HOUSE PROPERTY 4.20

30. Mr. L took loan from a bank for ₹ 1,00,000 on 1/1/2019 @ 12% p.a. for the construction of the house which is self-
occupied. Construction of the house got completed on 1/1/2023 and full amount of loan was paid back on 31/1/2022.
Compute interest allowed as deduction under section 24(b) for AY 2023-2024.
(a) ₹ 19,400 (c) ₹ 7,400
(b) ₹ 12,000 (d) ₹ 49,000

31. Which is the charging section of Income under head House property?
(a) 28 (c) 22
(b) 23 (d) 56

32. When a house property is let-out for a monthly rent of ₹ 25,000 during the P/Y 2022-23 and maintenance expenses
by way of salary to sweeper and watchman is ₹ 6,000 p.m. each, the income from house property would be:
(a) ₹ 2,28,000 (c) ₹ 2,10,000
(b) ₹ 3,00,000 (d) ₹ 2,50,000

33. Where the property is self-occupied for a part of the year and let out for remaining part of the year, expected rent
shall be computed for
(a) Let out period (c) 12 months
(b) 6 months (d) None of the above

34. In which of the following case GAV of the house is taken as nil.
(a) When the house is self-occupied by the assessee
(b) When assessee could not occupy the house because of employment and had to reside in other building not
belonging to him
(c) Both a) and b)
(d) None of the above

35. Mr. Zen owns a flat in Mumbai which was let out by him in the previous year 2022-23 on a rent of ₹ 20,000 p.m. upto
December, 2022 and for ₹ 30,000 p.m. thereafter. The annual municipal value is of ₹ 3,00,000. Fair rent is ₹ 2,50,000
and Standard Rent is ₹ 2,90,000. The Gross Annual Value of the flat shall be taken as:
(a) ₹ 2,70,000 (c) ₹ 2,50,000
(b) ₹ 3,00,000 (d) ₹ 2,90,000

36. Santnam purchased in October, 2022, a flat in Chennai, to be used for his own residential purposes with the financial
assistance of housing loan taken from PNB Housing Finance Ltd. He has paid interest on such loan till March, 2022 of ₹
1,78,780. The amount of interest paid on such loan allowed u/s 24(b) shall be:
(a) ₹ 1,50,000 (c) ₹ 2,00,000
(b) ₹ 1,78,780 (d) ₹ 30,000

37. What would be the amount of deduction available u/s 24(b) for previous year 2022-23 in case of let out property as
per following detail:
Amount of loan (for construction of property): ₹ 12,00,000
Date of loan: 01.01.2016
Rate of interest: 12%
Date of completion of construction: 31.1.2018
(a) ₹ 1,80,000 (c) ₹ 2,01,600
(b) ₹ 2,00,000 (d) ₹ 1,44,000

38. Where the landlord provides facilities along with the house which are inseparable, then such rent shall be taxable u/h:
(a) House Property (c) Salary
(b) Capital gains (d) Income from Other Sources/PGBP
CA SHREY RATHI HOUSE PROPERTY 4.21

39. Mr. Kamal had two children Sumit and Sushmita (married with Aman) of age 14 & 16 respectively and wife name Anu.
In which of the following case he will not be considered as deemed owner?
(a) Transfer of property to Anu for inadequate consideration
(b) Transfer of property to Sushmita for inadequate consideration
(c) Transfer of property to Sumit for inadequate consideration
(d) None of the above

40. The amount realized shall be taxable only if assessee is owner of the building. Discuss the validity of the statement.
(a) Valid, the assessee shall be taxable in the owner of the building
(b) Invalid, assessee shall be chargeable to tax irrespective of fact whether assessee is owner of building or not
(c) Partly valid
(d) None of the above

41. ......................... is the rent fixed by Rent Control Act.


(a) Fair rent (c) Actual rent
(b) Realisable rent (d) Standard rent

42. Ms. Anamika, a resident individual, owns a house property in London for which loan of $ 80,000 has been borrowed at
an interest rate of 7%. She has let out the property and had paid the interest of $ 5,600 for the previous year 2022-23
on which TDS of $ 560 was deducted. The amount of interest on loan eligible for deduction when 1 $ = ₹ 70 is
(a) ₹ 3,25,800 (c) ₹ 3,29,000
(b) ₹ 3,52,800 (d) ₹ 3,92,000

43. If a single unit of property is self-occupied for few months and let out for the other months it shall be treated as
(a) Self-occupied property (c) Let out property
(b) Unoccupied property (d) Self-occupied property and Let out property

44. Mr. J has taken a house on rent and sublets the same to Mr. A. Income of Mr. J from such house property shall be
taxable under the head:
(a) income from house property (c) income from other sources
(b) income from salary (d) shall not be taxed at all

45. Where standard rent is not applicable GAV shall be higher of—
(a) fair rent and municipal value (c) municipal value and actual rent
(b) fair rent and municipal value and actual rent (d) none of the above

46. Ms. L let out a property for ₹ 20,000 per month during the A/Y 2023-2024. The municipal tax on the let-out property
was enhanced retrospectively. Hence, she paid ₹ 60,000 as municipal tax which included arrears of municipal tax of ₹
45,000. Her income from house property is
(a) ₹ 1,80,000 (c) ₹ 1,26,000
(b) ₹ 1,57,500 (d) ₹ 1,36,500

47. In the year 2019-20, Puneet did not receive ₹ 1,00,000 from the defaulting tenant. Out of this, only 60,000 was
allowed as deduction in 2019-20. Puneet recovered ₹ 70,000 in the year 2022-23. The amount taxable u/s 25A shall
be:
(a) ₹ 49,000 (c) ₹ 70,000
(b) ₹ 21,000 (d) ₹ 28,000

48. Which of the following conditions needs to be satisfied to charge income u/h house property?
(a) The property should consist of any building
(b) The assessee should be the owner of such property
(c) The property should not be used by the owner for the purpose of any business or profession, the profits of which
are chargeable to income tax.
(d) All of the above
CA SHREY RATHI HOUSE PROPERTY 4.22

49. In case of disputed ownership relating to property, who is generally considered to be the owner of the property?
(a) The person receiving the rent (c) The court
(b) The income tax authority (d) None of the above

50. Mr. Raghav has three houses for self-occupation. What would be the tax treatment for A.Y. 2023-24 in respect of
income from house property? (ICAI MCQ Booklet)
(a) One house, at the option of Mr. Raghav, would be treated as self-occupied. The other two houses would be
deemed to be let out.
(b) Two houses, at the option of Mr. Raghav, would be treated as self-occupied. The other house would be deemed
to be let out.
(c) One house, at the option of Assessing Officer, would be treated as self-occupied. The other house would be
deemed to be let out.
(d) Two houses, at the option of Assessing Officer, would be treated as self-occupied. The other house would be
deemed to be let out.

MCQ’s Answers
1. (b) 2. (d) 3. (c) 4. (c) 5. (b) 6. (c) 7. (c) 8. (c) 9. (a) 10. (a)
11. (c) 12. (b) 13. (a) 14. (b) 15. (b) 16. (a) 17. (d) 18. (b) 19. (d) 20. (c)
21. (a) 22. (b) 23. (c) 24. (d) 25. (d) 26. (d) 27. (a) 28. (b) 29. (c) 30. (c)
31. (c) 32. (c) 33. (c) 34. (c) 35. (d) 36. (b) 37. (d) 38. (d) 39. (b) 40. (b)
41. (d) 42. (d) 43. (c) 44. (c) 45. (b) 46. (c) 47. (b) 48. (d) 49. (a) 50. (b)
CA SHREY RATHI PGBP 5.1

RATAN TATA

CHAPTER 5
PROFIT AND GAINS FROM
BUSINESS & PROFESSION

“Take the stones people throw at you,


and use them to build a monument.”

PRACTICAL QUESTIONS
Q 1: Mr. Jagan furnishes the following information for the A/Y 2023-24:
Profit & Loss A/c for the year ending 31st March 2023
Particulars (₹) Particulars (₹)
To Salaries 60,000 By Gross Profit 6,80,000
” Bad debts 16,000 ” Commission 10,000
” Reserve for bad debts 24,000 ” Discount received 6,000
” Fire insurance premium 4,000 ” Sundry trade receipts 16,000
” Advertising 16,000 ” Rent from house property 48,000
” Interest on capital 8,000
” Interest on bank loan 8,000
” Depreciation 48,000
” Net Profit 5,76,000
Total 7,60,000 Total 7,60,000
(i) The amount of depreciation allowable is ₹ 40,000.
Compute the income of Jagan from business & his gross total income for the A/Y 2023-24.

Sol 1: Computation of income of Mr. Jagan from business & his GTI:
Particulars (₹) (₹)
Net Profit as per Profit & Loss A/c 5,76,000
Add: Expenses debited to P/L A/c but not allowed under Income Tax Act
o Reserve for bad debts 24,000
o Interest on capital 8,000
o Depreciation (₹ 48,000 – ₹ 40,000) 8,000 40,000
Less: Income credited to P/L A/c but not taxable u/h PGBP
o Rent from house property 48,000
Income u/h PGBP 5,68,000
CA SHREY RATHI PGBP 5.2

Computation of GTI
Income u/h PGBP (as computed above) ₹ 5,68,000
Income from House Property [WN 1] ₹ 33,600
Gross Total Income ₹ 6,01,600
WN 1: Computation of income u/h House Property:
Rent received from house property = GAV ₹ 48,000
Less: Municipal taxes paid Nil
Net Annual Value ₹ 48,000
Less: Deductions u/s 24
(a) Statutory Deduction @ 30% ₹ 14,400
(b) Interest on Borrowed Capital Nil
Income u/h House Property ₹ 33,600

Q 2: Mr. Vishal gives you the following information for the year ended 31.03.2023:
S. No. Particulars (₹)
1. Net Profit as per P&L A/c (without allowing the following items) 5,20,000
2. Capital expenditure on family planning 70,000
3. Lump-sum consideration for purchase of technical know-how 1,20,000
4. Entertainment expenditure 40,000
5. Expenditure on acquisition of patent right 80,000
6. Expenditure on advertisement paid in cash 25,000
7. Provision for custom duty (He paid ₹ 30,000 on 27.06.2022 and ₹ 12,000 on 03.11.2022).
Due date of filing of return is 31.07.2022. 60,000
8. Amount paid to Delhi University for an approved research program in the field of social
sciences not connected with his business. 60,000
Compute his business income for the A/Y 2023-24.

Sol 2: Computation of business income of Mr. Vishal for the A/Y 2023-24:
Particulars (₹)
Net Profit as per Profit & Loss A/c (before allowing the following expenditures) 5,20,000
Less: Admissible expenses
• Depreciation on purchase of technical know-how (₹ 1,20,000 x 25%) 30,000
• Entertainment expenditure 40,000
• Depreciation on acquisition of patent rights (₹ 80,000 x 25%) 20,000
• Provision for custom duty (allowed to the extent it is paid on or before the due date of filing of
return) 30,000
• Amount paid to Delhi University for research in social sciences (100%) 60,000
Business income of Mr. Vishal 3,40,000
Notes:
1. Capital expenditure on family planning is allowed to a company only and not to any other person.
2. Technical know-how & patents are in the nature of capital assets & therefore shall not be allowed to be deducted,
however depreciation shall be allowed.
3. Expenditure on advertisement shall not be allowed as the payment in made in cash in excess of ₹ 10,000 u/s 40A(3).

Q 3: Examine, with reasons, the allowability of the following expenses under the Income Tax Act, 1961 while computing
income from business or profession for the Assessment year 2023-24:
(i) Payment of ₹ 50,000 by using credit card for fire insurance.
(ii) Purchase of oil seeds of ₹ 50,000 in cash from a farmer on a banking day.
(iii) Tax on non–monetary perquisite provided to an employee ₹ 20,000.
(iv) Salary payment of ₹ 2,00,000 outside India by a company without deduction for tax.
(v) Payment made in cash ₹ 30,000 to a transporter in a day for carriage of goods. [ICAI Study Module]
CA SHREY RATHI PGBP 5.3

Sol 3:
1. Allowance as deduction: Payment for fire insurance is allowable as deduction under section 36(1). Since payment
credit card is covered under Rule 6DD, which contains the exceptions to section 40A(3), disallowance under section
40A(3) is not attracted in the case .
2. Allowable as deduction: As per rule 6DD, in case the payment is, made for purchase of agricultural produce directly
to the cultivator, grower or producer of such agricultural produce, no disallowance under section 40A(3) is attracted
even though the cash payment for the expense exceeds ₹ 10,000. Therefore, in the given case, disallowance under
section 40A(3) is not attracted since, cash payment for purchase of oil seeds is made directly to the farmer .
3. Not allowable as deduction: Income tax of ₹ 20,000 paid by the employer in respect of non-monetary perquisites
provided to its employees is exempt in the hands of the employee under section 10(10CC). As per section 40(a)(v),
such income tax paid by the employer is not deductible while computing business income.
4. Not allowable as deduction: Disallowance under section 40(a)(iii) is attracted in respect of salary payment of ₹
2,00,000 outside India by a company without deduction of tax at source.
5. Allowable as deduction: The limit for attracting disallowance under section 40A(3) for payment otherwise than by
way of account payee cheque or account payee bank draft or ECS is ₹ 35,000 in case of payment made for plying,
hiring or leasing goods carriage. Therefore in the present case, disallowance under section 40A(3) is not attracted for
payment of ₹ 30,000 made in cash to a transporter for carriage of goods .

Q 4: Mr. Sivam, a retail trader in Cochin gives the following Trading and Profit & Loss A/c for the year ended 31.03.2023:
Particulars (₹) Particulars (₹)
To Opening stock 90,000 By Sales 1,12,11,500
To Purchases 1,10,04,000 By Closing Stock 1,86,100
To Gross Profits c/d 3,03,600
Total 14,00,000 Total 14,00,000
To Salary 60,000 By Gross Profit b/d 3,06,000
To Rent and rates 36,000 By Income from UTI 2,400
To Interest on loan 15,000
To Depreciation 1,05,000
To Printing and stationery 23,200
To Postage and telegrams 1,640
To Loss on sale of shares (short-term
capital loss) 8,100
To Other general expenses 7,060
To Net Profit 50,000
Total 3,06,000 Total 3,06,000
Additional Information:
(a) It was found, some stocks were omitted to include in both the opening and closing stock, the values of which were ₹
9,000 and ₹ 18,000 respectively.
(b) Salary includes ₹ 10,000 paid to his brother, which is unreasonable to the extent of ₹ 2,000.
(c) The whole amount of printing and stationery was paid in cash.
(d) The depreciation provided in Profit & Loss A/c ₹ 1,05,000 was based on following information: The WDV of Plant &
Machinery is ₹ 4,20,000. A new plant under same block of depreciation of 15% was brought on 01.08.2022 for ₹
70,000. Two old plants were sold on 01.12.2022 for ₹ 50,000.
(e) Rent and rates includes goods & services tax liability of ₹ 4,300 paid on 17.04.2023.
(f) Other business receipts include ₹ 2,200 received as refund as custom duty relating to 2021-22.
(g) Other general expenses include ₹ 2,000 paid as donation to a public charitable trust.
Compute his business income. Also advise him, whether he should opt for presumptive income u/s 44AD assuming all his
receipts are received in account payee cheque. [ICAI Study Module]
CA SHREY RATHI PGBP 5.4

Sol 4: Computation of business income of Mr. Tendulkar for the previous year ended 31.03.2023:
Particulars (₹) (₹)
Net Profit as per Profit & Loss Account 50,000
Add: Expenses debited to P/L A/c but not allowed under Income Tax Act
• Loss on sale of shares (short-term) 8,100
• Unreasonable payment to brother 2,000
• Printing & stationary expenses (cash payment more than ₹ 10,000 is not admissible 23,200
u/s 40A(3))
• Depreciation [WN 1] (₹ 1,05,000 – ₹ 66,000) 39,000
• Donation to a public charitable trust is not a business expense 2,000 74,300
Less: Income credited to P/L A/c but not taxable u/h PGBP
• Income from UTI 2,400
Add: Incomes not recorded but taxable u/h PGBP
• Omission of closing stock 18,000
Less: Expenses deductible u/h PGBP but not shown in the P/L A/c
• Omission of opening stock 9,000
Income u/h PGBP of Mr. Tendulkar 1,30,900

WN 1: Calculation of depreciation u/s 32:


Written down value as on 01.04.2022 ₹ 4,20,000
(+) Acquisitions on 01.08.2022 ₹ 70,000
(-) Sale of assets ₹ 50,000
Written down value as on 31.03.2023 ₹ 4,40,000
Depreciation @ 15% as per Income Tax laws ₹ 66,000
Presumptive income u/s 44AD: As Mr. Tendulkar receives all his receipts through account payee cheques, his income shall
be 6% of turnover.
Income u/s 44AD = ₹ 1,12,11,500 x 6% = ₹ 6,72,690.
Advise: As presumptive income is higher than the normal income, Mr. Tendulkar should not follow presumptive basis of
income.

Q 5: Examine with reasons, whether the following statement are true or false, with regards to the provision of the Income
Tax Act, 1961:
(a) Payment made in respect of a business expenditure incurred on 16 th February, 2022 for ₹ 25,000 through a bearer
cheque is hit by the provision of section 40A(3).
(b) It is a condition precedent to write off in the books of account, the amount due from debtor to claim deduction for
bad debt.
(c) Failure to deduct tax at source from the amounts payable to a resident as rent or royalty, will result in disallowance
while computing the business income where the resident payee has not paid the tax due on such income.
[ICAI Study Module]

Sol 5:
(a) True: In order to escape the disallowance specified in section 40A(3), payment in respect of the business expenditure
ought to have been made through an account payee cheque. Payment through a bearer cheque will attract
disallowance under section 40A(3).
(b) True: It is mandatory to write off the amount due from a debtor as not receivable in the books of account, in order the
claim the same as bad debt under section 36(1)(vii).
(c) True: Section 40(a)(ia) provides that failure to deduct tax at source from rent or royalty payable to a resident will
result in disallowance of 30% of such expenditure, where the resident payee has not paid the tax due on such income.
CA SHREY RATHI PGBP 5.5

Q 6: Following is the details provided by Mr. Anish Kumar. Compute the income chargeable u/h PGBP from the following
details:
Profit & Loss A/c for the year ending 31st March 2023
Particulars (₹) Particulars (₹)
To Repairs on building 1,30,000 By Gross Profit 6,01,000
To Advertisement 51,000 By Income Tax Refund 4,500
To Amount paid to scientific By Interest from company deposits 6,400
research association approved 1,00,000 By Dividends 3,600
u/s 35(1)(ii)
To Interest 1,10,000
To Travelling Expenses 1,30,000
To Misc. Expenses 550
To Net Profit 93,950
Total 6,15,500 Total 6,15,500
Following additional information is furnished:
(1) Repairs on building includes ₹ 95,000 being cost of raising a compound wall for his own business premises.
(2) Interest payments include interest of ₹ 12,000 payable outside India to a resident Indian on which tax has not been
deducted and penalty of ₹ 24,000 for contravention of GST Act.
(3) Travelling expenses include ₹ 20,000 for a trip with cousins.

Sol 6: Computation of business income of Mr. Anish for the A/Y 2023-24:
Particulars (₹) (₹)
Net Profit as per Profit & Loss A/c 93,950
Add: Expenses debited to P/L A/c but not allowed under Income Tax Act
➢ Repairs of building (raising a compound wall is of capital nature) 95,000
➢ Interest payable outside India on which TDS has not been deducted 12,000
➢ Penalty for contravention of GST Act 24,000
➢ Travelling expenses (trip with cousins is of personal nature) 20,000 1,51,000
Less: Incomes credited to Profit & Loss A/c but not taxable u/h PGBP
➢ Income tax refund (not taxable, as income tax paid is not allowed deduction) 4,500
➢ Interest from company deposits (taxable u/h other sources) 6,400
➢ Dividends 3,600 14,500
Income u/h PGBP for Mr. Anish 2,30,450
Note: Depreciation on raising a compound wall is not taken. Students can give alternative view charging depreciation.

Q 7: Mr. Prajju is a registered medical practitioner who keeps his books on cash basis and provides you the following
details:
Receipt and Payment A/c for the year ending 31st March 2023
Particulars (₹) Particulars (₹)
To Balance b/d 2,700 By Costs of medicines 20,000
To Loan from bank 6,000 By Surgical equipment 6,000
To Sale of medicines 30,500 By Motor car 12,000
To Consultation fees 10,000 By Car expenses 1,800
To Visiting fees 8,000 By Salaries 1,200
To Interest on investment 9,000 By Rent of dispensary 1,200
To Dividend on shares 7,200 By General expenses 600
To Sale of building 15,000 By Personal expenses 3,600
To Sale of furniture 5,000 By Life insurance premium 2,000
By Interest on bank loan for investment 360
By Insurance of property 400
By Fixed deposit in bank 30,000
By Balance c/d 14,240
Total 93,400 Total 93,400
CA SHREY RATHI PGBP 5.6

Compute his income from profession taking into account the following further information:
1. 1/3rd of the motor car expenses is in respect of his personal use.
2. The original cost of the building was ₹ 20,000 and written down value of furniture as on 1st April, 2022 was ₹ 4,000.
There was no other asset in his block.
3. The rate of depreciation on motor car and on surgical equipments is 15%. An old car was purchased in June 2022
while surgical equipments were purchased in January 2023.
4. Outstanding consultancy fees and outstanding salaries are ₹ 30,000 and ₹ 1,900 respectively. Further, medicines
valuing ₹ 9,000 were sold on credit which are not recorded above.

Sol 7: Computation of income from profession of Mr. Prajju for the year ending 31.03.2023:
Particulars (₹) (₹)
Income from profession:
➢ Sale of medicines 30,500
➢ Consultation fees 10,000
➢ Visiting fees 8,000
Total Receipts taxable u/h PGBP (A) 48,500
Expenses of profession:
➢ Cost of medicines 20,000
➢ Depreciation on surgical equipments (₹ 6,000 x 15% x 50%) 450
➢ Depreciation on motor car (₹ 12,000 x 15% x 2/3) 1,200
➢ Car expenses (₹ 1,800 x 2/3) 1,200
➢ Salaries 1,200
➢ Rent of dispensary 1,200
➢ General expenses 600
Total Expenses deductible u/s PGBP (B) 25,850
Income from Profession for Mr. Prajju (A-B) 22,650
Notes:
1. As 1/3rd of the car is used for personal purpose, all expenses relating to the car shall restricted to 2/3 rd portion (i.e.
business purpose).
2. Sale of building and furniture shall not be dealt with as the entire block has ceased to exist, the gains or loss as the
case may be will be treated as per the provisions of capital gains chapter.
3. Outstanding consultancy fees, outstanding salaries & credit sale of medicines shall be ignored as Mr. Prajju is
following cash basis of accounting.

Q 8: Mr. Batla is the owner of a small business unit. He gives you the following details and requests you to compute his
business income for the assessment year 2023-24:
(i) Computed net profit after charging the following items ₹ 27,500
(ii) Provision for reserves debited to P/L A/c
 Provision for doubtful debts ₹ 15,000
 Depreciation reserve ₹ 20,000
(iii) House hold expenses ₹ 46,000
(iv) Other charitable donations ₹ 30,000
(v) Cheques issued for purchases ₹ 60,000
(vi) Advertisement gifts to 50 customers at a cost of ₹ 100 each.
(vii) Audit fee charged ₹ 20,000, including expenses on income-tax assessment ₹ 15,000.
(viii) Trademarks purchased for ₹ 70,000 during the previous year.
(ix) Income credited to P/L A/c were bank interest on FD ₹ 5,000.
(x) Opening stock is valued at cost plus 10% basis, whereas closing stock was valued at cost minus 10% basis. Opening
stock valued was ₹ 66,000; closing stock valued was ₹ 72,000.
CA SHREY RATHI PGBP 5.7

Sol 8: Computation of business income of Mr. Batla for the A/Y 2023-24:
Particulars (₹) (₹)
Computed Net Profit (after charging certain expenditures) 27,500
Add: Expenses deducted but not allowed under the Income Tax Act
▪ Provision for doubtful debts 15,000
▪ Depreciation reserve 20,000
▪ Household expenses 46,000
▪ Other charitable donations 30,000
▪ Trademarks acquired during the year (₹ 70,000 – 25%) 52,500
▪ Valuation of stock [(₹ 66,000 x 10/110) + (₹ 72,000 x 10/90)] 14,000 1,77,500
Less: Income credited to P/L A/c but not taxable u/h PGBP
▪ Interest on Bank FD 5,000
Income u/h PGBP for Mr. Batla 2,00,000

Q 9: Mr. Ranjeet, a manufacturer in Bhopal gives you the following Trading and Profit & Loss A/c for the year ending
31.03.2023.
Trading and Profit & Loss A/c
Particulars (₹) Particulars (₹)
To Opening stock 71,000 By Sales 32,00,000
To Purchases 16,99,000 By Closing stock 2,00,000
To Wages 2,70,000
To Gross profit 13,60,000
Total 34,00,000 Total 34,00,000
To Office expenses 3,26,000 By Gross Profit 13,60,000
To State GST penalty paid 5,000 By Dividend from domestic companies 15,000
To State GST paid 1,10,000 By Agricultural income 1,80,000
To General expenses 54,000
To Interest on loan to bank 60,000
To Depreciation 2,00,000
To Net Profit 8,00,000
Total 15,55,000 Total 15,55,000
Additional information provided by Mr. Ranjeet is as under:
(i) Office expenses include ₹ 46,000 paid as commission to the sister of the assessee. The commission amount at the
market rate is ₹ 36,000.
(ii) The assessee paid ₹ 33,000 in cash to a transport carrier on 19.11.2022. This amount is included in general expenses.
(iii) A sum of ₹ 4,000 p.m. was paid as salary to a staff throughout the year and this has not been recorded in the books
of account.
(iv) Interest on loan actually paid upto 31.03.2023 was ₹ 20,000 and the balance was paid on 3rd December, 2023.
(v) Housing loan principal repaid during the year was ₹ 50,000 and it relates to a residential property occupied by him.
Interest on housing loan was ₹ 23,000. Housing loan was taken from Axis Bank. These amounts were not dealt with
in the Profit and Loss A/c given above.
(vi) Depreciation allowable under the Act is to be computed on the basis of following information:
Plant & Machinery (Rate of depreciation – 15%)
Opening WDV (as on 01.04.2022) ₹ 12,00,000
Acquisitions during the year (used for more than 180 days) ₹ 2,00,000
Acquisitions during the year (used for less than 180 days) ₹ 2,00,000
Compute the business income of Mr. Ranjeet for the A/Y 2023-24.
CA SHREY RATHI PGBP 5.8

Sol 9: Computation of income from business of Mr. Ranjeet for the A/Y 2023-24:
Particulars (₹) (₹)
Net Profit as per Profit & Loss A/c 8,00,000
Add: Expenses debited to P/L A/c but not allowed under Income Tax Act
o State GST penalty paid (illegal) 5,000
o Unreasonable commission to sister u/s 40A(2) 10,000
o Interest on loan (allowed deduction on payment basis u/s 43B) (Expenses shall be
allowed to the extent it is paid on or before the due date) (₹ 60,000 – ₹ 20,000) 40,000 55,000
Less: Income credited to P/L A/c but not taxable u/h PGBP
o Dividend from Domestic Company 15,000
o Agriculture Income 1,80,000 1,95,000
Less: Expenses deductible u/h PGBP but not shown in the P/L A/c
o Salary to staff not recorded (₹ 4,000 x 12) 48,000
o Depreciation [WN 1] (₹ 2,85,000 – ₹ 2,00,000) 85,000 1,33,000
Income u/h PGBP for Mr. Ranjeet 5,27,000

WN 1: Calculation of Depreciation:
Normal Depreciation
Opening WDV as on 01.04.2022 ₹ 12,00,000 x 15% = ₹ 1,80,000
Acquisitions during the year (used for more than 180 days) ₹ 2,00,000 x 15% = ₹ 30,000
Acquisitions during the year (used for less than 180 days) ₹ 2,00,000 x 15% x 50% = ₹ 15,000
₹ 2,25,000
Additional Depreciation, as Mr. Ranjeet is a manufacturer:
Acquisitions during the year (used for more than 180 days) ₹ 2,00,000 x 20% = ₹ 40,000
Acquisitions during the year (used for less than 180 days) ₹ 2,00,000 x 20% x 50% = ₹ 20,000
₹ 60,000
Total Depreciation as per Income Tax = ₹ 2,25,000 + ₹ 60,000 = ₹ 2,85,000
Notes:
1. Payment to transport operator in cash is admissible upto ₹ 35,000 u/s 40A(3). Therefore, payment of ₹ 33,000 shall
be admissible.
2. Housing loan will not have any impact on business income as it does not relate to the business of the assessee. It shall
be dealt in House property chapter.

Q 10: Mr. Kunal, a proprietor, engaged in the business of generation of power, furnishes the following particular
pertaining to P.Y. 2022-23. Compute the depreciation allowable under section 32 for A.Y 2023-24, while computing his
income under the head “profits and gains of business or profession.” The proprietor has opted for the depreciation
allowance on the basis of written down value.
S. No. Particulars (₹)
1. Opening written down value of plant and machinery (15% block) as on 01.04.2022. (Purchase 5,78,000
value ₹ 8,00,000)
2. Purchase of second hand machinery (15% block) on 29.12.2022 for business purpose. 2,00,000
3. Machinery Y (15% block) purchased and installed on 12.07.2022 for purpose of power 8,00,000
generation.
4. Acquired and installed for use a new air pollution control equipment on 31.07.2022. 2,50,000
5. New air conditioner purchased and installed in office premises on 08.09.2022. 3,00,000
6. New machinery Z (15% block) acquired and installed on 23.11.2022 for the purpose of 3,25,000
generation of power.
7. Sale value of an old machinery X, sold during the year (purchase value ₹ 4,80,000, WDV as 3,10,000
01.04.2022 ₹ 3,46,800)
[ICAI Study Module]
CA SHREY RATHI PGBP 5.9

Sol 10: Computation of depreciation allowance u/s 32 for the A/Y 2023-24:
Particulars (₹) P & M (15%) (₹) P & M (40%)
(₹)
Opening WDV as on 01.04.2022 5,78,000 -
Add: Plant & Machinery acquired during the year
o Second hand machinery 2,00,000
o Machinery y 8,00,000
o Air conditioner for office 3,00,000
o Machinery Z 3,25,000 16,25,000
o Air pollution control equipments - 2,50,000
22,03,000 2,50,000
Less: Assets sold during the year 3,10,000 -
Written down value before charging depreciation 18,93,000 2,50,000
Normal Depreciation
Depreciation on plant & machinery put to use for less than 180
days @ 7.5% (i.e. 50% of 15%)
o Second-hand machinery (₹ 2,00,000 x 7.5%) 15,000
o Machinery Z (₹ 3,25,000 x 7.5%) 24,375 39,375
15% on the balance WDV being put to use for more than 180 days
(₹ 13,68,000 x 15%) 2,05,200
40% on air pollution control equipment (₹ 2,50,000 x 40%) 1,00,000
Additional Depreciation
Machinery Y (₹ 8,00,000 x 20%) 1,60,000
Machinery Z (₹ 3,25,000 x 10%, being 50% of 20%) 32,500 1,92,500
Air pollution control equipment (₹ 2,50,000 x 20%) 50,000
Total Depreciation 4,37,075 1,50,000
Notes:
(1) Power generation equipments qualify for claiming additional depreciation in respect of new plant and machinery.
(2) Additional depreciation is not allowed in respect of second-hand machinery & air conditioner installed in office
premises.
(3) The balance 50% additional depreciation in respect of Machinery Z of ₹ 32,500 (10% of ₹ 3,25,000) can be claimed
as deduction in subsequent financial year 2023-24.

Q 11: Mr. Sameer (age: 44 years), a resident individual, furnishes the following information for the A/Y 2023-24:
Profit & Loss A/c for the year ending 31st March 2023
Particulars (₹) Particulars (₹)
To Salary & wages 72,000 By Gross profit 8,71,000
” Conveyance expenses 15,600 ” Commission received 1,87,000
” Other general expenses 21,200 ” Sundry receipts 22,000
” Bad debts written off 3,400 ” Short-term profit on sale of investments 60,000
” Reserve for losses 5,000
” Advertisement expenses ₹ 3,000
(+) Outstanding ₹ 1,800 4,800
” Finance charges 32,500
” Interest on Sameer’s Capital 3,300
” Depreciation 27,000
” Provision for goods and services tax and
custom duty 11,000
” Lump-sum consideration for acquiring
patent on 14th Jan, 2023 56,000
Net Profit 8,88,200
Total 11,40,000 Total 11,40,000
CA SHREY RATHI PGBP 5.10

Other Information:
1. Depreciation according to income tax provision is ₹ 31,000.
2. Salary to staff includes payment of ₹ 18,000 to a relative out of which ₹ 7,000 is unreasonable.
3. Other general expenses include:
(a) expenditure of ₹ 3,200 incurred by Sameer on training of his employees,
(b) commission of ₹ 5,000 for securing a business order and
(c) compensation of ₹ 6,000 paid to an employee while terminating his service in the business interest.
4. Out of outstanding goods & services tax and custom duty, ₹ 2,500 is paid on 15th June 2023 and ₹ 6,500 is paid on 18th
August 2023. The balance is not yet paid. Due date of filing return of income is 31st July 2023.
Determine income u/h PGBP of Mr. Sameer for the A/Y 2023-24 assuming that insurance premium paid by him on his life
insurance policy is ₹ 8,000.

Sol 11: Computation of business income of Mr. Sameer for the A/Y 2023-24:
Particulars (₹) (₹)
Net Profit as per Profit & Loss A/c 8,88,200
Add: Expenses debited to P/L A/c but not allowed under Income Tax Act
(a) Salary to a relative (unreasonable payment u/s 40A(2)) 7,000
(b) Reserve for losses 5,000
(c) Interest on Sameer’s capital (interest on own capital is not allowed) 3,300
(d) Provision for goods & services tax & custom duty (allowed only to the extent it is
paid on or before the due date of filing of return u/s 139(1) [₹ 11,000 – ₹ 2,500] 8,500
(e) Lump-sum consideration for purchase of patents 56,000 79,800
Less: Income credited to P/L A/c but not taxable u/h PGBP
▪ Short-term profit on sale of investments 60,000
Less: Expenses deductible u/h PGBP but not shown in the P/L A/c
▪ Depreciation as per Income Tax [₹ 31,000 – ₹ 27,000] 4,000
▪ Depreciation on Patent acquired on 18th January 2022 (₹ 56,000 x 25% x 50%) 7,000 11,000
Income u/h PGBP for Mr. Sameer 8,97,000

Notes:
1. Expenditure of ₹ 3,200 incurred on training of his employees; commission of ₹ 5,000 for securing a business order &
compensation of ₹ 6,000 paid to an employee for terminating his service in the business interest are all business
expenditures and therefore shall be admissible.
2. Premium paid for life insurance policy is a personal expenditure and therefore shall not be admissible. However,
deduction u/s 80C is allowed of such premium from the Gross Total Income of the assessee.

Q 12: Mr. Arijeet, a resident, furnishes you the following information and asks you to compute his business income for the
A/Y 2023-24.
Profit and Loss A/c for the year ending 31st March, 2023
Particulars (₹) Particulars (₹)
To Office expenses 11,000 By Gross Profit 3,78,000
To Telephone security deposited 8,000 By Sundry receipts 8,000
To Salary to staff 42,000
To Depreciation 28,000
To Travelling expenses 43,000
To Loss of cash by theft 5,000
To Special reserve 7,500
To Diwali expenses 7,100
To Interest and legal expenses 44,000
To Sundry expenses 8,500
To Net Profit 1,81,900
Total 3,86,000 Total 3,86,000
CA SHREY RATHI PGBP 5.11

Additional information:
1. Salary to staff includes payment of ₹ 12,000 out of India on which tax has not been deducted at source.
2. Depreciated value of machinery on 1st April, 2022 is ₹ 1,10,000 (Depreciation rate – 15%)
(a) A machinery whose w.d.v on 1st April, 2022 is ₹ 17,440 is sold during the P/Y for ₹ 11,000.
(b) A machinery (cost price – ₹ 20,000) whose w.d.v on 1st April, 2022 is ₹ 2,350 is sold for ₹ 15,000 during the
previous year.
(c) During the year, Arijeet acquired a plant for ₹ 1,22,670 which is eligible for depreciation @ 15%. The plant is
installed and put to use on 17th July, 2022.
3. Travelling expenses include ₹ 10,000 being hotel expenditure in respect of a personal visit to Chennai for 6 days.
4. Legal expenses include the following payments:
(a) Payment of ₹ 5,000 to an employee for filing an appeal.
(b) Payment of ₹ 12,000 to Anandita, non-employee, for preparation of final accounts.
(c) Payment of ₹ 20,000 to Mohit, a CA for obtaining tax advice.
5. Sundry expenses include expenditure of ₹ 1,000 on maintenance of guest house in Delhi for the purpose of carrying
on the business and ₹ 4,000 being employer’s contribution towards ESI(PF) out of which ₹ 600 is paid after the due
date of submission of return of income.
6. Expenditure on Diwali includes a gift of ₹ 2,000 for his wife.
7. Interest includes a payment of ₹ 3,000 out of India on which tax has not been deducted.

Sol 12: Computation of business income of Mr. Arijeet for the A/Y 2023-24:
Particulars (₹) (₹)
Net Profit as per Profit & Loss Account 1,81,900
Add: Expenses debited to P/L A/c but not allowed under Income Tax Act
❖ Salary to staff outside India on which TDS has not been deducted shall not be 12,000
admissible u/s 40(a)
❖ Travelling expenses to Chennai (personal visit) 10,000
❖ Special Reserve 7,500
❖ Diwali gift to wife (personal nature) 2,000
❖ Interest paid outside India on which TDS has not been deducted shall not be
admissible u/s 40(a) 3,000
❖ Sundry expenses (employer’s contribution towards ESI(PF) after the due date shall
not be allowed as per the provisions of Section 43B, it shall be allowed in the year of
payment) 600 35,100
Less: Expenses deductible u/h PGBP but not shown in the P/L A/c
❖ Depreciation [WN 1] (₹ 31,000 – ₹ 28,000) 3,000
Income u/h PGBP for Mr. Arijeet 2,14,000
WN 1: Calculation of depreciation u/s 32:
Written down value as on 01.04.2022 ₹ 1,10,000
(+) Acquisition on 17th July 2022 ₹ 1,22,670
(-) Sale of assets during the year (11,000 + 15,000) ₹ 26,000
Written down value as on 31.03.2023 ₹ 2,06,670
Depreciation @ 15% as per Income Tax laws ₹ 31,000
Notes:
1. Telephone security deposited has been assumed irrecoverable, therefore shall be allowed deduction.
2. Payments to an employee for filing an appeal; payment for preparation of final accounts & payment to a CA for tax
advise are business expenditures & shall be allowed deduction u/s 37(1).
CA SHREY RATHI PGBP 5.12

Q 13: Mr. Nishant is a businessman in Mangalore. Compute his business income on the basis of the following data for the
year ending 31st March 2023.
Profit & Loss Account for the year ending 31st March 2023
Particulars (₹) Particulars (₹)
To Opening stock 1,35,000 By Sales 78,80,000
” Purchases 65,77,700 ” Closing stock 2,25,000
” Salaries & wages 2,64,000
” General expenses 77,600
” Household expenses 55,000
” Income tax for 2021-22 21,800
” Advertisement expenses 41,500
” Interest on own capital 18,000
” Reserve for bad debts 7,400
” Depreciation on furniture 12,200
” Dividend tax 1,100
” Unapproved Gratuity Fund 6,000
To Net Profit 8,87,700
Total 81,05,000 Total 81,05,000
Other Information:
1. Closing stock and opening stock has been consistently valued at 10% below cost.
2. Depreciation on furniture as per tax laws is ₹ 9,800.
3. Amount of sales includes ₹ 47,850, being value of goods withdrawn for the use of Nishant’s family members. These
goods were purchased at cost of ₹ 31,250. Market value of such goods is ₹ 49,750.
4. Household expenses include a contribution of ₹ 3,000 towards PPF.
5. Salaries & wages include an expenditure of ₹ 22,000 which is paid in cash.

Sol 13: Computation of income u/h PGBP for Mr. Nishant for the A/Y 2023-24:
Particulars (₹) (₹)
Net Profit as per Profit & Loss Account 8,87,700
Add: Expenses debited to P/L A/c but not allowed under Income Tax Act
o Salaries & wages (payment in excess of ₹ 10,000 in cash shall be inadmissible as per 22,000
section 40A(3))
o Household expenses & PPF (personal nature) 55,000
o Income tax of 2021-22 (inadmissible u/s 40(a), being personal expense) 21,800
o Interest on own capital (not allowed) 18,000
o Reserve for bad debts (no reserve is allowed, only bad debts written off as 7,400
irrecoverable shall be admissible)
o Depreciation is allowed as per Income Tax laws (₹ 12,200 – ₹ 9,800) 2,400
o Dividend tax 1,100
o Unapproved gratuity fund (only payment to approved gratuity fund within the time
limits is admissible) 6,000 1,33,700
Less: Expenses deductible u/h PGBP but not shown in the P/L A/c
o Undervaluation of opening stock (₹ 1,35,000 x 10/90) 15,000
Add: Incomes not recorded but taxable u/h PGBP
o Undervaluation of closing stock (₹ 2,25,000 x 10/90) 25,000
Less: Income credited to P/L A/c but not taxable u/h PGBP
o Goods withdrawn for personal included in sales (₹ 47,850 – ₹ 31,250) 16,600
Income u/h PGBP for Mr. Nishant 10,14,800

Note: Market value of goods withdrawn for personal use is irrelevant for computation of income u/h PGBP.
CA SHREY RATHI PGBP 5.13

Q 14: Examine with reasons, for the following statements, whether the following statements are true or false having
regards to the provision of the Income Tax Act, 1961;
1. For a dealer in share and securities transaction tax paid in a recognized stock exchange is permissible business
expenditure.
2. The mediclaim premium paid to GIC by Mr. Lomesh for his employees, by a draft, on 27.12.2022 is a deductible
expenditure under section 36.
3. Under section 35DDA, amortization of expenditure incurred under eligible Voluntary Retirement Scheme at the time
of retirement alone, can be done.
4. An existing assessee engaged in trading activities, can claim additional deprecation under section 32(1)(iia) in respect
of new plant acquired and installed in the trading concern, where the increase in value of such plant as compared to
the approved base year is more than 10%. [ICAI Study Module]

Sol 14:
1. True: Section 36(1)(xv) allows a deduction of the amount of securities transaction tax paid by the assessee in respect
of taxable securities transaction entered into in the course of business during the previous year as deduction from the
business income of a dealer in shares and securities.
2. True: Section 36(1)(ib) provides deduction in respect of premium paid by an employer to keep in force an insurance
on the health of his employees under a scheme framed in this behalf by GIC or any other insurer. The medical
insurance premium can be paid by any mode other than cash to be eligible for deduction under section 36(1)(ib).
3. False: Expenditure incurred in making payment to the employee in connection with his voluntary retirement either in
the year of retirement or in any subsequent year, will be entitled to deduction in 5 equal annual instalments
beginning from the year in which each payment is made to the employee.
4. False: Additional deprecation can be claimed only in respect of eligible plant and machinery acquired and instalment
by an assessee engaged in the business of manufacture or production of any article or thing or in the business of
generation or transmission or distribution of power. In this case, the assessee is engaged in trading activities and the
new plant has been acquired and installed in a trading concern. Hence, the assessee will not be entitled to claim
additional deprecation under section 32(1)(iia).

Q 15: Mr. Manohar who maintains books of accounts on cash basis is a law consultant. He provides the following
information for the A/Y 2023-24.
Receipt and Payment A/c for the year ending 31st March 2022
Particulars (₹) Particulars (₹)
To Balance b/d 18,600 By Purchase of a printer 14,000
To Fees from clients for: By Car expenses 28,800
• 2021-22 1,23,700 By Sundry expenses 56,600
• 2022-23 9,68,400 By Salary to staff for:
• 2023-24 1,48,300 • 2021-22 23,600
To Presents from clients 36,600 • 2022-23 1,32,000
To Interest free loan from a client for • 2023-24 32,000
purchase of a car 1,75,000 By Expenses relating to house property:
To Winning from lottery 28,000 • Municipal tax 4,000
To Interest from UTI 7,400 • Repairs & insurance 5,000
To Rent from house property 72,000 By Car purchased on 23rd Nov 2022 8,12,000
To Share of income from a firm 45,000 By Office repairs 27,700
By Interest on loan 14,400
By Telephone expenses 8,600
By Electricity expenses 12,200
By Legal expenses 31,800
By Income tax payment 5,500
By Life insurance premium 42,000
By Balance c/d 3,72,800

Total 16,23,000 Total 16,23,000


CA SHREY RATHI PGBP 5.14

Other information:
(a) Car is partly used for business purpose (60%) and partly for private purpose (40%).
(b) Payment of income tax includes interest for late filing of return to the extent of ₹ 850.
(c) Salary to staff includes payment in cash of ₹ 8,000 & ₹ 10,000 to two different employees on the same day.
(d) Municipal taxes include payment of ₹ 2,200 for the P/Y 2021-22.
Determine income u/h PGBP of Mr. Manohar for the A/Y 2023-24.

Sol 15: Computation of professional income of Mr. Manohar for the year ending 31.03.2023 who maintains his accounts
on cash basis:
Particulars (₹) (₹)
Income from profession:
(1) Fees from clients
➢ 2021-22 1,23,700
➢ 2022-23 9,68,400
➢ 2023-24 1,48,300
(2) Presents from clients 36,600
Total Receipts taxable u/h PGBP (A) 12,77,000
Expenses of profession:
1. Depreciation on printer (₹ 14,000 x 40%) 5,600
2. Car expenses (₹ 28,800 x 60%) 17,280
3. Sundry expenses 56,600
4. Salaries
➢ 2021-22 23,600
➢ 2022-23 1,32,000
➢ 2023-24 32,000
5. Depreciation on car (₹ 8,12,000 x 15% x 50% x 60%) 36,540
6. Office repairs 27,700
7. Interest on loan 14,400
8. Telephone expenses 8,600
9. Electricity expenses 12,200
10. Legal Expenses 31,800
Total Expenses deductible u/s PGBP (B) 3,98,320
Income from Profession for Mr. Manohar (A-B) 8,78,680
Notes:
1. Depreciation on printer has been fully allowed assuming it has been acquired and put to use for more than 180 days.
2. Interest on loan has been assumed to be paid for loan taken for the purpose of profession.
3. Car has been partly used for business (60%) and partly for personal purpose (40%). Therefore, all expenses relating to
car (i.e. car expenses & depreciation on car) shall be allowed only to the extent of 60%.
4. Salary to staff in cash of ₹ 8,000 & ₹ 10,000 to two different employees on the same day is permissible as section
40A(3) deals with cash payment upto ₹ 10,000 for an expenditure made to a person in a day. As the payment does
not exceed ₹ 10,000, nothing shall be disallowed.
5. Income tax payment and life insurance premium paid are personal expenses and therefore shall not be allowed. Even
interest paid for late filing of return is inadmissible u/s 40(a).
6. Expenses relating to house property such as municipal taxes and repairs of house property shall be dealt within the
house property chapter.
CA SHREY RATHI PGBP 5.15

Q 16: Examine with reasons, the allowability of the following expenses incurred by Mr. Manav, a wholesale dealer of
commodities, under the Income Tax Act, 1961 while computing profit and gains from business or profession for the A/Y
2023-24.
(1) Construction of school building in compliance with CSR activities amounting to ₹ 5,60,000
(2) Purchase of building for setting up a warehousing facility for storage of food grains amounting to ₹ 4,50,000.
(3) Interest on loan paid to Mr. X (a resident) ₹ 50,000 on which tax has not been deducted. the sales for the P/Y 2020-21
was ₹ 202 lakhs.
(4) Commodities Transaction Tax paid ₹ 20,000 on sale of bullion. [ICAI Study Module]

Sol 16: Allowability of the expenses incurred by Mr. Manav, wholesale dealer in commodities, while computing profits and
gains from business or profession.
(1) Construction of school building in compliance with CSR activities: Under section 37(1) only expenditure not being in
the nature of capital expenditure or personal expense and covered under section 30 to 36, and incurred wholly and
exclusively for the business is allowed as a deduction while computing business income.
However, any expenditure incurred by an assessee on the activities relating to corporate social responsibility referred to in
section 135 of the Companies Act, 2013 shall not be deemed to have incurred for the purpose of business and hence, shall
not be allowed as deduction under section 37.
Accordingly, the amount of ₹ 5,60,000 incurred by Mr. Manav, towards construction of school building in compliance with
CSR activities shall not be allowed as deduction under section 37.

(2) Purchase of building for setting up a ware housing facility for storage of food grains: Mr. Manav would be eligible for
investment–linked tax deduction under section 35AD @ 100% in respect of amount of ₹ 4,50,000 invested in
purchase of building for setting up a warehousing facility for storage of food grains which commences operation on or
after 1st April, 2009 (P.Y. 2022-23 in this case). Therefore, the deduction under section 35AD while computing business
income would be ₹ 4,50.000.

(3) Interest on loan paid to Mr. X (a resident) ₹ 50,000 on which tax has not been deducted: As per section 194A Mr.
Manav, being an individual is required to deduct tax at source on the amount of interest on loan paid to Mr. X since
his turnover during the P/Y 2020-21 exceeds the monetary limit of ₹ 100 lacs.
Therefore, ₹ 15000 being 30% of ₹ 50000 would be disallowed under section 40(a)(ia) while computing the business
income of Mr. Manav for non-deduction of tax at source under section 194A on interest of ₹ 50,000 paid by it to Mr. X.
The balance ₹ 35,000 would be allowed as deduction under section 36(1)(iii), assuming that the amount was borrowed for
the purposes of business.

(4) Commodities transaction tax of ₹ 20,000 paid on sale of bullion: Commodities transaction tax paid in respect of
taxable commodities transaction entered into in the course of business during the previous year is allowable as
deduction, provided the income arising from such taxable commodities transaction in included in the income
computed under the head “Profits and Gains of Business or Profession.” Taking that income from this commodities
transaction is included while computing the business income of Mr. Manav, the commodity transaction tax of ₹
20,000 paid is allowable as deduction under section 36(1)(xvi).
CA SHREY RATHI PGBP 5.16

Q 17: Ms. Akanksha furnishes the following information for the A/Y 2023-24:
Profit & Loss A/c for the year ending 31st March 2023
Particulars (₹) Particulars (₹)
To Office expenses 22,000 By Gross profit 4,58,000
” Sundry expenses 36,000 ” Sundry receipts 31,000
” Extension of Building 14,000 ” Bad debts recovered (not allowed as
” Depreciation on Plant & Machinery 18,500 deduction earlier) 12,000
” Salary to staff 48,000 ” Custom duty recovered from the
” Bonus 8,000 Government (earlier allowed as 18,500
” Contribution towards: deduction)
• RPF 15,500 ” Gift received from mother 22,500
• Unapproved Gratuity Fund 3,200
” Goods & Services Tax 31,300
” Provision for Goods & Services Tax 18,200
” Payment to approved research
association for carrying scientific 24,000
research
” Net Profit 3,03,300
Total 5,42,000 Total 5,42,000
Other Information:
1. Payment to scientific research association for carrying on scientific research is not related to the business of
Akanksha. Besides she purchases a plant of ₹ 40,000 for the purpose of carrying on scientific research related to her
business. Neither the cost of plant nor depreciation thereon is debited to P/L A/c.
2. Out of bonus of ₹ 8,000, ₹ 3,000 is paid during the year 2022-23, ₹ 2,500 is paid by 31st July 2023 (being the due date
of furnishing return of income) and the balance of ₹ 2,500 is however paid on 12th December 2023.
3. Depreciation on plant & machinery and extension of building as per the income tax provisions is ₹ 15,000.
4. GST of ₹ 31,300 includes (a) interest for late payment of tax: ₹ 1,800 and (b) penalty for evading tax: ₹ 4,300.
5. Provision for GST represents an outstanding GST liability which is however paid on 29th June 2023.
6. Salary to staff includes a payment of pension of ₹ 4,000 to the widow of a former employee.
Ascertain the income u/h PGBP of Ms. Akanksha for the A/Y 2023-24 assuming that she deposits ₹ 40,000 in PPF A/c
during the P/Y 2022-23.
CA SHREY RATHI PGBP 5.17

Sol 17: Computation of income u/h PGBP for Ms. Akanksha for the A/Y 2023-24:
Particulars (₹) (₹)
Net Profit as per Profit & Loss Account 3,03,300
Add: Expenses debited to P/L A/c but not allowed under Income Tax Act
• Extension of Building 14,000
• Depreciation on Plant & Machinery and Building as per Income Tax Laws ( ₹ 18,500 – 3,500
₹ 15,000)
• Bonus (Payment after the due date shall not be allowed as per the provisions of 2,500
Section 43B, it shall be allowed in the year of payment)
• Contribution towards Unapproved Gratuity Fund (only approved funds are allowed) 3,200
• GST (Penalty of illegal nature is inadmissible u/s 40(a)) 4,300
Less: Income credited to P/L A/c but not taxable u/h PGBP 27,500
• Bad debts recovered (as it was not allowed deduction in the earlier previous years, it
shall not become taxable in the year of receipt) 12,000
• Gift received from mother (gift from relative is not taxable) 22,500 34,500
Less: Expenses deductible u/h PGBP but not shown in the P/L A/c
• Payment to Scientific Research Association not related to the business of Akanksha
(100% expenditure allowed) – Since 100% deduction is claimed, no further treatment ---
is required.
• Purchase of plant for in-house research (100% deduction is allowed as per section
40,000 40,000
35)
Income u/h PGBP for Ms. Akanksha 2,56,300
Notes: Payment to the widow of a former employee is an admissible expense u/s 37.

MULTIPLE CHOICE QUESTIONS


1. Expenses debited to P/L A/c but not allowed under the Income Tax Act shall be ………….. while computing income u/h
PGBP.
(a) Subtracted (c) Added
(b) No Treatment (d) None of the above

2. For an assessee engaged in business and profession, any sum received under an agreement for not carrying any
activity in relation to any business is taxable under head.....
(a) PGBP (c) Other Sources
(b) Capital Gains (d) Salary

3. Which of the following income is chargeable to tax under head Profits and gains from business and profession?
(a) Cash Compensatory Support and Duty Drawback
(b) Sum received under a key man insurance policy
(c) Speculative income
(d) All of the above

4. Where assessee has sublet a part of the premises, the allowance under section 30 would be confined to _______.
(a) Rent recovered from the sub – tenant.
(b) Rent paid by assessee.
(c) Difference between rent paid by assessee and the rent recovered from the sub – tenant.
(d) None of the above

5. Interest from bank FD should be credited to P/L A/c while computing PGBP income
(a) True (c) False
(b) Partly True (d) None of the above

6. The illegality of a business, profession or vocation exempts its profits from tax.
(a) True (b) False
CA SHREY RATHI PGBP 5.18

7. Mr. Anurag uses the premises partly for his craft business and partly for his residential purpose. So, _____ part of
expenses attributable to that part of premises used for purpose of ______ will be allowed as deduction under Section
30.
(a) Full; business (c) Proportionate; business
(b) Full; residential (d) Proportionate; residential

8. Section 43B & 35DDA are to be followed on ……………… bases of accounting.


(a) Cash (c) Method regularly followed by the assessee
(b) Mercantile (d) None of the above

9. Mr. Rohan, a proprietor engaged in manufacturing business, has acquired and installed the computer amounting to ₹
3,00,000 in the office premises on 02.01.2023. The additional depreciation as per the Income Tax Act, 1961 for the
A.Y. 2022-23 is:
(a) ₹ 30,000 (c) ₹ 60,000
(b) ₹ 45,000 (d) Nil

10. Mr. X acquires an asset in the year 2016-17 for the use for scientific research for ₹ 2,75,000. He claimed deduction
under section 35(1)(iv) in the previous year 2016-17. The asset was brought into use for the business of Mr. X in the
P.Y. 2022-23, after the research was completed. The actual cost of the asset to be included in the block of assets is –
(ICAI MCQ Booklet)
(a) Nil
(b) Market value of the asset on the date of transfer to business
(c) ₹ 2,75,000 less notional depreciation under section 32 upto the date of transfer
(d) Actual cost of the asset i.e. ₹ 2,75,000

11. Depreciation rate for oil wells is –


(a) 40% (c) 10%
(b) 15% (d) 30%

12. Mr. Abhishek, a proprietor engaged in manufacturing business, purchased new plant and machinery for ₹ 8,00,000 on
18.4.2022 and put to use on 18.12.2022. Also, Rate of depreciation is 15%. The amount of depreciation as per the
Income Tax Act, 1961 for the A.Y. 2023-24 is:
(a) ₹ 1,20,000 (c) ₹ 80,000
(b) ₹ 60,000 (d) ₹ 1,40,000

13. Mr. Gopal purchased a car on 18.04.2019 for ₹ 9,00,000 for personal use. It is now brought into professional use on
14.06.2022, when its market value was ₹ 4,50,000. Assume rate of depreciation is 15%. The amount of depreciation
for A.Y. 2023-24 is:
(a) ₹ 1,35,000 (c) ₹ 1,14,750
(b) ₹ 67,500 (d) ₹ 57,375

14. When all the assets of the block are sold and sale value is less than the wdv, then such deficit shall be termed as:
(a) Short-term capital gain (c) Short-term capital loss
(b) Long-term capital gain (d) Long-term capital loss

15. Mr. Rishabh carries on his handicraft business in the building which is taken on lease by him on 04.04.2022. Mr.
Rishabh incurred ₹ 2,00,000 by way of capital expenditure for renovation and improvement of the building. Whether
depreciation be charged on ₹ 2,00,000?
(a) Yes (b) No

16. When an asset on which depreciation is allowed is transferred and block does not cease to exist, how should it be
treated?
(a) Cost of asset shall be reduced from the block of asset
(b) Sale proceeds of asset shall be reduced from block of asset
(c) Profit shall be computed as per section 50 and chargeable under head capital gain.
(d) None of the above
CA SHREY RATHI PGBP 5.19

17. Unabsorbed depreciation can be carried forward


(a) 8 years (c) 4 years
(b) Indefinitely (d) Nil

18. An assessee purchased imported second hand machinery and contends that additional depreciation shall be allowed
on same. Is the contention of assessee correct?
(a) Invalid (c) Valid
(b) Partly valid (d) None of the above

19. An assessee who has a block of ₹ 250 lakhs of plant and machinery as on 1st April 2022. He sold the entire block for ₹
170 lakhs on 24th June 2022 and an asset of ₹ 20 lakhs was acquired on 6th November 2022. On what amount should
depreciation be charged and what shall be the amount of capital gain?
(a) Nil, Nil (c) ₹ 100 lakhs, Nil
(b) Nil, (₹ 100 lakhs) (d) Nil, ₹ 100 lakhs

20. Additional depreciation shall not be allowed in which of the following case
(a) Assessee engaged in business of manufacture or production of any article or thing
(b) Assessee engaged in business of generation or generation or distribution of power
(c) Where assessee has purchased a plant and machinery which is already used
(d) All of the above

21. An assessee engaged in business of generation and distribution of power claims depreciation on SLM basis and sales
asset. The sales price of asset exceeds WDV of the asset, what shall be the tax treatment under head PGBP:
(a) Actual cost – WDV is taxable under head PGBP
(b) S.P. - WDV is taxable under head PGBP
(c) Least of a) and b)
(d) None of the above

22. Where assessee is engaged in business of generation/ generation & distribution of power. He can claim
depreciation......
(a) As per straight Line method (SLM) on each asset
(b) As per written down value (WDN) method on block of assets
(c) Assessee has option to claim depreciation either as per (a) or (b)
(d) None of the above

23. A Goods carriage for the purpose of presumptive income u/s 44AE shall be considered to be a heavy goods vehicle
where gross vehicle weight exceeds:
(a) 10,000 kgs (c) 15,000 kgs
(b) 12,000 kgs (d) 14,000 kgs

24. Deduction under section 35(1)(ii) is:


(a) Sum paid to a research association having an object the undertaking of scientific research
(b) Sum paid to a national laboratory / university / institution approved by Central Government
(c) Sum paid to a company to be utilised in scientific research
(d) None of the above

25. If there is a change of ownership, depreciation shall be apportioned on the basis of ……………..
(a) No. of days (c) No. of weeks
(b) No. of months (d) None

26. Where an assessee pays any sum to a university, college, or other approved institution to be used for research in
social science or statistical research. The research is not related to his business. The deduction allowed would be of
……..
(a) 150% u/s 35(1)(ii) (c) 100% u/s 35(1)(ii)
(b) 100% u/s 35(1)(iii) (d) 200% u/s 35(1)(iii)
CA SHREY RATHI PGBP 5.20

27. Which of the following incomes is not taxable under the head PGBP?
(a) Amount received on keyman insurance policy
(b) Interest received on bonds invested out of business profits
(c) Compensation received on breach of a business contract
(d) Salary received by a partner from the firm

28. An assessee carrying on business is receiving & paying all his transactions through banking channels shall get his books
of account audited where the turnover exceeds:
(c) ₹ 1,00,00,000 (c) ₹ 5,00,00,000
(d) ₹ 2,00,00,000 (d) ₹ 50,00,000

29. Which of the following expenditure does not fall u/s 43B to be allowed on payment basis only?
(a) Bonus or Commission
(b) Interest on loan for purchase of business asset
(c) Employer’s contribution to any provident fund
(d) None of the above

30. A real estate developer sold a flat for ₹ 70 lakhs. The SDV on the DOA & DOR was ₹ 80 lakhs & ₹ 85 lakhs respectively.
What shall be sale value in the hands of the developer u/s 43CA where he received ₹ 6 lakhs as advance in cash on the
DOA?
(a) ₹ 70,00,000 (c) ₹ 85,00,000
(b) ₹ 80,00,000 (d) ₹ 77,00,000

31. Which of the following is not a specified business u/s 35AD?


(a) Beekeeping and production of honey & beeswax
(b) Production of medicines for cure of cancer
(c) Production of fertiliser
(d) Cold chain facility

32. An assessee paid VRS as per the scheme of voluntary retirement amounting to ₹ 6,00,000 on 21st March 2023. How
much deduction of same shall be allowed to assessee for the previous year ending 31st March 2023?
(a) ₹ 6,00,000 (c) ₹ 1,00,000
(b) ₹ 1,50,000 (d) ₹ 1,20,000

33. What amount of deduction shall be allowed at the time of death of animal held otherwise than as stock-in-trade?
(a) Actual cost of animals – amount realised on sale
(b) Actual cost of animals – market value of animals on the date of sale
(c) Actual cost of animals
(d) None of the above

34. In case of company assessee, the total preliminary expenses incurred are allowed as deduction to extent of 5% of:
(a) the cost of the project
(b) the aggregate capital employed
(c) the cost of project or the capital employed whichever is higher
(d) the cost of project or the capital employed whichever is lower

35. Capital expenditure incurred on family planning amongst employees of a partnership firm is allowed as deduction
(a) in full (c) in 5 equal instalments
(b) in 10 equal instalments (d) Not allowed

36. Any expense on advertisement in any souvenir, brochure, tract pamphlet or the like published by a political party......
under head PGBP.
(a) Shall be allowed as deduction (c) Shall not be allowed as deduction
(b) May be allowed as deduction (d) None of the above
CA SHREY RATHI PGBP 5.21

37. Where any sum is paid or payable outside India to foreign resident on which tax has not been deducted. The amount
so paid...................
(a) Is allowed as deduction under the Income Tax Act in the year of payment
(b) Is never allowed as deduction under the Income Tax Act in the year of payment
(c) Is not allowed as deduction but shall be allowed in the year in which TDS is deducted
(d) None of the above

38. If penalty is in nature of compensation then it ……... under section 37(1)


(a) Is allowable as deduction (c) Is not allowable as deduction
(b) May be allowed as deduction (d) depends upon the assessing officer to allow it or not

39. An assessee makes a cash payment of ₹ 32,000 to a transport operator wants to claim deduction of such expenditure.
Is his claim valid?
(a) Valid (c) Invalid
(b) Valid upto ₹ 10,000. Rest shall be disallowed (d) None of the above

40. Calculation what amount is disallowed u/s 40(b) where book profit of the firm is ₹ 5,00,000 Remuneration paid to
working partner is ₹ 4,70,000.
(a) ₹ 4,70,000 (c) ₹ 1,80,000
(b) ₹ 80,000 (d) Nil

41. In case the book profit is negative, the maximum amount of remuneration distributable to partners shall be:
(a) ₹ 1,50,000 (c) ₹ 3,00,000
(b) ₹ 2,70,000 (d) Nil

42. With reference to Section 43B, any payment made after the due date of 1 st year in which the expense was incurred,
then it shall be ………………..
(a) allowed in the year of payment (c) allowed in the year to which the expense relates
(b) never allowed (d) None of the above

43. A person carrying on business shall get his accounts mandatorily audited if his turnover exceeds…
(a) ₹ 50,00,000 (c) ₹ 1,00,00,000
(b) ₹ 2,00,00,000 (d) ₹ 25,00,000

44. A person carrying on specified profession has to maintain prescribed books of accounts if his receipts exceeds
………………. in all the 3 preceding years.
(a) ₹ 50,000 (c) ₹ 1,20,000
(b) ₹ 1,50,000 (d) ₹ 2,00,000

45. An assessee carrying on business on presumptive income basis shall be liable to tax @ ………… of turnover.
(a) 8% of total turnover (c) 6% of total turnover
(b) 8% of bank turnover & 6% of cash turnover (d) 6% of bank turnover & 8% of cash turnover

46. Raju succeeded to the business of his father Ramu consequent to demise of Ramu 1/11/2022. Raju recovered ₹
30,000 due from a customer which was written off by late Ramu as bad debt and allowed as deduction. The amount
recovered is:
(a) Exempt from tax
(b) Fully taxable as business income
(c) ₹ 15,000 being 50% taxable as business income
(d) To be set off against current year bad debts

47. PQR Ltd. is located in a backward area in the State of Andhra Pradesh and acquired some machinery for ₹ 20 lakhs on
10/8/2022. It was put to use from 15/11/2022. How much would be the total depreciation in respect of the said
machinery?
(a) ₹ 3,00,000 (c) ₹ 4,00,000
(b) ₹ 7,00,000 (d) ₹ 3,50,000
CA SHREY RATHI PGBP 5.22

48. Dr. L is practicing MBBS and has gross receipt of ₹ 18,40,000. His presumptive income under section 44ADA would be:
(a) ₹ 1,47,200 (c) ₹ 92,000
(b) ₹ 9,20,000 (d) ₹ 4,60,000

49. Mr. Y has 5 goods carriage vehicles on 1/4/2022. He acquires 3 more vehicles from 11/9/2022. What is the
presumptive income under section 44AE if all are light goods carriage vehicles?
(a) ₹ 8,10,000 (c) ₹ 2,02,500
(b) ₹ 3,64,500 (d) ₹ 6,07,500

50. GC Ltd. engaged in manufacture of biomedicines in August, 2022 converted an equipment which was used for
scientific research purposes previously, for regular business use. The original cost of the plant is ₹ 15 lakhs which was
acquired in April, 2021. The company had claimed deduction at 100%. The plant used for scientific research would be
included in the block of assets now at a value for:
(a) Nil (c) ₹ 15,00,000
(b) ₹ 30,00,000 (d) ₹ 22,75,000

51. KBC Ltd. paid fees for technical services of ₹ 6 lakhs but omitted to deduct tax at source and such omission continued
till the due date for filing the return of income specified in section 139(1). The amount of expenditure liable for
disallowance would be -
(a) ₹ 1,80,000 (c) ₹ 6,00,000
(b) ₹ 1,20,000 (d) Nil

52. Dr. Rishabh has surgical equipment whose WDV as on 1/4/2022 was ₹ 4,10,000. He acquired some more equipment
in December 2022 for ₹ 3,50,000. He sold equipment in March 2023 for ₹ 2,00,000 whose original cost was ₹
1,70,000. The written down value of the block for the purpose of computing depreciation is:
(a) ₹ 5,90,000 (c) ₹ 5,60,000
(b) ₹ 7,30,000 (d) ₹ 4,30,000

53. Where an assessee doing a business incurs any expenditure in respect of which payments made to a person in a day
exceeds ₹ 10,000 should be paid through account payee cheque or demand draft to claim deduction for such
expenditure. This restriction does not apply to
(a) Payments made to RBI
(b) Payments made to cultivators
(c) Payment of terminal benefits to employees not exceeding ₹ 50,000
(d) All of the above

54. KL Industries Ltd. acquired a factory building for self-use in November, 2022. The value of land underneath the
building was ₹ 5 lakhs and value of building was ₹ 10 lakhs. The amount of eligible depreciation is:
(a) ₹ 1,50,000 (c) ₹ 25,000
(b) ₹ 1,00,000 (d) ₹ 50,000

55. Where an asset used for scientific research for more than three years is sold without having been used for other
purposes, then the sale proceeds to the extent of the cost of the asset already allowed as deduction under section 35
in the past shall be treated as—
(a) Business income (c) Long-term capital gain
(b) Short-term capital gain (d) Exempted income

56. PL Ltd. made provision of ₹ 12 lakhs for bonus payable for the year ended 31st March, 2023. It paid ₹ 7 lakhs on 3rd
July 2023, ₹ 3 lakhs on 30th September 2023, and ₹ 3 lakhs on 15th December, 2023. Due date for filing of return is 31st
October 2023. The amount eligible for deduction under section 43B would be:
(a) ₹ 10 lakhs (c) ₹ 12 lakhs
(b) ₹ 7 lakhs (d) ₹ 13 lakhs

57. Provisions of section 44AD for computation of presumptive income are not applicable to -
(a) Limited Liability Partnership (c) Resident Partnership Firm
(b) Resident Hindu Undivided Family (d) Resident Individual
CA SHREY RATHI PGBP 5.23

58. Under the Income Tax Act, 1961, interest on capital received by a partner from a partnership firm is chargeable under
the head—
(a) Profits and gains of business of profession (c) Income from other sources
(b) Capital gains (d) None of the above

59. In the case of companies, capital expenditure incurred for the purpose of promoting family planning amongst the
employees would be deductible to the extent—
(a) Equal to 1/4th in each year for 4 years (c) Equal to 1/6th in each year for 6 years
(b) Equal to 1/5th in each year for 5 years (d) Equal to 1/10th in each year for 10 years

60. The expenditure incurred on payment under voluntary retirement scheme shall be allowed as deduction in
(a) The previous year in which it is paid
(b) Equal instalments in 5 previous years starting from the previous year in which it is paid
(c) Not allowed at all
(d) Allowed to the extent of Rs. 5,00,000

61. If an eligible assesses is engaged in any profession referred to in section 44ADA and he had opted for presumptive
income scheme under section 44ADA, the assesses shall:
(a) Be entitled to deduction under section 30 to 37
(b) Not be entitled to deduction under section 30 to 37
(c) Not be entitled to deduction under section 30 to 37 except on account of interest on capital and loan from a
partner and remuneration to working partner as per section 40(b)
(d) None of the above

62. Where an assesses is carrying on a specified business referred to in section 35AD, he shall be allowed deduction
(a) Only for revenue expenditure
(b) Both the revenue and capital expenditure
(c) Both for revenue and capital expenditure other than goodwill, land and financial instruments
(d) Both for revenue and capital expenditure other than land, building and goodwill

63. In case an assesses is engaged in the business of plying, hiring or leasing goods carriage, presumption income scheme
under section 44AE is applicable if the assesses is the owner of maximum of
(a) 8 goods carriages (c) 10 goods carriages
(b) 12 goods carriages (d) 15 goods carriages

64. Which of following expenditure for which payment is made to a resident are disallowed to the extent of 30% unless
the TDS has been done:
(a) Interest, Royalty, Fee for technical services (c) Commission or brokerage or rent
(b) Payment to contractors (d) All expenses on which is tax in deductible at source

65. In case an eligible assesses is engaged in any business (other than plying, hiring or leasing of goods transport)
presumptive income scheme is applicable if the gross receipts / sales paid or payable to him in the previous year does
not exceed:
(a) ₹ 60 lakhs (c) ₹ 100 lakhs
(b) ₹ 50 lakhs (d) ₹ 200 lakhs

66. An assessee made a payment of ₹ 25,000 as a secret commission, prohibited by law, for some offensive purpose. Such
expenditure shall be considered as:
(a) Revenue expenditure (c) Capital expenditure
(b) Deferred revenue expenditure (d) Non-deductible expenditure

67. The business income of a company assesses before claiming ₹ 760,000 being 1/5th capital expenditure on family
planning is ₹ 740,000. The balance ₹ 20,000 shall be treated as:
(a) Business loss (c) Unabsorbed expenditure on family planning
(b) Terminal depreciation (d) None of the above
CA SHREY RATHI PGBP 5.24

68. Interest on capital from partner of a firm is allowed as deduction to the firm to the extent of:
(a) Rate mentioned in the partnership deed
(b) 12% p.a. even if it is not mentioned in partnership deed
(c) 12% p.a. or at the rate mentioned in partnership deed whichever is less
(d) Any rate at the choice of partner

69. Deduction under section 40(b)shall be allowed on account of salary/remuneration paid to:
(a) Any partner (c) Major partner only
(b) Working partner only (d) None of the above

70. In case the assesses follows mercantile system of accounting, bonus or commission payable to employee shall be
allowed as deduction on:
(a) Due basis
(b) Payment basis
(c) Due or payment basis, at the option of the assesses
(d) Due basis but subject to condition of section 43B.

71. Interest on money borrowed for the purpose of acquiring a capital asset pertaining to the period after the asset is put
to use is to be:
(a) Capitalized
(b) Treated as revenue expenditure
(c) Either capitalized or treated as revenue expenditure at the option of the assessee
(d) Capitalized to the extent of 50% and balance 50% is to be treated as revenue expenditure

72. Mr. W acquired an asset for ₹ 5,90,000 which included ₹ 90,000 as GST for which the assesses has claimed ITC. The
actual cost of acquisition to be included in the block of asset shall be:
(a) ₹ 5,00,000 (c) ₹ 5,90,000
(b) ₹ 6,50,000 (d) None of the above

73. Where the entire block of the asset is sold for a price more than the opening W.D.V. plus cost of assets, if any,
acquired during the year, the excess, amount shall be subject to:
(a) Balancing charge
(b) Short-term capital gain
(c) Long-term on short-term capital gain depending upon the period for which block is held.
(d) None of the above

74. Mr. V is doing agency business and has received a sum of ₹ 5,00,000 from his principal for termination of agency.
Compensation amount so received shall be:
(a) Exempt as it if capital receipt (c) Fully taxable under the head business and profession
(b) Taxable under the head other sources (d) None of the above

75. Mr. R owns two commercial vehicles. One vehicle is heavy good vehicles which can carry weight of 16 tonnes. This
was owned for 9 months and three days. The other vehicle is light goods vehicle which is owned for 11 months & 22
days. What is the income from business of Mr. R if he opts for presumptive scheme under section 44AE?
(a) ₹ 2,50,000 (c) ₹ 4,20,000
(b) ₹ 4,05,000 (d) ₹ 3,90,000

76. YT Ltd. purchased goods on credit from D Ltd. on 7th May 2022 for ₹ 86,000 for which payment of ₹ 5,000 is made in
cash on 12th May 2022; ₹ 40,000 by bearer cheque on 30th May 2022 & Rs. 41,000 by account payee cheque on 13th
June 2022. The amount of disallowance u/s 40A(3) is:
(a) ₹ 15,000 (c) ₹ 45,000
(b) ₹ 40,000 (d) ₹ 86,000

77. Cash payment of ₹ 17,000 & ₹ 6,000 was made by OPL Ltd. against Bill No. 1 & Bill No. 23 respectively to a contractor
engaged in plying of passenger carriage on 29th June 2022. How much amount is disallowed under section 40A(3)?
(a) ₹ 17,000 (c) ₹ 23,000
(b) ₹ 10,000 (d) Nil
CA SHREY RATHI PGBP 5.25

78. Interest on capital & interest on loan received by a partner from a firm is:
(a) Exempt for the partner under section 10(2A)
(b) Taxable for the partner as income from business and profession to the extent it is allowed as deduction to the
firm under section 40(b)
(c) Taxable for the partner as income from business and profession
(d) Taxable for the partner as income from business and profession on account of interest on capital to the extent it
is allowed and income from other sources on account of interest on loan

79. Any sum received by an employer from keyman insurance policy taken on the life of the employee shall be for the
employer as:
(a) Income from Salaries (c) Income from Other Sources
(b) Income from Business & Profession (d) Income from Capital Gains

80. Tax audit is compulsory in case a person is carrying on profession referred under section 44AB whose gross receipts
exceeds:
(a) ₹ 50 lakhs (c) ₹ 100 lakhs
(b) ₹ 150 lakhs (d) ₹ 200 lakhs

81. An Individual who has been carrying on non-specified profession is:


(a) not required to maintain any books of account
(b) required to maintain book of account of the current previous year if the gross receipts of such profession exceed
₹ 1,50,000.
(c) required to maintain books of account of the current previous year if the gross receipts of such profession of any
of three preceding previous year exceeded ₹ 10 lakhs.
(d) required to maintain book of account of the current previous year if in any of the preceding 3 previous years his
total income exceeded ₹ 2,50,000 or gross receipts exceeded ₹ 25 lakhs.

82. In the case of company assessee, the total preliminary expenses incurred are allowed as deduction to extent of 5% of:
(a) the cost of the project
(b) the aggregate capital employed
(c) the cost of project or the capital employed whichever is higher
(d) none of the above

83. Which of the following expenditure on scientific research is not allowed as deduction?
(a) Revenue expenses incurred during the previous year
(b) Revenue expenses on payment of salary to employees engaged in scientific research and purchase of material
used in scientific research incurred during three years immediately preceding the commencement of business
(c) Capital expenditure incurred on scientific research during the year related to the business
(d) Expenditure incurred on acquisition of land during the year for scientific research

84. Deduction of 100% for in-house research shall be allowed for the purchase of:
(a) Any assets (c) Any assets other than land
(b) Any assets other than land and buildings (d) Only Plant and machinery

85. Donation to approved university or college for social or statistical research shall be allowed as deduction to the extent
of
(a) 125% of the donation so made (c) 100% of the donation so made
(b) 175% of the donation so made (d) 200% of the donation so made

86. Brought forward unabsorbed capital expenditure on scientific research can be carried forward:
(a) indefinite period of time (c) 8 years
(b) 5 years (d) 10 years
CA SHREY RATHI PGBP 5.26

87. W.D.V. of block having 5 machines for which depreciation rate is 15% as on 1/4/2022 is ₹ 5,00,000. 1 new machine
amounting to ₹ 1,00,000 was acquired on 1/1/2023 and put to use on 1/2/2023. During the PY 2022-23, 2 old
machinery are sold for ₹ 5,40,000. The depreciation to be allowed for this block shall be:
(a) ₹ 9,000 (c) ₹ 4,500
(b) ₹ 5,000 (d) ₹ 5,400

88. Unabsorbed depreciation brought forward from an earlier year of a particular business can be set off from:
(a) the same business (c) any head of income
(b) any business income (d) any head of income except income from salary

89. Electricity companies are allowed depreciation on the basis of:


(a) Block of asset
(b) Each asset separately
(c) Each asset separately unless the assessee opts for block of asset system in the first previous year of its
commencement.
(d) Either on block of asset or each asset separately provided the option is exercised in the first previous year.

90. Mr. K, a retailer acquired furniture on 10th May 2022 for ₹ 10,000 in cash and on 15th May 2022, for ₹ 15,000 and ₹
20,000 by a bearer cheque and account payee cheque, respectively. Depreciation allowable for PY 2022-23 would be
(ICAI MCQ Booklet)
(a) ₹ 2,000 (c) ₹ 3,000
(b) ₹ 3,500 (d) ₹ 4,500

91. ABC Ltd. incurred capital expenditure of ₹ 1,50,000 on 1/4/2022 for acquisition of patents and copyrights. Such
expenditure is
(a) Eligible for deduction in 10 years from AY 2023-2024
(b) Eligible for deduction in 5 years from AY 2023-2024
(c) Subject to depreciation @ 25% under section 32
(d) Subject to depreciation @ 15% under section 32

92. Which of the following income is not chargeable as income of business or profession?
(a) Profits and gains of business carried by an assessee during the previous year
(b) Income derived by a trade, professional or similar association from specific services performed for its members
(c) Winnings from horse races
(d) Salary received by a partner of a firm from the firm in which he is a partner

93. In case of electricity companies charging depreciation on SLM basis on a single asset if such assets is sold for a price
more than its value but less than its historical cost then the assessee shall be chargeable for:
(a) balancing charge
(b) short-term capital gain
(c) short-term or long-term capital gain depending upon the period after which the block is transferred
(d) balancing charge or capital gains at the choice of the assessee

94. If a new machinery eligible for 15% rate of depreciation is purchased on 15/4/2021 and put to use for the purpose of
the business on 2/1/2023, depreciation would be allowable at the rate of:
(a) 7.5% (c) 15%
(b) 10% (d) 20%

95. Employer’s contribution to provident fund/superannuation fund/gratuity fund is allowed as deduction in computing
income under the head “Profits and gains of business or profession”, provided it has been paid-
(a) Before the end of the previous year
(b) On or before the due date by which the employer is required to credit an employee’s contribution to the
employee’s account in the relevant fund.
(c) On or before the due date for filing the return of income under section 139(1).
(d) Before the end of the relevant assessment year
CA SHREY RATHI PGBP 5.27

96. An assessee uses plant and machinery for the purpose of carrying on his business. Under section 31, he shall be
eligible for deduction on account of-
(a) Both capital and revenue expenditure on repairs
(b) Current repairs
(c) Current repairs plus 1/5th of capital expenditure on repairs.
(d) None of the above

97. The W.D.V. of a block (Plant and Machinery, rate of depreciation 15%) as an 1.4.2022 is ₹ 3,20,000. A second-hand
machinery costing ₹ 50,000 was acquired on 1.9.2022 but put to use on 1.11.2022. During January 2023, part of this
block was sold for ₹ 2,00,000. The depreciation for P.Y. 2022-23 would be- (ICAI MCQ Booklet)
(a) ₹ 21,750 (c) ₹ 25,500
(b) ₹ 21,125 (d) ₹ 12,750

98. Sakshita Ltd., has spent a sum of ₹ 30 lakhs towards meeting its corporate social Responsibility (CSR) obligation. The
amount of deduction available while computing the business income is:
(a) ₹ 30 lakh (c) ₹ 37.5 lakh
(b) Nil (d) ₹ 45 lakh

99. Zing Zang is an individual, manufacturing a product. He has turnover of ₹ 98,50,000 which is inclusive of amount of ₹
25 lakhs received through electronic clearing system. The accounts are not property maintained and you have advised
him to pay tax u/s 44AD of the Act. On how much income he will pay tax for P.Y. 2022-23:
(a) ₹ 7,88,000 (c) ₹ 7,38,000
(b) Manufacturers not allowed u/s 44AD (d) ₹ 5,91,000

100. Which is the charging section of income under the head profits and gains of business or profession?
(a) Section 15 (c) Section 28
(b) Section 24 (d) Section 17

101. An assessee paid insurance premium against risk of damage or destruction of stocks or stores used for the purposes
of his business or profession. Such expenditure shall be considered as;
(a) Revenue expenditure (c) Capital expenditure
(b) Deferred revenue expenditure (d) Illegal expenditure

102. Where an asset is purchased in last year & put to use in the current year, then depreciation shall be:
(a) fully allowed irrespective of its installation
(b) fully allowed only if used for more than 180 days, otherwise 50% of depreciation shall be allowed
(c) 50% of normal depreciation irrespective of its installation
(d) none of the above

103. DLF builders sold a flat for ₹ 5 crores to Mr. Chauhan. The stamp duty value of the property was ₹ 5.20 crores on the
date of agreement, however the stamp duty value on the date of registry was ₹ 5.40 crores. Mr. Chauhan paid ₹ 25
lakhs as advance on the date of booking by account payee cheque. Determine the full value of consideration as per
the relevant provisions of the act.
(a) ₹ 5.40 crores (c) ₹ 5.25 crores
(b) ₹ 5.20 crores (d) ₹ 5 crores

104. Mr. Kumar paid ₹ 40,000 as commission to his brother for securing a business order. He generally pays ₹ 25,000 to
other agents for securing such order. Discuss the validity of the payment.
(a) Whole ₹ 40,000 shall be allowed as expenditure
(b) Amount excessively paid (i.e. ₹ 15,000) shall be disallowed, rest ₹ 25,000 shall be allowed
(c) Whole ₹ 40,000 shall be disallowed
(d) Expenditure shall be allowed to the extent permitted by the assessing officer.
CA SHREY RATHI PGBP 5.28

105. Mr. Sharma acquired a building on 15.12.2020 for ₹ 12,00,000 and was using this building for residential purposes. He
later on decided to use this building for professional purposes on 14.10.2022. Compute his depreciation assuming the
rate of depreciation to be 10%.
(a) ₹ 1,20,000 (c) ₹ 51,300
(b) ₹ 1,02,600 (d) ₹ 60,000

106. Mr. Gupta made payment to a non-resident in the form of salary on which TDS was not deducted in the F/Y 2022-23.
However, TDS was deducted on such amount in the F/Y 2023-24. Discuss in which year Mr. Gupta can claim such
expenditure in his Profit & Loss Account?
(a) F/Y 2022-23
(b) F/Y 2023-24
(c) He cannot claim deduction of this expenditure even if he has deducted TDS later on
(d) The assessing officer will decide the year in which the expenditure can be claimed

107. Where the stock of a businessman is converted into his capital asset, then
(a) the FMV shall be considered to be the sale value
(b) the historical cost shall be considered to be the sale value
(c) the cost shall be reduced from the value of the purchases
(d) none of the above

108. Rate of depreciation for pollution control equipments is:


(a) 15% (c) 10%
(b) 30% (d) 40%

109. Which of the following statement in incorrect?


(a) Interest paid under the Income Tax Act, 1961 shall not be admissible as an expense
(b) Penalty paid under any GST law is an admissible expense
(c) Expenditure on legal proceedings under Custom law shall not be allowed
(d) Provisions/Reverses under Income Tax can be created

110. Which of the following assets are eligible for additional depreciation?
(a) Road transport vehicle (c) Second hand machinery installed in factory
(b) Water cooler installed in office premises (d) Computers installed in factory

111. Expenditure incurred by pharmaceutical companies in the form of gifts/freebies to medical practitioner shall be
(a) allowed as deduction (c) shall not be allowed deduction
(b) partially allowed as determined by the AO (d) 50% allowed & 50% disallowed

112. Mr. Kunal sells printers to Ms. Prachi on credit. Outstanding balance as on 1st April 2022 is ₹ 2,00,000. Sales made
during the year were ₹ 4,50,000. Payment made by Ms. Prachi during the year 2022-23 is ₹ 1,50,000. On 31st March
2022, Kunal writes off ₹ 70,000 as bad debts. However, in November 2022, Mr. Kunal received ₹ 4,60,000 as full &
final payment. Discuss the necessary treatment for the F/Y 2022-23.
(a) ₹ 30,000 shall be treated as bad debts recovered and therefore liable to tax
(b) No treatment shall be done
(c) ₹ 70,000 shall be shown as bad debts but no treatment of recovery
(d) whole ₹ 4,60,000 shall be treated as income and therefore liable to tax

113. Ms. Kiran Pal purchased goods on credit from his brother for ₹ 85,000 where as the market value of such goods was ₹
62,000. The payment was made by bearer cheque on 25th January 2023. Discuss the treatment of such expenditure.
(a) Full ₹ 85,000 shall be allowed as deduction
(b) ₹ 23,000 shall be disallowed u/s 40A(2) but ₹ 62,000 shall be allowed as deduction
(c) ₹ 23,000 shall be disallowed u/s 40A(2) & ₹ 62,000 shall be disallowed u/s 40A(3)
(d) ₹ 62,000 shall be disallowed u/s 40A(3) but ₹ 23,000 shall be allowed as deduction
CA SHREY RATHI PGBP 5.29

114. An asset of ₹ 2,00,000 was used by DFC Ltd. in scientific research. After 3 years, the asset ceased to be used in the
scientific research business and was sold for ₹ 2,40,000. Discuss the treatment of the sale of the scientific research
asset.
(a) ₹ 2,00,000 shall be taxable u/h PGBP & ₹ 40,000 u/h Capital Gains
(b) Whole ₹ 2,40,000 shall be taxable u/h PGBP
(c) Whole ₹ 2,40,000 shall be taxable u/h Capital Gains
(d) ₹ 1,20,000 shall be taxable u/h PGBP & ₹ 1,20,000 u/h Capital Gains

115. Where land or building or both are held as stock-in-trade, then the treatment of such land or building or both shall be
done under
(a) Section 43CA (c) Section 43BA
(b) Section 43AC (d) Section 44BC

116. Advertisement expenditure incurred was ₹ 1,20,000 out of which ₹ 30,000 was paid by cash, ₹ 40,000 through ECS &
the balance by Account Payee DD. The amount of expenditure deductible shall be:
(a) ₹ 40,000 (c) ₹ 1,20,000
(b) ₹ 50,000 (d) ₹ 70,000

117. Donation to Indian Institute of Technology for scientific research is eligible for ……….. deduction.
(a) 50% (c) 100%
(b) 150% (d) 200%

118. DF Ltd. has set up a manufacturing undertaking in the backward areas of Bihar in 2021-22. He has purchased
machinery worth 20 crores on 14th September 2021 but put to use on 20th January 2022. Compute the additional
depreciation for DF Ltd. for the P/Y 2022-23.
(a) ₹ 4 crores (c) Nil
(b) ₹ 1.7 crores (d) ₹ 2 crores

119. A power generating company is claiming depreciation on written down value basis. The block has only 1 asset whose
w.d.v value as on 1st April 2022 is ₹ 2,50,000. The company sold the asset for ₹ 2,20,000. Discuss the treatment
assuming rate of depreciation to be 15%.
(a) Terminal depreciation – ₹ 30,000 (c) Depreciation on ₹ 30,000 @ 15% = ₹ 4,500
(b) Short term capital loss - ₹ 30,000. (d) None of the above

120. PQ Ltd. commenced a specified business u/s 35AD on 25.06.2020 and purchased an asset worth ₹ 20,00,000 and put
to use on the same day. However, this asset was transferred from specified business to normal business on 29th
September 2022. Compute the amount taxable (if any) for PQ Ltd assuming the depreciation rate to be 15%.
(a) ₹ 20,00,000 (c) ₹ 17,00,000
(b) ₹ 14,45,000 (d) Not taxable at all

121. M/s ABC, an eligible assessee, following mercantile system of accounting, carrying on eligible business under section
44AD provides the following details: (ICAI MCQ Booklet)
 Total turnover for the financial year 2022-23 is ₹ 130 lakh
 Out of the above:
- ₹ 25 lakh received by A/c payee cheque during the financial year 2022-23;
- ₹ 50 lakh received by cash during the financial year 2022-23;
- ₹ 25 lakh received by A/c payee bank draft before the due date of filing of return;
- ₹ 30 lakh not received till due date of filing of return.
What shall be the amount of deemed profits of M/s ABC under section 44AD for A.Y. 2023-24?
(a) ₹ 10.4 lakh (c) ₹ 5.5 lakh
(b) ₹ 7.0 lakh (d) ₹ 9.4 lakh
CA SHREY RATHI PGBP 5.30

122. Mr. Shahid, a wholesale supplier of dyes, provides you with the details of the following cash payments he made
throughout the year –
• 12-06-2022: loan repayment of ₹ 27,000 taken for business purpose from his friend Kunal. The repayment also
includes interest of ₹ 5,000.
• 19-08-2022: Portable dye machinery purchased for ₹ 15,000. The payment was made in cash in three weekly
instalments.
• 26-01-2023: Payment of ₹ 10,000 made to electrician due to unforeseen electric circuit at shop
• 28-02-2023: Purchases made from unregistered dealer for ₹ 13,500
What will be disallowance under 40A(3), if any, if Mr. Shahid opts to declare his income as per the provisions of
section 44AD? (ICAI MCQ Booklet)
(a) ₹ 18,500 (c) ₹ 13,500
(b) ₹ 28,500 (d) Nil

123. An electricity company charging depreciation on straight line method on each asset separately, sells one of its
machinery in April, 2022 at ₹ 1,20,000. The WDV of the machinery at the beginning of the year i.e., on 1 st April, 2022
is ₹ 1,35,000. No new machinery was purchased during the year. The shortfall of ₹ 15,000 is treated as – (ICAI MCQ
Booklet)
(a) Terminal depreciation (c) Normal depreciation.
(b) Short-term capital loss (d) Any of the above, at the option of the assessee

124. K is a working partner in a firm on behalf of his HUF and the HUF has contributed ₹ 3,00,000 as its capital
contribution. Apart from this, K has given a loan of ₹ 50,000 to the firm in his individual capacity. The firm pays
interest as per market rate of 15% per annum on capital as well as loan. Compute the amount of interest that shall be
allowed to the firm while calculating its business income assuming that the interest is authorized by the partnership
deed. (ICAI MCQ Booklet)
(a) ₹ 42,000 (c) ₹ 52,500
(b) ₹ 51,000 (d) ₹ 43,500

125. Mr. C, aged 35 years, is a working partner in M/s BCD, a partnership firm, with equal profit-sharing ratio. During the
P.Y. 2022-23, the firm has paid remuneration to Mr. B, Mr. C and Mr. D, being the working partners of the firm, of ₹
2,00,000 each. The firm has paid interest on capital of ₹ 1,20,000 in to all the three partners and the same is within
the prescribed limit of 12%. The firm had a loss of ₹ 1,12,000 after debiting remuneration and interest on capital.
Note – Remuneration and interest on capital is authorized by the partnership deed.
You, being the CA of Mr. C, are in the process of computing his total income. What would be his taxable remuneration
from the firm? (ICAI MCQ Booklet)
(a) ₹ 2,00,000 (c) ₹ 1,27,600
(b) ₹ 1,51,600 (d) ₹ 1,50,000

MCQ’s Answers
1. (c) 2. (a) 3. (d) 4. (c) 5. (c) 6. (b) 7. (c) 8. (a) 9. (d) 10. (a)
11. (a) 12. (d) 13. (a) 14. (c) 15. (a) 16. (b) 17. (b) 18. (a) 19. (c) 20. (c)
21. (c) 22. (c) 23. (b) 24. (a) 25. (a) 26. (b) 27. (b) 28. (c) 29. (d) 30. (c)
31. (b) 32. (d) 33. (a) 34. (c) 35. (d) 36. (c) 37. (c) 38. (a) 39. (a) 40. (b)
41. (a) 42. (a) 43. (c) 44. (b) 45. (d) 46. (b) 47. (d) 48. (b) 49. (d) 50. (a)
51. (a) 52. (c) 53. (d) 54. (d) 55. (a) 56. (a) 57. (a) 58. (a) 59. (b) 60. (b)
61. (c) 62. (c) 63. (c) 64. (d) 65. (d) 66. (d) 67. (c) 68. (c) 69. (b) 70. (d)
71. (b) 72. (a) 73. (b) 74. (c) 75. (a) 76. (b) 77. (d) 78. (d) 79. (b) 80. (a)
81. (d) 82. (c) 83. (d) 84. (c) 85. (c) 86. (a) 87. (c) 88. (d) 89. (d) 90. (c)
91. (c) 92. (c) 93. (a) 94. (c) 95. (c) 96. (b) 97. (a) 98. (b) 99. (c) 100. (c)
101. (a) 102. (a) 103. (d) 104. (b) 105. (b) 106. (c) 107. (a) 108. (d) 109. (b) 110. (d)
111. (c) 112. (a) 113. (c) 114. (a) 115. (a) 116. (d) 117. (c) 118. (d) 119. (b) 120. (b)
121. (d) 122. (d) 123. (a) 124. (d) 125. (c)
CA SHREY RATHI PGBP 5.31

ICAI Case Based MCQs


1. “Lux Enterprises” a proprietorship firm of Mr. Lucifer Mornigstar, a resident individual, in Maharashtra engaged in
business of printing and publishing. The following details pertain to assets of the business:
Particulars Date of purchase Date of put to use Amount
Office building superstructure 30-09-2022 30-12-2022 1,85,00,000
constructed on leased land
BMW M4 convertible car 23-08-2019 25-08-2019 94,80,000
Machineries used in printing and 25-09-2022 15-10-2022 9,12,500
publishing process
Notes:
(1) Car is also used for personal purposes; disallowance for personal use may be taken at 20%.
(2) Written down value of Plant & Machinery (depreciable @ 15%) as on 01-04-2022 is ₹ 1,45,00,000.
Based on the facts of the case scenario given above, choose the most appropriate answer to the following questions,
assuming all the aforementioned assets are purchased through account payee cheque:

1.1 What would be the amount of depreciation allowable on plant and machinery (Depreciable @ 15%) for the
previous year 2022-23?
(a) ₹ 24,25,938 (c) ₹ 24,94,376
(b) ₹ 23,34,688 (d) ₹ 24,03,126

1.2 What would be the WDV of plant and machinery (Depreciable @ 15%) as on 01-04-2023?
(a) ₹ 1,29,86,562 (c) ₹ 1,30,77,812
(b) ₹ 1,29,18,124 (d) ₹ 1,30,09,375

1.3 What would the WDV of Office building superstructure constructed on leased land as on 01-04-2023?
(a) ₹ 1,85,00,000 (c) ₹ 1,75,75,000
(b) ₹ 1,66,50,000 (d) ₹ 1,57,25,000

1.4 What would be the amount of depreciation allowable on BMW M4 convertible car for the previous year 2022-23?
(a) ₹ 22,75,200 (c) ₹ 9,98,758
(b) ₹ 9,75,492 (d) ₹ 7,80,394

MCQ’s Answers
1.1 (b) 1.2 (c) 1.3 (c) 1.4 (c)

2. ABC & Co. is a partnership firm engaged in the business of sale of footwear. The partnership firm consist of three
partners – A, B & C. A & B are working partners and C is a sleeping partner. The firm is liable to tax audit under section
44AB of the Act. It has a book profit of ₹ 11,50,000.
Following payments were made to partners as authorised by the partnership deed:
▪ Remuneration to A & B - ₹ 32,000 p.m. to each partner
▪ Remuneration to C - ₹ 10,000 p.m.
▪ Interest on capital @ 19.5% to A & B - ₹ 18,500 p.a. to each partner
▪ Interest on capital @ 17% to C - ₹ 10,540 p.a.
The firm has following brought forward losses of past years:
A.Y. Business loss Unabsorbed depreciation Long-term capital loss
2020-21 26,000 17,600 5,300
2021-22 78,000 29,860 -
2022-23 1,05,670 54,180 13,470
Based on the facts of the case scenario given above, choose the most appropriate answer to the following questions:
CA SHREY RATHI PGBP 5.32

2.1 What amount of interest is allowable as deduction in the hands of firm while computing profits and gains from
business or profession?
(a) ₹ 29,040 (c) ₹ 47,540
(b) ₹ 22,769 (d) ₹ 30,209

2.2 What amount of remuneration not allowable as deduction in the hands of firm while computing profits and gains
from business or profession?
(a) ₹ 1,20,000 (c) ₹ 1,08,000
(b) Nil (d) ₹ 78,000

2.3 What is the due date of filing of return of income for Mr. A and Mr. C for the A.Y. 2022-23?
(a) 31st July 2023 for Mr. C and 30th September 2023 for Mr. A
(b) 31st July 2023 for Mr. C and 31st October 2023 for Mr. A
(c) 31st October 2023 for both Mr. A & Mr. C
(d) 31st October 2023 for Mr. C and 31st July 2023 for Mr. A

2.4 What would be the income under the head “Profit and gains from business or profession” in the hands of ABC & Co.
for the A.Y. 2023-24?
(a) ₹ 70,690 (c) ₹ 51,920
(b) ₹ 1,72,330 (d) ₹ 1,53,560

MCQ’s Answers
2.1 (d) 2.2 (a) 2.3 (c) 2.4 (b)
CA SHREY RATHI CAPITAL GAINS 6.1

MOTHER TERESA

CHAPTER 6
CAPITAL GAINS

“Kind words can be short and easy to


speak, but their echoes are truly
endless.”

PRACTICAL QUESTIONS
Q 1: Mr. Puneet is a sole proprietor. Due to a fire accident on 25.07.2022, stock worth ₹ 7,00,000 and machinery with
WDV as on 01.04.2022 ₹ 11,00,000 were damaged. During the process of safeguarding his machinery from further
damage, Puneet lost his gold chain worth ₹ 1,20,000 which he purchased in May, 2003.
Mr. Puneet received the following amounts from the insurance company on 19.01.2023:
(i) Towards loss of stock ₹ 5,50,000
(ii) Towards loss of machinery ₹ 8,80,000
(iii) Towards loss of gold chain ₹ 2,50,000
Compute income u/h capital gains for the A/Y 2023-24 for Mr. Puneet. CII for 2003-04: 109; 2022-23: 331.

Sol 1: Computation of capital gains for Mr. Sohan for the A/Y 2023-24:
Particulars Machinery (₹) Gold Chain (₹)
Sale consideration = Amount received as insurance claim 8,80,000 2,50,000
Less: Cost of acquisition for machinery (not indexed, being a depreciable asset) 11,00,000
Less: Indexed cost of acquisition for gold chain (₹ 1,20,000 x 331/109) 3,64,404
STCL / LTCL 2,20,000 1,14,404
Notes: Stock is not a capital asset and therefore shall not be dealt in the capital gains chapter.

Q 2: Mr. Mithun purchased 100 shares of M/s Goodmoney Co. Ltd. on 01.04.2005 at rate of ₹ 1,000 per share in public
issue of the company by paying securities transaction tax.
Company allotted bonus shares in the ratio of 1:1 on 01.12.2021. He has also received dividend of ₹ 10 per share on
01.05.2022.
He has sold all the share on 01.10.2022 at the rate of ₹ 4,000 per share through a recognized stock exchange and paid
brokerage of 1% and securities transaction tax of 0.02% to celebrate his 75 th birthday. The cost inflation index for F/Y
2005-06 and 2022-23 is 117 & 331 respectively.
Compute his income u/h Capital Gains for A/Y 2023-24 assuming the FMV as on 31st January 2018 is ₹ 2,350 per share.
[ICAI Study Module]
CA SHREY RATHI CAPITAL GAINS 6.2

Sol 2: Computation of income u/h Capital Gains of Mr. Mithun for A/Y 2023-24:
Particulars Original Shares Bonus Shares (₹)
(₹)
Period of holding 01.04.2005 – 01.12.2021 –
01.10.2022 01.10.2022
(LTCA) (STCA)
Sale consideration 4,00,000 4,00,000
Less: Brokerage @ 1% 4,000 4,000
Net sale consideration 3,96,000 3,96,000
Less: Cost of acquisition
o Original shares – [WN 1] 2,35,000
o Bonus Shares Nil
LTCG / STCG 1,61,000 3,96,000
WN 1: Cost of Acquisition for Original Shares
COA in relation to LTCA being equity shares on which STT is paid and acquired before 1st February 2018 shall be higher of:
(i) cost of acquisition of such asset; and ₹ 1,00,000
(ii) lower of
(a) the fair market value of such asset as on 31st January 2018; and ₹ 2,35,000
(b) sale value received or receivable as a result of transfer of the capital asset ₹ 4,00,000
Therefore, COA shall be ₹ 2,35,000. However, benefits of indexation shall not be available.
Notes:
(1) Since bonus shares are held for less than 12 months before sale, the gain arising there from is a short-term capital
gains chargeable to tax @ 15% as per section 111A. Further cost of bonus share will be Nil as such shares are allotted
after 01.04.2001.
(2) Brokerage paid is allowable since it is an expenditure incurred wholly and exclusively in connection with the transfer.
However, securities transaction tax is not allowable as deduction.

Q 3: On 21.07.2006, the Government of Haryana acquires a plot of land owned by YZ Ltd. for constructing police
headquarters. On 18.06.2022, YZ Ltd. receives ₹ 18,00,000 as compensation (cost of acquisition in September, 1993 – ₹
3,80,000; FMV as on 01.04.2001 – ₹ 3,75,000). Against the award an appeal is filed by YZ Ltd. and the High Court by its
order dated 08.10.2023 increases the award to ₹ 23,00,000. The additional compensation of ₹ 5,00,000 is received by YZ
Ltd. on 22.12.2023. The following investments are made by YZ Ltd.:
Date of Investment Amount (₹) Investment
18.07.2022 9,00,000 Residential house property
01.01.2024 3,00,000 Bonds of NHAI
Compute income u/h capital gains of YZ Ltd. for the A/Y 2023-24 and 2024-25. CII for F/Y 2006-07: 122; 2022-23: 331.

Sol 3: Computation of capital gains of YZ Ltd. for the A/Y 2023-24 and 2024-25:
A/Y 2023-24: Original compensation
Particulars (₹)
Sale consideration = compensation received on compulsory acquisition 18,00,000
Less: Indexed cost of acquisition (₹ 3,80,000 x 122/100) [WN 1 & 2] 4,63,600
Long-term Capital Gains 13,36,400
A/Y 2024-25: Enhanced compensation
Particulars (₹)
Sale consideration 5,00,000
Less: Indexed cost of acquisition Nil
LTCG before exemption u/s 54EC 5,00,000
Less: Exemption u/s 54EC for investment in the bonds of National Highway Authority of India within
6 months from the date of transfer (Capital Gains or Amount invested whichever is lower) [Note 4] 3,00,000
Long-term Capital Gains 2,00,000
CA SHREY RATHI CAPITAL GAINS 6.3

Notes:
1. Transfer shall conclude at the time of compulsory acquisition of the property. Therefore, indexation shall apply from
F/Y 2006-07 (i.e. 122).
2. Where an asset is acquired before 01.04.2001, then the assessee has the option to take actual cost of acquisition or
FMV as on 01.04.2001 whichever is higher as its cost of acquisition for the purpose of indexation.
3. Exemption u/s 54F is only available to an individual or HUF. Therefore, investment by YZ Ltd. in residential house shall
not be eligible for exemption.
4. In case of compulsory acquisition of the original asset, the period available for investment would be considered from
the date of receipt of such compensation and not from the date of transfer.

Q 4: Rocky enters into a partnership with Vaidik on 15.05.2022 to start hand bags business. The following assets were
introduced by Rocky as his capital contribution to the firm:
S. No. Particulars Land Plant & Machinery
1. Date of acquisition 18.07.1983 22.09.2004
2. Fair market value as on 15.05.2022 ₹ 35,00,000 ₹ 8,50,000
3. Amount recorded in the books of accounts ₹ 33,50,000 ₹ 6,40,000
4. Cost of acquisition ₹ 2,25,000 ₹ 15,80,000
5. WDV of Plant & Machinery as on 01.04.2022 - ₹ 4,20,000
6. FMV as on 01.04.2001 ₹ 6,90,000 -
On 24.03.2023 he purchased a residential house property for ₹ 4,00,000. Compute capital gains for the A/Y 2023-24. CII
for F/Y 2022-23: 331.

Sol 4: Computation of capital gains for Mr. Rocky for the A/Y 2023-24:
Particulars Land (₹) P/M (₹)
Sale consideration = Amount recorded in the books of the firm 33,50,000 6,40,000
Less: Indexed cost of acquisition for land (₹ 6,90,000 x 331/100) 22,83,900
Less: Cost of acquisition for plant & machinery 4,20,000
LTCG / STCG before exemption u/s 54F 11,62,700 2,20,000
Less: Exemption u/s 54F [(Capital Gains x Amount invested) / Net sale
consideration] [(₹ 11,62,700 x ₹ 4,00,000) / ₹ 33,50,000] 1,38,830 -
LTCG / STCG 10,23,870 2,20,000
Notes:
1. Where an asset is acquired before 01.04.2001, then the assessee has the option to take actual cost of acquisition or
FMV as on 01.04.2001 whichever is higher as its cost of acquisition for the purpose of indexation.
2. FMV on the date transfer is irrelevant.
3. Depreciable assets are always short-term capital asset, therefore indexation shall not apply even if they are held for
more than 36 months (except in case of SLM)

Q 5: Mr. Kapil who transfers his land and building on 12.12.2022, furnishes the following information:
(i) Net consideration received – ₹ 12,00,000.
(ii) Value adopted by stamp valuation authority, which was not contested by Mr. Kapil – ₹ 17,00,000.
(iii) Value ascertained by Valuation Officer on reference by Assessing Officer – ₹ 19,00,000.
(iv) Mr. Kapil purchased such land in 1999-00 for ₹ 80,000. FMV of land on 01.04.2001 was ₹ 1,20,000.
(v) A residential building was constructed on such land for ₹ 4,00,000 during the year 2005-06.
(vi) Brought forward short-term capital loss (incurred on sale of shares during the year 2017-18) is ₹ 30,000.
(vii) He purchased bonds of ₹ 40,000 of National Highway Authority of India on 31.01.2023.
Compute income u/h capital gains for Mr. Kapil for the A/Y 2023-24. CII for F/Y 2005-06: 117; 2022-23: 331.
CA SHREY RATHI CAPITAL GAINS 6.4

Sol 5: Computation of income u/h Capital Gains of Mr. Kapil for the A/Y 2023-24:
Particulars (₹)
Sale consideration 17,00,000
[As per section 50C, in case the actual sale consideration is lower than the stamp duty value fixed
by the stamp valuation authority, but the assessee deems it to be more than the FMV, then in such
case, it may be referred to the valuation officer provided the assessee has not disputed under any
court. Then the full value of consideration shall be the value determined by the Stamp Valuation
Authority or value determined by Valuation Officer whichever is lower] (₹ 17,00,000 or ₹ 19,00,000
whichever is lower)
Less: Indexed cost of acquisition (₹ 1,20,000 x 331/100) 3,97,200
Less: Indexed cost of improvement (₹ 4,00,000 x 331/117) 11,31,624
LTCG before exemption u/s 54EC 1,71,176
Less: Exemption u/s 54EC for investment in the bonds of National Highway Authority of India within
6 months from the date of transfer (Capital Gains or Amount invested whichever is lower) 40,000
LTCG on sale land & building 1,31,176
Less: Brought forward short-term capital loss of the F/Y 2016-17 30,000
Net LTCG on sale land & building 1,01,176
Notes:
1. Where an asset is acquired before 01.04.2001, then the assessee has the option to take actual cost of acquisition or
FMV as on 01.04.2001 whichever is higher as its cost of acquisition for the purpose of indexation.
2. As per section 74, short term capital loss can be carry forward and set off against any income under the head “Capital
gains”, be it long term or short term for 8 A/Ys. Therefore, in this case, short term capital loss of ₹ 30,000 can be set
off against long term capital gain of ₹ 1,31,176.

Q 6: Mr. Kumar is the owner of a residential house which was purchased in September, 2004 for ₹ 9,50,000. He sold the
said house on 5th August, 2022 for ₹ 24,00,000. Valuation as per stamp valuation authority of the said residential house
was ₹ 43,00,000. He invested ₹ 5,00,000 in NHAI Bonds on 12th January 2023. He purchased a residential house on 5th July
2023 for ₹ 10,00,000.
You are requested to calculate the taxable income for the A/Y 2023-24. Cost inflation index for F/Y 2004-05 and 2022-23
are 113 and 331 respectively.

Sol 6: Computation of total income of Mr. Kumar for the A/Y 2023-24:
Particulars (₹) (₹)
Sale price of the residential house 43,00,000
[As per Section 50C, where the stamp duty value (i.e. ₹ 43,00,000) exceeds 110% of
the sale consideration (₹ 23,00,000 x 110% = ₹ 25,30,000) shown by the assessee,
the SDV shall be considered as full value of consideration]
Less: Indexed cost of Acquisition (₹ 9,50,000 x 331/113) 27,82,743
LTCG before exemption u/s 54 & 54EC 15,17,257
Less: Exemption u/s 54 (for investment in residential house property) 10,00,000
Less: Exemption u/s 54EC (for investment in NHAI Bonds within 6 months from the
date of transfer) 5,00,000 15,00,000
Long–term Capital Gains 17,257

Q 7: Mr. Roy aged 55 years owned a residential house in Ghaziabad. It was acquired by Mr. Roy on 10.10.2007 for ₹
24,00,000. He sold it for ₹ 65,00,000 on 04.11.2022. The stamp valuation authority of the State fixed value of the property
at ₹ 75,00,000. The assessee paid 2% of the sale consideration as brokerage on sale on the sale of the said property.
Mr. Roy acquired a residential house property at Kolkata on 10.12.2022 for ₹ 7,00,000 and deposited ₹ 3,00,000 on
10.04.2023 and ₹ 5,00,000 on 15.06.2023 in the capital gains bonds of Rural Electrification Corporation Ltd. He deposited
₹ 4,00,000 on 06.07.2023 and ₹ 9,00,000 on 01.11.2023 in the capital gains deposit scheme in a Nationalized Bank for
construction of an additional floor on the residential house property in Kolkata.
Compute the capital gain chargeable to tax for the A/Y 2023-24 assuming he does not have any other income. Cost
Inflation Index for the Financial year 2007-08: 129 and Financial year 2022-23: 331.
CA SHREY RATHI CAPITAL GAINS 6.5

Sol 7: Computation of capital gains chargeable to tax in the hands of Mr. Roy for the A/Y 2023-24:
Particulars (₹) (₹)
Gross Sale Consideration on transfer of residential house 75,00,000
[As per Section 50C, where the stamp duty value (i.e. ₹ 75,00,000) exceeds 110% of the
sale consideration (₹ 65,00,000 x 110% = ₹ 71,50,000) shown by the assessee,
therefore the SDV shall be considered as full value of consideration]
Less: Brokerage @ 2% of actual sale consideration of ₹ 65,00,000 1,30,000
Net sale consideration 73,70,000
Less: Indexed cost of acquisition [₹ 24,00,000 x 331/129] 61,58,140
Long Term Capital Gain before exemption u/s 54 & 54EC 12,11,860
Less: Exemption under section 54
o Acquisition of residential house property at Kolkata on 10.12.2022 (i.e., within the
prescribed time of two years from 04.11.2022, being the date of transfer of 7,00,000
residential house to Ghaziabad).
o Amount deposit in Capital Gains Accounts Scheme on or before the due date of
filing return of income for construction of additional floor on the residential house
property at Kolkata. Since Mr. Roy has no other source of income, his due date for 4,00,000
filing return of income is 31st July 2023.
Therefore ₹ 4,00,000 deposited on 06.07.2023 will be eligible for exemption whereas ₹
9,00,000 deposited on 01.11.2023 will not be eligible for exemption under section 54.
Exemption under section 54EC
o Amount deposited in capital gains bonds of RECL within six months from the date
of transfer (i.e. on or before 03.05.2023) would qualify for exemption.
Therefore, in this case ₹ 3,00,000 deposited in capital gains for exemption under
section 54EC, whereas ₹ 5,00,000 deposited on 15.06.2023 would not quality for
exemption. 3,00,000
Total exemption shall be restricted to LTCG 14,00,000 12,11,860
Long term capital gain Nil

Q 8: Ms. Priya had purchased 500 equity shares in AB Ltd. at a cost of ₹ 30 per share (brokerage 1%) in January, 1996. She
got 50 bonus shares in October, 2000. She again got 550 bonus shares by virtue of her holding in March, 2005. FMV of the
shares of AB Ltd. on 1st April, 2001 was ₹ 50 per share. In January, 2023 she transferred all her shares @ ₹ 240 per share
(brokerage 2%). Compute the capital gains taxable in the hands of Ms. Priya for the P/Y 2022-23 assuming AB Ltd. is an
unlisted company and STT was not applicable at the time of sale. CII for F/Y 2004-05: 113; F/Y 2022-23: 331.

Sol 8: Computation of capital gains in the hands of Ms. Priya for the P/Y 2022-23:
Particulars 500 original 50 bonus 550 bonus
shares (₹) shares (₹) shares (₹)
Gross sale value @ ₹ 240 per share 1,20,000 12,000 1,32,000
Less: Brokerage @ 2% 2,400 240 2,640
Net sale value 1,17,600 11,760 1,29,360
Less: Indexed cost of acquisition
500 original shares: [(500 x ₹ 50) x 331/100] 82,750
50 bonus shares: [(50 x ₹ 50) x 331/100] 8,275
550 bonus shares Nil
Long-term capital gains 34,850 3,485 1,29,360
Total LTCG ₹ 1,67,695
Notes:
1. Where an asset is acquired before 01.04.2001, then the assessee has the option to take actual cost of acquisition or
FMV as on 01.04.2001 whichever is higher as its cost of acquisition for the purpose of indexation.
2. Brokerage paid at the time of purchase shall not be considered as FMV as on 01.04.2001 has been taken as COA for
indexation.
3. COA for bonus shares is nil. But any bonus shares allotted before 01.04.2001 are eligible for FMV as on 01.04.2001.
Therefore, COA for bonus shares acquired in October, 2000 is taken as ₹ 50 per share.
CA SHREY RATHI CAPITAL GAINS 6.6

Q 9: Ms. Harshita purchased a land at a cost of ₹ 35 lakhs in the financial year 2003-04. She started her real estate
business on 1st April, 2011, and converted the said land into stock-in–trade of her business on the said date, when the fair
market value of the land was ₹ 210 lakhs.
She constructed 15 flats of equal size, quality and dimension. Cost of construction of each flat is ₹ 10 lakhs. Construction
was completed in February, 2023. She sold 10 flats at ₹ 30 lakhs per flat in March, 2023. The remaining 5 flats were held in
stock as on 31st March, 2023.
She invested ₹ 50 lakhs in bonds issued by National Highways Authority of India on 31st March, 2023 and another ₹ 50
lakhs in bonds of Rural Electrification Corporation Ltd. in April 2023.
Compute the amount of chargeable capital gain and business income in the hands of Ms. Harshita arising from the above
transaction for Assessment year 2023-24 indicating clearly the reasons for treatment for each item. [Cost Inflation Index:
F/Y 2003-04: 109; FY 2011-12: 184; FY 2022-23: 331] [ICAI Study Module]

Sol 9: Computation of capital gains and business income of Harshita for A/Y 2023-24:
Particulars (₹)
Capital Gains
Fair market value of land on the date of conversion deemed as the full value of consideration for the 2,10,00,000
purpose of section 45(2)
Less; Indexed cost of acquisition [₹ 35,00,000 x 184/109] 59,08,257
1,50,91,743
Proportionate capital gains arising during A/Y 2023-24 [₹ 1,50,91,743 x 2/3] 1,00,61,162
Less: Exemption under section 54EC 50,00,000
Capital gains chargeable to tax for A/Y 2023-24 50,61,162
Particulars (₹)
Business Income
Sale price of flats [10 x ₹ 30 lakhs] 3,00,00,000
Less: Cost of flats = Fair market value of land on the date of conversion [₹ 210 lakhs x 2/3] 1,40,00,000
Less: Cost of construction of flats [10 x ₹ 10 lakhs] 1,00,00,000
Business income chargeable to tax for A/Y 2023-24 60,00,000
Notes:
1. The conversion of a capital asset into stock-in-trade is treated as a transfer under section 2(47). It would be treated as
a transfer in the year in which the capital asset is conversion into stock-in–trade.
2. As per section 45(2), the capital gains arising from the transfer by way of conversion of capital assets into stock-in-
trade will be chargeable to tax only in the year in which the stock-in-trade is sold. However, indexation benefit would
be available only up to the year of conversion of capital asset into stock-in-trade and not up to the year of sale of
stock-in-trade.
3. For the propose of computing capital gains in such cases, the fair market value of the capital asset on the date on
which it was converted into stock-in-trade shall be deemed to be the full value of consideration received or accruing
as a result of the transfer of the capital asset. In this case, since only 2/3 of the stock-in-trade (10 flats out of 15 flats)
is sold in the P/Y 2022-23, only proportionate capital gains (i.e., 2/3rd) would be chargeable to tax in the A/Y 2023-24.
4. On sale of such stock-in-trade, business income would arise. The business income chargeable to tax would be the
difference between the price at which the stock-in-trade is sold and the fair market value on the date of conversion of
the capital asset into stock-in-trade.
5. In case of conversion of capital asset into stock-in-trade and subsequent sale of stock-in-trade, the period of 6 months
is to be reckoned from the date of sale of stock-in-trade for the purpose of exemption u/s 54EC. In this case, since the
investment in bonds of NHAI has been made within 6 months of sale of flats, the same qualifies for exemption under
section 54EC with respect to long-term capital gains arising in any financial year, the maximum deduction under
section 54EC would be ₹ 50 lakhs, whether the investment in bonds of NHAI or RECL are made in the same financial
year or next financial year or partly in the same financial year and partly in the next financial year. Therefore, even
though investment of ₹ 50 lakhs has been made in bonds of NHAI during the P/Y 2022-23 and investment of ₹ 50
lakhs has been made in bonds of RECL during the P/Y 2023-24, both within the stipulated six months period, the
maximum deduction allowable for A/Y 2023-24, in respect of long-term capital gain arising on sale of long-term capital
asset(s) during the P/Y 2022-23, is only ₹ 50 lakhs.
CA SHREY RATHI CAPITAL GAINS 6.7

Q 10: Mr. Malik owns a factory building on which he had been claiming deprecation for the past few years. It is the only
asset in the block. The factory building and land appurtenant thereto were sold during the year. The following details are
available:
Particular (₹)
Building completed in September, 2009 for 10,00,000
Land appurtenant thereto purchased in April, 2002 for 12,00,000
Advance received from a prospective buyer for land in May, 2003, forfeited in favour of assessee, as 50,000
negotiations failed.
WDV of the building block as on 01.04.2022 8,74,800
Sale value of factory building in November, 2022 8,00,000
Sale value of appurtenant land in November, 2022 40,00,000
The assessee is ready to invest in long term specified assets under section 54EC, within specified time.
Compute the amount of taxable capital gain for the A/Y 2023-24 and the amount to be invested under section 54EC for
availing the maximum exemption. CII for F/Y 2002-03: 105; F/Y 2003-04: 109; F/Y 2022-23: 331.

Sol 10: Computation of taxable capital gain of Mr. Malik for A/Y 2023-24:
Particulars (₹) (₹)
Factory Building:
Sale price of building 8,00,000
Less: WDV as on 01.04.2022 8,74,800
Short–term capital loss on sale of building (-) 74,800

Land appurtenant to the above building:


Sale value of land 40,00,000
Less: Indexed cost of acquisition (₹ 11,50,000 x 331/105) [WN 1] 36,25,238
Long–term capital gains on sale of land 3,74,762
Net long-term capital gain 2,99,962
Investment under section 54EC:
Exemption under section 54EC is available if the capital gains arising from transfer of a long–term capital asset are
invested in bonds of National Highways Authority of India and Rural Electrification Corporation Ltd. within 6 months from
the date of transfer. As per section 54EC, the amount to be invested for availing the maximum exemption is the net
amount of capital gain arising from transfer of long-term capital asset, which is ₹ 2,99,962 in this case.
Notes:
1. Where advance money has been received by the assessee, and retained by him, as a result of failure of the
negotiation, section 51 will apply. The advance retained by the assessee will reduce the cost of acquisition. Indexation
is to be done on the cost of acquisition so arrived at after reducing the advance money forfeited i.e. ₹ 12,00,000 – ₹
50,000 = ₹ 11,50,000. It may be noted that in cases where the advance money is forfeited on or after 01.04.2014, the
amount so forfeited would be taxable under the head “Income from Other Sources” and such amount will not be
deducted from the cost of acquisition of such asset while calculating capital gains.
2. Factory building on which depreciation has been claimed, is a depreciable asset. Profit/loss arising on sale is deemed
to be short term capital gain/loss as per section 50, and no indexation benefit is available.
3. Land is not a depreciable asset, hence section 50 will not apply. Being a long-term capital asset (held for more than 24
months), indexation benefit is available.
4. As per section 74, short term capital loss can be set off against any income under the head “Capital gains”, be it long
term or short term. Therefore, in this case, short term capital loss of ₹ 74,800 can be set off against long term capital
gain of ₹ 3,74,762.

Q 11: Compute the amount of capital gains for the P/Y 2022-23 from the following information:
(i) Mr. Harmeet sold his car for ₹ 1,97,000 on 28.09.2022. It was purchased by him 21.07.2011 for ₹ 6,98,000. He
spent ₹ 33,000 by way of capital repairs on it on 31.01.2017.
CA SHREY RATHI CAPITAL GAINS 6.8

(ii) He had applied for 5,000 shares of AB Ltd. and paid ₹ 5 per share at the time of application. The shares were
allotted to him on 17.06.2022. These shares were forfeited by AB Ltd. on 15.11.2022 as he failed to pay the call
money.
(iii) He holds 5,000 shares of XY Ltd. which he sold on 20.10.2022 for ₹ 200 per share and paid brokerage of ₹ 4,000.
Out of the 5,000 shares sold by him, 2,500 shares were purchased by him on 19.02.2001 for ₹ 1,00,000. The
balance shares were bonus shares which were allotted to him on 19.12.2012. FMV as on 01.04.2001 was ₹ 48 per
share.
(iv) He purchased long term specified assets notified by the Central Government for ₹ 3,00,000 from the above sale
proceed of shares on 30.12.2022.
Assume Harmeet is not a dealer in shares and such shares are unlisted shares and the car was for his personal use.

Sol 11: Computation of capital gains for Mr. Harmeet for the A/Y 2023-24:
Particulars 5000 forfeited 2500 original 2500 bonus
shares (₹) shares (₹) shares (₹)
Gross Sale Value: Original & Bonus shares (2500 x ₹ 200) each Nil 5,00,000 5,00,000
Less: Brokerage (divided proportionately) - 2,000 2,000
Net Sale Value Nil 4,98,000 4,98,000
Less: Cost of acquisition for forfeited shares (5,000 x ₹ 5) 25,000
Less: Indexed cost of acquisition for original shares (2,500 x ₹ 48
x 331/100) 3,97,200 -
Short-term capital loss / Long-term capital gains (25,000) 1,00,800 4,98,000
Long-term capital gains 5,73,800
Less: Exemption u/s 54EE for investments in the long term
specified assets notified by the Central Government 3,00,000
Income u/h Capital Gains 2,73,800
Notes:
1. Where an asset is acquired before 01.04.2001, then the assessee has the option to take actual cost of acquisition or
FMV as on 01.04.2001 whichever is higher as its cost of acquisition for the purpose of indexation.
2. COA for bonus shares is nil as assessee has not incurred any cost while acquiring such shares.
3. Short-term capital loss can be set off against short-term capital gains or long-term capital gains u/s 74. As there is a
long-term capital gains of ₹ 5,98,800, such loss shall be set off from such gains.

Q 12: Mr. S sold on 30.10.2022 an urban agricultural land, which he has been using for agricultural purposes for several
years for ₹ 32,50,000. He had acquired that land in April, 2007 for ₹ 11,14,000.
He purchased rural agricultural land for ₹ 2,00,000 on 28.02.2023 which was sold for ₹ 3,00,000 on 15.06.2023.
Further, a sum of ₹ 5,00,000 was invested by him in purchase of residential property on 26.05.2023. He owned only one
house property before this date. The new house property was sold on 31.08.2023 for ₹ 6,00,000.
Compute capital gains for the A/Y 2023-24 and A/Y 2024-25. CII for F/Y 2007-08: 129; F/Y 2022-23: 331.

Sol 12: Computation of capital gains for Mr. S for the A/Y 2023-24 and A/Y 2024-25:
A/Y 2023-24:
Particulars (₹)
Sale value 32,50,000
Less: Indexed cost of acquisition (11,14,000 x 331/129) 28,58,403
Long-term Capital Gains 3,91,597
Less: Exemption u/s 54B for purchase of rural agricultural land = Capital gains or amount invested
whichever is lower (₹ 3,91,597 or ₹ 2,00,000 whichever is lower) 2,00,000
Less: Exemption u/s 54F [(Capital Gains x Amount invested) / Net sale consideration] [(₹ 3,91,597 x ₹
5,00,000 / ₹ 32,50,000)] 60,246
Income u/h Capital Gains 1,31,351
CA SHREY RATHI CAPITAL GAINS 6.9

A/Y 2024-25:
(i) Sale of rural agricultural land for 3,00,000 on 15.06.2023 will not have any tax implications as rural agricultural land is
not a capital asset.
(ii) Sale of new house property for 6,00,000 on 31.08.2023 will have the following tax implications:
(a) LTCG exempted in A/Y 2023-24 shall be taxable in the A/Y 2024-25, being ₹ 60,246.
(b) STCG = Sale value – cost of acquisition = ₹ 6,00,000 – ₹ 5,00,000 = ₹ 1,00,000.

Q 13: Mr. Arvind purchased 500 listed equity shares at ₹ 70 each on 14.08.2005. On 23.09.2008 he was given bonus shares
by the company on the basis of 2 shares for every 5 shares held. On 07.04.2022 he was given a right share to acquire 300
shares @ ₹ 50 per share. He acquired 2/3rd of the right shares offered and sold the balance 1/3rd @ ₹ 20 per share on
15.05.2022. The right shares were allotted to him on 29.07.2022. All the shares were sold on 20.03.2023 @ 160 each.
Assume shares are not sold through recognised stock exchange.
Compute capital gains of Mr. Arvind for the P/Y 2022-23. CII for F/Y 2005-06: 117; F/Y 2008-09: 137; F/Y 2022-23: 331.

Sol 13: Computation of capital gains of Mr. Arvind for the A/Y 2023-24:
Particulars 500 Original 200 Bonus 200 Acquired 100 renounced
shares (₹) shares (₹) right shares (₹) right shares (₹)
(LTCA) (LTCA) (STCA) (STCA)
Sale value [₹ 160 per share / ₹ 20 per shares 80,000 32,000 32,000 2,000
for renounced right shares]
Less: Indexed cost of acquisition / COA
o Original shares [(500 x ₹ 70) x 331/117) 99,017
o Bonus shares Nil
o Right shares acquired: (200 x ₹ 50) 10,000
o Right shares renounced Nil
LTCL/LTCG/STCG (-) 19,017 32,000 22,000 2,000
Net LTCG/STCG 12,983 24,000
Note: Cost of acquisition for bonus shares and renounced right shares shall ne nil as assessee has not incurred any cost
while acquiring such shares.

Q 14: Mr. Janardan inherited a house in Jodhpur under will of his father in June, 2006. The house was purchased by his
father in October, 1987 for ₹ 2,45,000. He invested an amount of ₹ 7,00,000 in construction of one more floor in this
house in July, 2008. The house was sold by him in December, 2022 for ₹ 37,50,000. The valuation adopted by the stamp
value authority for charge of stamp duty was ₹ 47,25,000 which was not contested by the buyer, but as per assessee’s
requests, made a reference to the Valuation Officer. The value determined by the valuation officer was ₹ 49,70,000.
Brokerage @ 1% of sale consideration was paid on sale. The market value of house as on 01.04.2001 was ₹ 4,70,000.
You are required to compute capital gain chargeable to tax for A/Y 2023-24. [CII for F/Y 2008-09: 137; F/Y 2022-23: 331]

Sol 14: Computation of capital gains for Mr. Janardan for the A/Y 2023-24:
Particulars (₹)
Gross sale consideration 47,25,000
[As per section 50C, in case the actual sale consideration is lower than the stamp duty value fixed by
the stamp valuation authority, but the assessee deems it to be more than the FMV, then in such case,
it may be referred to the valuation officer provided the assessee has not disputed under any court.
Then the full value of consideration shall be value determined by the Stamp Valuation Authority or
value determined by Valuation Officer whichever is lower] (₹ 47,25,000 or ₹ 49,70,000 whichever is
lower)
Less: Brokerage @ 1% of actual sale consideration (₹ 37,50,000 x 1%) 37,500
Net sale consideration 46,87,500
Less: Indexed cost of acquisition (₹ 4,70,000 x 331/100) 15,55,700
Less: Indexed cost of improvement (₹ 7,00,000 x 331/137) 16,91,241
Income u/h Capital Gains “Long-term capital gains” 14,40,559
CA SHREY RATHI CAPITAL GAINS 6.10

Notes:
1. Where an asset is acquired before 01.04.2001, then the assessee has the option to take actual cost of acquisition or
FMV as on 01.04.2001 whichever is higher as its cost of acquisition for the purpose of indexation.
2. Brokerage shall be computed on the actual sale consideration declared by the assessee and not on the value
determined as per section 50C.

Q 15: Mr. Dewang submits you the following information about the assets he sold during the P/Y 2022-23:
S. No. Particulars Date of Cost (₹) FMV as on Date of sale Sale price (₹)
Acquisition 1.4.2001 (₹)
1. Urban Agricultural 01.08.1994 1,70,000 3,10,000 15.05.2022 16,50,000
Land
2. Shares (Listed) 12.11.2021 50,000 - 19.07.2022 90,000
3. Jewellery 28.03.2004 1,30,000 - 22.10.2022 4,80,000
4. House Property 09.12.1991 8,00,000 18,00,000 18.03.2023 62,00,000
He deposited a sum of ₹ 6,00,000 on 22.02.2023 out of the sale of agriculture land in the capital gain scheme as he
intended to buy another agricultural land. Out of the sale proceeds of house property, he invested ₹ 1,50,000 on the
purchase of another residential house property on 15.05.2023. The shares were sold through recognised stock exchange.
Compute capital gains for the A/Y 2023-24. [CII for F/Y 2003-04: 109; 2022-23: 331]

Sol 15: Computation of capital gains for Mr. Dewang for the A/Y 2023-24:
Particulars Urban Agri. Listed shares Jewellery (₹) House Property
Land (₹) (₹) (LTCA) (₹)
(LTCA) (STCA) (LTCA)
Sale value 16,50,000 90,000 4,80,000 62,00,000
Less: Indexed cost of acquisition / cost of
acquisition
Urban Agri. Land: (₹ 3,10,000 x 331/100) 10,26,100
Listed shares 50,000
Jewellery (₹ 1,30,000 x 331/109) 3,94,771
House Property: (₹ 18,00,000 x 331/100) 59,58,000
LTCG/STCG 6,23,900 40,000 85,229 2,42,000
Less: Exemption u/s 54B for investment in
agricultural land 6,00,000
Less: Exemption u/s 54 for investment in new
residential house property 1,50,000
Net LTCG/STCG 23,900 40,000 85,229 92,000
Total LTCG 2,01,129
Total STCG 40,000

Note: Where an asset is acquired before 01.04.2001, then the assessee has the option to take actual cost of acquisition or
FMV as on 01.04.2001 whichever is higher as its cost of acquisition for the purpose of indexation.

Q 16: Mr. T sells non-listed shares of a company on 27.11.2022 for ₹ 16,00,000 whose cost of acquisition when acquired
on 11th July, 2010 was ₹ 3,75,000. On 1st July, 2023, he invested ₹ 9,00,000 in the bonds of NHAI for claiming exemption
u/s 54EC. On 31st July, 2024, he deposits ₹ 13,00,000 in a bank account u/s 54F for construction of a house in future.
Construction of the house is completed on 12th October, 2024. Total investment of ₹ 12,50,000 is financed by withdrawing
from the deposit account.
Determine the income for the A/Y 2023-24 & 2025-26 assuming T does not own any other house. [CII for 2010-11: 167;
2022-23: 331]
CA SHREY RATHI CAPITAL GAINS 6.11

Sol 16: Computation of income of Mr. T for the A/Y 2023-24 & A/Y 2025-26:
A/Y 2023-24:
Particulars (₹)
Sale value 16,00,000
Less: Indexed cost of acquisition (₹ 3,75,000 x 331/167) 7,43,263
Long-term Capital Gains 8,56,737
Less: Exemptions u/s 54EC & 54F [WN 1] -
Income u/h Capital Gains – LTCG 8,56,737

WN 1: Exemption u/s 54EC for investment in the bonds of NHAI shall be made within 6 months from the date of transfer
(i.e. till 26.05.2023), which didn’t happen as Mr. T purchased such bonds on 1st July, 2023. Therefore, exemption u/s 54EC
shall not be allowed.
Exemption u/s 54F for investment in residential house property shall be allowed where an assessee deposits with Capital
Gains Account Scheme on or before the due date of filing of return of income (i.e. on or before 31.07.2023). Mr. T
deposited on 31st July, 2024 and therefore shall not be allowed exemption u/s 54F.

A/Y 2025-26: As no exemption u/s 54F was allowed in the F/Y 2022-23, it would have become taxable in the year 2022-23
itself. Therefore, there shall be no tax implications in the year in which such amount is withdrawn from the Capital Gains
Account Scheme.

Q 17: Mr. A purchased a house property on 18th March, 1997 for ₹ 1,05,000. He entered into an agreement with Mr. B for
the sale of house property on 14th October, 2002 and received an advance of ₹ 25,000. However, since Mr. B did not remit
the balance amount, Mr. A forfeited the advance. Later on, he gifted the house property to his brother Mr. C on 22nd May,
2004.
Following renovations were carried out by Mr. A and Mr. C to the house property:
By Mr. A during the F/Y 1999-00 ₹ 10,000
By Mr. A during the F/Y 2003-04 ₹ 50,000
By Mr. C during the F/Y 2004-05 ₹ 1,90,000
The FMV of the property as on 01.04.2001 is ₹ 4,50,000.
Mr. C entered into an agreement with Mr. D for the sale of the house property on 11 th December, 2006 and received an
advance of ₹ 80,000. The said amount was forfeited by Mr. C since Mr. D could not fulfil the terms of the agreement.
Finally, the house was sold by Mr. C to Mr. E on 12th February, 2023 for a consideration of ₹ 25,00,000.
Compute the capital gains chargeable to tax in the hands of Mr. C for the P/Y 2022-23. CII for F/Y 2003-04: 109; 2004-05:
113; F/Y 2022-23: 331]
Sol 17: Computation of capital gains chargeable for Mr. C for the A/Y 2023-24:
Particulars (₹) (₹)
Sale consideration 25,00,000
Less: Indexed cost of acquisition [(₹ 4,50,000 – ₹ 80,000) x 331/100] 12,24,700
Less: Indexed cost of improvement
• 2003-04 (₹ 50,000 x 331/109) 1,51,835
• 2004-05 (₹ 1,90,000 x 331/113) 5,56,549 7,08,384
Income u/h Capital Gains (LTCG) 5,66,916
Notes:
1. Where an asset is acquired before 01.04.2001, then the assessee has the option to take actual cost of acquisition or
FMV as on 01.04.2001 whichever is higher as its cost of acquisition for the purpose of indexation.
2. Cost of improvement incurred before 01.04.2001 shall be ignored as FMV as on 01.04.2001 is allowed to be taken as
Cost of Acquisition
3. Advance money forfeited by the previous owner is irrelevant, therefore advance money forfeited by A (i.e. ₹ 25,000 is
irrelevant)
4. Advance money forfeited by the current owner shall be deducted from the cost and then the balance shall be
indexed.
CA SHREY RATHI CAPITAL GAINS 6.12

Q 18: Mr. Martin, a resident individual, sold his residential house property on 08.06.2022 for ₹ 70 lakhs which was
purchased by him for ₹ 20,50,000 on 05.05.2006.
He paid ₹ 1 lakhs as brokerage for the sale of said property. The stamp duty valuation assessed by sub- registrar was ₹ 82
lakhs.
He bought another house property on 25.12.2022 for ₹ 15 lakhs.
He deposited ₹ 5 lakhs on 10.11.2022 in the capital gain bond of National Highway Authority of India (NHAI).
He deposited another ₹ 10 lakhs on 10.07.2023 in the capital gain deposit scheme with SBI for construction of additional
of additional floor of house property.
Compute income under the head “Capital Gains” for A/Y 2023-24. CII for F/Y 2006-07: 122 and 2022-23: 331.

Sol 18: Computation of income u/h Capital Gains for Mr. Martin for the A/Y 2023-24:
Particulars (₹) (₹)
Sale Consideration
[As per Section 50C, where the stamp duty value (i.e. ₹ 82,00,000) exceeds 110% of the 82,00,000
sale consideration (₹ 70,00,000 x 110% = ₹ 77,00,000) shown by the assessee, therefore
the SDV shall be considered as full value of consideration]
Less: Expenses in connection with transfer (brokerage paid for sale of property) 1,00,000
Net sale consideration 81,00,000
Less: Indexed cost of acquisition [₹ 20,50,000 x 331/122] 55,61,885
Long-term Capital Gains before exemptions u/s 54 & 54EC 25,38,115
Less: Exemption under section 54:
o Purchase of new residential house property within two years from the date of sale of 15,00,000
residential house
o Deposit in capital Gains Accounts Scheme on or before the due date of filing of return
of income u/s 139(1) for construction floor on such house property. 10,00,000 25,00,000
Less: Exemption under section 54EC:
Investment in capital gains bond of NHAI within 6 months from the date of transfer (i.e.
before 08.12.2022) (limited to ₹ 38,115) 38,115
Long-term Capital Loss Nil
Note: Exemption u/s 54 is available in respect of reinvestment of capital gains on sale of residential house in one
residential house in India. In this case, exemption would be available for amount invested in purchase of new residential
house and amount deposited for construction of addition floor in the same house, since they together constitute one
residential house.

MULTIPLE CHOICE QUESTIONS


1. Capital asset excludes all assets except
(a) Stock in trade (c) Jewellery
(b) Personal effect (d) Rural agricultural land

2. Cost of acquisition in case of bonus shares allotted before 1/4/2001 will be:
(a) Face value on the date of allotment (c) Market value as on 1/4/2001
(b) Nil (d) Current market value

3. Mr. KP has received a sum of ₹ 3,40,000 as interest on enhanced compensation for compulsory acquisition of land by
State Government in May, 2022, of this, only ₹ 12,000 pertains to the current year and the rest pertains to earlier
years. The amount chargeable to tax for the PY 2022-2023 u/h Capital Gains would be:
(a) ₹ 12,000 (c) ₹ 3,40,000
(b) Nil (d) ₹ 1,70,000
CA SHREY RATHI CAPITAL GAINS 6.13

4. For claiming exemption under section 54EC, an assessee has to invest the resultant capital gains within a specified
period. Which of the following is not eligible for such investment?
(a) National Highway Authority of India Limited (c) Bonds of NABARD
(b) Rural Electrical Corporation of India Limited (d) None of the above

5. Long term capital gains on zero coupon bonds are chargeable to tax
(a) @ 20% computed after indexation of such bonds
(b) @ 10% computed without indexation of such bonds
(c) Higher of (a) or (b)
(d) Lower of (a) or (b)

6. Which of the following is not required for charging income tax on capital gains?
(a) The transfer must have been affected in the relevant assessment year
(b) There must be a gain arising on transfer of capital asset
(c) Capital gains should not be exempt under section 54
(d) Capital gains should not be exempt under section 54EC

7. In order to enjoy exemption under section 54EC, the resultant long-term capital gains should be invested in specified
bonds within a period of …………….. from the date of transfer
(a) 36 months (c) 6 months
(b) 4 months (d) 12 months

8. Residential house is sold for ₹ 90 lakhs and the long-term capital gains computed are ₹ 50 lakhs. The assesses bought
two residential houses for ₹ 30 lakhs and ₹ 20 lakhs respectively. The amount eligible for exemption under section 54
would be:
(a) ₹ 50 lakhs (c) ₹ 30 lakhs
(b) ₹ 20 lakhs (d) Nil

9. Mr. SJ entered into an agreement with Mr. D for sale of a building for ₹ 20 lakhs in June, 2020. Mr. SJ received
advance of ₹ 2 lakhs. Subsequently the agreement was cancelled and Mr. SJ forfeited the advance money. The
advance money is:
(a) To be reduced from the cost of acquisition
(b) To be reduced from indexed cost of acquisition
(c) Taxable as capital gains
(d) Taxable as income under the head income from other sources

10. Under which section the assesses has to reinvest the entire amount of net consideration to claim full exemption for
the long-term capital gains earned during a previous year
(a) Section 54EC (c) Section 54B
(b) Section 54F (d) Section 54D

11. Cost of acquisition of securities held with depository is to be computed by


(a) Average cost method (c) Last in first out (LIFO)
(b) First in first out (FIFO) method (d) Weighted average cost method

12. Long term capital gains on sale of a long-term capital asset on 15th October, 2022 is ₹ 105 lakh. The assessee invested
₹ 50 lakhs in RECI bonds on 31s1 March, 2022 and ₹ 55 lakhs in NHAI bonds on 18th May, 2022. The amount of
exemption eligible under section 54EC is:
(a) Nil (c) ₹ 55 Lakhs
(b) ₹ 50 lakhs (d) ₹ 105 lakhs

13. Mr. G purchased a car for his personal use for ₹ 5,00,000 in April, 2022 and sold the same for ₹ 5,50,000 in July, 2022.
The taxable capital gains would be:
(a) Nil (c) ₹ 50,000
(b) ₹ 5,50,000 (d) ₹ 4,00,000
CA SHREY RATHI CAPITAL GAINS 6.14

14. On 1/6/2022 Mr. R transferred his vacant land to Mr. D for ₹ 12 lakhs. The land was acquired on 1/9/2019 for ₹ 3
lakhs. If indexation is applied, the indexed cost of acquisition would be ₹ 3.30 lakhs. The taxable capital gain would be:
(a) Long term capital gain ₹ 8.70 lakh (c) Long term capital gain ₹ 9 Lakh
(b) Short term capital gain ₹ 9 lakhs (d) Short term capital gain ₹ 8.70 lakhs

15. Short-term capital gain is gain arising from the transfer of a land and building which is held by the assessee for not
more than:
(a) 36 months from the date of its acquisition (c) 24 months from the date of its acquisition
(b) 12 months from the date of its acquisition (d) 48 months from the date of its acquisition

16. Distribution of assets by a partnership at the time dissolution of firm shall be regarded as a transfer and subject to
capital gain:
(a) in the hands of the firm
(b) in the hands of the partners
(c) in the hands of both firm as well as partners
(d) neither in the hands of a firm nor in the hands of partners

17. Distribution of assets at the time of complete partition of H.U.F. shall


(a) be regarded as a transfer in the hands of H.U.F. for capital gain purpose
(b) be regarded as a transfer in the hands of coparceners
(c) not be regarded as transfer in the hands of H.U.F.
(d) neither be regarded as transfer in hands H.U.F. nor in the hands of coparceners

18. Cost of improvement of tenancy rights, route permits or loom hours shall be:
(a) always taken as Nil
(b) always considered irrespective of period when it was incurred
(c) considered when incurred on or after 1/4/2001
(d) considered when incurred before 1/4/2001

19. Conversion of capital asset into stock in trade will result into capital gain of the previous year:
(a) in which such conversion took place
(b) In which such converted asset is sold or transferred
(c) Both of the above
(d) none of the above

20. Where a partner transfers any capital asset into the business of firm the sale consideration of such asset to the
partner shall be:
(a) market value of such asset on the date of such transfer
(b) price at which it was recorded in the books of the firm
(c) cost of such asset to the partner
(d) price which is mutually decided by partner

21. If the goodwill of a business, right to manufacture or produce, tenancy rights, route permit or loom hours is acquired
before 1/4/2001 the cost of acquisition of such asset shall be:
(a) Cost for which it was acquired by the assessee
(b) FMV as on 1/4/2001
(c) Always taken as Nil
(d) Higher of (a) or (b)

22. The cost of acquisition of the shares given under the scheme of employee stock option plan shall be:
(a) FMV of the equity shares which has been taken into account for the purpose of valuation of perquisite.
(b) FMV of the equity shares which has been taken into account for the purpose of valuation of perquisite as reduced
by the amount paid by the employee.
(c) always taken as Nil
(d) Price at which it was offered to employee
CA SHREY RATHI CAPITAL GAINS 6.15

23. The cost of acquisition of the right shares to a person who purchased the right to acquire share from the existing
shareholder shall be:
(a) market value of right shares are offered
(b) price at which these shares are offered
(c) price at which these shares are offered plus the amount paid to the person renouncing the right
(d) always taken as NIL

24. If any advance money received by the assessee under the agreement of transfer which could not be matured is
forfeited before 1/4/2014 then such money shall
(a) Be taxable as the income of other sources in the year it is forfeited
(b) Be deducted from the cost of acquisition of such asset after doing indexation
(c) Be deducted from the cost of acquisition of such asset before doing indexation
(d) it shall be ignored in all cases

25. Where a capital asset other than urban agricultural land is compulsorily acquired then the capital gain shall arise in
the previous year:
(a) of compulsory acquisition (c) in which part or full consideration is received
(b) in which full consideration is received (d) in any year at the discretion of the government

26. In the case of compulsory acquisition, the indexation of cost of acquisition or cost of improvement shall be done till
the:
(a) Previous Year of compulsory acquisition (c) in which part or full compensation received
(b) In which the full compensation received (d) in any year at the discretion of the government

27. In case of compulsory acquisition, if an assessee receives enhanced compensation then enhanced compensation is
taxable as:
(a) Short-term capital gain
(b) long-term capital gain
(c) short term or long term capital gain depending upon the original capital gain of compulsory acquisition
(d) any kind of capital gains to be decided by the government

28. In case of compulsory acquisition if initial compensation or enhanced compensation is received by legal heir due to
death of assessee, then capital gain shall:
(a) not be taxable in the hands of legal heir
(b) taxable for the dead assessee
(c) be taxable in the hands of legal heir
(d) for initial compensation the legal heir will be taxable as representative assessee and for enhanced compensation
he shall be himself taxable.

29. For claiming exemption under section 54, the assesses should purchase residential property:
(a) 2 years after the date of transfer
(b) 3 years after the date of transfer
(c) within one year before or two years after the date of transfer
(d) one year before and 3 years after the date of transfer

30. If the assessee wishes to deposit money under capital gain scheme for claiming exemption under section 54, it should
be deposited:
(a) within six months from the date of transfer
(b) within six months from the end of the relevant previous year
(c) within the due date of furnishing the ITR u/s 139(1)
(d) within six months or within due date of furnishing the ITR, whichever is earlier

31. If a new house property for which exemption was claimed under section 54 is transferred within 3 years:
(a) Capital gain exempt under section 54 earlier shall be separately taxable as capital gains
(b) The entire capital gain on new transfer shall be taxable
(c) Capital gain exempt under section 54 earlier shall be reduced from cost of acquisition of new house property
(d) Capital gain exempt under section 54 earlier shall be added to the cost of acquisition of new house property
CA SHREY RATHI CAPITAL GAINS 6.16

32. The exemption u/s 54B is allowed to:


(a) Any assessee (c) Individual or HUF both
(b) Individual only (d) HUF only

33. For claiming exemption under section 54B the assessee should acquire:
(a) urban agricultural land (c) any of (a) or (b)
(b) rural agricultural land (d) none of (a) or (b)

34. Exemption under section 54EC shall be available for transfer of:
(a) Any long-term capital asset
(b) Residential house property
(c) Land or building or both
(d) Any long term capital asset other than residential house property

35. Under section 54EC the assessee shall be allowed exemption:


(a) of capital gain invested subject to maximum of ₹ 50 lakhs per financial year
(b) proportionate to the net consideration price invested
(c) to the extent of the capital gain invested
(d) of capital gain invested subject to maximum of ₹ 50 lakhs in aggregate for the FY and of next FY

36. Exemption u/s 54F is available


(a) to the extent of amount invested
(b) proportionate to the net sale consideration so invested
(c) to the extent of amount actually invested
(d) none of the above

37. For claiming exemption under section 54EC amount to the extent of the capital gain subject to maximum of ₹ 50 lakhs
should be invested:
(a) within 2 years from the date of transfer
(b) within 3 years from the date of transfer
(c) within six months from the date of transfer
(d) within six months of transfer or before the due date of furnishing the return of income, whichever is earlier

38. For claiming exemption under section 54F the amount to the extent of net sale consideration to be invested in the
completion of the construction house property within:
(a) two years from the date of transfer
(b) three years from the date of transfer
(c) one year before or two years after the date of transfer
(d) one year before or three years after the date of transfer

39. Where after depositing the amount under capital gain scheme, the individual assessee has died, the amount lying in
the capital gain scheme:
(a) shall be taxable in the hands of legal heir
(b) should be utilized by the legal heir for the specified purpose
(c) shall be exempt in the hands of legal heir
(d) shall be taxable in the hands of the person who deposited the amount

40. Long term capital gain on sale of listed equity oriented mutual fund on which STT has been paid shall be:
(a) Taxable @ 10% irrespective of amount of LTCG
(b) Taxable @ 20% irrespective of amount of LTCG
(c) Taxable @ 10% after allowing exemption of ₹ 1,00,000 out of LTCG
(d) Taxable @ 20% after allowing exemption of ₹ 1,00,000 out of LTCG
CA SHREY RATHI CAPITAL GAINS 6.17

41. Transfer of capital asset under a gift or will


(a) shall be regarded as transfer in the hands of donor
(b) shall not be regarded as transfer in the hands of donor
(c) shall be regarded as transfer in the hands of donee
(d) shall not be regarded as transfer in the hands of donee

42. Which of the following is capital assets:


(a) A maruti dealer holding cars for sale
(b) A maruti dealer has a honda city car for his personal use.
(c) Jewellery held by a jeweller which has been held as SIT
(d) Jewellery held by a jeweller for his personal use

43. Which of the following assets is long term capital assets:


(a) Car used for 5 years for personal purposes before the date of sale.
(b) Jewellery held for 10 years for personal use before its date of sale.
(c) House property held by a property dealer for sale for 4 years before sale.
(d) Shares held by Mr. J as investment and sold 11 months after date of purchase.

44. Mr. KG owns two machineries in the block of assets which is eligible for depreciation at the rate of 15%. The WDV of
the block as on 1/4/2022 was ₹ 65,000. No other asset was acquired in this block during the year. One of these
machines was sold during the previous year for ₹ 75,000. Compute the capital gain for the PY 2022-23.
(a) Short term capital gain of ₹ 10,000 in hands of Mr. KG
(b) Short term capital loss of ₹ 10,000 in hands of Mr. KG
(c) Long term capital gain of ₹ 10,000 in hands of Mr. KG
(d) No capital gain as depreciation would be allowed on one of the machines left with Mr. KG

45. Mr. J purchased a house for ₹ 20 lakhs on 1/2/2021. On 1/1/2022 he had constructed one additional floor at the cost
of ₹ 5 lakhs. On 1/1/2023 this house has been sold off for ₹ 51 lakhs and selling expenses have been ₹ 1 lakh.
Calculate Capital Gains for the PY 2022-23.
(a) ₹ 14,00,000 (c) ₹ 50,00,000
(b) ₹ 23,00,000 (d) ₹ 25,00,000

46. Mr. J purchases a house property in December 2006 for ₹ 10,25,000 and an amount of ₹ 7,05,000 was spent on the
improvement and repairs of the property in March 2011. The property was proposed to be sold to Mr. Z in the month
of May, 2022 and an advance of ₹ 40,000 was taken from him. As the entire money was not paid in time, Mr. J
forfeited the advance and subsequently sold the property to Mr. Y in the month of March, 2023 for ₹ 40,00,000. The
fair market value of the property on April 1, 2001 was ₹ 11,00,000. Compute Capital Gain chargeable to tax for PY
2022-23. [CII for F/Y 2006-07: 122; 2010-11: 167; 2022-23: 331]
(a) (₹ 1,78,278) (c) ₹ 2,46,650
(b) (₹ 3,81,762) (d) (₹ 69,753)

47. Which of the following would be regarded as transfer?


(a) Transfer of a capital asset in a scheme of reverse mortgage
(b) Transfer of a capital asset under a gift or will or an irrevocable trust
(c) Transfer by way of conversion of equity shares from preference shares
(d) Redemption of Zero Coupon Bonds

48. Short-term capital gains arising on transfer of listed shares on which STT is paid at the time of transfer, would be
chargeable to tax-
(a) At the rate of 10% (c) At the rate of 15%
(b) At the rate of 20% (d) At the rate of 5%
CA SHREY RATHI CAPITAL GAINS 6.18

49. Distribution of assets at the time of liquidation of a company-


(a) Is not transfer in the hands of the company or the shareholders
(b) Is not a transfer in the hands of the company but capital gains are chargeable to tax on such distribution in the
hands of the shareholders
(c) Is not a transfer in the hands of the shareholders but capital gains are chargeable to tax on such distribution in the
hands of the company
(d) Is a transfer both in the hands of shareholders and company

50. Unlisted preference shares would be long-term capital asset only if it is


(a) Held for more than 12 months immediately preceding the date of transfer
(b) Held for more than 24 months immediately preceding the date of transfer
(c) Held for more than 30 months immediately preceding the date of transfer
(d) Held for more than 36 months immediately preceding the date of transfer

51. Capital gain on transfer of depreciable asset would be-


(a) Long term capital gain, if held for more than 36 months
(b) Long term capital gain, if held for more than 24 months
(c) Long term capital gain, if held for more than 12 months
(d) Short term capital gain, irrespective of the period of holding

52. For an assessee, who is a salaried employee who invests in equity shares, what is the benefit available to him on sale
and acquisition of 100 listed shares of X Ltd. on which securities transaction tax has been paid. Assume that the
assessee held the shares for 14 months before sale? (ICAI MCQ Booklet)
(a) Rebate under section 88E is allowable in respect of securities transaction tax paid
(b) Securities transaction tax paid is treated as expenses of transfer and deducted from sale consideration.
(c) Capital gains is taxable at a concessional rate of 10% on such capital gains exceeding ₹ 1 lakh
(d) Capital gains is taxable at concessional rate of 15%.

53. Under section 50C, the stamp duty value is taken as the full value of consideration only if-
(a) The asset transferred is building and the actual consideration is less than the SDV
(b) The asset transferred is either land or building or both and the SDV exceeds the actual consideration
(c) The asset transferred is either land or building or both and the SDV value exceeds 110% of the actual
consideration.
(d) The asset transferred is land and the actual consideration is less than the SDV

54. Under section 54F, capital gains are exempted if


(a) Long-term capital gain arising on transfer of residential house is invested in acquisition of one residential house
situated in or outside India
(b) Long-term capital gain arising on transfer of a capital asset other than a residential house is invested in acquisition
of one residential house situated in or outside India.
(c) Net sale consideration on transfer of a capital asset other than a residential house is invested in acquisition of one
residential house situated in India
(d) Short term or long- term capital gain arising on transfer of a capital asset other than a residential house is
invested in acquisition of one residential house situated in India

55. Mr. Karan, a senior citizen mortgaged his residential property with the bank, under a reverse mortgage scheme. He
was getting monthly instalments from the bank as loan. Discuss the treatment of such receipt by Mr. Karan.
(a) Any amount received from the bank under reverse mortgage scheme is exempt u/s 10(43).
(b) Amount received under reverse mortgage scheme shall be fully taxable in the hands of the recipient.
(c) Amount received under reverse mortgage scheme shall be partly taxable & partly exempt.
(d) None of the above

56. The Cost of Acquisition of a capital asset earlier taxable as gift u/h Income from Other Sources shall be:
(a) Nil
(b) The amount taxable u/s 56(2)(x) u/h Income from Other Sources
(c) The amount taxable u/s 56(2)(x) u/h Income from Other Sources less ₹ 50,000.
(d) None of the above
CA SHREY RATHI CAPITAL GAINS 6.19

57. The Cost Inflation Index for the F/Y 2022-23 is:
(a) 317 (c) 100
(b) 289 (d) 331

58. Mr. Yam acquired shares of ABC Ltd., a listed company on 13 th October 2014 for ₹ 5,00,000. The FMV of such shares
as on 31st January 2018 is ₹ 5,40,000. It’s indexed cost of acquisition is ₹ 6,20,000. The shares were sold on 28th
December 2022 for ₹ 7,00,000. The amount taxable u/h Capital Gains shall:
(a) ₹ 2,00,000 (c) ₹ 1,60,000
(b) ₹ 80,000 (d) ₹ 1,00,000

59. Mr. Akash purchased goodwill on 25th November 1997 for ₹ 22,000. FMV of the goodwill as on 01.04.2001 was ₹
57,000. He sold the goodwill on 16th December 2022 for ₹ 3,50,000. Compute his capital gains.
(a) ₹ 2,77,180 (c) ₹ 1,61,330
(b) ₹ 3,28,000 (d) ₹ 3,50,000

60. Mr. Sundar’s residential house met with accidental fire on 23rd July 2017. Further he lost household appliances worth
₹ 1,40,000 and a gold chain worth ₹ 2,40,000 in this fire. The house was purchased on 3rd July 2005 for ₹ 5,00,000
whereas the household appliances & gold chain were purchased in the year 2013-14 & 2016-17 respectively. Sundar
received insurance claim on 12th October 2022. The claim amount is ₹ 12,80,000 for the house, ₹ 1,30,000 for
household appliances and ₹ 2,10,000 for the gold chain. Compute income u/h Capital Gains. CII for the P/Y 2005-06:
117; 2013-14: 220; 2016-17: 264; 2017-18: 272 & 2022-23: 331.
(a) ₹ 77,607 (c) ₹ 53,419
(b) ₹ 87,607 (d) (₹ 164,530)

61. For the purpose of slump sale, the cost of acquisition shall be:
(a) Net worth (i.e. total value of assets less external liabilities)
(b) FMV of the assets on the date of slump sale
(c) Historical cost of the assets less current liabilities
(d) None of the above

62. The treatment of Joint Development Agreement has been devised under
(a) Section 45(5) (c) Section 45(4)
(b) Section 45(5A) (d) Section 46A

63. For the purpose of computing the stamp duty value as per Section 50C, the date of agreement shall be considered
provided
(a) Part payment has been made by cash on or before the date of agreement
(b) Part payment has been made by account payee cheque/draft/ECS after the date of agreement
(c) Full payment has been made by cash after the date of agreement but before the date of registration
(d) Part payment has been made by account payee cheque/draft/ECS before the date of agreement

64. Mr. Kanchan’s property was compulsorily acquired on 12th February 2016 and a compensation of ₹ 23,00,000 was
paid to him on 19th July 2017. He purchased such property on 9th November 2014 for ₹ 16,00,000. Kanchan filed a suit
challenging the quantum of deduction. The court ordered additional compensation of ₹ 6,00,000. Such compensation
was received on 19.11.2022. He incurred an expenditure of ₹ 40,000 for recovery of additional compensation. Discuss
the treatment for the P/Y 2022-23.
(a) STCG – ₹ 12,60,000 (c) LTCG – 5,60,000
(b) STCG – ₹ 7,00,000 (d) STCG – 5,60,000

65. In case of demerger of a company, the cost of acquisition of shares of demerged company shall be:
(a) Cost of acquisition of share in demerged company x Book value of assets / Net worth of the demerged company
(b) Nil
(c) Original cost – (a)
(d) FMV of the shares in resulting company
CA SHREY RATHI CAPITAL GAINS 6.20

66. Mr. Pappu purchased 300 shares of XYZ Ltd., an unlisted company on 23rd May 2012 for ₹ 10,500. He further
purchased 200 shares on 15th October 2015 for ₹ 15,000. He sold 400 shares on 22nd July 2022 @ ₹ 105 each.
Compute his capital gains. CII for the P/Y 2012-13: 200; 2015-16: 254 & 2022-23: 331.
(a) ₹ 14,849 (c) ₹ 16,500
(b) ₹ 9,760 (d) ₹ 24,000

67. Ms. Kirti purchased debentures on 25th January 2001 for ₹ 33,000. FMV of the debentures as on 1st April 2001 was ₹
34,500. She sold the debentures on 14th March 2023 for ₹ 87,000. Compute her Capital Gains. CII for P/Y 2001-02:
100; 2022-23: 331.
(a) ₹ 52,500 (c) (₹ 27,195)
(b) (₹ 22,230) (d) ₹ 54,000

68. When can an assessing officer refer the case to a valuation officer?
(a) the market value of the asset exceeds by more than 25% of the value of the asset claimed by the assessee
(b) the market value of the asset exceeds by more than ₹ 25,000 of the value claimed by the assessee
(c) the value of the asset claimed by the assessee exceeds by more than 15% of the market value of the asset
(d) both (b) & (c)

69. Under Reverse Mortgage Scheme, where the loan is disbursed directly to the Reverse Mortgager, the maximum
period of payment by the lender shall be:
(a) 10 years (c) 20 years
(b) 15 years (d) 25 years

70. Exemption on compulsory acquisition of urban agricultural land u/s 10(37) is allowed when
(a) The land was held by individual or HUF
(b) Such land was being utilised for agricultural purpose during 2 years by himself or parents
(c) Both (a) & (b) are satisfied
(d) The land was held by company with no restriction of time limit of its usage

71. AB Ltd., a listed company bought back its shares from one of its shareholders at 250 per share on 12th July 2022
whereas the FMV was 236 per share. Mr. Anand purchased 250 shares of the company @ 143 per share on 25 th
September 2021. Compute capital gains in the hands of Mr. Anand.
(a) ₹ 26,750 (c) ₹ 23,250
(b) ₹ 25,000 (d) None of the above

72. The term transfer has been defined under


(a) Section 2(47) (c) Section 2(42A)
(b) Section 2(29A) (d) Section 2(14)

73. Where an asset received by a shareholder on the liquidation of the company is further sold, the cost of acquisition of
such asset shall be:
(a) The cost for which it was originally acquired by the company
(b) The written down value of the asset in the hands of the company on the date of distribution
(c) The value as determined by the liquidator
(d) The FMV as of the asset as on the date of distribution of the asset

74. What will be the full value of consideration as per Sec. 50C where the stamp duty value of the property is ₹ 159 lakhs
and the sale consideration received is ₹ 150 lakhs?
(a) ₹ 150 lakhs (c) ₹ 200 lakhs
(b) ₹ 157.5 lakhs (d) ₹ 159 lakhs

75. Where an individual transfer his land under Joint Development Agreement to a developer, the capital gains tax
liability shall arise in the year in which
(a) The transfer was made by such individual to the developer
(b) The assessing officer decides the year depending upon the completion of the property
(c) The certificate of completion is issued by the competent authority
(d) As mutually decided between the individual & the developer
CA SHREY RATHI CAPITAL GAINS 6.21

76. The cost of acquisition of a capital asset earlier used as stock in trade shall be the
(a) Cost for which it was originally acquired
(b) FMV as on the date on conversion from stock in trade to a capital asset
(c) Average of cost and FMV
(d) Amount calculated as per LIFO method

77. Under section 54EC, capital gains on transfer of land or building or both are exempted if invested in the bonds issued
by NHAI & RECL or other notified bond- (ICAI MCQ Booklet)
(a) Within a period of 6 months after the date of such transfer
(b) Within a period of 6 months from the end of the relevant previous year
(c) Within a period of 6 months from the end of the previous year or the due date for filing the return of income
under section 139(1), whichever is earlier
(d) At any time before the end of the relevant previous year

78. What will be the year of chargeability in case of dissolution of partnership firm
(a) P/Y of initiation of dissolution (c) P/Y of distribution of capital assets
(b) P/Y of finalisation of dissolution (d) P/Y of approval from the partners

79. Cost of acquisition of forfeited shares shall be:


(a) Price paid to the company (c) FMV as on the date of the forfeiture
(b) Nil (d) Price paid to the company – refund from the company

80. Mr. X, a property dealer, converted one of his house properties into stock in trade on 01.07.2022 and which was
subsequently sold for a consideration of ₹ 10,00.000 on 30.12.2022. The fair market value of the property as on
01.07.2022 is ₹ 8,00,000 and as on 30.12.2022 is ₹ 8,50,000. Determine the full value consideration on date of
conversion.
(a) ₹ 10,00,000 (c) ₹ 8,00,000
(b) ₹ 8,50,000 (d) ₹ 50,000

81. Mr. A (aged 45 years) sold an agricultural land for ₹ 52 lakhs on 04-10-2022 which was acquired at a cost of ₹ 49.25
lakhs on 13-09-2021 situated at 7 kms from the jurisdiction of municipality having population of 4,00,000 and also
sold another agricultural land for ₹ 53 lakhs on 12-12-2022 acquired at a cost of ₹ 46 lakhs on 15-02-2021 situated at
1.5 kms from the jurisdiction of municipality having population of 12,000. What would be the amount of capital gain
chargeable to tax in the hands of Mr. A for the assessment year 2023-24? Cost inflation index for F.Y. 2019-20: 289,
2020-21: 301, 2022-23: 331. (ICAI MCQ Booklet)
(a) Short-term capital gain of ₹ 9.75 lakhs (c) Long-term capital gain of ₹ 2,54,325
(b) Short-term capital gain of ₹ 7 lakhs (d) Long-term capital gain of ₹ 2,67,531

82. Mr. Kashyap has acquired a building from his friend on 10-10-2022 for ₹ 15,00,000. The stamp due value of the
building on the date of purchase is ₹ 16,20,000. Income chargeable to tax in the hands of Mr. Kashyap is (ICAI MCQ
Booklet)
(a) ₹ 70,000 (c) Nil
(b) ₹ 50,000 (d) 1,20,000

MCQ’s Answers
1. (c) 2. (c) 3. (b) 4. (c) 5. (d) 6. (a) 7. (c) 8. (a) 9. (d) 10. (b)
11. (b) 12. (b) 13. (a) 14. (a) 15. (c) 16. (a) 17. (d) 18. (a) 19. (b) 20. (b)
21. (a) 22. (a) 23. (c) 24. (c) 25. (c) 26. (a) 27. (c) 28. (c) 29. (c) 30. (c)
31. (c) 32. (c) 33. (c) 34. (c) 35. (d) 36. (b) 37. (c) 38. (b) 39. (c) 40. (c)
41. (b) 42. (d) 43. (b) 44. (a) 45. (d) 46. (a) 47. (d) 48. (c) 49. (b) 50. (b)
51. (d) 52. (c) 53. (c) 54. (c) 55. (a) 56. (b) 57. (d) 58. (c) 59. (a) 60. (b)
61. (a) 62. (b) 63. (d) 64. (d) 65. (c) 66. (a) 67. (a) 68. (b) 69. (c) 70. (c)
71. (a) 72. (a) 73. (d) 74. (a) 75. (c) 76. (b) 77. (a) 78. (c) 79. (a) 80. (c)
81. (b) 82. (c)
CA SHREY RATHI INCOME FROM OTHER SOURCES 7.1

STEVE JOBS

CHAPTER 7
INCOME FROM OTHER SOURCES

“Your time is limited, so don’t


waste if living someone’s else life”

PRACTICAL QUESTIONS
Q 1: Ms. Sonam is a businesswoman. For the P/Y 2022-23, her business income is ₹ 16,00,000. During the P/Y 2014-15, a
plot of land owned by Sonam was compulsorily acquired by UP Govt. Initial compensation of ₹ 14,50,000 was received by
Sonam in 2016-17. On Sonam’s appeal UP High Court increased the compensation from ₹ 14,50,000 to ₹ 18,00,000. On 1st
August 2022, she received additional compensation of ₹ 3,50,000 along with interest of ₹ 50,000. Sonam has spent ₹
42,000 to get enhanced compensation.
On 18th January 2023, Sonam purchased a painting from a friend for ₹ 2,25,000 whose market value is ₹ 6,00,000. On 19th
March 2023, she purchased a second hand car for ₹ 1,18,000 whose market value is ₹ 1,85,000. Sonam annually deposits
₹ 30,000 in PPF.
Find out the net income of Ms. Sonam for the P/Y 2022-23.

Sol 1: Computation of net income of Ms. Sonam for the P/Y 2022-23:
Particulars (₹) (₹)
Income from business 16,00,000
Enhanced compensation received from UP Govt. on 1st August 2022 3,50,000
(-) Expenses incurred (₹ 42,000 x ₹ 3,50,000 / ₹ 4,00,000) 36,750 3,13,250
Interest on enhanced compensation 50,000
(-) Fixed deduction @ 50% 25,000 25,000
Purchase of a painting for inadequate consideration shall be taxable if the difference
between the fair market value and purchase price exceeds ₹ 50,000. (₹ 6,00,000 – ₹ 3,75,000
2,25,000)
Purchase of second hand car shall not be taxable as it is not considered a property. -
Gross Total Income 23,13,250
Less: Deduction u/s 80C – PPF (to be discussed in the chapter deductions) 30,000
Net income of Ms. Sonam for the P/Y 2022-23 22,83,250
CA SHREY RATHI INCOME FROM OTHER SOURCES 7.2

Q 2: The following incomes are received by Mr. Randeep during the P/Y 2022-23:
(₹)
1. Director’s fees 2,400
2. Income from agricultural land in China 6,000
3. Ground rent for land in Indore 9,000
4. Interest on Postal Savings Bank A/c 2,850
5. Interest on deposit with Industrial Finance Corporation of India 700
6. Dividend from a foreign company 500
7. Rent from sub-letting a house 26,000
8. Repairs paid for the sub-let house 11,000
9. Other expenses on sub-let house 200
10. Winnings from horse races (gross) 18,000
11. Interest on securities (gross) 6,600
You are required to calculate Income from Other Sources of Mr. Randeep for the A/Y 2023-24.

Sol 2: Computation of income from other sources for Mr. Randeep for the A/Y 2023-24:
S. No. Particulars (₹)
1. Director’s fees 2,400
2. Income from agricultural land in China 6,000
3. Ground rent for land in Indore 9,000
4. Interest on Postal Saving Bank A/c [exempt upto ₹ 3,500] -
5. Interest on deposit with Industrial Finance Corporation of India 700
6. Dividend from a foreign company 500
7. Rent from sub-letting a house ₹ 26,000
(-) Repairs paid for the sub-let house ₹ 11,000
(-) Other expenses on sub-let house ₹ 200 14,800
8. Winnings from horse races (gross) 18,000
9. Interest on securities (gross) 6,600
Income from Other Sources for Mr. Randeep for the A/Y 2023-24 58,000

Q 3: Discuss the tax treatment of the following transactions for the P/Y 2022-23:
1. Ayush receives the following house properties from his friends without any consideration:
Name of friends Date of gift Location of property Stamp Duty Value (₹)
Nidhi 26.09.2022 Chennai 40,000
Ankita 15.02.2023 Kashmir 2,90,000
2. Ritu purchases jewellery and painting from Sunita for ₹ 5,00,000 and ₹ 2,70,000 respectively whose FMV were ₹
5,40,000 and ₹ 4,60,000.

Sol 3:
1. Gift of immovable property without consideration: Where any immovable property is received by any person
without consideration, the stamp duty value of which exceeds ₹ 50,000, the stamp duty value of such property shall
be taxable in the hands of recipient.
Chennai: As the stamp duty value of the property is ₹ 40,000 (i.e. it does not exceed ₹ 50,000), therefore it shall not be
taxable for Ayush.
Kashmir: As the stamp duty value of the property is ₹ 2,90,000 (i.e. exceeding ₹ 50,000), therefore it shall be taxable for
Ayush.
Note: It shall be noted that in case of immovable property, the limit of ₹ 50,000 is applicable for each property.
CA SHREY RATHI INCOME FROM OTHER SOURCES 7.3

2. Gift of movable property with inadequate consideration: where any movable property is acquired for a consideration
which is less than the aggregate FMV of the property by an amount exceeding ₹ 50,000, the aggregate FMV of such
property as exceeds such consideration shall be taxable in the hands of recipient.
Movable Property FMV (₹) Consideration (₹) Benefit (₹)
Jewellery 5,40,000 5,00,000 40,000
Painting 4,60,000 2,70,000 1,90,000
Total value of Gift 2,30,000
Note: It shall be noted that in case of movable property, aggregate of gift shall be taken into consideration. Therefore, in
this case, the whole of ₹ 2,30,000 shall be taxable in the hands of Ritu.

Q 4: Mr. Sisodia, a resident individual submits the following particulars of his income for the year ended 31.03.2023:
1. Royalty income from a coal mine - ₹ 40,000
(spent 3,000 for collecting such royalty income)
2. Agricultural income from Canada - ₹ 25,000
3. Salary as a member of Parliament - ₹ 3,000 p.m.
4. He took a residential house on rent for ₹ 10,000 p.a. half of which is sublet for ₹ 1,200 p.m.
5. Dividend received from a cooperative society - ₹ 6,000
(paid ₹ 1,100 for collecting dividend)
Compute Mr. Sisodia’s income from other sources for the A/Y 2023-24.

Sol 4: Computation of income from other sources of Mr. Sisodia for the A/Y 2023-24:
S. No. Particulars (₹) (₹)

1. Royalty income from a coal mine 40,000


(-) Collection charges 3,000 37,000
2. Agricultural income from Canada 25,000
3. Salary as a member of Parliament (₹ 3,000 x 12) 36,000
4. Sub-letting of residential house (₹ 1,200 x 12) 14,400
(-) Rent paid to the landlord (₹ 10,000 x ½) 5,000 9,400
5. Dividend received from a cooperative society 6,000
(-) Collection charges 1,100 4,900
Income from Other Sources of Mr. Sisodia 1,12,300

Q 5: Mr. Ronnie let out his house property to Ms. Mahi for a monthly rent of ₹ 10,000. The house has been let out with
the facilities, i.e. Fooding; Furniture; Electricity and Telephone.
Actual expenditure incurred by Mr. Ronnie for providing such facilities is as follows:
Fooding: ₹ 1,400 p.m.; Furniture repairs: ₹ 500 p.m.; Depreciation on furniture: ₹ 6,000 p.a., Depreciation on room: ₹
6,000 p.a.; Electricity and telephone: ₹ 600 p.m.
Find the taxable income of Mr. Ronnie for the A/Y 2023-24, in the following cases:
Case 1: Rent is not segregated with such facilities.
Case 2: Rent is segregated as under and Mahi is ready to accept the property without above facilities. For Fooding: ₹ 2,000
p.m.; Furniture: ₹ 3,000 p.m.; Room: ₹ 4,000 p.m.; Electricity and telephone: ₹ 1,000 p.m.
CA SHREY RATHI INCOME FROM OTHER SOURCES 7.4

Sol 5: Computation of taxable income of Mr. Ronnie for the A/Y 2023-24:
Case 1: Rent is not segregated with such facilities. Where rent is not segregated, then the entire rent shall be taxable u/h
income from other sources.
Particulars (₹)
Composite rent received (₹ 10,000 x 12) 1,20,000
Less: Actual expenditure incurred by Mr. Ronnie
Fooding (₹ 1,400 x 12) ₹ 16,800
Furniture repairs (₹ 500 x 12) ₹ 6,000
Depreciation on furniture ₹ 6,000
Depreciation on room ₹ 6,000
Electricity and telephone (₹ 600 x 12) ₹ 7,200 42,000
Income from Other Sources 78,000
Case 2: Rent is segregated. Income arising from room shall be taxable u/h house property whereas income arising from
fooding, furniture & electricity and telephone shall be taxable u/h income from other sources.
House Property income:
Particulars (₹)
Gross Annual Value (₹ 4,000 x 12) 48,000
Less: Municipal taxes paid by the owner -
Net Annual Value 48,000
Less: Deduction u/s 24
(a) Standard deduction @ 30% 14,400
(b) Interest on borrowed capital -
Income u/h House Property 33,600
Income from Other Sources:
Particulars (₹)
Income from fooding, furniture & electricity and telephone [(2,000 + 3,000 + 1,000) x 12] 72,000
Less: Actual expenditure incurred by Mr. Ronnie
Fooding (₹ 1,400 x 12) ₹ 16,800
Furniture repairs (₹ 500 x 12) ₹ 6,000
Depreciation on furniture ₹ 6,000
Electricity and telephone (₹ 600 x 12) ₹ 7,200 36,000
Income from Other Sources 36,000
Total income = ₹ 33,600 + ₹ 36,000 = ₹ 69,600.

Q 6: From the following particulars submitted by Mr. X, compute his income from other sources for the A/Y 2023-24.
(₹)
1. Director’s meeting fees received from Y Ltd. 3,000
2. Agricultural income from land situated in India 32,000
3. Agricultural income from Nepal 15,000
4. Interest:
(a) from Bank on FDR 10,000
(b) on Post Office Saving Account 2,600
(c) on Government securities 1,200
(d) on Public Provident Fund A/c 7,000
(e) on National Saving Certificate VIII issue 3,000
5. Dividend from ACT Ltd. declared on 21.09.2022 700
6. Lottery prize received after TDS – ₹ 28,000. He spent ₹ 2,000 for the purchase of lottery tickets.
7. Rent received from sub-letting of a flat - ₹ 12,000 (rent paid to landlord for the flat is ₹ 6,000). Mr. X spent ₹ 600 for
realising the rent.
CA SHREY RATHI INCOME FROM OTHER SOURCES 7.5

Sol 6: Computation of income from other sources of Mr. X for the A/Y 2023-24:
S. No. Particulars (₹) (₹)
1. Director’s meeting fees received from Y Ltd. 3,000
2. Agricultural income from land situated in India [exempt u/s 10(1)]
3. Agricultural income from Nepal 15,000
4. Interest on:
 Bank FDR 10,000
 Post Office Saving Account (exempt upto ₹ 3,500 for individual account) -
 Government securities 1,200
 Public Provident Fund (exempt) -
 National Saving Certificate VIII issue 3,000 14,200
5. Dividend from ACT Ltd. 700
6. Lottery income (net) (₹ 28,000 x 100/70) (expenses relating to lottery are not 40,000
allowed as deduction)
7. Rent received from sub-letting of a flat 12,000
(-) Rent paid to landlord 6,000
(-) Expenses for realising the rent 600 5,400
Income from Other Sources for Mr. X 78,300

Q 7: Mr. Sandeep holds the following securities on 1st April 2022:


1. 6.5% MP Government Bonds (date of payment of interest: 1st February) - ₹ 8,00,000
2. 8% Non-listed debentures of KLM Ltd. (dates of payment of interest: 15th July and 15th January) - ₹ 50,000
3. 9% debentures of PQR Ltd. (dates of payment of interest: 25th June and 25th December) - ₹ 30,000. On 17th November
2022, Sandeep sells debentures of PQR Ltd. at par.
His business income ₹ 6,55,000, Post Office Saving Bank interest is ₹ 5,500 and he has received a gift of ₹ 85,000 in foreign
currency from a friend on 1st May, 2022 on his marriage anniversary.
You are required to calculate taxable income of Mr. Sandeep for the A/Y 2023-24.

Sol 7: Computation of taxable income of Mr. Sandeep for the A/Y 2023-24:
S. No. Particulars (₹)
1. Income from business 6,55,000
2. Income from other sources:
❖ MP Government Bonds (₹ 8,00,000 x 6.5%) 52,000
❖ Non-listed debentures of KLM Ltd. [(₹ 50,000 x 8% x ½) + (₹ 50,000 x 8% x ½)] 4,000
❖ Debentures of PQR Ltd. (₹ 30,000 x 9% x ½) 1,350
❖ Post Office Saving Bank Interest (exempt upto ₹ 3,500 for individual accounts) 2,000
❖ Gift in foreign currency (fully taxable as the gift exceeds ₹ 50,000) 85,000
Taxable income of Mr. Sandeep 7,99,350
Notes: Interest on debentures of PQR Ltd. shall not be computed for 25 th December as they were sold on 17th November,
2022. As they were sold at par, there will be no capital gains also.

Q 8: Ms. Asha provides the following information for the A/Y 2023-24:
(₹)
i. Dividend on equity shares from an Indian company 8,800
ii. Winnings from lottery: amount received – ₹ 70,000, & TDS – ₹ 30,000 1,00,000
iii. Interest received on Government securities held as investment 15,000
iv. Family pension received 50,000
v. Dividend on preference shares from an Indian company 6,000
CA SHREY RATHI INCOME FROM OTHER SOURCES 7.6

She incurred the following expenses:


1. Interest paid on amount borrowed for purchasing equity shares 3,000
2. Lottery tickets purchased 5,000
3. Collection charges in respect of interest on Government securities @ 5% on amount collected.
4. Collection charges on preference shares 1,000
You are required to calculate Income from Other Sources of Ms. Asha for the A/Y 2023-24.

Sol 8: Computation of Income from Other Sources for Ms. Asha for the A/Y 2023-24:
S. No. Particulars (₹) (₹)
1. Dividend on equity shares from an Indian company 8,800
Less: Interest paid on amount borrowed for purchasing equity shares (limited
to 20% of the amount of dividend received) 1,760 7,040
2. Winning from lottery (net) (Note 2) 70,000
Add: TDS 30,000 1,00,000
3. Interest received on Government securities 15,000
Less: Collection charges @ 5% 750 14,250
4. Family pension received 50,000
Less: Deduction of 15,000 or 33.33% of such pension whichever is lower 15,000 35,000
5. Dividend on preference shares from an Indian company 6,000
Less: Collection charges on preference 1,000 5,000
Income from Other Sources for Ms. Asha 1,61,290
Note: Any expenditure relating to lottery is inadmissible. Only expenses relating to owing and maintaining race horses are
allowable.

Q 9: The Balance Sheet of KPMG Ltd. is as under:


Liabilities (₹) Assets (₹)
Equity share capital (including bonus Cash 7,00,000
shares of ₹ 1,00,000 issued out of 5,00,000
reserves Other Assets 2,00,000

Reserves and surplus 4,00,000


Total 9,00,000 Total 9,00,000
The company distributed debentures of ₹ 6,00,000 to his shareholders. Compute the amount of deemed dividend u/s
2(22)(b).

Sol 9: Any distribution of debentures by the company to its shareholders shall be considered as deemed dividend u/s
2(22)(b) to the extent of accumulated profits whether capitalised or not.
Deemed dividend u/s 2(22)(b) = Amount of debentures distributed to its shareholders (i.e. ₹ 6,00,000) (but restricted to
reserves and surplus + issue of bonus shares) (i.e. ₹ 4,00,000 + ₹ 1,00,000)
Deemed dividend u/s 2(22)(b) = ₹ 5,00,000

Q 10: An enterprise engaged in manufacturing of sports items discontinued its activities and decided to lease out its
factory building, plant & machinery and furniture from 01.04.2022 on a consolidated lease rent of ₹ 50,000 per month.
Compute the income for the A/Y 2023-24 of assessee from the following information:
(₹)
(i) Interest received on deposits 1,00,000
(ii) Brokerage paid on loans taken (promissory notes) 2,000
(iii) Interest paid on loans, which were given as deposits on interest to others 75,000
(iv) Expenses incurred on repairs of building, plant & machinery 25,000
(v) Fire insurance on repairs of building, plant & machinery 12,000
CA SHREY RATHI INCOME FROM OTHER SOURCES 7.7

(vi) Depreciation for the year 1,47,000


(vii) Legal fees paid to advocate for drafting and registering the lease agreement 1,500
(viii) Factory license fees paid for the year 1,000
(ix) There is an unabsorbed depreciation of ₹ 2,75,000 for A/Y 2020-21 & 2021-22
(x) Interest paid includes an amount of ₹ 25,000 remitted outside India on which TDS was not deducted.

Sol 10: Computation of income for the A/Y 2023-24:


S. No. Particulars (₹) (₹)
(i) Consolidated lease rent on letting of factory building, plant & machinery &
furniture (₹ 50,000 x 12) 6,00,000
(-) Expenses incurred on repairs of building, plant & machinery 25,000
(-) Fire insurance on repairs of building, plant & machinery 12,000
(-) Depreciation for the year 1,47,000
(-) Legal fees paid to advocate for drafting and registering the lease agreement 1,500
(-) Factory license fees paid for the year 1,000 4,13,500
(ii) Interest received on deposits 1,00,000
(-) Brokerage paid on loans taken (promissory notes) 2,000
(-) Interest paid on loans (interest paid outside India on which tax has not been
deducted shall not be allowed deduction u/s 40A(2)) (₹ 75,000 – ₹ 25,000) 50,000 48,000
4,61,500
Less: Unabsorbed depreciation for A/Y 2020-21 & 2021-22 (deductible against
any head except salary) 2,75,000
Total income 1,86,500

Q 11: Discuss the taxability of the following in the hands of the recipient u/s 56(2)(x) of the Income Tax Act, 1961:
1. Akhil HUF received ₹ 85,000 in cash from niece (i.e. Daughter of Akhil’s sister) of Akhil. Akhil is the karta of the HUF.
2. Nitisha, a member of her father’s HUF, transferred a house property to the HUF without consideration. The stamp
duty value of the house property is ₹ 9,00,000.
3. Mr. Akshat received 100 shares of A Ltd. from his friend on the occasion of his 30 th Marriage Anniversary. The fair
market value of the shares on that date was ₹ 100 per share. He also received jewellery worth 45,000 (FMV) from his
nephew on the same day.
4. Kishan HUF gifted a car to son of Karta for achieving good marks in CA exams. The FMV of the car is ₹ 5,25,000.
5. A sum of 1,20,000 was received as gift from non-relatives by Raj on the occasion of the marriage of his son Pravin.
[ICAI Study Module]
Sol 11: Computation of taxability u/s 56(2)(x) in the hands of the recipient:
S. No. Taxable / Non- Amount liable Reason
taxable to tax (₹)
1. Taxable 85,000 Sum of money exceeding ₹ 50,000 received without consideration from
a non-relative is taxable. Daughter of Akhil’s sister is not a relative of
Akhil HUF, since she is not a member Akhil HUF.
2. Non-taxable - Immovable property received without consideration by a HUF from its
relative is not taxable. Since Nitisha is a member of HUF, she is a relative
of the HUF. However, income from such asset would be included in the
hands of Nitisha (refer chapter clubbing of income)
3. Taxable 55,000 As per the provisions of Section 56(2)(x), in case the aggregate fair
market value of property, other than immovable property, received
without consideration exceeds ₹ 50,000, the whole of the aggregate
value shall be taxable. In this case, the aggregate fair market value of the
shares (₹ 10,000) + jewellery (₹ 45,000) exceeds ₹ 50,000. Hence the
entire amount of ₹ 55,000 shall be taxable.
4. Non-taxable - Car is not included in the definition of the property for the purpose of
Section 56(2)(x), therefore the same shall not be taxable.
CA SHREY RATHI INCOME FROM OTHER SOURCES 7.8

5. Taxable 1,20,000 The exemption from applicability of Section 56(2)(x) would be available
if gift is received on the occasion of marriage of the individual himself. In
this case, since gift is received by Mr. Raj from a non-relative on the
occasion of marriage of his son, it would be taxable in the hands u/s
56(2)(x).

Q 12: Mr. Rocky is a professor by profession. From the following information given below, determine the income of Mr.
Rocky for the A/Y 2023-24:
(i) Income of Rocky from teaching profession is ₹ 12,00,000 during the P/Y 2022-23.
(ii) On 13th June, 2022, he purchased jewellery from a person (other than registered dealer) for ₹ 7,00,000 (FMV is ₹
8,50,000). He sold the jewellery on 24th August, 2022 for ₹ 8,80,000.
(iii) Mr. Rocky received ₹ 2,80,000 in cash on the occasion of his marriage. ₹ 2,50,000 was gifted by Rocky’s relatives
and the balance by his friends.
(iv) He received a computer from his paternal uncle on his birthday worth ₹ 45,000. He also received ₹ 32,000 in cash
on his birthday from his friends and ₹ 21,000 from his grandmother.
(v) Rocky received ₹ 2,00,000 under a will of a person known to him.
(vi) Interest on National Defence Gold Bonds amounts to ₹ 8,600 which was credited to his bank account on
31.03.2022.
(vii) He purchased an immovable property from a non-relative person for a sum of ₹ 20,00,000, the stamp duty value of
which is ₹ 22,50,000.
(viii) He also purchased a second hand car for ₹ 1,90,000 (market value being ₹ 2,60,000).
(ix) Interest received from Post Office Saving Bank Account – ₹ 10,000.
(x) He deposits ₹ 40,000 annually to PPF.

Sol 12: Computation of income of Mr. Rocky for the A/Y 2023-24:
S. No. Particulars (₹)
1. Income from teaching profession 12,00,000
2. Capital gains on sale of jewellery (₹ 8,80,000 – ₹ 8,50,000) (Refer Note) 30,000
3. Income from other sources
o Purchase of jewellery for inadequate consideration (₹ 8,50,000 – ₹ 7,00,000) 1,50,000
o Gift in money on the occasion of marriage is not taxable whether it is received from -
relative or non-relative.
o Gift of computer from paternal uncle on his birthday shall not be taxable as it is not a -
property for Section 56(2)(x)
o Gift of ₹ 32,000 in money from friends shall not be taxable as it does not exceed ₹ 50,000. -
o Gift of ₹ 21,000 from grandmother is not taxable as grandmother falls in relative. -
o Receipt of money under a will is not taxable -
o Interest on National Defence Gold Bonds (exempt) -
o Purchase of an immovable property from a non-relative person for inadequate
consideration [Being in excess of ₹ 50,000 or 10% of consideration] (₹ 22,50,000 – ₹ 2,50,000
20,00,000)
o Purchase of second hand car shall not be taxable as it is not a property for Section 56(2)(x) -
o Interest received on Post Office Saving Bank Interest ₹ 10,000
Less: Exemption for individual account ₹ 3,500 6,500
Gross Total Income 16,36,500
Less: Deductions u/s VI-A
❖ 80C – PPF 40,000
❖ 80TTA – Interest on Post Office Saving Bank Account (discussed in chapter deductions) 6,500
Total Income 15,90,000
Note: Cost of jewellery for capital gains shall be FMV on the date of purchase as the inadequacy on such purchase is
taxable u/h Income from other sources.
CA SHREY RATHI INCOME FROM OTHER SOURCES 7.9

Q 13: Examine the following transactions in the context of Income Tax Act 1961:
(a) Mr. B transferred 500 shares of RT (P) Ltd. to QK (P) Ltd. on 19.11.2022 for ₹ 3,00,000 when the market price was ₹
5,00,000. The indexed cost of acquisition of shares for Mr. B was computed at ₹ 4,45,000. The transfer was not
subject to STT. Determine the taxability of Mr. B & QK (P) Ltd.
(b) Mr. Chezian is employed in a company with taxable salary of ₹ 5,00,000. He received a cash gift of ₹ 1,00,000 from
Atma Charitable Trust (registered under section 12AA) in December 2022 for meeting his medical expenses. Is the
cash gift so received taxable in the hands of Mr. Chezian? [ICAI Study Module]

Sol 13:
(a) Any movable property received for inadequate consideration by any person is chargeable to tax u/s 56(2)(x), if the
difference between aggregate FMV of the property and consideration exceeds ₹ 50,000. Thus, shares received by QK
(P) Ltd. from Mr. B for inadequate consideration is chargeable to tax u/s 56(2)(x) to the extent of ₹ 2,00,000.
As per Section 50CA (refer capital gains chapter), since, the consideration is less than the fair market value of
unquoted shares of RT (P) Ltd., fair market value of shares of the company would be deemed to be the full value of
consideration. Therefore, the full value of consideration (₹ 5,00,000) less the indexed cost of acquisition (₹ 4,55,000)
would result in long-term capital gains of ₹ 55,000 in the hands of Mr. B.

(b) The provisions of Section 56(2)(x) would not apply to any sum of money or any property received from any trust or
institution registered u/s 12AA. Therefore, the cash gift of ₹ 1,00,000 received from Atma Charitable Trust, being a
trust registered u/s 12AA, for meeting medical expenses would not be chargeable to tax u/s 56(2)(x) in the hands of
Mr. Chezian.

Q 14: Examine under which heads the following incomes are taxable:
(a) Rental income in case property held as stock-in-trade for 3 years
(b) Dividend on shares in case of a dealer in shares
(c) Salary received by a partner from his partnership firm
(d) Rental income of machinery
(e) Winning from lotteries by a person having the same as business activity
(f) Salaries payable to a Member of parliament
(g) Receipts without consideration
(h) In case of retirement, interest on employee’s contribution if provident fund is unrecognized.
(i) Rental income in case of a person engaged in the business of letting out of properties.

Sol 14:
S. No. Particulars Head of Income
(a) Rental income in case property held as stock-in-trade Income from House Property
for 3 years.
(b) Dividend on shares in case of a dealer in shares Income from Other Sources
(c) Salary received by a partner from his partnership Profits and gains of business or profession
firm.
(d) Rental income of machinery Profits and gains of business or profession/ Income
from other sources
(e) Winning from lotteries by a person having the same Income from Other Sources
as business activity
(f) Salaries payable to a Member of parliament Income from Other Sources
(g) Receipts without consideration Income from Other Sources
(h) In case of retirement, interest on employee’s Income from Other Sources
contribution if provident fund is unrecognized.
(i) Rental income in case of a person engaged in the Profits and gains of business or profession
business of letting out of properties.
Note: As per Section 56(2(ii), rental income of machinery would be chargeable to tax under the head “Income from Other
Sources”, if the same is not chargeable to income-tax under the head “Profits and gains of business or profession”.
CA SHREY RATHI INCOME FROM OTHER SOURCES 7.10

Q 15: Mr. Aiyaz has taken a loan of ₹ 1,00,000 @ 10%. The amount was invested by him in securities. During the year he
has received gross interest of ₹ 18,000 and has paid collection charges to the bank ₹ 400. He has paid interest ₹ 10,000 on
the loan taken by him for investment. He has also provided the following information regarding the P/Y 2022-23:
(i) Received cash gifts ₹ 1,71,000, out of which ₹ 1,00,000 was received on the occasion of marriage; ₹ 40,000 from
relatives and the remaining from non-relatives on the occasion of birthday.
(ii) He bought 250 equity shares of a listed company from another friend for ₹ 75,000, whereas the market value of
such shares is ₹ 1,95,000 on the date of purchase.
(iii) A mobile phone worth ₹ 55,000 was gifted by his non-resident friend.
Determine the amount of income chargeable to tax under the head income from other sources for the A/Y 2023-24.

Sol 15: Computation of income u/h other sources for Mr. Aiyaz for the A/Y 2023-24:
S. No. Particulars (₹) (₹)
1. Interest on securities 18,000
(-) Collection charges to the bank 400
(-) Interest on loan taken for investment 10,000 7,600
2. Cash gifts:
❖ On the occasion of marriage (not taxable without any limit) -
❖ Received from relatives (not taxable without any limit) -
❖ Received from non-relatives (₹ 1,71,000 – ₹ 1,00,000 – ₹ 40,000 = ₹ -
31,000) (not taxable as the amount of gift does not exceed ₹ 50,000) -
3. Purchase of equity shares from friend for inadequate consideration (₹ 1,95,000
– ₹ 75,000) 1,20,000
4. Mobile phone gifted by non-resident friend (not taxable as mobile phone is not
a property as per Section 56(2)(x)) -
Income from Other Sources for Mr. Aiyaz 1,27,600

Q 16: The following details have been furnished by Mrs. Hemali, pertaining to the year ended 31-3-2023:
a) Cash gift of ₹ 51,000 received from her friend on the occasion of her “Shastiaptha Poorthi” a wedding function
celebrated on her husband completing 60 years of age. This was also her 25th wedding anniversary.
b) On the above occasion, a diamond necklace worth ₹ 2 lacs was presented by her sister living in Dubai.
c) When she celebrated her daughter’s wedding on 21-2-2023, her friend assigned in Mrs. Hemali’s favour, a fixed
deposit held by the said friend in a scheduled bank; the value of the fixed deposit and the accrued interest on the said
date was ₹ 52,000.
Compute the income if any, assessable as income from other sources. [ICAI Study Module]

Sol 16:
a) Any sum of money received by an individual on the occasion of the marriage of the individual is exempt. This
provision is however, not applicable to a cash gift received during a wedding function celebrated on completion of 60
years of age.
The gift of ₹ 51,000 received from a non-relative is therefore, chargeable to tax under Section 56(2)(x) in the hands of
Mrs. Hemali, since the same exceeds ₹ 50,000.
.
b) The provision of Section 56(2)(x) are not attracted in respected of any sum of money or property received from a
relative. Thus, the gift of diamond necklace received from her sister is not taxable under section 56(2)(x), even though
jewellery falls within the definition of “property”.

c) To be exempt from applicability of section 56(2)(x) the property should be received on the occasion of the marriage of
the individual, not that of the individual’s son or daughter. Therefore, this exemption provision is not attracted in this
case.
Any sum of money received without consideration by an individual is chargeable to tax under Section 56(2)(x), if the
aggregate value exceeds ₹ 50,000 in a year. “Sum of money” has, however, not been defined under Section 56(2)(x).
CA SHREY RATHI INCOME FROM OTHER SOURCES 7.11

Therefore, there are two possible views in respect of value of fixed deposit assigned in favour of Mrs. Hemali:
1. The first view is that fixed deposit does not fall within the meaning of “sum of money” and therefore, the
provisions of Section 56(2)(vii) are not attracted. It may be noted that fixed deposit is also not included in the
definition of “property”.
2. However, another possible view is that fixed deposit assigned in favour of Mrs. Hemali falls within the meaning of
“sum of money” received.
Income assessable as “Income from other sources”
If the first view is taken, the total amount chargeable to tax as “Income from other sources” would be ₹ 51,000 being
cash gift received from a friend on her Shastiaptha Poorthi.
As per the second view the provision of Section 56(2)(x) would be attracted in respect of the fixed deposit assigned
and the “Income from other sources” of Mrs. Hemali would be ₹ 1,03,000 (₹ 51,000 + ₹ 52,000).

MULTIPLE CHOICE QUESTIONS


1. Income from letting of machinery, plant and furniture is –
(a) Always chargeable to tax under the head “profits and gains from business and profession”
(b) Always chargeable to tax under the head “Income from other sources”
(c) Chargeable under the head “income from other sources” only if not chargeable under the head “profits and gains
of business and profession”
(d) Chargeable to tax under the head “Income from house property”

2. In respect of winnings from lottery, crossword puzzle or races including horse race or card game etc.
(a) No deduction under Chapter VI-A is allowed and basic exemption limit cannot be exhausted
(b) No deduction under Chapter VI-A is allowed but unexhausted basic exemption can be exhausted
(c) Both deduction under Chapter VI- A and basic exemption are allowed
(d) Deduction under Chapter VI-A is allowed but basic exemption limit cannot be exhausted

3. The deduction allowable in respect of family pension taxable under “Income from other sources” is:
(a) 33.33% of the pension
(b) 30% of the pension or ₹ 15,000, whichever is less
(c) 33.33% of the pension or ₹ 15,000, whichever is less
(d) 30% of the pension

4. Mr. Kashyap has acquired a building from his friend on 10.10.2022 for ₹ 15,00,000. The stamp duty value of the
building on the date of purchase is ₹ 16,70,000. Income chargeable to tax u/h Other Sources in the hands of Mr.
Kashyap shall be:
(a) ₹ 1,70,000 (c) ₹ 50,000
(b) Nil (d) ₹ 1,50,000

5. Ganesh received ₹ 60,000 from his friend his friend on the occasion of his birthday
(a) The entire amount of ₹ 60,000 is taxable (c) ₹ 50,000 is taxable
(b) The entire amount is exempt (d) ₹ 10,000 is taxable

6. Mr. X, aged, 61 years, received dividend of ₹ 12,00,000 from a domestic company in P/Y 2022-23. Such income shall
be taxable in the hands of Mr X to the extent of:
(a) The entire amount of ₹ 12,00,000 (c) ₹ 2,00,000
(b) Nil. As exempt u/s 10(34) (d) ₹ 10,00,000

7. In respect of lottery income earned by an individual-


(a) No deduction under Chapter VI-A is allowed but loss under other heads can be set-off against such income
(b) No deduction under Chapter VI-A is allowed and no loss can be set-off against such income
(c) Both deduction under Chapter VI-A and set-off of losses against such income are allowed
(d) Deduction under chapter VI-A is allowed but set-off of losses under other heads against such income is not
allowed
CA SHREY RATHI INCOME FROM OTHER SOURCES 7.12

8. Mr. Y has received a sum of ₹ 51,000 on 24.10.2022 from relatives on the occasion of his marriage.
(a) Entire ₹ 51,000 is chargeable to tax (c) Only ₹ 1,000 is chargeable to tax
(b) Entire ₹ 51,000 is exempt from tax (d) Only 50% i.e., ₹ 25,500 is chargeable to tax

9. Mr. Mayank has received a sum of ₹ 75,000 on 19.11.2022 from his friend on the occasion of his marriage
anniversary. (ICAI MCQ Booklet)
(a) Entire ₹ 75,000 is chargeable to tax (c) Entire ₹ 75,000 is exempt from tax
(b) Only ₹ 25,000 is chargeable to tax (d) Only 50% i.e., ₹ 37,500 is chargeable to tax

10. The deduction in respect of interest on enhanced compensation of ₹ 1,50,000 received during the previous year 2022-
23, would be-
(a) ₹ 1,50,000, being 100% of ₹ 1,50,000 (c) Nil
(b) ₹ 45,000, being 30% of ₹ 1,50,000 (d) ₹ 75,000, being 50% of ₹ 1,50,000

11. Mr. Y received the following income during PY 2022-23. Determine his Income from other sources:
1) Director’s fees ₹ 5,000
2) Income from agricultural land in Pakistan ₹ 15,000
3) Rent from let-out land in Jaipur ₹ 20,000
4) Interest on deposit with HDFC Bank ₹ 1,000
5) Divided from Indian company ₹ 5,000.
(a) ₹ 21,000 (c) ₹ 46,000
(b) ₹ 31,000 (d) ₹ 41,000

12. Mr. X, aged 61 years received of ₹ 12,00,000 as dividend from ABC Ltd. In the P/Y 2022-23. Interest on loan taken for
the purpose of investment in ABC Ltd., is ₹ 3,00,000. Income included in the hands of Mr. X for P.Y. 2022-23 would be:
(ICAI MCQ Booklet)
(a) ₹ 12,00,000 (c) ₹ 9,00,000
(b) ₹ 9,60,000 (d) Nil

13. Under a lucky draw competition, Mr. P won a car which has FMV of ₹ 7,00,000. What amount is taxable in his hands?
(a) ₹ 7,00,000 (c) ₹ 4,90,000
(b) ₹ 2,10,000 (d) Nil

14. A watch has been gifted to an individual which has fair market value of ₹ 1,00,000. The fair market value of watch is
(a) Exempt since received from a relative
(b) Not taxable since watch is not movable property within the definition of section 56(2)
(c) Taxable under head other sources
(d) None of the above

15. GGC Private Limited gives a loan of ₹ 75,00,000 to Mr. J, who is not a shareholder. Mr. J uses the amount of loan for
the benefit of Mr. A who is shareholder in GGC Private Limited holding 15% shares. In this case, amount taxable as
deemed dividends in the hands of Mr. J will be ...................... and that in hands of Mr. A will be ......................
(a) Nil, Nil (c) Nil, 75,00,000
(b) 75,00,000, Nil (d) 75,00,000, 75,00,000

16. Mr. Kunal has received three gifts from his three friends
1) ₹ 55,000 in cash
2) Land with market value ₹ 5,00,000, value for purpose of charging stamp duty ₹ 4,00,000
3) Jewellery with market value ₹ 3,00,000 What is the amount taxable as income from other sources.
(a) ₹ 7,55,000 (c) ₹ 8,55,000
(b) ₹ 4,55,000 (d) ₹ 3,55,000
CA SHREY RATHI INCOME FROM OTHER SOURCES 7.13

17. Which of the following statement is true?


(a) Money received as gift shall be taxable in the hands of recipient if it exceeds ₹ 50,000
(b) Stamp duty value of the immovable property shall be taxable if it is given without consideration and the stamp
duty value exceeds ₹ 1,00,000
(c) Fair market value of the movable property shall be taxable if it is given without consideration and the fair market
value exceeds ₹ 1,00,000
(d) None of the above

18. Which of the following movable property is not taxable under section 56(2) when transferred without consideration?
(a) Drawings (c) Paintings
(b) Cars (d) Sculptures

19. The interest on Post Office Savings Bank Account would be exempt from tax upto ............ for joint account.
(a) ₹ 3,500 (c) ₹ 7,000
(b) Nil (d) Without limit

20. The limit of ₹ 50,000 for computing limit for amount of cash received without consideration has to be seen for
(a) Separately for all transaction (c) Either of (a) and (b)
(b) Cumulatively for all transaction (d) None of (a) and (b)

21. Guest lecturer’s remuneration is taxable under which head of income


(a) Other sources (c) Salary
(b) House property (d) Profit & Gains from Business and Profession

22. Income from other sources is also known as ...................... head of income
(a) Residuary head (c) Useless head
(b) Complementary head (d) None of the these

23. Lottery income is taxable @ ………….


(a) 20% (c) Flat 30%
(b) Slab Rate (d) 30% after deducting all expenses

24. On 30th December 2022, Mr. Q gets by gift a commercial flat (stamp duty value is ₹ 25,00,000) from elder brother of
his father-in-law. The amount chargeable to tax in the hands of Mr. Q is
(a) ₹ 25,00,000 (c) ₹ 24,50,000
(b) ₹ 20,00,000 (d) Nil

25. Income by way of interest on securities earned by stock broker from securities held as inventory is chargeable under
the head of
(a) Business and Profession (c) House Property
(b) Salary (d) Other Sources

26. Bond washing transaction is given under:


(a) Section 98 (c) Section 95
(b) Section 94 (d) Section 112

27. When an advance received for transfer of capital asset is forfeited on or after 1/4/2014, such forfeited amount shall
be
(a) Deducted from the cost of the asset
(b) Exempt
(c) Taxable as income under the head income from other sources
(d) None of the above
CA SHREY RATHI INCOME FROM OTHER SOURCES 7.14

28. A private limited company engaged in manufacturing activity had general reserve of ₹ 20 lakhs. It granted a loan of ₹
5 lakhs to a director who held 13% shareholding cum voting rights in the company. The said loan was re-paid by him
before the end of the year. The amount of deemed dividend arising out of the above transaction is:
(a) ₹ 2,60,000 (c) ₹ 2,40,000
(b) ₹ 5,00,000 (d) Nil

29. If no regular system of accounting is followed by the assessee then interest on securities is taxable on:
(a) Due basis (c) Due or receipt basis at the option of the assesses
(b) Receipts basis (d) None of the above

30. When any sum of money which exceeds ₹ 50,000 is received without consideration then the whole of such sum shall
be taxable:
(a) In the hands of all assesses (c) In the hands of an individual
(b) In the hands of an individual or HUF (d) In the hands of all assesses other than a company

31. Mr. T traced a missing girl by spending ₹ 20,000. For this, he was awarded with a sum of ₹ 1,20,000. In this case the
award is taxable to the extent of
(a) ₹ 1,00,000 (c) ₹ 1,20,000
(b) ₹ 1,15,000 (d) Nil

32. Mr. K died in a train accident and after his death his family members were paid compensation of ₹ 5,00,000 by Central
Government. They have spent ₹ 89,000 on collecting this amount. In this case the compensation is taxable to the
extent of
(a) ₹ 5,00,000 (c) ₹ 4,11,000
(b) ₹ 5,89,000 (d) Nil

33. Mr. R received the following gifts during the PY 2022-23. Determine the amount of taxable gift under the head
‘income from other sources’
1) ₹ 50,000 from his employer
2) ₹ 1,00,000 from mother’s sister
3) ₹ 10,000 from his friend on the occasion of his marriage
4) ₹ 60,000 in the form of scholarship from a registered charitable trust
(a) Nil (c) ₹ 60,000
(b) ₹ 70,000 (d) ₹ 2,10,000

34. GC Pvt. Ltd. is a closely held company and has received from Mr. M, shares of another closely held company but
without any consideration
(a) The whole of the fair market value of the shares shall be taxable
(b) The whole of the FMV shall is taxable if it exceeds ₹ 50,000
(c) The whole of FMV shall be exempt
(d) The whole of the cost of such shares shall be exempt

35. Mr. B received ₹ 80,000 by way of gift from friends upon retirement from service. The amount of gift chargeable to
income-tax would be
(a) Nil (c) ₹ 30,000
(b) ₹ 70,000 (d) ₹ 80,000

36. Ms. J received dividend of ₹ 80,000 for her equity shareholding in GGC Ltd. (a listed domestic company). She paid
interest of ₹ 12,500 for the amounts borrowed for investment in those shares. The taxable dividend income in hands
of Ms. J would be:
(a) ₹ 80,000 (c) Nil
(b) ₹ 67,500 (d) ₹ 92,500
CA SHREY RATHI INCOME FROM OTHER SOURCES 7.15

37. Dividends declared by Unit Trust of India or mutual fund is:


(a) Fully exempt in the hands of unit holders
(b) Fully taxable in the hands of unit holders
(c) Taxable but deduction is allowed under section 80C from GTI
(d) Fully exempt in the hands of the unit holders except when it is chargeable to tax under the provisions of section
115BBDA

38. Mr. Veer received following gifts on the occasion of his birthday. Determine the amount taxable as Income from other
sources from the following information:
1) Cash gift from elder brother ₹ 30,000
2) Gold chain from younger sister which has market value of ₹ 38,000
3) Cash gifts from friends ₹ 45,000
4) Purchased shares from younger brother for ₹ 1,00,000 when the market value of the shares was ₹ 1,35,000.
(a) ₹ 1,48,000 (c) ₹ 1,18,000
(b) ₹ 80,000 (d) Nil

39. An assessee received family pension of ₹ 90,000, what amount is taxable as incomer from Other Sources?
(a) ₹ 60,000 (c) ₹ 75,000
(b) ₹ 45,000 (d) None of the above

40. Which of the following are not relative of an individual for the purpose of section 56(2)(x)?
(a) Brother of the father
(b) Sister of spouse of the individual
(c) Cousin sister of the individual
(d) Brother in law of the individual

41. Mr. MKD purchases a land on 27.10.2022 for which the stamp duty value is ₹ 70,00,000 and the full value of
consideration paid is ₹ 65,50,000, value determined by valuation officer is ₹ 70,50,000. Determine the amount taxable
under IFOS.
(a) ₹ 4,50.000 (c) ₹ 50,000
(b) ₹ 5,00,000 (d) Nil

42. Identify which is not taxable under the head ‘Income from Other Sources’?
(a) Advance money forfeited on failure of negotiations in a transfer of a capital asset after 01.04.2014
(b) Sum received on maturity of Keyman insurance policy by an employee
(c) Income from letting out of machinery in the course of business
(d) Immovable property received without consideration

43. A Ltd transferred an immovable property to B Ltd, an Indian company on the event of amalgamation without
consideration for which the stamp duty value on the date of transfer is ₹ 90,00,000. Identify the tax treatment of this
transaction.
(a) Entire ₹ 90,00,000 is taxable (c) Greater than ₹ 50,000 is taxable
(b) 50% of the stamp duty value is taxable (d) Not taxable

44. While comparing the sale consideration with the stamp duty value in respect of transfer of immovable property for
inadequate consideration, the stamp duty value on the:
(a) date of registration shall be considered
(b) date of agreement shall be taken provided the advance is made by any mode other than cash on or before the
date of agreement
(c) date of registration shall be considered provided some advance is made in cash
(d) date of agreement whether or not some advance is made

45. Which of the following interest on bonds/funds are exempt from tax?
(a) National Defence Gold Bonds (c) Public Provident Fund
(b) Power Finance Corporation (d) All of the above
CA SHREY RATHI INCOME FROM OTHER SOURCES 7.16

46. Expenses not deductible u/h Income from other sources are stated under
(a) Section 56 (c) Section 58
(b) Section 57 (d) Section 59

47. Mr. H (aged 40 years) received ₹ 40,500 as interest on fixed deposit from ICICI Bank. The income to be taxed u/h
Income from other sources will be:
(a) ₹ 45,000 (c) ₹ 40,500
(b) Nil (d) ₹ 40,000

48. Mr. N received ₹ 13,40,000 as dividend from foreign company. Discuss its taxability.
(a) Fully exempt
(b) Exempt upto 10,00,000. Balance taxable as per slab rate.
(c) Exempt upto 10,00,000. Balance taxable @ 10% as per section 115BBDA.
(d) Fully taxable

49. Mr. Z gifted two immovable properties to his friend Mr. D. The stamp duty value of both the properties are ₹ 42,000
& ₹ 55,000 respectively. The amount table in the hands of Mr. D shall be:
(a) ₹ 97,000 (c) ₹ 47,000
(b) ₹ 55,000 (d) Nil

50. Mr. Anand received ₹ 28,000 as interest on enhanced compensation. He spent ₹ 10,000 on receiving such amount.
The taxable amount for Mr. Anand shall be:
(a) ₹ 28,000 (c) ₹ 14,000
(b) ₹ 18,000 (d) Not taxable

51. Ms. Shalini received interest on enhanced compensation of ₹ 5,00,000. Out of this interest, ₹ 1,50,000 relates to the
previous year 2020-21, ₹ 1,90,000 relates to previous year 2021-22 and ₹ 1,60,000 relates to previous year 2022-23.
She paid ₹ 1,00,000 to her advocate for his efforts in the matter. What amount would be taxable in P.Y. 2022-23 and
taxable, if any, under which head of income. (ICAI MCQ Booklet)
(a) ₹ 2,50,000 under the head “Income from other sources”
(b) ₹ 4,00,000 under the head “Income from other sources”
(c) ₹ 1,60,000 under the head “Income from other sources”
(d) ₹ 1,60,000 under the head “Capital gains”

MCQ’s Answers
1. (c) 2. (a) 3. (c) 4. (a) 5. (a) 6. (a) 7. (b) 8. (b) 9. (a) 10. (d)
11. (c) 12. (b) 13. (a) 14. (b) 15. (a) 16. (a) 17. (a) 18. (b) 19. (c) 20. (b)
21. (a) 22. (a) 23. (c) 24. (a) 25. (a) 26. (b) 27. (c) 28. (b) 29. (c) 30. (a)
31. (d) 32. (d) 33. (a) 34. (b) 35. (d) 36. (b) 37. (b) 38. (d) 39. (c) 40. (c)
41. (d) 42. (b) 43. (d) 44. (b) 45. (d) 46. (c) 47. (a) 48. (d) 49. (b) 50. (c)
51. (a)
CA SHREY RATHI CLUBBING OF INCOME 8.1

MICHELLE OBAMA

CHAPTER 8
CLUBBING OF INCOME

“You can’t make decisions based on


fear and the possibilities of what
might happen”

PRACTICAL QUESTIONS
Q 1: Income of Z and Mrs. Z for the previous year is as under:
Mr. Z (₹) Mrs. Z (₹)
1. Gross salary from ABC Ltd. 3,60,000 Nil
2. Business Income 12,00,000 2,50,000
3. Income from other sources 80,000 35,000
Z is employed by ABC Ltd. without any technical or professional qualification. Mrs. Z holds 22% equity share capital in ABC
Ltd. from 18th February 2023.
Find out the net income of Z and Mrs. Z for the A/Y 2023-24.

Sol 1: Computation of net income of Z and Mrs. Z for the A/Y 2023-24:
S. No Particulars Mr. Z (₹) Mrs. Z (₹)
1. Gross salary from ABC Ltd. – As Mrs. Z holds 22% equity share capital in ABC
Ltd. (i.e. substantial interest), any salary from such concern received by Mr. Z
without any technical or professional qualification shall be taxable in the hands
of Mrs. Z ₹ 3,60,000
Less: Standard deduction u/s 16(ia) ₹ 50,000 3,10,000
2. Business income 12,00,000 2,50,000
3. Income from other sources 80,000 35,000
Net income 12,80,000 5,95,000

Q 2: A sole proprietor business was started by Mrs. Geetu. As on 01.04.2021 her capital in business was ₹ 3,00,000. Her
mother-in-law gifted ₹ 2,00,000 on 31.05.2021 which was also invested by her in the business on the same date. Mrs.
Geetu earned profits from her business for the financial year 2021-22: ₹ 1,50,000 and financial year 2022-23: ₹ 3,90,000.
Compute the income to be clubbed in the hands of her mother-in-law for the A/Y 2022-23 and 2023-24.

Sol 2: Computation of income to be clubbed in the hands of mother-in-law for A/Y 2022-23 & A/Y 2023-24:
Any income arising from assets transferred by an individual to her daughter in law shall be taxable in the hands of the
transferor. If transferred asset is invested by daughter in law in any business, the following amount shall be clubbed with
the income of the transferor:
CA SHREY RATHI CLUBBING OF INCOME 8.2

Income from business x Amount invested out of asset transferred on the first day of the P/Y
Total investment of transferee on the first day of the P/Y
A/Y 2022-23: Nil, as mother-in-law did not transfer any amount on the first day of the P/Y.
A/Y 2023-24: ₹ 3,90,000 x ₹ 2,00,000 = ₹ 1,20,000
₹ 6,50,000
Total investment for A/Y 2023-24: Self investment + Amount gifted by mother-in-law + Profit earned during the F/Y 2021-
22.
Total investment = ₹ 3,00,000 + ₹ 2,00,000 + ₹ 1,50,000 = ₹ 6,50,000.

Q 3: A and B are minor sons of X and Mrs. X. Business income of X is ₹ 4,00,000. Income from house property of Mrs. X is ₹
2,40,000. Income of A and B from teaching is ₹ 1,20,000 and ₹ 90,000 respectively. Besides income from teaching, A and B
received interest on debentures (debentures were purchased out of income from teaching) of ₹ 25,000 and ₹ 1,400
respectively.
A and B have also received the following birthday gifts:
(i) Gift received by A from X’s friend – ₹ 70,000.
(ii) Gift received by A from X’s sister – ₹ 38,000.
(iii) Gift received by B from his grandmother – ₹ 45,000.
Find out the income of X, Mrs. X, A and B.

Sol 3: Computation of income of X, Mrs. X, A and B:


Particulars X (₹) Mrs. X (₹) A (₹) B (₹)
Business income 4,00,000
Income from house property 2,40,000
Income from teaching (no clubbing as income is earned due to skill 1,20,000 90,000
& talent)
Income of minor children to be included in the income of that
parent whose income (before such clubbing) is greater. However,
that parent shall be allowed an exemption u/s 10(32) of ₹ 1,500
p.a. or income of each minor child whichever is lower.
Minor Child A:
➢ Interest on debentures ₹ 25,000
➢ Gift from X’s friend (non-relative) ₹ 70,000
₹ 95,000
Less: Exemption u/s 10(32) ₹ 1,500 93,500
Minor Child B:
➢ Interest on debentures ₹ 1,400
Less: Exemption u/s 10(32) ₹ 1,400 -
Net Income 4,93,500 2,40,000 1,20,000 90,000
Note: Gift received from X’s sister & grandmother shall not be taxable, being relatives.

Q 4: Mr. Q is an employee of RST Ltd. and has substantial interest in the company. His salary is ₹ 25,000 p.m. Mrs. Q is also
working in that company at a salary of ₹ 10,000 p.m. with professional qualification.
Mr. Q also receives ₹ 30,000 as income from securities. Mrs. Q owns a house property which she has let out. Rent
received from such house property is ₹ 14,000 p.m.
Mr. & Mrs. Q have 3 minor children: twin daughters and one son. Income of the twin daughters is ₹ 2000 p.m. & ₹ 3,000
p.a. respectively and that of his son is ₹ 1,200 p.a.
Compute the income of Mr. Q & Mrs. Q.
CA SHREY RATHI CLUBBING OF INCOME 8.3

Sol 4: Computation of income of Mr. Q & Mrs. Q:


S. No Particulars Mr. Q (₹) Mrs. Q (₹)
1. Salary from RST Ltd.:
• Mr. Q (having substantial interest): ₹ 25,000 x 12 ₹ 3,00,000
Less: Standard deduction u/s 16(ia) ₹ 50,000 2,50,000
• Mrs. Q’s salary will not be added in Mr. Q income as she possesses
professional qualification: ₹ 10,000 x 12 ₹ 1,20,000
Less: Standard deduction u/s 16(ia) ₹ 50,000 70,000
2. Income from house property:
Rent received = NAV (₹ 14,000 x 12) ₹ 1,68,000
(-) Standard Deduction @ 30% ₹ 50,400 1,17,600
3. Income from securities 30,000
4. Income of minor children to be included in the income of that parent whose
income (before such clubbing) is greater. However, that parent shall be
allowed an exemption u/s 10(32) of ₹ 1,500 p.a. or income of each minor child
whichever is lower.
Daughter 1: (₹ 2,000 x 12) – ₹ 1,500 ₹ 22,500
Daughter 2: ₹ 3,000 – ₹ 1,500 ₹ 1,500
Son: ₹ 1,200 – ₹ 1,200 Nil 24,000
Total Income 3,04,000 1,87,600

Q 5: Mrs. Kasturi transferred her immovable property to ABC Co. Ltd. subject to a condition that out of the rental income,
a sum of 1,30,000 p.a. shall be utilised for the benefit of her son’s wife. Mrs. Kasturi claims that the amount of 1,30,000
(utilized by her son’s wife) should not be included in her total income as she is no longer the owner of the property.
Examine with reasons whether the contention of Mrs. Kasturi is valid in law. [ICAI Study Module]

Sol 5: The clubbing provisions u/s 64(1)(viii) are attracted in case of transfer of any asset, directly or indirectly, otherwise
than for inadequate consideration, to any person to the extent to which the income from such asset is for the immediate
or deferred benefit of son’s wife. Such income shall be included in computing the total income of the transferor, i.e. Mrs.
Kasturi. Therefore, the contention of Mrs. Kasturi is, hence, not valid in law.
Note: It has been assumed that the transfer was done for inadequate consideration, and therefore the clubbing provisions
are attracted. However, if it is assumed that the transfer was for adequate consideration, the provisions of section
64(1)(viii) would not be attracted.

Q 6: Mr. D and his wife Mrs. D furnish the following information:


(₹)
(i) Salary income of Mrs. D 4,60,000
(ii) Income of minor son ‘B’ who suffers from disability specified u/s 80U 1,21,000
(iii) Income of minor daughter ‘C’ from singing 98,000
(iv) Income from profession of Mr. D 7,50,000
(v) Cash gift received by ‘C’ from friend of Mrs. D on his birthday 45,000
(vi) Income of minor married daughter ‘A’ from company deposit 30,000
Compute the total income of Mr. D & Mrs. D for the A/Y 2023-24.
CA SHREY RATHI CLUBBING OF INCOME 8.4

Sol 6: Computation of total income of Mr. D & Mrs. D for the A/Y 2023-24:
S. No Particulars Mr. D (₹) Mrs. D (₹)
(i) Salary income - 4,60,000
(ii) Income of minor son ‘B’ shall not be clubbed as he is suffering from disability
specified u/s 80U - -
(iii) Income of minor daughter ‘C’ from singing (income of minor child from skill,
manual work or talent shall not be clubbed) - -
(iv) Professional income 7,50,000 -
(v) Cash gift received by ‘C’ from Mrs. D’s friend (not taxable as the cash gift does
not exceed ₹ 50,000 u/s 56(2)(x)) - -
(vi) Income of minor married daughter ‘A’ from company deposits shall be
included in the income of that parent whose income (before such clubbing) is
greater. However, that parent shall be allowed an exemption u/s 10(32) of ₹ 28,500 -
1,500 p.a. or income of each minor child whichever is lower. (₹ 30,000 – ₹
1,500)
Total Income 7,78,500 4,60,000

Q 7: Mr. Vaibhav started a proprietary business on 01.04.2021 with a capital of 5,00,000. He incurred a loss of 2,00,000
during the year 2021-22. To overcome the financial position, his wife Mrs. Kirti, a software engineer gave a gift of 5,00,000
on 01.04.2022, which was immediately invested by Vaibhav in his business. He earned a profit of 4,00,000 during the year
2022-23. Compute the amount to be clubbed in the hands of Mrs. Kirti for the A/Y 2023-24.
Will your answer change if the said amount was given as loan to Vaibhav instead of gift? [ICAI Module]

Sol 7: Section 64(1)(iv) of the Income-tax Act, 1961 provides for the clubbing of income in the hands of the individual, if
the income earned is from the assets (other than house property) transferred directly or indirectly to the spouse of the
individual, otherwise than for adequate consideration or in connection with an agreement to live apart.
In this case Mr. Vaibhav received a gift of ₹ 5,00,000 on 1.4.2022 from his wife Mrs. Kirti, which he invested in his business
immediately. The income to be clubbed in the hands of Mrs. Kirti for the A/Y 2023-24 is computed as under:
Particulars Mr. Vaibhav’s capital Capital contribution out of Total (₹)
contribution (₹) gift from Kirti (₹)
Capital as on 1.4.2022 3,00,000 5,00,000 8,00,000
(5,00,000 - 2,00,000)
Profit for P/Y 2022-23 to be apportioned on
the basis of capital employed on the first day 1,50,000 2,50,000 4,00,000
of the previous year i.e. as on 01.04.2022 (4,00,000 x 3/8) (4,00,000 x 5/8)
(3:5)

Therefore, the income to be clubbed in the hands of Mrs. Kirti for the A/Y 2023-24 is ₹ 2,50,000.
In case Mrs. Kirti gave the said amount of ₹ 5,00,000 as a loan, then clubbing provisions would not get attracted.
Note: The provisions of Section 56(2)(x) would not be attracted in the hands of Mr. Vaibhav, since he has received such
sum of money from a relative i.e. his wife.

Q 8: During the P/Y 2022-23, the following transactions occurred in respect of Mr. X:
(a) During the year Mr. X got a salary of ₹ 10,000 p.m. while his wife received a salary of ₹ 15,000 p.m.
(b) Mr. X had a fixed deposit of ₹ 5,00,000 in SBI. He instructed the bank to credit the interest on the deposit @ 9% for
the whole year to the saving bank account of Mr. P, son of his brother, to help him in education.
(c) Mr. X holds 62% share in a partnership firm. His wife received a commission of ₹ 35,000 from the firm for
promoting the sales of the firm. Mrs. X possesses technical and professional qualification.
(d) Mr. X gifted a flat to Mrs. X on 1st May, 2022. During the previous year the flat had income u/h House Property ₹
52,000 to Mrs. X.
CA SHREY RATHI CLUBBING OF INCOME 8.5

(e) Mr. X gifted ₹ 3,00,000 to his minor son Mr. A who invested the same in a business and he got a share of income of
₹ 30,000 from the investment.
(f) Ms. B, minor married daughter of X derived an income of ₹ 45,000 through a business activity involving application
of her skill and talent.
Compute the total income of Mr. X, Mrs. X and their minor children.

Sol 8: Computation of total income of Mr. X, Mrs. X, Mr. A & Ms. B:


S. No. Particulars X (₹) Mrs. X (₹) A (₹) B (₹)
(a) Gross salary income
Mr. X 1,20,000
Less: Standard deduction u/s 16(ia) 50,000 70,000
Mrs. X 1,80,000
Less: Standard deduction u/s 16(ia) 50,000 1,30,000
(b) Transfer of interest without transfer of asset shall be 45,000
taxable in the hands of the transferor u/s 60. (₹ 5,00,000
x 9%)
(c) Remuneration received by a spouse having technical and 35,000
professional qualification from a partnership firm in
which the other spouse has substantial interest shall not
be taxable in the hands of the substantial interest holder
spouse.
(d) Gift of house property by Mr. X to his spouse shall be 52,000
taxable in the hands of the transferor.
(e) Return on investment received by minor son out of the
funds gifted by his father be included in the income of 28,500
that parent whose income (before such clubbing) is
greater. However, that parent shall be allowed an
exemption u/s 10(32) of ₹ 1,500 p.a. or income of each
minor child whichever is lower. (₹ 30,000 – ₹ 1,500)
(f) Income of minor married daughter from a business
involving skill and talent shall not be clubbed. 45,000
Total Income 1,95,500 1,65,000 Nil 45,000

Q 9: Determine the Gross Total Income of P and his wife Q from the following particulars for the year ending 31.03.2023:
1. P and his wife Q are partners in a firm carrying on electronics business, their respective shares of profit being ₹
90,000 and ₹ 60,000.
2. A house property in the name of P was transferred to his wife on 01.12.2022 for adequate consideration. The
property was let out throughout the year at ₹ 30,000 p.m.
3. Debentures of a company of ₹ 1,40,000 and ₹ 1,12,000 purchased three years ago are in the names of P and Q
respectively, on which interest is receivable @ 10% p.a. Q had in the past transferred ₹ 70,000 out of her income to P
for the purchase of the debentures in P’s name.
4. P had transferred ₹ 50,000 to Q in the year 2016 without any consideration which was given as a loan by her to M.
She earned ₹ 20,000 as interest during the earlier previous years which was also given as loan to M. During the P/Y
2022-23, she received interest @ 10% p.a. on ₹ 70,000.
5. P transferred ₹ 75,000 to a trust, the income accruing from its investment as interest amounted to ₹ 7,500, out of
which ₹ 5,000 shall be utilised for the benefit of son’s wife and ₹ 2,500 for the benefit of his son’s minor child.
6. Their 15 years old daughter was admitted to the benefits of another firm, from which she received ₹ 70,000 as her
share of profit in the firm and ₹ 90,000 as interest on capital. The capital was invested out of the minor’s own funds
amounting to ₹ 9,00,000.
CA SHREY RATHI CLUBBING OF INCOME 8.6

Sol 9: Computation of total income of Mr. P & Mrs. Q


S. No Particulars Mr. P (₹) Mrs. Q (₹)
1. Share of profit in a firm is taxable in the hands of the firm and therefore shall - -
not be taxable in the hands of partners.
2. Transfer of house property for adequate consideration by P to Q shall not be 1,68,000 84,000
clubbed.
P: (₹ 30,000 x 8) – standard deduction @ 30%
Q: (₹ 30,000 x 4) – standard deduction @ 30%
3. Interest on debentures. Transfer of ₹ 70,000 by Q to P for purchase of 7,000 18,200
debentures shall be clubbed in the hands of Q.
P: (₹ 1,40,000 – ₹ 70,000) x 10%
Q: (₹ 1,12,000 + ₹ 70,000) x 10%
4. Transfer of ₹ 50,000 by P to Q from which interest has accrued shall be taxable 5,000 2,000
in the hands of P, however interest on accrued interest shall not be clubbed
and therefore shall be taxable in the hands of Q.
5. Transfer of an asset by an individual to a trust for the benefit of his daughter-
in-law shall be taxable in the hands of the individual. However, no clubbing 5,000 -
shall be done where the benefit reaches his son’s minor child (i.e. grandchild).
Share of profit received by minor child from a firm shall not be taxable as
6. income of firm is taxable in its own hands. But interest on capital shall be
included in the income of that parent whose income (before such clubbing) is 88,500 -
greater. However, that parent shall be allowed an exemption u/s 10(32) of ₹
1,500 p.a. or income of each minor child whichever is lower. (₹ 90,000 – ₹
1,500)
Total Income 2,73,500 1,04,200

Q 10: Mr. B is the Karta of a HUF, whose members derive income as given below:
(i) Income from B’s profession – ₹ 55,000.
(ii) Mrs. B’s salary as fashion designer – ₹ 76,000.
(iii) Minor Son ‘D’ (interest on fixed deposits with a bank which were gifted to him by his uncle) – ₹ 10,000.
(iv) Minor Daughter P’s earnings from sports – ₹ 85,000.
(v) D’s winnings from lottery (gross) – ₹ 1,40,000
Discuss the tax implications in the hands of Mr. B and Mrs. B regarding the above incomes.

Sol 10: Treatment of various incomes in the hands of Mr. B & Mrs. B:
(i) Income of ₹ 55,000 from Mr. B’s profession shall be taxable in the hands of Mr. B u/h PGBP.
(ii) Salary of ₹ 76,000 received by Mrs. B as a fashion designer shall be taxable u/s Salaries in the hands of Mrs. B.
(iii) Income from fixed deposit of ₹ 10,000 arising to the minor son shall be clubbed in the income of that parent whose
income (before such clubbing) is greater. However, that parent shall be allowed an exemption u/s 10(32) of ₹ 1,500
p.a. or income of each minor child whichever is lower. As income of mother is greater, ₹ 8,500 shall be included in
her income.
(iv) Income of ₹ 85,000 arising to the minor daughter P from sports shall not be included in the hands of the parent,
since such income has arisen on account of an activity involving application of her skill.
(v) Income of ₹ 1,40,000 arising to minor son ‘D’ from lottery shall be included in the income of Mrs. B, since her income
is greater than the income of Mr. B before including the income of minor child.

Q 11: Mr. Sarthak is a member of HUF. It consists of himself, his wife Juhi and his major son Arjun and his minor daughter
Aditi. Sarthak transferred his house property acquired through his personal income to the hug without any consideration.
On 01-10-2022, HUF is partitioned and such property being divided equally. Net Annual Value of the property for the
previous Year 2022-23 is ₹ 1,00,000. Determine the tax implications. [PYQ – May 2022]
CA SHREY RATHI CLUBBING OF INCOME 8.7

Sol 11:
Particulars (₹)
Since Mr. Sarthak, who is a member of the HUF, transfers the house property acquired by him out of his 35,000
personal income to the HUF without any consideration, the income from such property would continue
to be included in his total income upto the date of partition. Accordingly, income from such property for
six months upto the date of partition i.e., 30.9.2022 (6/12 x ₹ 70,000 [Net Annual Value of ₹ 1,00,000
less deduction under section 24(a) @ 30%) would be included in the total income of Mr. Sarthak.
Since the HUF was partitioned on 1.10.2021, the income derived from such converted house property as
is received by Mr. Sarthak’s spouse, Juhi, on partition will be deemed to arise to Mr. Sarthak from house
property transferred indirectly by him to her and consequently, such income would also be included in
the total income of Mr. Sarthak. Accordingly, Mr. Sarthak’s share (25%) and Juhi’s share (25%) would be
included in the total income of Mr. Sarthak.
o Sarthak’s Share [25% of ₹ 35,000 (₹ 70,000 x 6/12)] 8,750
o Juhi’s Share [25% of ₹ 35,000] included in the total income of Sarthak 8,750
Income from house property includible in the income of Mr. Sarthak 52,500
o 25% share of Sarthak’s minor daughter, Aditi, i.e., ₹ 8,750, being 25% of ₹ 35,000, would be included
in the total income of Mr. Sarthak or Juhi, whosoever’s total income, before including Aditi’s income,
is higher. Such parent shall be entitled to an exemption of ₹ 1,500 under section 10(32).
o 25% share of Sarthak’s major son, Arjun, i.e., ₹ 8,750, being 25% of ₹ 35,000, would be included in
Arjun’s total income. Distribution of house property on partition of HUF is not a transfer for levy of
capital gains tax.

MULTIPLE CHOICE QUESTIONS


1. Income of a minor child who is suffering from disability of the nature as specified in Section 80U is
(a) to be assessed in the hands of the minor child
(b) to be clubbed with the income of that parent whose total income, before including minor’s income, is higher
(c) completely exempt from tax
(d) to be clubbed with the income of father

2. Income arising to a minor married daughter is


(a) to be assessed in the hands of the minor married daughter
(b) to be clubbed with the income of that parent whose total income, before including minor’s income, is higher
(c) completely exempt from tax
(d) to be clubbed with the income of her husband

3. Where a member of a HUF has transferred his self-acquired property for inadequate consideration into joint family
property, income arising therefrom is
(a) taxable as the income of the transferor member.
(b) taxable in the hands of the HUF.
(c) taxable in the hands of the Karta of the HUF.
(d) exempt from tax.

4. If the converted property is subsequently partitioned among the members of the family, the income derived from
such converted property as is received by the spouse of the transferor will be taxable – (ICAI MCQ Booklet)
(a) as the income of the Karta of the HUF
(b) as the income of the spouse of the transferor
(c) as the income of the HUF
(d) as the income of the transferor-member

5. Mr. J gifts a sum of ₹ 1,00,000 to his brother’s wife Mrs. B. Mr. B gifts a sum of ₹ 1,00,000 to Mrs. J. From the sum
gifted to her, Mrs. B invests in a fixed deposit, income therefrom is ₹ 10,000. Aforesaid ₹ 10,000 will be included in
the total income of
(a) Mr. A (c) Mrs. B
(b) Mrs. A (d) Mr. B
CA SHREY RATHI CLUBBING OF INCOME 8.8

6. Income of a minor child from a fixed deposit with a bank, made out of income earned from scholarship is -
(a) to be assessed in the hands of the minor child
(b) to be clubbed with the income of that parent whose total income, before including minor’s income, is higher
(c) completely exempt from tax
(d) to be clubbed with the income of father

7. Mr. Kumar transfers income of ₹ 51,000 from rent to his major son without transfer of house property. Rent of ₹
51,000 is -
(a) taxable in the hands of the transferor - father
(b) taxable in the hands of his son
(c) taxable in the hands of the that parent whose total income is higher
(d) exempt from tax

8. Mr. GJ and Mrs. GJ hold 15% and 10% shares in a concern and both are employed by that firm getting the monthly
salary of ₹ 20,000 respectively. Their remuneration does not match their technical or professional skills or
experience. Apart from the salary Income Mr. GJ has earned ₹ 2,00,000 from cloth business whereas Mrs. GJ has
earned ₹ 2,00,000 as rent of the house property. Mr. GJ has invested ₹ 50,000 in PPF account and another ₹ 50,000
in NSC. While Mrs. GJ has invested ₹ 1,00,000 in NSC and has donated ₹ 10,000 to PMNRF. Whose total income will
be higher for clubbing of remuneration of both of them?
(a) Mr. GJ (c) Both have equal income
(b) Mrs. GJ (d) None of the above

9. If there is a transfer of asset which is not revocable during the life time of the transferee, income arising from such
asset shall be included in the income of:
(a) transferor
(b) transferee
(c) transferee till his death and thereafter in the hands of the transferor
(d) transferor or transferee at their mutual consent

10. For the provisions of clubbing of incomes to calculate substantial interest we consider the holding of:
(a) the individual only (c) the individual along with his relatives
(b) the individual & his spouse taken together (d) the individual & his spouse taken separately

11. Mr. Kunal gifts ₹ 1,00,000 to his wife, Mrs. Priyanka on 10/4/2022, who invested the same in the business of cloth
which is being run by her for last 3 years. Mrs. Priyanka earns ₹ 20,000 as profits from such business for the PY 2022-
23. What amount will be clubbed in the income of Mr. Kunal:
(a) ₹ 20,000 (c) ₹ 1,20,000
(b) ₹ 10,000 (d) NIL

12. When income of minor child is clubbed in the income of the parent, such parent will be allowed exemption of:
(a) ₹ 1,500 p.m.
(b) ₹ 1,500 p.a. per minor child
(c) to the extent of actual income clubbed or ₹ 1,500 p.a. per minor child, whichever is less.
(d) No exemption is allowed

13. Mr. K transfers the house property to his minor child without adequate consideration, then income such house
property shall be:
(a) clubbed in the income of Mr. K as per section 27 and exemption of ₹ 1,500 shall be allowed
(b) clubbed in the income of Mr. K as per section 27 but exemption of ₹ 1,500 shall not be allowed
(c) clubbed in the income of Mr. K as per section 64(1A) but exemption of ₹ 1,500 shall not be allowed
(d) clubbed in the income of Mr. K as per section 64(1A) and exemption of ₹ 1,500 shall be allowed
CA SHREY RATHI CLUBBING OF INCOME 8.9

14. If both of the parents of child have died then his income shall be:
(a) exempt from tax.
(b) taxable for the guardian who maintains the child
(c) shall be taxable for child who has earned it
(d) taxable for dead parents.

15. If the marriage of the parents does not subsist, the income of the minor child shall be clubbed in the income of:
(a) father or mother who is loved more
(b) parent who maintains the child
(c) father or mother whose income is higher
(d) neither with mother nor with father, but shall be taxable for child itself

16. Mr. Pathak as on 1/10/2021 transferred shares without consideration to his fiancée, Ms. Kumari. They got married
on 1/4/2022. For the PY 2022-23, Income from share shall be assessed in hands of:
(a) Mr. Pathan (c) Any of the above, with their mutual consent
(b) Ms. Kumari (d) Neither of the above

17. Mr. Alok formed a trust for the benefit of his wife. The income of trust meant for the benefit of Mrs. Roopam shall be
assessed in the hands of
(a) Mrs. Roopam (c) Both for them partially
(b) Mr. Alok (d) None of them

18. Income from asset transferred to spouse is taxable u/s


(a) 64(1)(iv) (c) 64(1)(vii)
(b) 64(1)(vi) (d) 64(1)(viii)

19. For the purpose of clubbing of income of the specified person in the income of the individual under section 64, the
word ‘income’ includes .....................?
(a) Loss (c) Interest
(b) Expenses (d) All of the above

20. At the time of fixation of marriage of his son, Mr. Satish gifted land to his would be daughter in law. The marriage
was held in the month subsequent to the date of transfer. The income accruing on land after marriage shall be
taxable in the hands of
(a) Mr. Satish (c) Equally in hands of (a) and (b)
(b) Daughter in law (d) None of the above

21. If there is revocable transfer of an asset by any person to another person, any income arising from such asset shall be
included in the income of:
(a) Transferor (c) Both transferor & transferee
(b) Transferee (d) None of the above

22. Clubbing provisions under section 64(1)(vi) are applicable where the asset is transferred by an individual without an
adequate consideration to:
(a) Daughter’s husband (c) Minor child
(b) Daughter in law (d) Spouse

23. Mr. Mayank is a Chartered Accountant and is working as Accounts Officer in GGC Pvt. Ltd. on a salary of ₹ 20,000
p.m. He got married to Mrs. Parul who holds 25% shares of this company. What will be the impact of salary paid to
Mr. Mayank by the company in the hands of Mrs. Parul:
(a) 100% salary to be clubbed (c) No amount be clubbed
(b) 50% salary to be clubbed (d) 25% salary be clubbed
CA SHREY RATHI CLUBBING OF INCOME 8.10

24. Mr. Shiva gifted a let-out building which fetches rental income on ₹ 10,500 per month to his son’s wife on
1/11/2022. The municipal Tax of ₹ 6,000 on the property was paid on 10/1/2023. The total income from all other
sources (computed) amounts to ₹ 2,60,000 except income from above said property. His total income chargeable to
tax is:
(a) ₹ 3,11,450 (c) ₹ 3,80,000
(b) ₹ 2,92,550 (d) ₹ 3,44,000

25. Mr. Jagga transferred his let out residential property to his wife by way of gift on 1/4/2022. During the PY 2022-23,
she earned rental income of ₹ 30,000 per month. She made fixed deposit in a bank out of such rental income and
earned interest income during the year of ₹ 21,000. The total amount of income liable for clubbing in the hands of
Mr. Jagga for the PY 2022-23 is:
(a) Nil (c) ₹ 2,52,000
(b) ₹ 21,000 (d) ₹ 2,73,000

26. The provisions relating to income from self-occupied property converted to joint family property shall be dealt under
(a) Section 64(1A) (c) Section 64(2)
(b) Section 65 (d) Section 64(1)(ii)

27. For the purposes of Section 64(1)(ii), an individual shall be deemed to have substantial interest if he along with his
relatives is entitled to atleast ……….. of the voting power.
(a) 10% (c) 15%
(b) 5% (d) 20%

28. Mr. Lallu transferred debentures of ABC Ltd. to his wife, Mrs. Laali for no consideration. Later on the wife sold the
debentures and purchased a residential property out of the sale consideration. The house property was further
rented out to a tenant. The rental income shall be taxable in the hands of:
(a) Mr. Lallu (c) Equally between both of them
(b) Mrs. Laali (d) Will be decided by the assessing officer

29. Mr. Bhavesh has 4 children. The first child who is suffering from specified disease has earned ₹ 35,000 by way of
interest from fixed deposit. The second child won ₹ 50,000 from a quiz competition. The third child earned ₹ 40,000
as share of profit from a firm & the fourth child was gifted ₹ 51,000 from his mother’s sister. Compute the amount to
be clubbed in the hands of Mr. Bhavesh assuming the income of the spouse is lower.
(a) ₹ 48,500 (c) ₹ 87,000
(b) Nil (d) ₹ 1,37,500

30. In which of the following cases, where an asset is transferred by wife to husband will not be clubbed:
(a) Transfer is done with adequate consideration
(b) Transfer is under an agreement to live apart
(c) Relationship didn’t exist either at the time of transfer of the asset or at the time of accrual of income
(d) All of the above

31. Mr. Aarav gifted a house property valued at ₹ 50 lakhs to his wife, Geetha, who in turn has gifted the same to her
daughter-in-law Deepa. The house was let out at ₹ 25,000 per month throughout the P.Y. 2022-23. Compute income
from house property for A.Y. 2023-24. In whose hands is the income from house property chargeable to tax? (ICAI
MCQ Booklet)
(a) ₹ 3,00,000 in the hands of Mr. Aarav (c) ₹ 2,10,000 in the hands of Geetha
(b) ₹ 2,10,000 in the hands of Mr. Aarav (d) ₹ 2,10,000 in the hands of Deepa
CA SHREY RATHI CLUBBING OF INCOME 8.11

32. Ram owns 500, 15% debentures of Reliance Industries Ltd. of ₹ 500 each. Annual interest of ₹ 37,500 was declared on
these debentures for P.Y. 2022-23. He transfers interest income to his friend Shyam, without transferring the
ownership of these debentures. While filing return of income for A.Y. 2023-24, Shyam showed ₹ 37,500 as his income
from debentures. As tax advisor of Shyam, do you agree with the tax treatment done by Shyam in his return of
income? (ICAI MCQ Booklet)
(a) Yes, since interest income was transferred to Shyam therefore, after transfer it becomes his income.
(b) No, since Ram has not transferred debentures to Shyam, interest income on the debentures is not taxable income
of Shyam
(c) Yes, if debentures are not transferred, interest income on debentures can be declared by anyone, Ram or Shyam,
as taxable income depending upon their discretion.
(d) No, since Shyam should have shown the income as interest income received from Mr. Ram and not as interest
income earned on debentures.

33. Mrs. Shivani, wife of Mr. Anurag, is a partner in a firm. Her capital contribution of ₹ 5 lakhs to the firm as on 1-4-2022
included ₹ 3.5 lakhs contributed out of gift received from Anurag. The firm paid interest on capital of ₹ 50,000 and
share of profit of ₹ 60,000 during the F.Y. 2022-23. The entire interest has been allowed as deduction in the hands of
the firm. Which of the following statements is correct? (ICAI MCQ Booklet)
(a) Share of profit is exempt but interest on capital is taxable in the hands of Mrs. Shivani.
(b) Share of profit is exempt but interest of ₹ 39,286 is includible in the income of Mr. Anurag and interest of ₹
10,714 is includible in the income of Mrs. Shivani.
(c) Share of profit is exempt but interest of ₹ 35,000 is includible in the income of Mr. Anurag and interest of ₹
15,000 is includible in the income of Mrs. Shivani.
(d) Share of profit to the extent of ₹ 42,000 and interest on capital to the extent of ₹ 35,000 is includible in the hands
of Mr. Anurag.

34. Mr. Arvind gifted a house property to his wife, Ms. Meena and a flat to his daughter-in-law, Ms. Seetha. Both the
properties were let out. Which of the following statements is correct? (ICAI MCQ Booklet)
(a) Income from both properties is to be included in the hands of Mr. Arvind by virtue of section 64.
(b) Income from property gifted to wife alone is to be included in Mr. Arvind’s hand by virtue of section 64.
(c) Mr. Arvind is deemed owner of house property gifted to Ms. Meena and Ms. Seetha.
(d) Mr. Arvind is deemed owner of property gifted of Ms. Meena. Income from property gifted to Ms. Seetha would
be included in his hands by virtue of section 64.

35. On 20-10-2022, Pihu (minor child) gets a gift of ₹ 20,00,000 from her father’s friend. On the same day, the amount is
deposited as fixed deposit in Pihu’s bank account. On the said deposit, interest of ₹ 13,000 was earned during the P.Y.
2022-23. In whose hands the income of Pihu shall be taxable? also compute the amount of income that shall be
taxable. (ICAI MCQ Booklet)
(a) Income of ₹ 20,11,500 shall be taxable in the hands of Pihu’s father.
(b) Income of ₹ 20,13,000 shall be taxable in the hands of Pihu’s father.
(c) Income of ₹ 20,11,500 shall be taxable in the hands of Pihu’s father or mother, whose income before this clubbing
is higher
(d) Income of ₹ 20,13,000 shall be taxable in the hands of Pihu’s father or mother, whose income before this clubbing
is higher

MCQ Answers
1. (a) 2. (b) 3. (a) 4. (d) 5. (d) 6. (b) 7. (a) 8. (a) 9. (c) 10. (c)
11. (d) 12. (c) 13. (b) 14. (c) 15. (b) 16. (b) 17. (b) 18. (a) 19. (d) 20. (b)
21. (a) 22. (b) 23. (c) 24. (d) 25. (c) 26. (c) 27. (d) 28. (a) 29. (b) 30. (d)
31. (b) 32. (b) 33. (c) 34. (d) 35. (c)
CA SHREY RATHI SET OFF AND CARRY FORWARD OF LOSSES 9.1

DHIRUBHAI AMBANI

CHAPTER 9
SET OFF AND CARRY FORWARD OF
LOSSES
“Our dreams have to be bigger. Our
ambitious higher. Our commitment
deeper & our efforts greater”

PRACTICAL QUESTIONS
Q 1: Mr. P, a resident individual, furnishes the following particulars of his income and other details for the previous year
2022-23:
S. No. Particulars (₹)
(i) Income from salary 18,000
(ii) Net annual value of house property 70,000
(iii) Income from business 80,000
(iv) Income from speculative business 12,000
(v) Long term capital gain on sale of land 15,800
(vi) Loss on maintenance of race horse 9,000
(vii) Loss on gambling 8,000
Depreciation allowable under the Income Tax Act 1961, comes to ₹ 8,000, for which no treatment is given above.
The other details of unabsorbed deprecation and forward losses (pertaining to A/Y 2022-23) are:
S. No. Particulars (₹)
(i) Unabsorbed depreciation 9,000
(ii) Loss from speculative business 16,000
(iii) Short term capital loss 7,800
Compute the gross total income of Mr. P for the Assessment year 2023-24, and the amount of loss that can or cannot be
carried forward. [ICAI Study Module]
CA SHREY RATHI SET OFF AND CARRY FORWARD OF LOSSES 9.2

Sol 1: Computation of Gross Total Income of Mr. P for the A/Y 2023-24:
Particulars (₹) (₹)
Income from salary 18,000
Income from House property
Net Annual Value 70,000
Less: Statutory deduction u/s 24(a) @ 30% 21,000 49,000
Income from business and profession
a) Income from business 80,000
Less: Current Year deprecation 8,000
72,000
Less: Unabsorbed deprecation 9,000 63,000
b) Income from speculative business 12,000
Less: Brought forward speculative loss 12,000 Nil
Balance loss of ₹ 4,000 (i.e. ₹ 16,000 - ₹ 12,000) can be carried forward to the next year)
Income from capital gain
Long term capital gain on sale of land 15,800
Less: Brought-forward short-term capital loss 7,800 8,000
Gross Total Income 1,38,000
Amount of loss to be carried forward to the next year
Particulars (₹)
Loss from speculative business (to be carried forward as per section 73) 4,000
Loss on maintenance of race horses (to be carried forward as per section 74A) 9,000
Notes:
1. Loss on gambling can neither be set-off nor be carried forward.
2. As per section 74A(3), the loss incurred on maintenance of race horse cannot be set-off against income from any
other source other than the activity of owning and maintaining race horses. Such loss can be carried forward for a
maximum period of 4 assessment years.
3. Speculative business loss can set-off against income from speculative business of the current year and the balance loss
can be carried forward to A/Y 2023-24. It may be noted that speculative business loss can be carried forward for a
maximum of 4 years as per section 73(4).

Q 2: Mr. Siddhant submits the following details of his income for the A/Y 2023-24.
Particulars (₹)
Income from salary 3,00,000
Loss from let out-house property (-) 40,000
Income from sugar business 50,000
Loss from iron ore business brought forward (discontinued in P/Y 2017-18) (-) 1,20,000
Short term capital loss (-) 60,000
Long term capital gains 40,000
Dividend 5,000
Income received from lottery wining (Gross) 50,000
Wining from card games 6,000
Agricultural income 20,000
Short term capital loss under section 111A (-) 10,000
Bank interest 5,000
Calculate gross total income and losses to be carried forward assuming that he does not opt for the provisions of Section
115BAC. [ICAI Study Module]
CA SHREY RATHI SET OFF AND CARRY FORWARD OF LOSSES 9.3

Sol 2: Computation of Gross Total Income of Mr. Siddhant for the A/Y 2023-24:
Particulars (₹) (₹)
Salaries
Income from salary 3,00,000
Less: Loss from house property set-off against salary income u/s 71 (40,000) 2,60,000
Profits and gains of Business or Profession
Income from sugar business 50,000
Less: Brought forward loss from iron-ore business set-off u/s 72(1) (50,000) Nil
Balance business loss of ₹ 70,000 of P/Y 2017-18 to be carried forward to A/Y 2024-
25.
Capital Gains
Long term capital gain 40,000
Less: Short-term capital loss set-off (40,000) Nil
Balance short-term capital loss of ₹ 20,000 to be carried forward
Short-term capital loss of ₹ 10,000 under section 111A also to be carried forward
Income from Other Sources
Dividend (fully taxable in the hands of shareholders) 5,000
Winnings from lottery 50,000
Winnings from card games 6,000
Bank interest 5,000 61,000
Gross Total Income 3,21,000
Losses to be carried forward to A/Y 2024-25:
Loss of iron-ore business brought forward of P/Y 2016-17 (₹ 1,20,000 – ₹ 50,000) 70,000
Short term capital loss (₹ 20,000 + ₹ 10,000) 30,000
Notes:
1. Agriculture income is exempt u/s 10(1).
2. It is presumed that loss from iron-ore business relates to P.Y. 2017-18, the year in which the business was
discontinued.

Q 3: Mr. A furnishes you the following information for the year ended 31-03-2023.
S. No. Particulars (₹)
(i) Income from plying of vehicles (computed as per books) (He owned 5 goods vehicles (other than 3,20,000
heavy goods vehicle) throughout the year
(ii) Income from retail trade of garments (computed as per books) (Sales turnover ₹ 1,35,70,000) 7,50,000
Mr. A has declared income on presumptive basis under section 44AD for the first time in A/Y
2022-23. Assume 10% of the turnover during the previous year 2022-23 was received in cash and
the balance through account payee cheque and all the payments in respect of expenditure were
also made through A/C Payee cheque or debit card.
(iii) He has brought forward deprecation relating to A.Y. 2020-21 1,00,000
Compute taxable income of Mr. A and his tax liability for the A/Y 2023-24 with reasons for your computation, assuming
that he does not opt for section 115BAC. [ICAI Study Module]

Sol 3: Computation of total income and tax liability of Mr. A for the A/Y 2023-24:
Particulars (₹)
Income from retail trade – as per books (See Note 1 below) 7,50,000
Income from playing of vehicles – as per books (See Note 2 below) 3,20,000
10,70,000
Less: Set off of brought forward deprecation relating to A/Y 2020-21 1,00,000
Total Income 9,70,000
Tax Liability 1,06,500
Add: Health and Education Cess @ 4% 4,260
Total Tax Liability 1,10,760
CA SHREY RATHI SET OFF AND CARRY FORWARD OF LOSSES 9.4

Notes:
1. Income from retail trade: Presumptive business income under section 44AD is ₹ 8,41,340 i.e. 8% of ₹ 13,57,000 being
10% of the turnover received in cash and 6% of 1,22,13,000 being the amount of turnover received through A/c payee
cheque. However, the income computed as per books is ₹ 7,50,000 which is to be further reduced by the amount of
unabsorbed deprecation of ₹ 1,00,000, since the income computed as per books is lower than the income deemed
under section 44AD, the assessee can adopt the income as per books.
However, if he does not opt for presumptive taxation under section 44AD, he has to get his books of account audited
under section 44AB, since his turnover exceeds ₹ 1 crore (the enhanced limit of ₹ 10 crores would not be available,
since more than 5% of the turnover is received in cash). It may further be noted that he cannot opt for section 44AD
for the next five assessment years, if he does not opt for section 44AD this year.
2. Income from plying of vehicles: Income calculated u/s 44AE(1) would be ₹ 7,500 x 12 x 5 which is equal to ₹ 4,50,000.
However, the income from plying of vehicles as per books ₹ 3,20,000, which is lower than the presumptive income of
₹ 4,50,000 calculated as per section 44AE(1). Hence, the assessee can adopt the income as per books i.e. ₹ 3,20,000,
provided he maintains books of accounts as per section 44AA and gets his accounts audited and furnishes an audit
report as required under section 44AB.
3. It is to be further noted that in both the above cases, had presumptive income provisions been opted, all deduction
under section 30 to 38, including depreciation would have been deemed to have been given full effect to and no
further deduction under those section would be allowable.

Q 4: Mr. Batra furnishes the following details for year ended 31.03.2023:
Particulars (₹)
Short term capital gain 1,40,000
Loss from speculative business 60,000
Long term capital gain on sale of land 30,000
Long term capital loss on sale of shares (securities transaction tax has not been paid) 1,00,000
Income from business of textile (after allowing current year deprecation) 50,000
Income from activity of owning and maintaining race horses 15,000
Income from salary 1,00,000
Loss from house property 40,000
Following are the brought forward losses:
(1) Losses from activity of owning and maintaining race horses pertaining to A.Y. 2020-21 - ₹ 25,000.
(2) Brought forward loss from business of textile ₹ 60,000 loss pertains to A.Y. 2015-16.
Compute gross total income of Mr. Batra for the assessment year 2023-24 assuming that he does not opt for Section
115BAC. Also state the eligible carry forward losses for the assessment year 2024-25. [ICAI Study Module]

Sol 4: Computation of gross total income of Mr. Batra for the A.Y. 2023-24:
Particulars (₹) (₹)
Salaries 1,00,000
Less: Current year loss from house property (40,000) 60,000
Profit and gains of business or profession
Income from textile business 50,000
Less: Loss from textile business brought forward from A.Y. 2015-16 50,000 Nil
Balance business loss of A/Y 2015-16 – ₹ 10,000 [Note 1]
Capital gain
Short term capital gain 1,40,000
Long term gain on sale of land 30,000
Less: long term capital loss on sale of shares 30,000 Nil
Loss to be carried forward to A/Y 2024-25 – ₹ 70,000 [Note 2]
Income from activity of owning and maintaining race horses
Less: loss from activity of owning and maintaining race horses brought forward from 15,000
A.Y. 2020-21.
Loss to be carried forward to A/Y 2024-25 – ₹ 10,000 [Note 3] 15,000 Nil
Gross Total Income 2,00,000
CA SHREY RATHI SET OFF AND CARRY FORWARD OF LOSSES 9.5

Losses to be carried forwards A/Y 2024-25:


Particulars (₹)
Current year losses from speculative business [Note 4] 60,000
Current year long term capital loss on sale of shares 70,000
Loss from activity of owing and maintaining of race horse pertaining to A.Y. 2020-21 10,000
Notes:
1. As per section 72(3), business loss can be carried forward for a maximum of 8 assessment years immediately
succeeding the assessment year for which the loss was first computed. Since, eight years period for carry forward of
business loss of A/Y 2015-16 expired with A/Y 2023-24, the balance unabsorbed business loss of ₹ 10,000 cannot be
carried forward to A/Y 2024-25.
2. Long term capital loss on sale of such shares can be set-off against long term capital gain on sale of land. The balance
loss of ₹ 70,000 can not be set-off against short term capital gain or against any other head of income. The same has
to be carried forward for set-off against long term capital gain for a maximum of 8 assessment years.
3. As per section 74A(3), the expenses incurred on maintenance of race horses can not be set-off against income from
any source other than the activity of owing of maintaining race horses. Such loss can be carried forward for a
maximum period of 4 assessment years.
4. Loss from speculation business can not be set-off against any income other than profit and gains of anther speculation
business such loss can, however, be carried forward maximum of 4 years as per section 73(4) to be set-off against
income from speculation business.

Q 5: The income of Mr. Prince from business is determined as ₹ 3,50,000 for the P/Y 2022-23 without making the
following adjustments:
(₹)
(a) Depreciation allowance for the current year 80,000
(b) Unabsorbed depreciation brought forward for the A/Y 2016-17 30,000
(c) Long-term capital loss for the current year 15,000
(d) Unabsorbed business loss brought forward from the A/Y 2011-12 45,000
(e) Unabsorbed speculation loss brought forward from the A/Y 2020-21 12,000
(f) Short-term capital loss for the current year 28,000
Compute the total income of Prince for the A/Y 2023-24.

Sol 5: Computation of gross total income of Prince for the A/Y 2023-24:
Particulars (₹)
Business Income 3,50,000
(-) Depreciation allowance for the current year 80,000
(-) Unabsorbed depreciation brought forward for the A/Y 2016-17 30,000
Gross Total Income 2,40,000
Notes:
1. Business loss can be carried forward and set off for a period of 8 assessment years from the year in which the loss
occurred u/s 72. Therefore, brought forward loss of A/Y 2011-12 can’t be set off as period of 8 years have lapsed.
2. Long-term capital loss can be set-off only against long-term capital gains. Therefore, long term capital loss ₹ 15,000
has to be carried forward for set-off against long term capital gains, if any, during A/Y 2024-25.
3. Speculative business loss can be set-off against income from speculative business of the current year and the balance
loss can be carried forward to A/Y 2024-25. It may be noted that speculative business loss can be carried forward for a
maximum of 4 years as per section 73(4).
4. Short-term capital loss can be set off against short-term capital gains or long-term capital gains u/s 74. As there is
neither short-term capital gains nor long-term capital gains, such loss shall be carried forward to the A/Y 2024-25.
CA SHREY RATHI SET OFF AND CARRY FORWARD OF LOSSES 9.6

Q 6: Ms. Geeta, a resident individual provides the following details of her income/losses for the year ended 31.3.2023:
(i) Salary received as a partner from a partnership firm ₹ 7,50,000. The same was allowed to the firm.
(ii) Loss on sale of shares listed in BSE ₹ 3,00,000. Shares were held for 15 months and STT paid on sale and acquisition.
(iii) Long-term capital gain on sale of land - ₹ 5,00,000.
(iv) ₹ 51,000 received in cash from friends in party.
(v) ₹ 55,000, received towards dividend on listed equity shares of domestic companies.
(vi) Brought forward business loss of A/Y 2020-21 - ₹ 12,50,000.
Compute gross total income of Mr. Geeta for the A/Y 2023-24 and ascertain the amount of loss that can be carried
forward. [ICAI Study Module]

Sol 6: Computation of Gross Total Income of Ms. Geeta for the A/Y 2023-24:
Particulars (₹) (₹)
Profits and gains of business and profession
Salary received as a partner from a partnership firm is taxable under the head “Profits 7,50,000
and gains of business and profession”
Less: Brought forward business loss of Assessment year 2020-21 to be set-off against 7,50,000 Nil
business income. (Note 1)
Capital Gains:
Long term capital gains on sale of land 5,00,000
Short-term capital loss on sale of shares (Note 2) 3,00,000 2,00,000
Income from Other sources
Cash gift received from friends – since the value of cash gift exceeds ₹ 50,000, the entire 51,000
sum is taxable.
Dividend received from a domestic company is fully taxable in the hands of shareholders 55,000 1,06,000
Gross Total Income 3,06,000
Notes:
1. Balance brought forward business loss of assessment year 2020-21 of ₹ 5,00,000 has to be carried forward to the next
year.
2. Short-term capital loss can be set off against short-term capital gains or long-term capital gains u/s 74. As there is
long-term capital gains, such loss shall be set off from such income.

Q 7: The following are the details relating to Mr. Srivatsan, a resident Indian, aged 57 years for the year ended 31.03.2023:
(₹)
(a) Income from salaries 2,20,000
(b) Loss from house property 1,90,000
(c) Loss from garments business 2,40,000
(d) Income from speculative business 30,000
(e) Loss from specified business covered u/s 35AD 20,000
(f) Long-term capital gains from sale of urban land 2,50,000
(g) Loss from card games 32,000
(h) Income from betting 45,000
(i) Life insurance premium paid 1,20,000
Compute his total income & show the items eligible for carry forward, assuming that he does not opt for the provisions of
Section 115BAC. [ICAI Study Module]
CA SHREY RATHI SET OFF AND CARRY FORWARD OF LOSSES 9.7

Sol 7: Computation of total income & losses to be carried forward of Mr. Udit for the A/Y 2023-24:
Particulars (₹) (₹)
Income from salaries 2,20,000
(-) Loss from house property (₹ 1,90,000 – ₹ 40,000) 1,50,000 70,000
Profits & Gains from Business & Profession
Income from speculative business 30,000
(-) Loss from garments business (₹ 2,40,000 but restricted to ₹ 30,000) 30,000 -
Capital Gains:
Long-term capital gains from sale of urban land 2,50,000
(-) Loss from garments business (₹ 2,40,000 – ₹ 30,000) 2,10,000
40,000
(-) Loss from house property 40,000 -
Income from Other Sources: Income from betting (lottery) 45,000
Gross Total Income 1,15,000
GTI excluding lottery 70,000
Less: Deductions u/c VI-A: 80C (Life insurance premium) (₹ 1,20,000 but restricted to
₹ 70,000) 70,000
Total Income excluding lottery Nil
Add: Income from betting (Lottery) 45,000
Total Income 45,000
Notes:
1. Loss from card games (lottery) is not deductible from any income whatsoever. Therefore, such loss shall lapse.
2. Loss from Non-speculative business loss after being set off from speculative income can be set off against income
from any head other than salary. There is no order of inter-head adjustment under the Income Tax Act, 1961,
therefore it shall be done in such a manner which is more beneficial to the assessee. It has been deducted from Short-
term capital gains. Similarly, loss from house property can be set off from any head including salary. First it has been
deducted from Short-term capital gains and the balance has been deducted from salary income as deductions are
allowed from salary income and not Short-term capital gains.
3. Loss from specified business covered u/s 35AD shall only be set off against income from specified business. Such loss
shall be carried forward to A/Y 2024-25. However, there is no time constraint.
Note: The solution of this question has been modified taking into consideration the best possible scenario for the
assessee. The students can follow the solution as provided by ICAI also.

Q 8: Mr. Feroz submits the following information relevant for the A/Y 2023-24:
(₹)
(a) Interest on Non-Govt. securities held as investment 1,50,000
(b) Business profits (before claiming following depreciation) 2,28,000
(c) Current depreciation allowance 58,000
(d) Current scientific research expenditure 24,000
(e) Unabsorbed business losses of the previous years:
2000-01 ₹ 11,000
2015-16 ₹ 9,000
2016-17 ₹ 15,000 35,000
(f) Dividend from a foreign company 1,20,000
(g) Long term capital loss brought forward from the A/Y 2020-21 40,700
Determine the net income of Mr. Feroz for the A/Y 2023-24.
CA SHREY RATHI SET OFF AND CARRY FORWARD OF LOSSES 9.8

Sol 8: Computation of net income of Mr. Feroz for the A/Y 2023-24:
Particulars (₹) (₹)
Profits & Gains from Business & Profession
Business income before depreciation 2,28,000
(-) Current year depreciation 58,000
(-) Current scientific research expenditure 24,000
(-) Unabsorbed business losses of previous years
 2015-16 ₹ 9,000
 2016-17 ₹ 15,000 24,000 1,22,000
Income from Other Sources
Interest on Non-Govt. securities held as investment 1,50,000
Dividend from a foreign company 1,20,000 2,70,000
Net Income 3,92,000
Notes:
1. Business loss can be carried forward and set off for a period of 8 assessment years from the year in which the loss
occurred u/s 72. Therefore, brought forward loss of A/Y 2000-2001 can’t be set off as period of 8 years have lapsed.
2. Long-term capital loss can be set-off only against long-term capital gains. Therefore, long term capital loss ₹ 40,700
has to be carried forward for set-off against long term capital gains, if any, during A/Y 2024-25.

Q 9: Compute the gross total income of Mr. F for the A/Y 2023-24 from the information given below:
Particulars (₹)
Net income from house property 1,25,000
Income from business (before providing for deprecation) 1,35,000
Short term capital gains on sale of shares 56,000
Long term capital loss from sale of property (brought forward from A/Y 2020-21) (90,000)
Income from tea business 1,20,000
Dividends from Indian companies carrying on agricultural operations (Gross) 80,000
Current year deprecation 26,000
Brought forward business loss (loss incurred six years ago) (45,000)
[ICAI Study Module]

Sol 9: Computation of Gross Total Income of Mr. F for the A/Y 2023-24:
Particulars (₹) (₹)
Income from house property 1,25,000
Income from business
Profits before deprecation 1,35,000
Less: Currents year deprecation 26,000
Less: Brought forward business loss 45,000
64,000
Income from tea business (40% is business Income) 48,000 1,12,000
Income from capital gains: Short-term capital gains 56,000
Income from Other Sources: Dividend income (taxable in the hands of shareholders) 80,000
Gross Total Income 3,73,000
Notes:
1. Dividend from Indian companies is taxable at normal rates of tax in the hands of resident shareholders.
2. 60% of the income from tea business is agricultural income and therefore, exempt from tax.
3. Long-term capital loss can be set-off only against long-term capital gains. Therefore, long term capital loss ₹ 90,000
brought forward from A/Y 2020-21 cannot be set-off in the A/Y 2023-24, since there is no long-term capital gains in
that year. It has to be carried forward for set-off against long term capital gains, if any, during A/Y 2024-25.
CA SHREY RATHI SET OFF AND CARRY FORWARD OF LOSSES 9.9

Q 10: Mr. Garg, a resident individual, furnishes the following particulars of his income and other details for the P/Y 2022-
23.
S. No. Particulars (₹)
(1) Income from salary 15,000
(2) Income from business 66,000
(3) Long term capital gain on sale of land 10,800
(4) Loss from owning & maintenance from race horses 15,000
(5) Loss from gambling 9,100
The other details of unabsorbed depreciation and brought forward losses pertaining to A/Y 2022-23 are as follows:
S. No. Particulars (₹)
(1) Unabsorbed depreciation 11,000
(2) Loss from speculative business 22,000
(3) Short term capital loss 9,800
Compute the total income of Mr. Garg for the A/Y 2023-24 and the amount of loss, if any that can be carried forward or
not. [ICAI Study Module]

Sol 10: Computation of total income of Mr. Garg for the A/Y 2023-24:
Particulars (₹) (₹)
Income from Salary 15,000
Profits and Gains of Business or Profession 66,000
Less: Unabsorbed depreciation brought forward from A/Y 2022-23 (unabsorbed
depreciation can be set off against any head of income) 11,000 55,000
Capitals Gains
Long term capital gain on sale of land 10,800
Less: Brought forward short-term capital loss [Short-term capital loss can be set off against
both short term capital gains and long-term capital gains as per section 74(1) 9,800 1,000
Gross Total Income 71,000

Amount of loss to be carried forward to A/Y 2024-25:


S. No. Particulars (₹)
(1) Loss from speculative business [to be carried forward as per section 73] 22,000
[Loss from a speculative business can be set off only against income from another speculative
business. Since there is no income from speculative business in the current year, the entire loss
of ₹ 22,000 brought forward from A/Y 2022-23 for set off against speculative business income of
that year. It may be noted that speculative business loss can be carried forward for a maximum
of four years as per section 73(4), i.e. upto A/Y 2026-27]

(2) Loss on maintenance of race horses [to be carried forward as per section 74A] 15,000
[As per section 74A(3), the loss incurred in the activity of owing and maintaining race horses in
any assessment year cannot be set off against income from any other sources other than the
activity of owing and maintaining race horses. Such loss can be carried forward for a maximum
of four assessment years i.e, upto A/Y 2027-28]

(3) Loss from gambling can neither be set off nor be carried forward. 9,100

Q 11: Ms. Disha, gives you the following particulars of her income / losses for the year ended 31.03.2023:
(a) Profit of business of consumer and house-hold appliances ₹ 50,000
(b) Loss of business of readymade garments ₹ 10,000
(c) Brought forward loss of catering business which was closed in the A/Y 2019-20 ₹ 15,000
(d) Short-term loss on sale of securities and shares ₹ 15,000
(e) Profit from speculative transaction entered during the year ₹ 12,500
CA SHREY RATHI SET OFF AND CARRY FORWARD OF LOSSES 9.10

(f) Loss of speculative transactions of A/Y 2014-15 not set off till A/Y 2019-20 ₹ 17,000
(g) Agricultural income ₹ 22,000
(h) Winning in card games ₹ 20,000
(i) Medical insurance premium paid by cheque ₹ 8,000
Compute the gross total income of Ms. Disha for the A/Y 2023-24.

Sol 11: Computation of gross total income of Ms. Disha for the A/Y 2023-24:
Particulars (₹) (₹)
Profit & Gains from Business & Profession
Profit of business of consumer and house-hold appliances 50,000
(-) Loss of business of readymade garments 10,000
(-) Brought forward loss of catering business which was closed in the A/Y 2017-18 15,000
(+) Profit from speculative transaction entered during the year 12,500 37,500
Income from Other Sources: Winning in card games 20,000
Gross Total Income 57,500
GTI excluding winnings from card games 37,500
Less: Deductions u/c VI-A: 80D: Medical insurance premium 8,000
Total Income excluding winnings from card games 29,500
Add: Winnings from card games 20,000
Total Income 49,500
Notes:
1. Short-term capital loss can be set off against short-term capital gains or long-term capital gains u/s 74. As there is
neither short-term capital gains nor long-term capital gains, such loss of ₹ 15,000 shall be carried forward to the A/Y
2024-25.
2. Agricultural income is exempt u/s 10(1).
3. Speculative business loss of A/Y 2014-15 can be set off till the A/Y 2018-19. Since such time has lapsed, it shall not be
allowed to be carried forward.

Q 12: Compute the total income of Mr. Mrigank for the P/Y 2021-22 & 2022-23 from the following information:
P/Y 2021-22 (₹) P/Y 2022-23 (₹)
Income from salary 1,80,000 Income from salary 2,00,000
Income from house property 60,000 Income from house property 90,000
Loss from non-speculative business 2,50,000 Income from non-speculative 70,000
Income from speculative business 40,000 Loss from speculative business 50,000
Short-term capital gains 30,000 Short-term capital loss 20,000
Long-term capital gains 80,000 Long-term capital gain 60,000
Interest income 10,000 Interest income 12,000
CA SHREY RATHI SET OFF AND CARRY FORWARD OF LOSSES 9.11

Sol 12: Computation of total income of Mr. Mrigank for the P/Y 2021-22 & P/Y 2022-23:
P/Y 2021-22:
Particulars (₹) (₹)
Income from Salary 1,80,000
Income from House Property 60,000
(-) Loss from non-speculative business [₹ 2,10,000 (i.e. ₹ 2,50,000 – ₹ 40,000) but
restricted to ₹ 60,000) 60,000 -
Profits & Gains from Business & Profession
Income from speculative business 40,000
(-) Loss from non-speculative business (₹ 2,50,000 but restricted to ₹ 40,000) 40,000
Capital Gains: -
Long-term capital gains 80,000
(-) Loss from non-speculative business [₹ 1,50,000 (i.e. ₹ 2,50,000 – ₹ 40,000 – ₹
60,000) but restricted to ₹ 80,000) 80,000
Short-term capital gains 30,000 -
(-) Loss from non-speculative business [₹ 70,000 (i.e. ₹ 2,50,000 – ₹ 40,000 – ₹
60,000 – ₹ 80,000) but restricted to ₹ 30,000) 30,000
Income from other sources: Interest income 10,000 -
(-) Loss from non-speculative business [₹ 40,000 (i.e. ₹ 2,50,000 – ₹ 40,000 – ₹
60,000 – ₹ 80,000 – ₹ 30,000) but restricted to ₹ 10,000) 10,000 -
Total Income 1,80,000
Notes: Loss from Non-speculative business loss after being set off from speculative income can be set off against income
from any head other than salary. There is no order of inter-head adjustment under the Income Tax Act, 1961, therefore it
shall be done in such a manner which is more beneficial to the assessee. (Other authors may have different view). Further
loss of ₹ 30,000 remaining shall be carried forward to the A/Y 2023-24.

P/Y 2022-23:
Particulars (₹) (₹)
Income from Salary 2,00,000
Income from House Property 90,000
Profits & Gains from Business & Profession:
Non-speculative income 70,000
(-) Non-speculative business loss of A/Y 2022-23 30,000 40,000
Capital Gains:
Long-term capital gains 60,000
Short-term capital loss 20,000 40,000
Income from Other Sources: Interest income 12,000
Total Income 3,82,000
Note: Speculative business loss can be set-off against income from speculative business of the current year and the
balance loss can be carried forward to A/Y 2024-25. It may be noted that speculative business loss can be carried forward
for a maximum of 4 years as per section 73(4).

Q 13: Simran, engaged in various types of activities, gives the following particulars of her income for the year ended
31.3.2023:
S. No. Particulars (₹)
(a) Profits of business of consumer and house hold products 50,000
(b) Loss of business of readymade garments 10,000
(c) Brought forward loss of catering business which was closed in A/Y 2020-21 15,000
(d) Short-term capital loss on sale of securities and shares 15,000
(e) Profit of speculative transaction entered into during the year 12,500
(f) Loss of speculative transaction of A/Y 2017-18 15,000
Compute the total income of Simran for the A/Y 2023-24.
CA SHREY RATHI SET OFF AND CARRY FORWARD OF LOSSES 9.12

Sol 13: Computation of total income of Simran for the A/Y 2023-24:
Particulars (₹) (₹)
Profit of business of consumer and house-hold products 50,000
Less: Loss of business of readymade garments for the year adjusted under section 70(1) 10,000
40,000
Less: Brought forward loss of catering business closed in A/Y 2020-21 set-off against
business income for the current year as per section 15,000 25,000
Profit of speculative transaction 12,500
Total Income 37,500
Notes:
1. Loss of speculative transaction of A/Y 2017-18 is not allowed to be set-off against the profit of speculative transaction
of the A/Y 2023-24, since as per the provision of section 73(4), loss can be carried for set-off for a maximum period of
4 year only i.e. up to A/Y 2021-22.
2. Short term capital loss of ₹ 15,000 on sale of securities and shares has to be carried forward as per section 74 since
there is no income under the head Capital Gains for the A/Y 2023-24. The loss is to be carried forward for set off in
future years against income chargeable under the hade Capital Gains. Such loss can be carried forward for a maximum
period of 8 assessment years.

Q 14: Compute the total income of Mr. Ranjeet from the following data:
(₹)
1. Income under the head Salaries 70,000
2. Income from House Property (A) 60,000
3. Income from House Property (B) (70,000)
4. Speculation income 20,000
5. Garments business income (1,30,000)
6. Income from activity of owning and maintaining race horses (1,50,000)
7. Income from agricultural business (1,25,000)
8. Short-term capital gain 30,000
9. Long-term capital gain (1,00,000)
10. Income from lottery 10,000
11. Income from horse races 1,70,000
12. Dividend income from non-domestic company 90,000
13. Interest on securities 20,000
CA SHREY RATHI SET OFF AND CARRY FORWARD OF LOSSES 9.13

Sol 14: Computation of total income of Mr. Ranjeet for the A/Y 2023-24:
Particulars (₹) (₹)
Income u/h Salaries 70,000
(-) Loss from House Property (B) [i.e. ₹ 10,000 (₹ 70,000 – ₹ 60,000)] 10,000 60,000
Income u/h House Property
Income from House Property (A) 60,000
(-) Loss from House Property (B) (₹ 70,000 but restricted to ₹ 60,000) 60,000 -
Profit and Gains from Business & Profession
Speculation income 20,000
(-) Loss from garments business (₹ 1,30,000 but restricted to ₹ 20,000) 20,000 -
Capital Gains
Short-term Capital Gains 30,000
(-) Loss from garments business [₹ 1,10,000 (i.e. ₹ 1,30,000 – ₹ 20,000) but restricted
to ₹ 30,000) 30,000 -
Income from Other Sources
Income from lottery 10,000
Income from horse races ₹ 1,70,000
(-) Loss from activity of owning and maintaining race horses ₹ 1,50,000 20,000
Dividend income from non-domestic company 90,000
Interest on securities 20,000
1,40,000
(-) Loss from garments business [₹ 80,000 (i.e. ₹ 1,30,000 – ₹ 20,000 – ₹ 30,000)]
Such losses shall only be set off to the extent of dividend & interest income (i.e. ₹
90,000 + ₹ 20,000) as no set-off is admissible against lottery or income from horse
races] 80,000 60,000
Total Income 1,20,000
Notes:
1. Loss from house property after being set off from house property income can be set off against any other head. There
is no order of inter-head adjustment under the Income Tax Act, 1961, therefore it shall be done in such a manner
which is more beneficial to the assessee. Such losses have been set off against salary income as no other losses can be
set off against salary income.
2. Long-term capital loss can be set-off only against long-term capital gains. Therefore, long term capital loss ₹ 1,00,000
has to be carried forward for set-off against long term capital gains, if any, during A/Y 2024-25.

Q 15: Mr. Arvind furnishes the following particulars of his incomes and losses for the A/Y 2023-24. Compute his total
income.
(₹)
1. Income from house property 1,40,000
2. Profits from medicine business 50,000
3. Speculative business profit 30,000
4. Share of profits from a firm 34,000
5. Long-term capital gains on sale of house property 20,000
6. Long-term capital loss on sale of jewellery 7,000
7. Short-term capital gains 15,000
8. Interest on securities 20,000

Following are the brought forward items of the assessment year 2022-23:
1. Unabsorbed depreciation 6,000
2. Speculation loss 40,000
3. Loss from short-term capital asset 24,000
4. Loss from long-term capital asset 9,000
CA SHREY RATHI SET OFF AND CARRY FORWARD OF LOSSES 9.14

Sol 15: Computation of total income of Mr. Arvind for the A/Y 2023-24:
Particulars (₹) (₹)
Income from house property 1,40,000
Profit & gains from Business & Profession:
Income from medicine business 50,000
(-) Unabsorbed depreciation (A/Y 2022-23) 6,000 44,000
Speculative business profit 30,000
(-) Speculation loss of A/Y 2022-23 (₹ 40,000 but restricted to ₹ 30,000) 30,000 -
Capital Gains:
Long-term capital gains on sale of house property 20,000
(-) Long-term capital loss on sale of jewellery 7,000
(-) Long-term capital loss of A/Y 2022-23 9,000
4,000
(-) Short-term capital loss (₹ 9,000 but limited to ₹ 4,000) 4,000 -
Short-term capital gains 15,000
(-) Short-term capital loss of A/Y 2022-23 (₹ 24,000 but restricted to ₹ 15,000) 15,000 -
Income from other sources: Interest on debentures 20,000
Total Income 2,04,000
Notes:
1. Speculative business loss can be set-off against income from speculative business of the current year and the balance
loss of ₹ 10,000 can be carried forward to A/Y 2023-24. It may be noted that speculative business loss can be carried
forward for a maximum of 4 years as per section 73(4).
2. Short-term capital loss can be set off against short-term capital gains or long-term capital gains u/s 74. As there was
long-term capital gains of ₹ 4,000 remaining after set off of long-term capital losses, such short-term capital loss can
be set off against such income. The balance of 5,000 shall be carried forward to the A/Y 2024-25.

Q 16: Mr. Maheshwari submits the following particulars of his income for the A/Y 2023-24. You are required to find his
total income:
(₹)
(i) Income from let out property 12,000
(ii) Loss from self-occupied property 14,000
(iii) Profits from non-speculative business 45,600
(iv) Speculative income 8,000
(v) Short-term capital gains 26,000
(vi) Long-term capital gains 4,000
(vii) Winnings from card games 8,000

The items brought forward for set-off are:


(a) Loss from non-speculative business of A/Y 2016-17 which is discontinued 13,000
(b) Loss from another non-speculative business of A/Y 2012-13 9,000
(c) Loss in card games of A/Y 2014-15 4,000
(d) Speculation loss of A/Y 2019-20 24,000
(e) Short-term capital loss of A/Y 2015-16 12,000
(f) Long-term capital loss of A/Y 2011-12 14,000
CA SHREY RATHI SET OFF AND CARRY FORWARD OF LOSSES 9.15

Sol 16: Computation of total income of Mr. Maheshwari for the A/Y 2023-24:
Particulars (₹) (₹)
House Property
Income from let out property 12,000
(-) Loss from self-occupied property (₹ 14,000 but limited to ₹ 12,000) 12,000 -
Profits & Gains from Business & Profession
Profits from non-speculative business 45,600
(-) Loss from non-speculative business of A/Y 2016-17 which is discontinued 13,000
(-) Loss from self-occupied property (₹ 14,000 – ₹ 12,000) 2,000 30,600
Speculative income 8,000
(-) Speculation loss of A/Y 2019-20 (₹ 24,000, but limited to ₹ 8,000) 8,000 -
Capital Gains
Long-term capital gains 4,000
Short-term capital gains 26,000
(-) Short-term capital loss of A/Y 2015-16 12,000 14,000
Income from Other Sources:
Winnings from card games (lottery) 8,000 8,000
Total Income 56,600
Notes:
1. Loss from house property after being set off from house property income can be set off against any other head. There
is no order of inter-head adjustment under the Income Tax Act, 1961, therefore it shall be done in such a manner
which is more beneficial to the assessee. Such losses have been set off against non-speculative business income.
2. Loss from non-speculative business of A/Y 2012-13 of ₹ 9,000 & Long-term capital loss of A/Y 2011-12 of ₹ 14,000
shall lapse as the time limit for set-off has ended.
3. Loss in card games is not eligible for set off or carry forward & set off.

Q 17: Mr Aditya furnishes the following details for the year ended 31-03-2023:
Particulars (₹)
Loss from speculative business A 25,000
Income from speculative business B 5,000
Loss from specified business covered under section 35AD 20,000
Income from salary 3,00,000
Loss from house property 2,50,000
Income from trading business 45,000
Long-term capital gain from sale of urban land 2,00,000
Long term capital loss on sale of shares (STT not paid) 75,000
Following are the brought forward losses:
(1) losses from owing and maintaining of race horses pertaining to A/Y 2020-21 ₹ 2,000.
(2) Brought forward loss from trading business 5,000 relating to A/Y 2017-18.
Compute the total income of Mr. Aditya and show items eligible for carry forward. [ICAI Study Module]
CA SHREY RATHI SET OFF AND CARRY FORWARD OF LOSSES 9.16

Sol 17: Computation of total income of Mr. Aditya for A.Y. 2023-24:
Particulars (₹) (₹)
Salaries
Income from salary 3,00,000
Less: Loss from house property 2,00,000 1,00,000
Profit and Gains of Business or Profession
Income from trading business 45,000
Less: Brought forward loss from trading business of A/Y 2017-18 can be set off against
current year income trading business as per section 72(1), since the eight years time limit 5,000 40,000
as specified under section 72(3), with in which set off is permitted, has not expired.
Income from speculative business B 5,000
Less: loss from speculative business A set off as per section 73(1) (₹ 25,000, but limited to -
₹ 5,000) 5,000
Capital Gains
Long term capital on gain of urban land 2,00,000
Less: long term capital loss on sale of shares (STT not paid) set of as per section 74(1) 75,000 1,25,000
Total Income 2,65,000
Items eligible for carried forward to A/Y 2023-24:
Particulars (₹)
Loss from house property: As per section 71(3A) loss from house property can be set off against any other 50,000
head of income to the extent of ₹ 2,00,000 only.
As per section 71B, balance loss not set off can be carried forward to the next year for set off against
income from house property of that year. It can be carried forward for a maximum of eight assessment
years i.e., upto A/Y 2031-32, in this case.
Loss from speculative business A: Loss from speculative business can be set off only against profit from any
other speculation business. As per section 73(2), balance loss not set off can be carried forward to the next
20,000
year. such loss can be carried forward for a maximum of for assessment year i.e. upto A/Y 2026-27, in this
case as specified under section 73(4).
Loss from specified business: Loss from specified business u/s 35AD can be set off only against profit of any
other specified business if loss cannot be set off the same of has to be carried forward to the subsequent 20,000
year for set off against income specified business if any in that year as per section 73A(2) such loss can be
carried forward indefinitely for set off against profit of any specified business.
Loss from the activity owing and maintaining race horses: Losses from the activity of owing and maintaining
race horses (current year or brought forward) can be set of only against income from the activity of owing 2,000
and maintaining race horses. If it cannot be so set off it has to be carried forward to the next year for the set
off against income from the activity of owing and maintaining race horses, if any in that year. It can be
carried forward for a maximum of four assessment years i.e. up to A/Y 2024-25 in this case as specified
under section 74A(B).

Q 18: Compute the total income of Mr. Krishna for the A/Y 2023-24 from the following particulars:
Particulars (₹)
Income from business before adjusting the following items:
(a) Business loss brought forward from A/Y 2018-19 1,75,000
(b) Currents year deprecation 70,000
(c) Unabsorbed deprecation of earlier year 40,000
Income from house property (Gross Annual Value) 1,55,000
Municipal taxes paid 4,32,000
Mr. Krishna sold a plot at Noida on 12th September, 2022 for a consideration of ₹ 6,40,000, which had been 32,000
purchased by him on 20th December, 2020 at a cost of ₹ 4,10,000
Long-term capital gain on sale of debentures 60,000
CA SHREY RATHI SET OFF AND CARRY FORWARD OF LOSSES 9.17

Sol 18: Computation of Income of Mr. Krishna for the A/Y 2023-24:
Particulars (₹) (₹)
Income from House Property
Gross Annual Value 4,32,000
Less: Municipal taxes paid 32,000
Net Annual Value (NAV) 4,00,000
Less: Deduction under section 24
(a) 30% of NAV 1,20,000
(b) Interest on housing loan - 2,80,000
Profit and Gains of Business or Profession
Income from business 1,75,000
Less: Current year depreciation u/s 32(1) 40,000
1,35,000
Less: Set-off brought forward business loss of A/Y 2018-19 u/s 72 70,000
65,000
Less: Unabsorbed depreciation set-off [See Note 2] 65,000 Nil
Capital Gains
Long term capital gain on sale of debentures 60,000
Less: Unabsorbed depreciation set-off [See Note 2] 60,000 Nil
Short term capital gain on sale of land [See Note 1] 2,30,000
Less: Unabsorbed depreciation set-off [See Note 2] 30,000 2,00,000
Total Income 4,80,000
Notes:
1. Since land is held for a period of less than 24 months, the gain of ₹ 2,30,000 arising from sale of such land is a short-
term capital gain.
2. Brought forward unabsorbed depreciation can be adjusted against any head of income. However, it is most beneficial
to set-off unabsorbed depreciation first against long term capital gains, since it is taxable at a higher rate of 20% (the
other income of the assessee falling in the 5% slab rate). Therefore, unabsorbed depreciation is first set-off against
long term capital gain to the extent of ₹ 60,000. The remaining unabsorbed depreciation is adjusted against business
income to the extent of ₹ 65,000 and balance of ₹ 30,000 is adjusted against short term capital gains.
In the alternative, the balance of ₹ 30,000 may also be set-off against income from house property, in which case, the
net income house property would be ₹ 2,50,000 and short-term capital gains would be ₹ 2,30,000. The gross total
income and total income would, however, remain unchanged.
CA SHREY RATHI SET OFF AND CARRY FORWARD OF LOSSES 9.18

Q 19: Mr. Rajat submits the following information for the financial year ending 31st March, 2023. He desires that you
should:
(a) Compute the total income and
(b) Ascertain the amount of losses that can be carried forward.
S. No. Particulars (₹)
(i) He has two houses:
(a) House No. l – Income after all statutory deductions 72,000
(b) House No. ll - Current year loss (30,000)
(ii) He has three proprietary businesses:
(a) Textile Business:
(i) Discontinued from 31st October, 2022- Current year loss 40,000
(ii) Brought forward business loss of A/Y 2018-19 95,000
(b) Chemical Business:
(i) Discontinued from 1st March, 2020 – hence no profit/loss Nil
(ii) Bad debts allowed in earlier years recovered during this year 35,000
(iii) Brought forward business loss of A.Y. 2019-20 50,000
(c) Leather Business: profit for the current year 1,00,000
(d) Shares of profit in a firm in which he is partner since 2008 16,550
(iii) He made the following capital gains/losses
(a) Short-term capital gain 60,000
(b) Long-term capital loss 35,000
(iv) Contribution to LIC towards premium 10,000
[ICAI Study Module]

Sol 19: Computation of total income of Mr. Rajat for the A/Y 2023-24:
Particulars (₹) (₹)
Income from House Property
House No. 1: Income 72,000
(-) House No. 2: Loss 30,000 42,000
Profits and Gains of Business or Profession
Profit from leather business 1,00,000
Bad debts recovered taxable under section 41(A) 35,000
1,35,000
Less: Current year loss of textile business 40,000
95,000
Less: Brought forward business loss of textile business for A.Y. 2018-19 set off against the
business income of current year 95,000 Nil
Capital Gains
Short-term capital gain 60,000
Gross Total Income 1,02,000
Less: Deduction u/c VI-A: 80C: LIC premium paid 10,000
Total Income 92,000

Statement of losses to be carried forward to A/Y 2024-25:


Particulars (₹)
Business loss of A/Y 2019-20 to be carried forward u/s 72 50,000
Long term capital loss of A/Y 2023-24 to be carried forward u/s 74 35,000
Notes:
1. Share of profit from firm of ₹ 16,550 is exempt under section 10(2A).
2. Long-term capital loss cannot be set off against short-term capital gains. Therefore, it has to be carried forward to the
next year to be set-off against long-term capital gains of that year.
CA SHREY RATHI SET OFF AND CARRY FORWARD OF LOSSES 9.19

MULTIPLE CHOICE QUESTIONS


1. Loss under the head of house property:
(a) can be carry forwarded for 8 years (c) can be carry forwarded for 4 years
(b) can’t be carried forward (d) can be carry forwarded for infinite years

2. Mr. A incurred short-term capital loss of ₹ 10,000 on sale of shares through the National Stock Exchange. Such loss -?
(ICAI MCQ Booklet)
(a) Can be set-off against both short term capital gains
(b) Can be set-off against both short term capital gains and long-term capital gains.
(c) Can be set-off against any head of income.
(d) Not allowed to be set-off.

3. Which of the following is correct?


(a) LTCL can be set off from LTCG only
(b) LTCL can be set off from STCG only
(c) Loss of business can be set off from salary income
(d) Loss of speculation business can be set off from income from normal business

4. Loss on account of owing & maintaining the race horses can be set off from:
(a) any business income (c) Income from horse races
(b) any income under the head other sources (d) Income of owing and maintaining of race horses

5. Loss on account of owing & maintaining the race horse can be carried forward:
(a) for 8 assessment years (c) indefinitely
(b) for 4 assessment years (d) nothing is mentioned in law

6. Losses from speculative business, losses of owning and maintaining race horses and losses of specified business under
section 35AD can be carried forward and set off from income
(a) of any business (c) any source or any head except income from salary
(b) any type of capital gain (d) only from respective nature of income

7. Compute the taxable income of Mr. Kamal If his Income from salary is ₹ 4,00,000 and Loss from house property ₹
2,70,000
(a) ₹ 4,00,000 (c) ₹ 3,65,000
(b) ₹ 1,30,000 (d) ₹ 2,00,000

8. During the PY 2022-23, Mr. John has following incomes and brought forward losses:
Short term capital gains on sale of shares (STT not paid) ₹ 1,50,000, Long term capital loss of AY 2021-22 (₹ 96,000),
Short term capital loss of AY 2020-21 (₹ 37,000). What is the capital gain taxable in the hands of Mr. John and how
much loss can be carried forward to the AY 2024-2025?
(a) ₹ 1,13,000, Nil (c) ₹ 1,13,000, ₹ 96,000
(b) ₹ 17,000, Nil (d) ₹ 54,000, ₹ 37,000

9. The first item in order of priority of set off between unabsorbed depreciation, capital expenditure on scientific
research, current year depreciation, and brought forwarded business loss is?
(a) Unabsorbed depreciation (c) Current year depreciation
(b) Capital expenditure on scientific research (d) Brought forwarded business loss

10. Speculation loss can be carried forward for the maximum of:
(a) 8 assessment years (c) 10 assessment years
(b) 4 assessment years (d) unlimited number of years

11. Inter head set off can be availed under section 71 before availing the intra head set off under section 70.
(a) True
(b) False
CA SHREY RATHI SET OFF AND CARRY FORWARD OF LOSSES 9.20

12. During the PY 2022-23, Mr. Jonny has following income and brought forward losses:
Long term capital gain ₹ 1,75,000, Long term capital loss of AY 2021-22 (₹ 96,000), Short term capital loss of AY 2020-
2021 (₹ 37,000).
What is the capital gain taxable in the hands of Mr. Jonny and how much loss can be carried forward to the AY 2024-
2025?
(a) ₹ 79,000, Nil (c) Nil, Nil
(b) ₹ 1,38,000, ₹ 96,000 (d) ₹ 42,000, Nil

13. Calculate speculative income: Speculative in FOREX (₹ 2,00,000) Speculative in Shares ₹ 5,00,000
(a) ₹ 3,00,000 (c) ₹ 2,50,000
(b) ₹ 2,00,000 (d) ₹ 5,00,000

14. Mathur Storage (P) Ltd. engaged in chain cold storage (which is a specified business u/s 35AD) has brought forward
business loss of ₹ 12 lakhs relating to assessment year 2020-2021. During the previous year 2022-23, its income from
the said business is ₹ 9 lakhs. It also has profit from trade in food grains of ₹ 6 lakhs. The total income of the company
for the PY 2022-2023 is:
(a) ₹ 15 lakhs (c) ₹ 6 lakhs
(b) ₹ 9 lakhs (d) ₹ 3 lakhs

15. No loss can be set-off against


(a) Income from salaries (c) Income from capital gains
(b) Income from house property (d) Winnings from lotteries, etc.

16. Bobby discontinued wholesale trade in medicines from 1st July, 2020. He recovered ₹ 1,50,000 in October, 2022 being
a bad debt which was written-off and allowed in assessment year 2021-2022. He has eligible brought forward
business loss of wholesale trade in medicines of ₹ 1,70,000. The consequence of bad debt recovery is that
(a) It is chargeable to tax
(b) It is eligible for set-off against brought forward business loss
(c) The brought forward business loss is taxable now
(d) 50% of the amount recovered now is taxable

17. A partnership firm with 4 equal partners has brought forward depreciation of ₹ 3 lakh and business loss of ₹ 3 lakhs
relating to assessment year 2022-23, On 1st April, 2022, two partners retired. The amount that assessee firm can set-
off against its income for the assessment year 2023-24 would be:
(a) Unabsorbed depreciation of ₹ 3 lakhs plus brought forward business loss of ₹ 3 lakhs
(b) Unabsorbed depreciation ‘nil’ plus brought forward business loss Rs. 3 lakhs
(c) Unabsorbed depreciation ₹ 3 lakhs plus brought forward business Loss ‘nil’
(d) Unabsorbed depreciation ₹ 3 lakhs plus brought forward business loss of ₹ 1.50 lakhs

18. Loss from specified business referred in section 35AD can be carried forward and set off from
(a) Income of any business and profession (c) Any head of income
(b) Income of a specified business only (d) Any heads of income other than salary

19. Brought forward loss from house property can be set-off


(a) Against any head of income to the extent of ₹ 2,00,000
(b) Against income from house property to the extent of ₹ 2,00,000
(c) Against income from house property without any limit
(d) Against any head of income without any limit

20. Mr. X incurred long-term capital loss from sale of listed shares in recognized stock exchange and STT is paid at the
time of acquisition and sale of such shares. Such loss
(a) can be set-off only against long-term capital gains
(b) can be set-off against both short-term capital gains and long-term capital gains
(c) can be set-off against any head of income.
(d) is not allowed to be set-off
CA SHREY RATHI SET OFF AND CARRY FORWARD OF LOSSES 9.21

21. Calculate total income. Agriculture income (₹ 2,00,000); Trading business ₹ 3,00,000
(a) ₹ 2,00,000 (c) ₹ 5,00,000
(b) ₹ 3,00,000 (d) ₹ 1,00,000

22. Compute the total income and losses to be carried forward of Mr. Johar for the PY 2022-23. Loss from profession ₹
1,05,000; short term capital loss on the sale of property ₹ 55,000; Long term capital gains on sale of shares ₹ 2,05,000
(STT not paid); Winnings from lotteries ₹ 1,00,000; Loss from horse races in Mumbai ₹ 40,000
(a) ₹ 2,50,000, ₹ 1,45,000 (c) ₹ 1,45,000, ₹ 40,000
(b) ₹ 1,45,000, Nil (d) ₹ 3,05,000, ₹ 2,00,000

23. According to section 80, no loss shall be carried forward for which ITR has not been filed under section 139(3). The
exceptions are (ICAI MCQ Booklet) [Refer chapter return of income]
(a) only loss under the head “Capital Gains” under section 74
(b) Loss under head “Capital Gains” and unabsorbed depreciation carried forward u/s 32(2)
(c) Loss from house property and unabsorbed depreciation carried forward u/s 32(2)
(d) Loss from house property; unabsorbed depreciation carried forward u/s 32(2) and the loss of specified business
referred under section 35AD

24. Loss of specified business under section 35AD can be carried forward:
(a) for 8 assessment years (c) indefinite period of time
(b) for 4 assessment years (d) nothing is mentioned in law

25. Where there is permissible inter-head adjustment, it should be set off first against:
(a) Income which has higher rate of tax (c) Income which has lower rate of tax
(b) Income as decided by the assessing officer (d) Income which is exempt from tax

26. Brought forward loss from house property of ₹ 3,10,000 of A.Y. 2021-22 is allowed to be set-off against income from
house property of A.Y. 2023-24 of ₹ 5,00,000 to the extent of – (ICAI MCQ Booklet)
(a) ₹ 2,00,000 (c) ₹ 2,50,000
(b) ₹ 3,10,000 (d) ₹ 1,00,000

27. Any loss arising from transfer of a personal capital asset on which gains are exempt from tax shall:
(a) not be allowed to be set off against any income
(b) be allowed to be set off against LTCG
(c) be allowed to be set off against STCG & LTCG both
(d) be allowed to be set off against all income except salary income & lottery

28. Rohan incurred loss of ₹ 3 lakh in the P.Y. 2022-23 in retail trade business. Against which of the following income
during the same year, can he set-off such loss? (ICAI MCQ Booklet)
(a) Profit of ₹ 1 lakh from wholesale cloth business
(b) Long-term capital gains of ₹ 1.50 lakhs on sale of land
(c) Speculative business income of ₹ 40,000
(d) All of the above

29. Rathore Ltd. provides you with following information:


Income from business before current year depreciation – ₹ 2,50,000
Current year depreciation – ₹ 40,000
Brought forward business loss of P/Y 2020-21 – ₹ 1,00,000
Unabsorbed capital expenditure on scientific research – ₹ 90,000
Unabsorbed capital expenditure on family planning – ₹ 80,000
What will be the amount to be carried forward to the next Assessment Year?
(a) Unabsorbed capital expenditure on scientific research – ₹ 60,000
(b) Unabsorbed capital expenditure on family planning – ₹ 60,000
(c) Current year depreciation of 40,000 & Unabsorbed capital expenditure on family planning – ₹ 20,000
(d) Brought forward business loss of P/Y 2020-21 – ₹ 60,000
CA SHREY RATHI SET OFF AND CARRY FORWARD OF LOSSES 9.22

30. Ms. Payal is running a textile business and has incurred a loss of ₹ 4,70,000 during the P/Y 2022-23. She has received
interest on investment in Public Provident Fund ₹ 30,000, earned ₹ 10,000 p.m. as salary from a part time job & ₹
20,000 from lottery income. Compute his total income for the P/Y 2022-23.
(a) (₹ 3,40,000) (c) ₹ 1,70,000
(b) ₹ 1,30,000 (d) ₹ 90,000

31. Virat run a business of manufacturing of shoes since the P.Y. 2020-21. During the P.Y. 2020-21 and P.Y. 2021-22, Virat
had incurred business losses. For P.Y. 2022-23, he earned business profit (computed) of ₹ 3 lakhs. Considering he
may/may not have sufficient business income to set off his earlier losses, which of the following order of set off shall
be considered: (He does not have income from any other source) (ICAI MCQ Booklet)
(a) First adjustment for loss of P.Y. 2020-21, then loss for P.Y. 2021-22 and then unabsorbed depreciation, if any.
(b) First adjustment for loss of P.Y. 2021-22, then loss for P.Y. 2020-21 and then unabsorbed depreciation, if any.
(c) First adjustment for unabsorbed depreciated, then loss of P.Y. 2021-22 and then loss for P.Y. 2020-21 if any
(d) First adjustment for unabsorbed depreciated, then loss of P.Y. 2020-21 and then loss for P.Y. 2021-22, if any

32. Mr. Ravi incurred loss of ₹ 4 lakh in the P.Y. 2022-23 in leather business. Against which of the following incomes
earned during the same year, can he set-off such loss?
(i) Profit of ₹ 1 lakh from apparel business
(ii) Long-term capital gains of ₹ 2 lakhs on sale of jewellery
(iii) Salary income of ₹ 1 lakh
Choose the correct answer: (ICAI MCQ Booklet)
(a) First from (ii) and thereafter from (i); the remaining loss has to be carried forward.
(b) First from (i) and thereafter from (ii) and (iii).
(c) First from (i) and thereafter from (iii); the remaining loss has to be carried forward.
(d) First from (i) and thereafter from (ii); the remaining loss has to be carried forward.

33. During the A.Y. 2022-23, Mr. A has loss of ₹ 8 lakhs under the head “Income from house property” which could not be
set off against any other head of income as per the provisions of section 71. The due date for filing return of income
u/s 139(1) in case of Mr. A has already expired and Mr. A forgot to file his return of income within the said due date.
However, Mr. A filed his belated return of income for A.Y. 2022-23. Now, while filing return of income for A.Y. 2023-
24, Mr. A wish to set off the said loss against income from house property for the P.Y. 2022-23. Determine whether
Mr. A can claim the said set off. (ICAI MCQ Booklet)
(a) No, Mr. A cannot claim set off of loss of ₹ 8 lakhs during A.Y. 2023-24 as he failed to file his return of income u/s
139(1) for A.Y. 2022-23.
(b) Yes, Mr. A can claim set of loss of ₹ 2 lakhs out of ₹ 8 lakhs from its income from house property during A.Y. 2023-
24 if any, and the balance has to be carried forward to A.Y. 2024-25.
(c) Yes, Mr. A can claim set of loss of ₹ 2 lakhs out of ₹ 8 lakhs from its income from any head during A.Y. 2023-24
and the balance has to be carried forward to A.Y. 2024-25.
(d) Yes, Mr. A can claim set off of loss of ₹ 8 lakhs during A.Y. 2023-24 from its income from house property, if any,
and the balance has to be carried forward to A.Y. 2024-25.

34. The details of income/loss of Mr. Kumar for A.Y. 2023-24 are as follows:
Particulars Amount (₹)
Income from Salary (Computed) 5,20,000
Loss from self-occupied house property 95,000
Loss from let-out house property 2,25,000
Loss from specified business u/s 35AD 2,80,000
Loss from medical business 1,20,000
Long term capital gain 1,60,000
Income from other sources 80,000
What shall be the gross total income of Mr. Kumar for A.Y. 2023-24? (ICAI MCQ Booklet)
(a) ₹ 4,40,000 (c) ₹ 1,60,000
(b) ₹ 3,20,000 (d) ₹ 4,80,000
CA SHREY RATHI SET OFF AND CARRY FORWARD OF LOSSES 9.23

MCQ’s Answers
1. (a) 2. (b) 3. (a) 4. (d) 5. (b) 6. (d) 7. (d) 8. (c) 9. (c) 10. (b)
11. (b) 12. (d) 13. (a) 14. (c) 15. (d) 16. (b) 17. (d) 18. (b) 19. (c) 20. (a)
21. (b) 22. (b) 23. (c) 24. (c) 25. (a) 26. (b) 27. (a) 28. (d) 29. (b) 30. (d)
31. (a) 32. (d) 33. (d) 34. (a)
CA SHREY RATHI DEDUCTIONS FROM GTI 10.1

USAIN BOLT

CHAPTER 10
DEDUCTIONS FROM GROSS TOTAL
INCOME

"You have to set yourself goals so you


can push yourself harder. Desire is the
key to success.”

PRACTICAL QUESTIONS
Q 1: Mr. Ishan who is person with severe disability submits you the following information and asks you to compute his
total income:
(₹)
(a) Salary (p.m.) 3,000
(b) Interest on Government securities 2,000
(c) Interest on saving bank account 44,000
(d) Long-term capital gain 2,25,000
(e) Deposit in PPF 10,000

Sol 1: Computation of total income of Mr. Ishan:


Particulars (₹) (₹)
Salary (3,000 x 12) 36,000
Less: Standard deduction u/s 16(ia) [to the extent of salary] 36,000 Nil
Capital Gains: Long-term capital gains 2,25,000
Income from Other Sources:
• Interest on Government securities 2,000
• Interest on saving bank account 44,000 46,000
Gross Total Income 2,71,000
GTI excluding LTCG 46,000
Less: Deductions u/c VI-A
o 80C: PPF 10,000
o 80TTA: Interest on saving bank account 10,000
o 80U: Person with severe disability 1,25,000 46,000
(limited to ₹ 46,000, being GTI excluding LTCG)
Total income (excluding long term capital gains) Nil
Add: Long-term capital gains 2,25,000
Total income (including long-term capital gains) 2,25,000
CA SHREY RATHI DEDUCTIONS FROM GTI 10.2

Q 2: Mr. Mark is totally blind (severe disability). From the following information determine his total income for the A/Y
2023-24.
(₹)
1. Salary from a publishing house for working as an artist 5,25,000
2. Special allowance 68,000
3. Remuneration for setting question papers 24,000
4. Interest on debentures 15,500
5. Dividend from a foreign company 3,800
6. Collection charges of question paper setting remuneration 4,000
7. Contribution towards recognised provident fund 40,000
8. Payment of insurance premium on his own life (sum assured: ₹ 60,000) 14,000
9. Contribution to Prime Minister’s National Relief Fund 28,500

Sol 2: Computation of total income of Mr. Mark for the A/Y 2023-24:
Particulars (₹) (₹)
Income u/h Salaries:
Salary from a publishing house for working as an artist 5,25,000
Special allowance 68,000
Gross salary 5,93,000
Less: Standard deduction u/s 16(ia) 50,000 5,43,000
Income from Other Sources:
Remuneration for setting question papers ₹ 24,000
(-) Collection charges ₹ 4,000 20,000
Interest on debentures 15,500
Dividend from a foreign country 3,800 39,300
Gross Total Income 5,82,300
Less: Deductions u/c VI-A
o 80C:
 Recognised Provident Fund 40,000
 Life Insurance Premium (limited to 15% of capital sum assured, in case
of blind person) (₹ 60,000 x 15%) 9,000
o 80G: Prime Minister’s National Relief Fund (Category A) (100% without any 28,500
qualifying limit)
o 80U: Person with severe disability 1,25,000 2,02,500
Total Income 3,79,800

Q 3: Examine the allowability of the following:


(1) Rajan has to pay to a hospital for treatment ₹ 62,000 and spent nothing for life insurance or for maintenance of
handicapped dependent.
(2) Raja, a resident Indian, has spent nothing for treatment in the pervious year and deposited ₹ 25,000 with LIC for
maintenance of handicapped dependant.
(3) Rajan has incurred ₹ 30,000 for medical insurance for his wife and ₹ 24,000 for his dependent mother aged 63 years.
(4) Payment of ₹ 50,000 by cheque to an electoral trust by an Indian company.

Sol 3: Computation of allowability of deductions:


(1) The deduction of ₹ 75,000 under section 80DD is allowed in full, irrespective of the amount of expenditure incurred or
paid by the assessee. If the expenditure is incurred in respect of a dependant with severe disability, the deduction
allowable is ₹ 1,25,000.
(2) Raja has deposited ₹ 25,000 for maintenance of handicapped dependent. The assessee is however, eligible to claim ₹
75,000, irrespective of the amount deposited with LIC. In the case of dependant with severe disability, the deduction
allowable is ₹ 1,25,000.
CA SHREY RATHI DEDUCTIONS FROM GTI 10.3

(3) Deduction u/s 80D shall be allowed to Rajan in the following manner:
Medical insurance for his wife (₹ 30,000 but restricted to ₹ 25,000) ₹ 25,000
Medical insurance for his dependent mother ₹ 24,000
Total medical insurance premium ₹ 49,000
(4) Amount paid by an Indian Company to an electoral trust is eligible trust is eligible for deduction under section 80GGB
from gross total income, since such payment is made otherwise than by way of cash.

Q 4: The GTI of Mr. Deepak is ₹ 30,00,000 which consist of the following:


(₹)
(a) Profit from profession 28,00,000
(b) Dividend from Indian company 2,00,000
The assessee made the following donations during the P/Y:
1. ₹ 5,000 to Jawaharlal Nehru Memorial Fund.
2. ₹ 6,000 to Prime Minister Drought Relief Fund.
3. ₹ 10,000 to Prime Minister National Relief Fund.
4. ₹ 2,000 to National Children Fund.
5. ₹ 80,000 to Municipal Corporation of Delhi, for opening a Girl’s school.
6. ₹ 22,000 for repairs of notified temple.
7. Shoes worth ₹ 15,000 were given to approved blind school.
8. ₹ 18,000 worth of books to school for distribution among poor students.
9. ₹ 21,000 given to a poor student for higher studies.
10. ₹ 51,000 for construction of a Dharamshala only for Hindus.
11. ₹ 2,00,000 to State of Delhi for Family Planning.
Compute the total income for the A/Y 2023-24 assuming ₹ 70,000 have been deposited in Public Provident Fund.

Sol 4: Computation of total income of Mr. Deepak for the A/Y 2023-24:
Particulars (₹) (₹)
Profit from profession 28,00,000
Dividend from Indian company 2,00,000
Gross Total Income 30,00,000
Less: Deductions u/c VI-A
o 80C: Public Provident Fund 70,000
o 80G [WN 1] 2,64,000 3,34,000
Total Income 26,66,000

WN 1: Computation of deduction u/s 80G:


Particulars of Donation Amount % of Donation u/s
donated (₹) deduction 80G (₹)
Prime Minister National Relief Fund (Category A) 10,000 100% 10,000
National Children Fund (Category A) 2,000 100% 2,000
Jawaharlal Nehru Memorial Fund (Category B) 5,000 50% 2,500
Prime Minister’s Drought Relief Fund (Category B) 6,000 50% 3,000
State of Delhi for promoting family planning (Category C) (subject to
qualifying limit) 2,00,000 100% 2,00,000
Municipal Corporation of Delhi for opening a girl’s school (Category D)
(subject to qualifying limit) (refer note below) 80,000
Notified temple (Category D) (subject to qualifying limit) (refer note 50% 46,500
below) 22,000
Total Deduction u/s 80G 2,64,000
CA SHREY RATHI DEDUCTIONS FROM GTI 10.4

Qualifying limit = 10% of Adjusted Gross Total Income


Qualifying limit = 10% of [Gross Total Income - Deductions u/s 80C to 80U except section 80G] = 10% of [₹ 30,00,000 – ₹
70,000] = ₹ 2,93,000
Note: Firstly, donation of ₹ 2,00,000 to State of Delhi for family planning qualifying for 100% deduction subject to
qualifying limit, has to be adjusted against qualifying limit. Donation to notified temple shall be 50% of the [balance
qualifying limit (i.e. ₹ 2,73,000 – ₹ 2,00,000 = ₹ 93,000) or amount of donation, being ₹ 1,02,000, whichever is lower].
Therefore, deduction for donation to notified temple shall be 50% of ₹ 93,000 = ₹ 46,500.
Notes:
1. Donation of shoes to blind school & distribution of books to blind school shall not be allowed deduction as they are
provided in kind.
2. Donation for construction of Dharamshala only for Hindus shall not be eligible as donation for the benefit of particular
community shall not be allowed.
3. Donation to poor student for higher studies is ineligible for deduction as such donation does not fall under any
category of Section 80G.

Q 5: Ms. Divyanka is author of one book of scientific nature and its print price is ₹ 500 per book and total number of
copies sold are 2000 and she has received royalty @ 50%. She has taken a loan from State Bank in November 2013 for
pursuing post-graduate degree and she repaid principal amount of ₹ 70,000 and interest of ₹ 20,000 to State Bank.
Payment of interest was given for the first time in financial year 2015-16. She has paid tuition fee of her daughter for
whole time education ₹ 33,000 in India. Compute total income for the P/Y 2022-23.

Sol 5: Computation of total income of Ms. Divyanka for the P/Y 2022-23:
Particulars (₹) (₹)
Royalty income on sale of books (2000 books x ₹ 500 x 50%) 5,00,000
Less: Deductions u/c VI-A:
o 80C: Tuition fee for her daughter 33,000
o 80E: Interest on educational loan 20,000
o 80QQB: Royalty income on books: Lower of the following:
➢ Royalty actually received – ₹ 5,00,000 (but limited to 15% of the value of
books sold) (2000 books x ₹ 500 x 15%) ₹ 1,50,000
➢ Limit specified ₹ 3,00,000 1,50,000 2,03,000
Total Income 2,97,000
Note: Interest on educational loan is allowed to be deducted for 8 years starting from the year from which the interest
repayment begins and not from the date of loan.

Q 6: Mr. Kevin, a non-resident supplies you the following information for the year ended 31.03.2023.
(₹)
1. Interest on bank deposits 59,000
2. Dividend on shares of foreign company received abroad 78,000
3. Interest from deposits in Indian companies (gross) 30,000
4. Income from horse races in India 17,500
5. Donation to Municipal Corporation of Delhi for family planning 15,000
6. Paid by cheque to insurance company for mediclaim for himself 2,000
7. Spent on medical treatment of his minor son who is physically handicapped 6,000
Compute total income of Mr. Kevin for the A/Y 2023-24.
CA SHREY RATHI DEDUCTIONS FROM GTI 10.5

Sol 6: Computation of total income of Mr. Kevin for the A/Y 2023-24:
Particulars (₹) (₹)
Income from Other Sources:
Interest on bank deposits 59,000
Dividend on shares of foreign company received abroad (only income received or -
earned in India is taxable in India for non-resident)
Interest on deposits in Indian companies (gross) 30,000
Income from horse races in India 17,500 1,06,500
Gross Total Income 1,06,500
Less: Income from lottery (horse races) 17,500
GTI excluding lottery (horse races) 89,000
Less: Deductions u/c VI-A:
o 80D: Mediclaim for himself 2,000
o 80G: [WN 1] 10,450
o 80DD: Treatment of dependent handicapped (not allowed to non-resident) - 12,450
Total Income excluding lottery (horse races) 76,550
Add: Income from lottery (horse races) 17,500
Total Income 94,050
WN 1: Computation of deduction under section 80G:
Donation to Municipal Corporation of Delhi falls under category (D) which is eligible to 100% deduction subject to
qualifying limit.
Qualifying limit = 10% of Adjusted Gross Total Income
= 10% of [GTI – Deductions u/s 80C to 80U except section 80G]
= 10% of [₹ 1,06,500 – ₹ 2,000]
= 10% of ₹ 1,04,500
= ₹ 10,450
Donation to Municipal Corporation of Delhi shall be 100% of [qualifying limit being ₹ 10,450 or amount of donation, being
₹ 15,000, whichever is lower]. Therefore, deduction for donation to Municipal Corporation of Delhi shall be 100% of ₹
10,450 = ₹ 10,450.

Q 7: Compute the total income of Mr. Ankit for the P/Y 2022-23 from the following information:
(₹)
(i) Basic salary 10,000 p.m.
(ii) Contribution by employee to recognised provident fund 20% of basic salary
(iii) Equal contribution was made by the employer
(iv) DA – 30% of basic salary (70% of which forms part of salary)
(v) Commission based on fixed % of turnover 30,000
(vi) Interest credited during the year @ 15% 45,000
(vii) Life insurance premium paid during the year 50,000
(viii) Purchase of bonds of NABARD 40,000
(ix) Investment in senior citizen saving scheme 10,000
(x) Annuity pension fund plan of insurance company 35,000
CA SHREY RATHI DEDUCTIONS FROM GTI 10.6

Sol 7: Computation of total income of Mr. Ankit for the P/Y 2022-23:
Particulars (₹) (₹)
Basic salary (₹ 10,000 x 12) 1,20,000
Employer’s contribution to recognised provident fund [WN 1] 2,976
Dearness allowance (₹ 1,20,000 x 30%) 36,000
Commission based on fixed % of turnover 30,000
Interest credited on RPF (₹ 45,000/15 x 5.5) 16,500
Gross Salary 2,05,476
Less: Standard Deduction u/s 16(ia) 50,000
Income u/h Salaries (GTI) 1,55,476
Less: Deductions u/c VI-A: 80C
o Own contribution towards RPF 24,000
o Life insurance premium 50,000
o Bonds of NABARD 40,000
o Investment in senior citizen saving scheme 10,000
1,24,000
80CCC: Annuity pension fund plan of insurance company 35,000
(limited to ₹ 1,50,000 as per section 80CCE) 1,59,000 1,50,000
Total Income 5,476
Total Income (rounded off) 5,480
WN 1: Computation of taxable employer’s contribution:
Taxable employer’s contribution = Contribution by employer – 12% [Basic salary + Dearness allowance (forming part of
salary) + Commission based on fixed % of turnover]
Taxable employer’s contribution = (₹ 1,20,000 x 20%) – 12% [₹ 1,20,000 + ₹ 25,200 + ₹ 30,000]
= ₹ 2,976.

Q 8: For the A/Y 2023-24, the Gross Total Income of Mr. Chaturvedi, a resident in India, was ₹ 8,18,240 which includes
Long-term capital gain of ₹ 2,45,000 and Short-term capital gain of ₹ 58,000. The Gross Total Income also includes interest
income of ₹ 12,000 from saving bank deposits with banks. Mr. Chaturvedi has invested in PPF ₹ 1,40,000 and also paid a
medical insurance premium ₹ 31,000. Mr. Chaturvedi also contributed ₹ 50,000 to public charitable trust eligible for
deduction u/s 80G by way of an account payee cheque. Compute the total income of Mr. Chaturvedi, who is 70 years old
as on 31.03.2023.

Sol 8: Computation of income of Mr. Chaturvedi for the A/Y 2023-24:


Particulars (₹) (₹)
Gross total income including long term capital gain 8,18,240
Less: Long term capital gain 2,45,000
GTI excluding LTCG 5,73,240
Less: Deduction under section Chapter VI-A:
o 80C: PPF deposit 1,40,000
o 80D (it is assumed that premium of ₹ 31,000 is paid by any mode other than cash) 31,000
o 80G (See notes 1 & 2 below) 19,512
o 80TTB 12,000 2,02,512
Total income (excluding long term capital gains) 3,70,728
Add: Long-term capital gains 2,45,000
Total income (including long-term capital gains) 6,15,728
Total income (rounded off) 6,15,730
CA SHREY RATHI DEDUCTIONS FROM GTI 10.7

Notes:
1. Computation of deduction under section 80G: Donation to charitable trust falls under category (D) which is eligible to
50% deduction subject to qualifying limit.
Qualifying limit = 10% of Adjusted Gross Total Income
= 10% of [GTI excluding LTCG – Deductions u/s 80C to 80U except section 80G]
= 10% of [₹ 5,73,240 – ₹ 1,83,000]
= 10% of ₹ 3,90,240
= ₹ 39,024
Donation to charitable trust shall be 50% of [qualifying limit being ₹ 39,024 or amount of donation, being ₹ 50,000,
whichever is lower]. Therefore, deduction for donation to notified temple shall be 50% of ₹ 39,024 = ₹ 19,512.
2. Deduction u/s 80G is allowed only if amount is paid by any mode other than cash, in case of amount exceeding ₹
2,000. Therefore, the contribution made to public charitable trust is eligible for deduction since it is made by way of
an account payee cheque.

Q 9: From the following information, compute the total income for the A/Y 2023-24 of Raju:
(a) Profits and Gains of Business and Profession ₹ 5,00,000
(b) Long-term Capital Gains ₹ 1,00,000
(c) Winning from lotteries (gross) ₹ 25,000
(d) Interest on listed securities ₹ 35,000
(e) Loss from house property ₹ 4,60,000

Raju has made the following payments during the P/Y 2022-23:
1. Medical Insurance Premium paid in cash for the health of his spouse ₹ 40,000
2. LIC premium for the policy taken for his wife (sum assured ₹ 4,00,000) ₹ 27,500
3. Subscription to notified bonds of NABARD ₹ 40,000
4. Swachh Bharat Kosh ₹ 20,000

Sol 9: Computation of total income of Mr. Raju for the A/Y 2023-24:
Particulars (₹) (₹)
Profits and Gains of Business & Profession 5,00,000
Less: Loss from house property (₹ 2,00,000 shall be set-off in this year as per section
71B, rest shall be carried forward to A/Y 2024-25) 2,00,000 3,00,000
Capital Gains: Long-term capital gains 1,00,000
Income from Other Sources:
• Winning from lotteries (gross) 25,000
• Interest on listed securities 35,000 60,000
Gross Total Income 4,60,000
Less: Long-term capital gains & lottery 1,25,000
GTI excluding LTCG & lottery 3,35,000
Less: Deductions u/c VI-A
o 80C
➢ LIC premium for the policy taken for his wife 27,500
➢ Subscription to notified bonds of NABARD 40,000 67,500
o 80D: Not available as health insurance premium is paid in cash -
o 80G: Swachh Bharat Kosh (Category A) (100%) 20,000
Total Income excluding LTCG & lottery 2,47,500
Add: Long-term capital gains & lottery 1,25,000
Total Income 3,72,500
CA SHREY RATHI DEDUCTIONS FROM GTI 10.8

Q 10: Examine the following statements with regard to the provision of the Income Tax Act, 1961:
1. During the financial year 2022-23, Mr. Amit paid interest on loan availed by him for his son’s higher education. His son
is already employed in a firm. Will Mr. Amit will get the deduction under section 80E?
2. Subscription to notified bonds of NABARD would qualify for deduction under section 80C.
3. In order to be eligible to claim deduction under section 80C, investment/contribution/subscription etc. in eligible or
approved modes, should be made from out of income chargeable to tax.
4. Where an individual repays a sum of ₹ 30,000 towards principal and ₹ 14,000 as interest in respect of loan taken from
a bank for pursuing eligible higher studies, the deduction allowable under section 80E is ₹ 44,000.
5. Ms. Sheela, widow of Mr. Satish (who was an employee of M/s. XYZ Ltd.), received ₹ 7 lakhs on 01.05.2022, being
amount standing to the credit of Mr. Satish in his NPS Account, in respect of which deduction has been allowed under
section 80CCD to Mr. Satish in the earlier previous years. such amount received by her as a nominee on closure of the
account is deemed to be her income for A/Y 2023-24.

Sol 10: Examinability of some statements are as under:


1. The statement is correct. The deduction u/s 80E available to an individual in respect of interest on loan taken for his
higher education or for the higher education of his relative i.e. spouse and children of the individual. Therefore, Mr.
Amit will get the deduction u/s 80E. It is immaterial that his son is already employed in a firm. This would not affect
Mr. Amit’s eligibility for deduction u/s 80E.
2. The statement is correct. U/s 80C, subscription to such bonds issued by NABARD (as the central Government may
notify in the official gazette) would qualify for deduction under section 80C.
3. The statement is not correct. There is no stipulation u/s 80C that the investment, subscription etc. should be made
from out of income chargeable to tax.
4. The statement is not correct. Deduction u/s 80E is in respect of interest paid on education loan. Hence, the deduction
will be limited to ₹ 14,000.
5. The statement is not correct. A proviso has been inserted in section 80CCD(3) to provide that the amount received by
the nominee, on closure of NPS account on the death of the assessee, shall not be deemed to be the income of the
nominee. Hence amount received by Ms. Sheela would not be deemed to be her income for A/Y 2022-23.

Q 11: The GTI of Ms. Anjali for the A/Y 2023-24 is ₹ 10,00,000. She made the following payments during the year 2022-23:
(i) LIP paid (policy value: ₹ 1,50,000) ₹ 20,000
(ii) PPF amount paid ₹ 80,00
(iii) Principal repayment of housing loan to HDFC Bank ₹ 50,000
(iv) Payment made to LIC pension fund ₹ 20,000
(v) Medical insurance premium for self, wife & dependent children ₹ 18,000
(vi) Mediclaim premium for parents (aged over 70 years) ₹ 35,000
Compute her total income for the A/Y 2023-24.
.
Sol 11: Computation of total income of Ms. Anjali for the A/Y 2023-24:
Particulars (₹) (₹)
Gross Total Income 10,00,000
Less: Deductions u/c VI-A
o 80C:
➢ LIP paid: ₹ 20,000 (limited to 10% of capital sum assured, being ₹ 1,50,000) 15,000
➢ PPF 80,000
➢ Principal repayment of housing loan 50,000
1,45,000
o 80CCC: Payment made to LIC pension fund 20,000
(Restricted to ₹ 1,50,000 as per section 80CCE) 1,65,000 1,50,000
o 80D:
➢ Medical insurance premium for self, wife & dependent children 18,000
➢ Mediclaim for parent (aged 70 years) 35,000 53,000
Total Income 7,97,000
CA SHREY RATHI DEDUCTIONS FROM GTI 10.9

Q 12: Mr. Rajmohan whose gross total income was ₹ 6,40,000 for the financial year 2022-23 furnishes you the following
information:
(i) Stamp duty paid on acquisition of residential house (self- occupied) ₹ 50,000.
(ii) Five year time deposit in an account under Post Office Time Deposit Rules, 1981 - ₹ 20,000.
(iii) Donation to a recognized charitable trust ₹ 25,000 which is eligible for deduction u/s 80G at the applicable rate.
(iv) Interest on loan taken for higher education of spouse paid during the year ₹ 10,000.
Compute the total income of Mr. Rajmohan for the Assessment Year 2023-24.

Sol 12: Computation of total income of Mr. Rajmohan for the A/Y 2023-24:
Particulars (₹) (₹)
Gross total income 6,40,000
Less: Deduction u/c VI-A
o 80C:
➢ Stamp duty paid on acquisition of residential house 50,000
➢ Five year time deposit with post office 20,000
o 80E: Interest on loan taken for higher education of spouse, being a relative. 10,000
o 80G (WN 1) 12,500 92,500
Total income 5,47,500
Notes: In case of deduction u/s 80G in respect of donation to charitable trust, the net qualifying amount has to be
restricted to 10% of adjusted total income, i.e., gross total income less deductions u/c VI-A except 80G. The adjusted total
income is, therefore, ₹ 5,60,000 (i.e. ₹ 6,40,000 – ₹ 80,000), 10% of which is ₹ 56,000, which is higher than the actual
donation of ₹ 25,000. Therefore, the deduction under section 80G would be ₹ 12,500, being 50% of the actual donation of
₹ 25,000.

Q 13: Mr. Shatrugan, a resident submits you the following particulars about his income for the P/Y 2022-23:
(₹)
1. Business Income 83,000
2. Interest on debentures 49,000
3. LTCG on transfer of gold 4,10,000
4. STCG on sale of shares taxable u/s 111A 20,000
5. Other short-term capital gains 10,000
6. Contributions towards public provident fund 40,000
7. Payment of medical insurance premium on own life 3,000
8. Donation to the national trust for welfare of persons with Autism 4,000
9. Donation to the fund set up by the Central Government for providing relief to
the victims of earthquake in Gujrat 3,000
10. Donation to Rajiv Gandhi Foundation 1,000
11. Donation to the Prime Minister’s Drought Relief Fund 5,000
12. Donation to approved public charitable institution 11,000
13. Donation to a poor boy for higher education 5,000
14. Donation of clothes to an approved institution 12,000
15. Donation to a charitable institution for construction of a rest house only for a
particular religious community 8,000
Determine the net income of Mr. Shatrugan for the A/Y 2023-24.
CA SHREY RATHI DEDUCTIONS FROM GTI 10.10

Sol 13: Computation of net income of Mr. Shatrugan for the A/Y 2023-24:
Particulars (₹) (₹)
Business income 83,000
Capital Gains:
 LTCG on transfer of gold 4,10,000
 STCG on sale of shares taxable u/s 111A 20,000
 Other short-term capital gains 10,000 4,40,000
Income from Other Sources: Interest on debentures 49,000
Gross Total Income 5,72,000
Less: LTCG on gold & STCG u/s 111A 4,30,000
GTI excluding LTCG on gold & STCG u/s 111A 1,42,000
Less: Deductions u/c VI-A
o 80C: Public provident fund 40,000
o 80D: Medical insurance premium on his own life 3,000
o 80G: (WN 1) 14,950 57,950
Total Income excluding LTCG on gold & STCG u/s 111A 84,550
Add: LTCG on gold & STCG u/s 111A 4,30,000
Total Income 5,14,550

WN 1: Computation of deduction u/s 80G:


Particulars of Donation Amount % of Donation u/s
donated (₹) deduction 80G (₹)
National trust for welfare of persons with Autism (Category A) 4,000 100% 4,000
Donation to the fund set up by the Central Government for providing
relief to the victims of earthquake in Gujrat (Category A) 3,000 100% 3,000
Rajiv Gandhi Foundation (Category B) 1,000 50% 500
Prime Minister’s Drought Relief Fund (Category B) 5,000 50% 2,500
Approved charitable institution (Category D) (subject to qualifying limit)
(refer note below) 11,000 50% 4,950
Total Deduction u/s 80G 14,950
Qualifying limit = 10% of Adjusted Gross Total Income
Qualifying limit = 10% of [Gross Total Income – LTCG on sale of gold – STCG on sale of shares u/s 111A - Deductions u/s
80C to 80U except section 80G] = 10% of [₹ 5,72,000 – ₹ 4,10,000 – ₹ 20,000 – ₹ 43,000] = ₹ 9,900
Donation of ₹ 11,000 to approved charitable institution qualifies for 50% deduction subject to qualifying limit. Deduction
shall be 50% of the [qualifying limit (i.e. ₹ 9,900) or amount of donation, being ₹ 11,000, whichever is lower]. Therefore,
deduction for donation to charitable institution shall be 50% of ₹ 9,900 = ₹ 4,950.
Notes:
1. Donation of clothes to approved institution shall not be allowed deduction as they are provided in kind.
2. Donation for construction of a rest house shall not be eligible as donation for the benefit of particular community shall
not be allowed.
3. Donation to poor student for higher studies is ineligible for deduction as such donation does not fall under any
category of Section 80G.

Q 14: Mr. Pankaj is an employee of a private limited company in Mumbai. He gets ₹ 18,000 p.m. as salary. Besides, the
employer provides a furnished housing facility (house being owned by the employer) at concessional rate (fair rent of
unfurnished house: ₹ 28,000; rent of furniture: ₹ 24,000; rent paid by Pankaj: ₹ 26,000); and medical allowance at the rate
of ₹ 500 p.m. The employer also provides sports club facility. A similar facility will cost Pankaj ₹ 7,000 per year.
CA SHREY RATHI DEDUCTIONS FROM GTI 10.11

Determine the net income of Pankaj for the A/Y 2023-24 after giving due considerations to the following:
(₹)
1. LTCG in respect of commercial building 17,000
2. LTCG in respect of non-listed shares 10,000
3. Winning from horse races (gross) 4,000
4. Winning from lottery (gross) 75,000
5. Interest from IFCI 8,000
6. Interest on company deposits 48,200
7. Insurance premium paid on a joint life policy on the life of Pankaj & his wife 26,000
(sum assured: ₹ 2,00,000)
8. Medical insurance on the life of Pankaj’s senior citizen father 66,000

Sol 14: Computation of net income of Pankaj for the A/Y 2023-24:
Particulars (₹) (₹)
Income u/h Salary [WN 1] 2,03,300
Income u/h Capital Gains:
o LTCG on sale of commercial building 17,000
o LTCG on sale of non-listed shares 10,000 27,000
Income from Other Sources:
o Winnings from horse races (gross) 4,000
o Winnings from lotteries (gross) 75,000
o Interest from IFCI Exempt
o Interest on company deposits 48,200 1,27,200
Gross Total Income 3,57,500
Less: LTCG + Lottery (₹ 27,000 + ₹ 79,000) 1,06,000
GTI excluding LTCG & Lottery 2,51,500
Less: Deductions u/c VI-A
• 80C: LIP on joint life policy (₹ 26,000, but limited to 10% of capital sum assured, 20,000
being ₹ 2,00,000)
• 80D: Medical insurance premium for his senior citizen father (₹ 66,000, but limited
to ₹ 50,000) 50,000 70,000
Total Income (excluding LTCG & Lottery) 1,81,500
Add: LTCG & Lottery 1,06,000
Total Income 2,87,500

WN 1: Computation of income u/h salary:


Particulars (₹)
Basic salary (₹ 18,000 x 12) 2,16,000
Medical allowance (₹ 500 x 12) 6,000
Sports club facility [Exempt] -
Rent free accommodation owned by the employer [15% of salary – amount recovered from the 7,300
employee] [{15% of (₹ 2,16,000 + 6,000)} – ₹ 26,000]
Rent of furniture 24,000
Gross Salary 2,53,300
Less: Standard deduction u/s 16(ia) 50,000
Income u/h Salaries 2,03,300
CA SHREY RATHI DEDUCTIONS FROM GTI 10.12

Q 15: Mr. Srinath declares ₹ 4,00,000 as his GTI for the A/Y 2023-24. The gross total income includes LTCG of ₹ 73,000 and
STCG u/s 111A of ₹ 27,000. The details of fund investment made during the year are:
(₹)
(i) Medical insurance premium paid by cheque for
(a) self 9,000
(b) for major son who is not dependent 1,000
(ii) Contribution made to:-
(a) Rajiv Gandhi Foundation 7,000
(b) Delhi University (declared as an institute of national eminence) 3,000
(c) Zila Saksharta Samiti 5,000
(d) An approved charitable institute by cheque 30,000
(e) Government for the purpose of promoting family planning 10,000
(f) Ganesh Temple in the society 9,000
Compute the total income of Mr. Srinath chargeable to tax for the A/Y 2023-24.

Sol 15: Computation of total income of Mr. Srinath for the A/Y 2023-24:
Particulars (₹) (₹)
Gross Total Income 4,00,000
Less: LTCG & STCG u/s 111A (₹ 73,000 + ₹ 27,000) 1,00,000
GTI excluding LTCG & STCG u/s 111A 3,00,000
Less: Deductions u/c VI-A
o 80D: Medical insurance premium for
❖ Self 9,000
❖ Major son who is not dependant (only dependant children are allowed) -
o 80G [WN 1] 31,050 40,050
Total Income (excluding LTCG & STCG u/s 111A) 2,59,950
Add: LTCG & STCG u/s 111A 1,00,000
Total Income 3,59,950
WN 1: Computation of deduction u/s 80G:
Particulars of Donation Amount % of Donation u/s
donated (₹) deduction 80G (₹)
Delhi University (declared as an institute of national eminence) 3,000 100% 3,000
(Category A)
Zila Saksharta Samiti (Category A) 5,000 100% 5,000
Rajiv Gandhi Foundation (Category B) 7,000 50% 3,500
Government for the purpose of promoting family planning (Category C) 10,000 100% 10,000
(subject to qualifying limit)
Approved charitable institution (Category D) (subject to qualifying limit) 30,000 50% 9,550
(refer below)
Total Deduction u/s 80G 31,050
Qualifying limit = 10% of Adjusted Gross Total Income
Qualifying limit = 10% of [Gross Total Income – LTCG – STCG u/s 111A - Deductions u/s 80C to 80U except section 80G] =
10% of [₹ 4,00,000 – ₹ 73,000 – ₹ 27,000 – ₹ 9,000] = ₹ 29,100
Firstly, donation of ₹ 10,000 to Government for family planning qualifying for 100% deduction subject to qualifying limit,
has to be adjusted against qualifying limit. Donation to approved charitable institution shall be 50% of the balance
qualifying limit (i.e. ₹ 29,100 – ₹ 10,000 = ₹ 19,100) or amount of donation, being ₹ 30,000, whichever is lower. Therefore,
deduction for donation to approved charitable institution shall be 50% of ₹ 19,100 = ₹ 9,550.
Note: Donation to Ganesh temple in the society does not qualify for deduction u/s 80G as it is not a notified temple by the
Government.
CA SHREY RATHI DEDUCTIONS FROM GTI 10.13

Q 16: Compute the total income for the A/Y 2023-24 of Ms. Roma, who has a gross total income of ₹ 15,00,000 for the A/Y
2023-24 and provides the following information about her investment/payments during the year 2022-23:
Particulars (₹)
Life Insurance premium paid (policy taken on 01-01-2012 and sum assured is ₹ 1,50,000) 35,000
Public Provident Fund contribution 1,50,000
Repayment oh housing loan to Bhartiya Mahila bank, Bangalore 20,000
Payment to L.I.C. pension fund 1,40,000
Mediclaim policy taken for self, wife and dependent children, premium paid by cheque 30,000
Medical Insurance premium paid by cheque for parents (Senior Citizen) 30,000

Sol 16: Computation of total income for the A/Y 2023-24 of Ms. Roma:
Particulars (₹) (₹)
Gross Total Income 15,00,000
Less: Deductions u/c VI-A: 80C
➢ Life insurance premium paid ₹ 35,000 (deduction restricted to 20% of the sum
assured since the policy was taken before 01.04.2012) (₹ 1,50,000 x 20%) 30,000
➢ Public Provident Fund
➢ Repayment of housing loan to Bhartiya Mahila Bank, Bangalore 1,50,000
20,000
[Restricted to a maximum of ₹ 1,50,000] 2,00,000
o 80CCC: Payment towards LIC Pension Fund 1,50,000
[As per section 80CCE, aggregate deduction u/s 80C and 80CCC, is restricted to ₹ 1,40,000
1,50,000] 2,90,000 1,50,000
o 80D: Medical insurance premium for: Self, wife & dependant children (₹ 30,000,
but restricted to ₹ 25,000) 25,000
Parents 30,000 55,000
Total Income 12,95,000

Q 17: Mr. Akhilesh provides you with the following information:


(₹)
(i) Income from salary 3,60,000
(ii) Income from house property 80,000
(iii) Profits and gains from shoes business 90,000
(iv) Long-term capital gain 70,000
(v) Short-term capital loss 30,000
(vi) Interest on debentures 14,000
Compute his total income for the A/Y 2023-24 by taking into consideration the following payments:
(a) Life insurance premium on his own life (sum assured: ₹ 3,00,000) 25,000
(b) Tuition fees paid for his two daughters in India for full time education 40,000
(c) Payment of medical insurance premium for:
• wife ₹ 9,000
• dependant daughter ₹ 12,000
• dependant sister ₹ 4,000
• dependant mother (aged 67 years) ₹ 19,000 44,000
(d) Expenses on medical treatment of a dependant, being a person with disability 26,000
(e) Payment of interest on account of loan taken from charitable institution for the education
of his son doing post graduate. 32,000
(f) Donation to Prime Minister’s National Relief Fund 8,000
(g) Donation to National Children Fund 6,000
(h) Donation to Government for promoting family planning 14,000
(i) Donation to a notified temple 21,000
CA SHREY RATHI DEDUCTIONS FROM GTI 10.14

Sol 17: Computation of total income of Mr. Akhilesh for the A/Y 2023-24:
Particulars (₹) (₹)
Income from salary 3,60,000
Income from house property 80,000
Profits and gains from shoes business 90,000
Capital Gains:
Long-term capital gains 70,000
(-) Short-term capital loss 30,000 40,000
Income from Other Sources: Interest on debentures 14,000
Gross Total Income 5,84,000
Less: Long-term capital gains 40,000
GTI excluding LTCG 5,44,000
Less: Deductions u/c VI-A
o 80C:
 Life insurance premium on his own life 25,000
 Tuition fees for full time education of his two daughters 40,000 65,000
o 80D: Medical insurance premium for:
 Wife 9,000
 Dependant daughter 12,000
 Dependant sister (not eligible for deduction) -
 Dependant mother (aged 67 years) 19,000 40,000
o 80DD: Treatment of a dependant, being a person with disability (actual 75,000
expenditure is irrelevant)
o 80E: Interest on loan taken from chartable institution for pursuing post-graduate 32,000
education
o 80G [WN 1] 37,600
Total Income (excluding LTCG) 2,94,400
Add: Long-term capital gains 40,000
Total Income 3,34,400
WN 1: Computation of deduction u/s 80G:
Particulars of Donation Amount % of Donation u/s
donated (₹) deduction 80G (₹)
Prime Minister’s National Relief Fund (Category A) 8,000 100% 8,000
National Children Fund (Category A) 6,000 100% 6,000
Government for the purpose of promoting family planning (Category C)
(subject to qualifying limit) 14,000 100% 14,000
Notified temple (Category D) (subject to qualifying limit) (refer below) 21,000 50% 9,600
Total Deduction u/s 80G 37,600

Qualifying limit = 10% of Adjusted Gross Total Income


Qualifying limit = 10% of [Gross Total Income – LTCG - Deductions u/s 80C to 80U except section 80G] = 10% of [₹ 5,84,000
– ₹ 40,000 – ₹ 2,12,000] = ₹ 33,200
Firstly, donation of ₹ 14,000 to Government for family planning qualifying for 100% deduction subject to qualifying limit,
has to be adjusted against qualifying limit. Donation to notified temple shall be 50% of the balance qualifying limit (i.e. ₹
33,200 – ₹ 14,000 = ₹ 19,200) or amount of donation, being ₹ 21,000, whichever is lower. Therefore, deduction for
donation to notified temple shall be 50% of ₹ 19,200 = ₹ 9,600.
CA SHREY RATHI DEDUCTIONS FROM GTI 10.15

MULTIPLE CHOICE QUESTIONS


1. Mr. Karan is author of books, omitted to claim deductions under section 80D and Section 80QQB from GTI while
submitting his ITR on 31/7/2023 which was the due date for the PY 2022-23. Later on Mr. Karan submitted revised ITR
on 09/12/2023 and claimed deduction of section 80D and section 80QQB.
(a) He will not be allowed deduction under section 80D and section 80QQB
(b) He will be allowed deduction under section 80D but not under section 80QQB
(c) He will be allowed deduction under section 80QQB but not under section 80D
(d) He will be allowed deduction under section 80D and section 80QQB

2. Deductions under section 80C to 80U are not allowed from


(a) long-term capital gain taxable under section 112
(b) long-term capital gain taxable under section 112A
(c) short-term capital gains taxable under Section 111A
(d) all of the above

3. Deduction u/s 80C in respect of Life Insurance Policy, Contribution of employee to Provident Fund, etc. is allowed to:
(a) Any assessee
(b) Individual assessee only
(c) Individual or HUF who may be resident or non-resident in India
(d) Individual or HUF who is resident in India

4. For claiming deduction u/s 80C in respect of life insurance premium, premium can be paid by assessee for:
(a) Himself and the spouse (c) Himself, spouse and dependent children
(b) Himself, spouse and minor children (d) Himself, spouse and any child

5. An assessee has paid life insurance premium of ₹ 45,000 during the previous year for a policy of ₹ 2,00,000 taken on
1/4/2015. He shall:
(a) Not be allowed any deduction under section 80C
(b) Be allowed deduction under section 80C to the extent of 10% of the capital sum assured
(c) Be allowed deduction for the entire premium
(d) Be allowed deduction under section 80C to the extent of 20% of the capital sum assured

6. For claiming deduction u/s 80C by an individual in respect of PPF contribution for any child, the child should be:
(a) Minor child
(b) Any child dependent on such individual
(c) Any child dependent or not dependent on such individual
(d) All of the above

7. If an assessee discontinues the life insurance policy before completion of 2 years then:
(a) No deduction shall be allowed in respect of the payment made in the year of termination.
(b) No deduction shall be allowed under section 80C in the year of termination.
(c) Amount of the deduction claimed due to insurance premium during past years shall be liable to tax in the year of
termination.
(d) No tax treatment has been given under income tax act.

8. The annual interest accrued on NSC VIII or NSC IX issue shall be:
(a) Exempt under section 10
(b) Taxable as income of other sources
(c) Taxable as income of other sources and interest so accrued shall also be eligible for deduction u/s 80C
(d) None of the above

9. Deduction under section 80C for tuition fee shall be allowed if such fee is paid to:
(a) Any university, college, school or other educational institution situated within India or outside India
(b) Any university, college, school or other educational institution situated within India
(c) Any university, college, school or other educational institution situated outside India
(d) Only to school situated within India or outside India
CA SHREY RATHI DEDUCTIONS FROM GTI 10.16

10. Deduction u/s 80C in respect of tuition fee is allowed to the maximum extent of:
(a) ₹ 12,000 per child for maximum of 2 children
(b) ₹ 12,000 p.m. per child for maximum of 2 children
(c) ₹ 50,000 p.a. per child upto unlimited children
(d) ₹ 50,000 p.a. for two children

11. Deduction under section 80C in respect of terms deposit with a scheduled bank is allowed if the term deposit is for a
period:
(a) Not less than 3 years (c) Not less than 7 years
(b) Not less than 5 years (d) Not less than 10 years

12. Deduction under section 80CCC is allowed to the maximum extent of:
(a) ₹ 2,00,000 (c) ₹ 1,50,000
(b) ₹ 1,00,000 (d) ₹ 2,50,000

13. Deduction under section 80CCC shall be allowed to:


(a) Individual who is a resident or non -resident of India
(b) Individual or HUF who is a resident of India
(c) Individual who is resident and ordinary resident of India
(d) Individual who is resident whether ordinary resident or non - resident HUF who is non -resident of India

14. Deduction section 80CCD is allowed to extent of:


(a) Employee’s contribution up to 10% of salary
(b) Employee’s contribution up to 15% salary
(c) Employee’s and employer’s contribution each up to 10% salary and in case of self-employed person up to 20% of
his gross total income
(d) Employee’s and employer’s contribution each up to 15% of salary

15. As per section 80CCE deduction u/s 80C, 80CCC and 80CCD cannot exceed:
(a) ₹ 1,00,000 including employer’s contribution to notified pension scheme referred to in section 80CCD
(b) ₹ 1,00,000 exclusive of employer’s contribution to notified pension scheme referred to in section 80CCD
(c) ₹ 1,50,000 including employer’s contribution to notified pension scheme referred to in section 80CCD
(d) ₹ 1,50,000 exclusive of employer’s contribution to notified pension scheme referred to in section 80CCD

16. Quantum of deduction under section 80EEA is:


(a) ₹ 50,000 (c) ₹ 1,00,000
(b) ₹ 1,50,000 (d) ₹ 1,25,000

17. Deduction under section 80D on amount of preventive health check-up is allowed if:
(a) It is paid by cheque (c) It is paid in any mode including cash
(b) It is paid by any mode other than cash (d) It is paid in cash

18. Where the Individual or his family or his parents or a member of HUF in case of HUF is a senior citizen the limit under
section 80D of ₹ 25,000 shall be substituted by-
(a) ₹ 40,000 (c) ₹ 30,000
(b) ₹ 20,000 (d) ₹ 50,000

19. For the deduction under section 80DD a person shall be known as severely disabled if he has disability of:
(a) More than 50% disability (c) More than 80% disability
(b) More than 75% disability (d) More than 25% disability

20. As per section 80A aggregate of all deductions from section 80C to section 80U cannot exceed
(a) Amount of GTI (c) Amount of casual incomes in GTI
(b) Amount of special incomes in GTI (d) Amount of capital gains in GTI
CA SHREY RATHI DEDUCTIONS FROM GTI 10.17

21. Deduction under section 80DDB shall be allowed for medical treatment of specified disease of:
(a) Any dependent relative (c) The assessee himself or any dependent relative
(b) Any dependent handicapped relative (d) None of the above

22. Deduction under section 80E is allowed on account of:


(a) Repayment of loan taken from certain specified institutions for higher education
(b) Repayment of loan and interest on loan taken from certain specified institutions for higher education
(c) Payment of interest on loan taken from certain specified institutions for higher education
(d) None of the above

23. The deduction u/s 80E is allowed for payment by way of interest on loan to the extent of:
(a) ₹ 2,00,000 (c) ₹ 1,00,000
(b) ₹ 1,50,000 (d) Any amount

24. Deduction u/s 80E for payment by way of interest on loan for higher education is allowed for:
(a) 5 years (c) 7 years
(b) 6 years (d) 8 years

25. The overall limit in case of deduction under section 80G is:
(a) 10% of gross total income (c) 25% of gross total income
(b) 10% of adjusted gross total income (d) 25% of adjusted gross total income

26. In case the donation under section 80G the maximum amount of donation in cash allowed is: (ICAI MCQ Booklet)
(a) ₹ 5,000 (c) ₹ 10,000
(b) ₹ 2,000 (d) ₹ 25,000

27. In Section 80G we use 10% of adjusted gross total income. This adjusted gross total income means:
(a) Total income less LTCG income, less all deductions except deduction of 80G
(b) Total income less LTCG income u/s 112, less LTCG income u/s 112A, less STCG income u/s 111A, less all
deductions except deduction of 80G
(c) Total income less LTCG income u/s 112, less LTCG income u/s 112A, less STCG income u/s 111A, less casual
incomes, less all deductions except deduction of 80G
(d) Total income less LTCG income, less STCG income of section 111A, less all deductions including deduction of 80G

28. Deduction in respect of rent paid u/s 80GG is allowed to:


(a) An individual
(b) An individual who is self employed
(c) Any individual who is self-employed or who is an employee but not entitled to HRA or Rent Free Accommodation
(d) Any individual who is self-employed or who is an employee but not entitled to HRA or Rent Free Accommodation
and who prays rent for his residential accommodation

29. The deduction under section 80GG shall be limited to:


(a) ₹ 1,50,000 p.a. (c) ₹ 24,000 p.a.
(b) ₹ 2,00,000 p.a. (d) ₹ 60,000 p.a.

30. Mr. Anil (aged 66 years) has earned 21,000 from saving bank account & 28,000 from fixed deposit receipts. How much
deduction shall be available to him u/s 80TTB?
(a) ₹ 10,000 (c) ₹ 21,000
(b) ₹ 49,000 (d) ₹ 50,000

31. Maximum deduction allowed u/s 80TTA shall be


(a) ₹ 10,000 p.a. (c) ₹ 40,000 p.a.
(b) ₹ 50,000 p.a. (d) ₹ 20,000 p.a.
CA SHREY RATHI DEDUCTIONS FROM GTI 10.18

32. Deduction u/s 80TTB is allowed to


(a) Any Individual (c) Any individual who is very senior citizen
(b) Any individual who is a senior citizen (d) Any individual who is not a senior citizen

33. Mr. Ramesh pays a rent of ₹ 5,000 per month. His total income is ₹ 2,80,000 (i.e. Gross Total Income as reduced by
deductions under Chapter VI-A except section 80GG). He is also in receipt of HRA. He would be eligible for a deduction
under section 80GG of an amount of
(a) ₹ 60,000 (c) ₹ 70,000
(b) ₹ 32,000 (d) Nil

34. Under section 80GGC, deduction is allowable in respect of contribution to political parties by
(a) any person other than local authority and every artificial juridical person wholly or partly funded by the
Government
(b) Local authority and every artificial juridical person wholly or partly funded by the Government
(c) An Indian company
(d) Any assessee

35. Mr. Srivastav, aged 72 years, paid medical insurance premium of ₹ 52,000 by cheque and ₹ 4,000 by cash during May,
2022 under a medical insurance scheme of the General insurance Corporation. The above sum was paid for insurance
of his own health. He would be entitled to a deduction under section 80D of a sum of- (ICAI MCQ Booklet)
(a) ₹ 30,000 (c) ₹ 52,000
(b) ₹ 50,000 (d) ₹ 56,000

36. Mr. Ramesh pays a rent of ₹ 5,000 per month. His total income is ₹ 2,80,000 (i.e. Gross Total income as reduced by
deductions under Chapter VI-A except section 80GG). He is also in receipt of HRA. He would be eligible for a deduction
under section 80GG of an amount of- (ICAI MCQ Booklet)
(a) ₹ 60,000 (c) ₹ 70,000
(b) ₹ 32,000 (d) Nil

37. The maximum deduction allowable under section 80EE for A.Y. 2023-24 is-
(a) ₹ 50,000 (c) ₹ 1,00,000
(b) ₹ 2,50,000 (d) ₹ 1,50,000

38. Which of the following is covered under section 80D of the Income Tax Act, 1961:
(a) Repayment of loan taken for higher education
(b) Medical treatment of handicapped dependent
(c) Medical insurance premium
(d) Reimbursement of medical expenses.

39. Bharat, engaged in business, claimed that he paid ₹ 10,000 per month by cheque as rent for his residence. He does not
own any residential building. His total income computed before deduction under section 80GG is ₹ 3,40,000. The
amount he can claim as deduction under section 80GG is-
(a) ₹ 24,000 (c) ₹ 1,20,000
(b) ₹ 34,000 (d) ₹ 60,000

40. Sudhan Ltd. Incorporated in April 2022 commenced commercial production from 01-06-2022. It deployed 100
employees who were employed for 260 days during the year and recruited 50 casual workmen who were employed for
100 days during the financial year 2022-23. The salary paid to 100 employees was ₹ 25 lakhs and salary paid to casual
workmen was ₹ 6 lakhs. The quantum of deduction under section 80JJAA is:
(a) ₹ 7.50 lakhs (c) ₹ 25 lakhs
(b) ₹ 9.30 lakhs (d) ₹ 6 lakhs

41. Mr. Rathi borrowed loan of ₹ 10 lakhs for higher education in India in the year 2009-10. He completed the course study
in 2012-13. He started repayment of the loan from April 2014. He paid interest of ₹ 41,000 and principal of ₹ 1,20,000
during the financial year 2022-23. The amount eligible for deduction under section 80E would be:
(a) ₹ 1,20,000 (c) ₹ 41,000
(b) ₹ 1,61,000 (d) Nil
CA SHREY RATHI DEDUCTIONS FROM GTI 10.19

42. Mr. Ganesh gave donation by way of cheque of ₹ 40,000 and by cash ₹ 5,000 to an approved charitable trust having
recognition under Section 80G. His gross total income for the previous year 2022-23 is ₹ 5 lakhs. The quantum of
deduction under section 80G would be:
(a) ₹ 45,000 (c) ₹ 40,000
(b) ₹ 22,500 (d) ₹ 20,000

43. The amount of deduction under section 80DD in respect of maintenance including medical treatment of a dependent
with 60% disability will be ₹ When no amount is actually spent on treatment by the resident
assessee and the handicapped person does not claim any deduction under section 80U.
(a) ₹ 50,000 (c) ₹ 1,00,000
(b) ₹ 75,000 (d) ₹ 1,25,000

44. Deduction for donation to National Fund for Control of Drug Abuse is eligible for
(a) 100% deduction without any qualifying limit (c) 50% deduction without any qualifying limit
(b) 100% deduction subject to qualifying limit (d) 50% deduction subject to qualifying limit

45. Section 80GGA allows 100% deduction for donation to:


(a) Approved research association, university, college to be used in scientific research
(b) Association engaged in any programme for rural development
(c) Association, university, college to be used in social science or statistical research
(d) All of the above

46. Mr. Suraj, a salaried employee earned a Salary income of ₹ 8,00,000 during the PY 2022-23. He also earned Short term
capital gains of ₹ 50,000 from sale of listed securities. He incurs rent expense of ₹ 10,000 p.m. What is the allowable
deduction under section 80GG for the AY 2023-24 assuming all the conditions are satisfied?
(a) ₹ 45,000 (c) ₹ 40,000
(b) ₹ 1,20,000 (d) ₹ 60,000

47. Mr. Kunal has incurred ₹ 80,000 by way of interest in respect of a loan taken on 1.4.2022 from an approved Financial
Institution for the higher studies of his son, Gaurav who is pursuing MBA in Germany. The allowable deduction under
section 80E for the AY 2023-24 is
(a) ₹ 50,000 (c) ₹ 80,000
(b) ₹ 40,000 (d) Nil

48. Deduction u/s 80EEB for interest payable on loan taken for purchase of electric vehicle, where such loan is sanctioned
between
(a) 01.04.2019 – 31.03.2020 (c) 01.04.2019 – 31.03.2022
(b) 01.04.2019 – 31.03.2021 (d) 01.04.2019 – 31.03.2023

49. Deduction in respect of employee cost of new workmen u/s 80JJAA shall not be allowed in respect of which of the
employees?
(a) Contract labour
(b) Employee who does not participate in the recognised provident fund
(c) Employee having salary of more than ₹ 25,000 p.m.
(d) All of the above

50. Deduction for a person with disability has been discussed under
(a) Section 80D (c) Section 80DD
(b) Section 80DDB (d) Section 80U

51. An individual has paid life insurance premium of ₹ 25,000 during the previous year for a policy of ₹ 1,00,000 taken on
1-4-2019. He shall- (ICAI MCQ Booklet)
(a) Not be allowed deduction u/s 80C
(b) Be allowed deduction of ₹ 20,000 u/s 80C
(c) Be allowed deduction of ₹ 25,000 under section 80C
(d) Be allowed deduct of ₹ 10,000 u/s 80C
CA SHREY RATHI DEDUCTIONS FROM GTI 10.20

52. In respect of loan of ₹ 40 lakhs sanctioned by SBI in March, 2022 for purchase of residential house intended for self-
occupation, compute the interest deduction allowable under the provisions of the Act for A.Y. 2023-24, assuming that
the disbursement was made on 1st June, 2022, the rate of interest is 8% p.a. and the loan sanctioned was 80% of the
stamp duty value of the property. (ICAI MCQ Booklet)
(a) ₹ 2,00,000 u/s 24 and ₹ 66,667 u/s 80EEA (c) ₹ 2,00,000 u/s 24 and ₹ 50,000 u/s 80EEA
(b) ₹ 1,50,000 u/s 80EEA and ₹ 1,16,667 u/s 24 (d) ₹ 2,00,000 u/s 24

53. Rajan, a resident Indian, has incurred ₹ 15,000 for medical treatment of his dependent brother, who is a person with
severe disability and has deposited ₹ 20,000 with LIC for his maintenance. For A.Y. 2023-24, Rajan would be eligible
for deduction under section 80DD of an amount equal to – (ICAI MCQ Booklet)
(a) ₹ 15,000 (c) ₹ 75,000
(b) ₹ 35,000 (d) ₹ 1,25,000

54. Mr. Shiva made donation of ₹ 50,000 to PM Cares Fund and ₹ 20,000 to Rajiv Gandhi Foundation by cheque. He made
a cash donation of ₹ 10,000 to a public charitable trust. The deduction allowable to him under section 80G for A.Y.
2023-24 is – (ICAI MCQ Booklet)
(a) ₹ 80,000 (c) ₹ 60,000
(b) ₹ 70,000 (d) ₹ 35,000

55. Mr. Ritvik has purchased his first house in Gwalior for self-occupation on 5-4-2021 for ₹ 45 lakhs (stamp duty value
being the same) with bank loan sanctioned on 30-3-2021 and disbursed on 3-4-21. He paid interest of ₹ 3.8 lakhs
during the P.Y. 2022-23. What is the tax treatment of interest by him? (ICAI MCQ Booklet)
(a) Interest of ₹ 2 lakh allowable u/s 24
(b) Interest of ₹ 2 lakh allowable u/s 24 and ₹ 1.8 lakhs allowable u/s 80EEA
(c) Interest of ₹ 2 lakh allowable u/s 24 and ₹ 1.5 lakhs allowable u/s 80EEA
(d) Interest of ₹ 1.5 lakh allowable u/s 24 and ₹ 1.5 lakhs allowable u/s 80EEA

56. Mr. Anuj is a businessman whose total income (after allowing deduction under Chapter VI-A except under section
80GG) for A.Y. 2022-23 is ₹ 5,95,000. He does not own any house property and is staying in a rented accommodation
in Patna for a monthly rent of ₹ 9,000. Deduction under section 80GG for A.Y. 2023-24 is - (ICAI MCQ Booklet)
(a) ₹ 48,500 (c) ₹ 60,000
(b) ₹ 1,48,750 (d) ₹ 1,08,000

MCQ Answers
1. (b) 2. (d) 3. (c) 4. (d) 5. (b) 6. (b) 7. (c) 8. (c) 9. (b) 10. (d)
11. (b) 12. (c) 13. (a) 14. (c) 15. (d) 16. (b) 17. (c) 18. (d) 19. (c) 20. (a)
21. (c) 22. (c) 23. (d) 24. (d) 25. (b) 26. (b) 27. (b) 28. (d) 29. (d) 30. (b)
31. (a) 32. (b) 33. (d) 34. (a) 35. (b) 36. (d) 37. (a) 38. (c) 39. (d) 40. (a)
41. (d) 42. (d) 43. (b) 44. (a) 45. (d) 46. (c) 47. (d) 48. (d) 49. (d) 50. (d)
51. (d) 52. (d) 53. (d) 54. (c) 55. (c) 56. (a)
CA SHREY RATHI INCOME EXEMPT FROM TAX 11.1

M S DHONI

CHAPTER 11
INCOME EXEMPT FROM TAX

"My goal is not to be better than anyone


else, but to be better than I used to be.”

PRACTICAL QUESTIONS
Q 1: Y Ltd. furnishes you the following information for the year ended 31-03-23:
Particulars (₹) in lacs
Total turnover of Unit A located in Special Economic Zone 100
Profit of the business of Unit A 30
Export turnover of Unit A 50
Total turnover of Unit B located in Domestic Tariff Area (DTA) 200
Profit of the business of Unit B 20
Compute deduction under Section 10AA for the A.Y. 2023-24, assuming that Y Ltd. commenced operation in SEZ and DTA
in the year 2018-19. [ICAI Study Module]

Sol 1: 100% of the profit derived from export of articles or things or services is eligible for deduction under Section 10AA,
since F.Y. 2022-2323 falls within the first five year period commencing from the year of manufacture or production of
articles or things or provision of services by the Unit in SEZ. As per Section 10AA(7), the profit derived from export of
articles or things or services shall be the amount which bears to the profits of the business of the undertaking, being the
Unit, the same proportion as the export turnover in respect of articles or things or services bears to the total turnover of
the business carried on by the undertaking.
Deduction under Section 10AA:
= Profit of the business of Unit A x Export Turnover of Unit A x 100%
Total Turnover of Unit A
= ₹ 30 lakhs x ₹ 50 lakhs x 100% = ₹ 15 lakhs
100 lakhs
Note: No deduction under Section 10AA is allowable in respect of profits of business of Unit B located in DTA.

Q 2: Examine with reasons in brief whether the following statements are true or false with reference to the provisions of
the Income-tax Act, 1961:
I. Pension received by a recipient of gallantry award, who was a former employee of Central Government, is exempt
from income-tax.
II. Mr. A, a member of a HUF, received ₹ 10,000 as his share from the income of the HUF. The same is to be included
in his chargeable income. [ICAI Study Module]
CA SHREY RATHI INCOME EXEMPT FROM TAX 11.2

Sol 2:
I. True: Section 10(18) exempts any income by way of pension received by individual who has been in service of
Central Government and has been awarded ‘Param Vir Chakra’ or “Mahavir Chakra” or “Vir Chakra” or such other
gallantry award as the Central Government, may, by notification in the Official Gazette, specify in this behalf.
II. False: Section 10(2) exempts any sum received by an individual as a member of a HUF where such sum has been
paid out of the income of the family. Therefore, Rs.10,000 should not be included in Mr. A’s chargeable income.

MULTIPLE CHOICE QUESTIONS


1. XYZ Ltd. has two units, one unit at Special Economic Zone (SEZ) and other unit at Domestic Tariff Area (DTA). The unit
in SEZ was set up and started manufacturing from 12-3-2014 and unit in DTA from 15-6-2017. Total turnover of XYZ
Ltd. and Unit in DTA is ₹ 8,50,00,000 and ₹ 3,25,000, respectively. Export sales of unit in SEZ and DTA is ₹ 2,50,00,000
and ₹ 1,25,00,000, respectively and net profit of Unit in SEZ and DTA is ₹ 80,00,000 and ₹ 45,00,000 respectively. XYZ
Ltd. would be eligible for deduction under section 10AA for P.Y. 2022-23 for- (ICAI MCQ Booklet)
(a) ₹ 38,09,524 (c) ₹ 23,52,941
(b) ₹ 19,04,762 (d) ₹ 11,76,471

2. Amount received from recognized provident fund after five years of continuous service is -
(a) Fully exempt u/s 10(12)
(b) Fully taxable as income of salary
(c) Taxable to the extent of employer’s contribution and interest thereon
(d) Taxable at the special rate of 10%

3. Any sum received by an individual as a member of HUF from the income of HUF shall be:
(a) Fully taxable as income of business and profession
(b) Fully exempt u/s 10(2)
(c) Included in the total income of the member of HUF for tax rate purpose
(d) Taxable at the special rate of 15%

4. Scholarship received by a student was ₹ 75,000 p.m. He spends ₹ 70,000 for meeting the cost of education. The
balance ₹ 5,000 is:
(a) ₹ 75,000 is taxable (c) ₹ 5,000 is taxable
(b) ₹ 75,000 is exempt (d) ₹ 5,000 is exempt

5. An author was awarded by Central Government a sum of ₹ 10,00,000 for writing a book in Hindi as first prize. Such
award was given by Central Government in the public interest. This award is:
(a) Casual income (c) fully taxable
(b) Fully exempt (d) exempt up to ₹ 2,50,000

6. Tax paid by the employer on non-monetary perquisites provided to employee shall be exempt:
(a) Under section 17(2) (c) Under section 10(10CC)
(b) Under section 10(14) (d) None of the above

7. Dividend received by a shareholder from foreign company is:


(a) Exempt u/s 10(34) (c) Exempt to the extent of ₹ 10,00,000
(b) Taxable as income from other sources (d) Taxable at the special rate of 15%

8. Any sum received under a Life Insurance Policy including bonus shall be exempt u/s 10(10D):
(a) In all kinds of policies
(b) In all kinds of policies except when received under a Key man Insurance Policy
(c) In all kinds of policies except when received under Key-man Policy or under a policy covered under section
80DD(3).
(d) In all kinds of policies except when received under Key-man Insurance Policy or such policy as is covered under
section 80DD(3) or policy issued, if the premium paid for any year exceeds 10% / 15% as the case may be, of
actual capital sum assured, except on death
CA SHREY RATHI INCOME EXEMPT FROM TAX 11.3

9. Any pension received by an individual or family pension received by any member of his family will be taxable in which
of the following cases:
(a) such individual is awarded Param Vir Chakra (c) such individual is awarded Vir Chakra
(b) such individual is awarded Maha Vir Chakra (d) such individual is awarded Bharat Ratna

10. Any amount received or receivable from the Central Government or a State Government or a local authority by an
individual or his legal heir by way of compensation on account of any disaster shall be:
(a) Taxable
(b) Fully exempt
(c) Exempt except to the extent the amount received or receivable has been allowed as deduction on account of any
loss or damage caused by such disaster.
(d) Exempt up to ₹ 10,00,000

11. In case of an individual, any income by way of interest on any money standing to his credit in a Non-Resident
(External) Account in any bank in India shall be
(a) Taxable for the person in whose name account is being operated
(b) Exempt for the person in whose name account is being operated
(c) Taxable for the person who withdraws amount from such account
(d) Exempt for the person who withdraws amount from such account

12. The quantum of deduction available under section 10AA in respect of profits and gains derived by a SEZ unit from
export of articles is-
(a) 100% of export profits for first 10 consecutive AYs and 50% for next 5 consecutive AYs
(b) 100% of export profits for first 5 consecutive AYs and 50% for next 10 consecutive AYs
(c) 100% of export profits for first 15 consecutive AYs
(d) 100% of export profits for first 5 consecutive AYs, 50% for export profits for next 5 consecutive AYs and upto 50%
of export profits for next 5 consecutive AYs, as is credited to Special Reserve Account.

13. Which of the following income is not exempt under section 10:
(a) Share of partner in total income of partnership firm
(b) Income from agriculture in Lahore
(c) Bonus of life insurance proceedings on maturity
(d) Income from mutual funds.

14. There is difference between ‘exemption’ and ‘deduction’ even though both result in tax reduction for assessee.
(a) True
(b) False

15. Government of India paid salary of ₹ 5 lakh and allowances/perquisites valued at ₹ 2.20 lakh to a person who is citizen
of India for the services rendered by him outside India during the previous year. His total income chargeable to tax
would be:
(a) ₹ 7,20,000 (c) ₹ 6,10,000
(b) ₹ 5,00,000 (d) Nil

16. Mr. Sankar received ₹ 50,000 as educational scholarship from Nehru Memorial Trust (a charitable trust). The
scholarship is to assist Mr. Sankar for pursuing M.A. (History) at Jawaharlal Nehru University, New Delhi. The amount
of scholarship liable to tax is:
(a) ₹ 50,000 (c) ₹ 25,000
(b) ₹ 10,000 (d) Nil

17. Section ……….. does not allow expenditure incurred by the assessee in relation to income exempt while calculating the
total income of the assessee.
(a) 13A (c) 14A
(b) 15A (d) 16A
CA SHREY RATHI INCOME EXEMPT FROM TAX 11.4

18. Where a unit located in Special Economic Zone is also transacting within India, then income deriving from such
transactions shall be:
(a) Taxable in India (c) Exempt in India
(b) Partially taxable partially exempt in India (d) 50% taxable in India & 50% exempt

19. XYZ Ltd. has one unit in SEZ and other unit in Domestic Tariff Area. The company provides the following information
for the P/Y 2022-23. Compute the amount exempt u/s 10AA if this F/Y is the 4th year of operation.
Particulars XYZ Ltd. (₹) DTA (₹)
Total Sales 80,00,000 25,00,000
Export Sales 45,00,000 15,00,000
Net Profit 6,50,000 1,70,000
(a) ₹ 3,65,625 (c) ₹ 2,61,818
(b) ₹ 3,54,545 (d) ₹ 1,77,273

20. Remuneration to a foreign national as employee of a foreign enterprise shall be exempt u/s 10(6) provided his stay in
India does not exceed
(a) 30 days (c) 75 days
(b) 60 days (d) 90 days

MCQ Answers:
1. (b) 2. (a) 3. (b) 4. (b) 5. (b) 6. (c) 7. (b) 8. (d) 9. (d) 10. (b)
11. (b) 12. (d) 13. (b) 14. (a) 15. (b) 16. (d) 17. (c) 18. (a) 19. (c) 20. (d)
CA SHREY RATHI AGRICULTUAL INCOME 12.1

SHAHRUKH KHAN

CHAPTER 12
AGRICULTURAL INCOME

"Success is not a good teacher,


failure makes you humble”

PRACTICAL QUESTIONS
Q 1: Mr. Ankit (age: 31 years) has non-agricultural income of ₹ 4,20,000 and agricultural income of ₹ 6,60,000 for the P/Y
2022-23. He deposits ₹ 40,000 in PPF account. Compute his tax liability for A/Y 2023-24.

Sol 1: Computation of tax liability of Mr. Ankit for the A/Y 2023-24:
As non-agricultural income of Mr. Ankit exceeds the maximum amount not chargeable to tax (i.e. first exemption limit) &
agricultural income exceeds ₹ 5,000 then he shall be liable to tax in the following manner:
Particulars Tax (₹)
Step 1: Calculate tax on agricultural income + non-agricultural income = ₹ 10,40,000 1,24,500
Step 2: Calculate tax on agricultural income + first exemption limit = ₹ 9,10,000 94,500
Step 3: Step 1 – Step 2 is tax payable 30,000
Step 4: Less: Rebate u/s 87A if the non-agricultural income does not exceed ₹ 5,00,000 12,500
17,500
Step 5: Health & Education Cess @ 4% 700
Tax Liability 18,200
Note: Deposit in PPF shall be deducted from non-agricultural income before applying steps for computation of tax.

Q 2: Mrs. Sonia (age: 71 years) submits the following information for the A/Y 2023-24:
Particulars (₹)
House property income 4,80,000
Income from other sources 27,000
Income from the business of growing and manufacturing tea in India (gross) 3,00,000
Expenditure on growing and manufacturing tea 35,000
Income from a farm house 50,000
Contribution towards National Saving Certificate VIII issue 60,000
Compute her tax liability for the A/Y 2023-24.
CA SHREY RATHI AGRICULTUAL INCOME 12.2

Sol 2: Computation of tax liability of Mrs. Sonia for the A/Y 2023-24:
Particulars Agricultural Non-
Income (₹) agricultural
Income (₹)
House property income 4,80,000
Income from other sources 27,000
Income from the business of growing and manufacturing tea in India (gross)
₹ 3,00,000
Less: Expenditure on growing and manufacturing tea ₹ 35,000
(to be apportioned in the ratio of 60:40) ₹ 2,65,000 1,59,000 1,06,000
Income from farm house 50,000
Gross Total Income 2,09,000 6,13,000
Less: Deduction u/s 80C: Contribution towards NSC VIII issue - 60,000
Agricultural/Non-agricultural (business) income 2,09,000 5,53,000
As non-agricultural income of Mrs. Sonia (aged 71 years) exceeds the maximum amount not chargeable to tax (i.e. first
exemption limit of ₹ 3,00,000) & agricultural income exceeds ₹ 5,000, then he shall be liable to tax in the following
manner:
Particulars Tax (₹)
Step 1: Calculate tax on agricultural income + non-agricultural income = ₹ 7,62,000 62,400
Step 2: Calculate tax on agricultural income + first exemption limit = ₹ 5,09,000 11,800
Step 3: Step 1 – Step 2 is tax payable 50,600
Step 4: Less: Rebate u/s 87A if the non-agricultural income does not exceed ₹ 5,00,000 -
50,600
Step 5: Add: Health & Education Cess @ 4% 2,024
Tax liability of Mrs. Sonia 52,624
Tax liability (rounded off to the nearest multiple of ₹ 10) 52,620

Q 3: Mr. B grows sugarcane and uses the same for the purpose of manufacturing sugar in his factory. 30% of sugarcane
produce is sold for ₹ 10 lacs, and the cost of cultivation of such sugarcane is ₹ 5 lacs. the cost of cultivation of the balance
sugarcane (70%) is ₹ 14 lacs and the market value of the same is ₹ 22 lacs. After incurring Rs.1.5 lacs in the manufacturing
process on the balance sugarcane, the sugar was sold for ₹ 25 lacs. Compute B’s business income and agricultural income.
[ICAI Study Module]

Sol 3: Computation of Business Income and Agricultural Income of Mr. B:


Particulars Business income Agricultural Income
(₹) (₹) (₹)
Sale of sugar
Business income
Sale Proceeds of sugar 25,00,000
Less: Market value of sugar (70%) 22,00,000
Less: Manufacturing exp. 1,50,000
1,50,000
Agricultural income
Market value of sugar (70%) 22,00,000
Less: Cost of cultivation 14,00,000 8,00,000
Sale of sugarcane
Agricultural Income
Sale proceeds of sugarcane (30%)
10,00,000
Less: Cost of cultivation
5,00,000 5,00,000
13,00,000
CA SHREY RATHI AGRICULTUAL INCOME 12.3

MULTIPLE CHOICE QUESTIONS


1. Definition of Agriculture Income is given under section:
(a) 2(1B) (c) 2(1C)
(b) 2(1D) (d) 2(1A)

2. Agriculture Income from outside India is:


(a) fully exempt from tax (c) partially chargeable to tax
(b) fully chargeable to tax (d) none of the above

3. There will be no partial integration of agricultural income with non-agricultural income, if the non-agricultural income
does not exceed:
(a) ₹ 5,000 (c) ₹ 10,000
(b) ₹ 50,000 (d) Basic exemption limit

4. Mr. Kumar has agricultural land in Delhi which he has sold and has earned long term capital gains of ₹ 10 Lakhs. Mr.
Kumar claims it to be his agricultural Income?
(a) He is correct (c) He is partially correct and partially incorrect
(b) He is not correct (d) None of the above

5. If any person earns income from allied activities like animal husbandry, dairy farming, fishery etc. then it is:
(a) Considered to be agricultural Income
(b) Considered to be income u/h PGBP.
(c) Partial agricultural income & partial non-agricultural income
(d) Any of the above depending on the assessee

6. A person holds shares of ABC Ltd which is an Indian company and the company is engaged in agricultural operation
and person has received dividends of ₹ 12,000 and he claims it is his agricultural Income. Discuss whether it is:
(a) fully taxable (c) fully exempt as it is dividend from Indian company
(b) fully exempt as an agricultural income (d) taxable beyond ₹ 10,000.

7. If any partner has received share of profits from a firm having Income from Agriculture activities.
(a) It will be considered to be agricultural Income and exempt from tax
(b) It will be considered to be Income from PGBP
(c) It will be considered to be Income from PGBP but exempt from tax u/s 10(2A)
(d) it will be considered as partial agricultural income & partial non-agricultural income

8. An assessee has incurred ₹ 1,00,000 on the cultivation of agricultural produce. 50% of the produce has been sold for ₹
1,10,000 and the balance 50% has been used by the assessee for his self-consumption, the agricultural income in this
case shall be:
(a) ₹ 1,10,000 (c) ₹ 60,000
(b) ₹ 55,000 (d) ₹ 1,00,000

9. Income derived from sugarcane plantation in USA but received in India shall be treated as:
(a) agricultural income and hence exempt
(b) agricultural income but taxable under the head income from other sources
(c) exempt as earned outside India.
(d) taxable as income from business and profession

10. Which of the following would be agricultural income—


(a) Income from breeding of livestock
(b) Income from poultry farming
(c) Rent received from land used for movie shooting
(d) Rent received from land used for grazing of cattle required for agricultural activities
CA SHREY RATHI AGRICULTUAL INCOME 12.4

11. The proportion of agricultural and business income in case of income derived from the sale of coffee grown and cured
by the assessee in India is—
(a) 65% and 35%, respectively (c) 75% and 25%, respectively
(b) 60% and 40%, respectively (d) 70% and 30%, respectively

12. If the assessee is engaged in the business of growing and manufacturing tea in India, the agriculture income in that
case shall be:
(a) 60% of the income from such business.
(b) 40% of the income from such business
(c) market value of the agricultural produce minus the expenses on cultivation of such agricultural produce
(d) none of the above

13. If the assessee uses its own agricultural produce for the purpose of manufacturing certain products other than tea,
rubber or coffee the cost of such agricultural produce for the purpose of computing business income of
manufacturing shall be:
(a) cost of producing such agriculture produce
(b) market value of such agriculture produce as on the date of use
(c) average of cost & market value
(d) any of the above depending on the assessee

14. If an assessee uses the agriculture produce grown by him for his own consumption then:
(a) the market value of such agriculture produce shall be treated as his agriculture income
(b) the market value of such agriculture product minus the cost of cultivation shall be treated as his agriculture
income
(c) nothing shall be treated as his agriculture income
(d) none of the above

15. If a partnership firm earns agriculture income then—


(a) it will be exempt in the hands of firm
(b) it will be exempt in the hands of firm but taxable in the hands of the partners
(c) it will be exempt in the hands of firm as well its partners
(d) it will be taxable in the hands of firm but exempt in the hands of the partners

16. Income derived from rubber plantation in Singapore but received in India shall be treated as:
(a) agricultural income and hence exempt
(b) agricultural income but taxable under the head income from other sources
(c) taxable in Singapore and thus not taxable in India
(d) 65% agricultural income and hence exempt & 35% non-agricultural income and hence taxable u/h PGBP

17. An individual aged 64 years has income from nursery of ₹ 4,00,000 & private tuition income of ₹ 3,00,000. Compute
his tax liability.
(a) Nil (c) ₹ 5,200
(b) ₹ 52,000 (d) None of the above

18. A HUF (Karta’s age is 66 years) has agricultural income of ₹ 2,80,000 and non-agricultural income of ₹ 4,50,000. The
HUF has invested ₹ 50,000 in PPF. Compute the tax liability of the HUF.
(a) Nil (c) ₹ 60,840
(b) ₹ 7,800 (d) ₹ 31,200

19. Agriculture Income is exempt provided the:


(a) land is situated in anywhere in India (c) land is situated in any rural area in India
(b) land is situated in any urban area in India (d) land is situated whether in India or outside India.
CA SHREY RATHI AGRICULTUAL INCOME 12.5

20. From the following information, compute agricultural income for Mr. Roy:
i. Income from farm house – ₹ 65,000
ii. Income from fisheries – ₹ 85,000
iii. Income from coffee grown, cured, roasted & grounded in Kerala – ₹ 1,00,000
iv. Income from tea grown in Srilanka – ₹ 50,000
v. Life insurance premium paid – ₹ 20,000
(a) ₹ 1,45,000 (c) ₹ 1,75,000
(b) ₹ 2,80,000 (d) ₹ 1,25,000

21. Which of the following statements is/are true in respect of taxability of agricultural income under the Income-tax Act,
1961? (ICAI MCQ Booklet)
(i) Any income derived from saplings or seedlings grown in a nursery is agricultural income exempt from tax u/s
10(1)
(ii) 60% of dividend received from shares held in a tea company is agricultural income exempt from tax u/s 10(1)
(iii) While computing income tax liability of an assessee aged 50 years, agricultural income is required to be added to
total income only if net agricultural income for the P.Y. exceeds ₹ 5,000 and the total income (including net
agricultural income) exceeds ₹ 2,50,000.
(iv) While computing income tax liability of an assessee aged 50 years, agricultural income is required to be added to
total income only if net agricultural income for the P.Y. exceeds ₹ 5,000 and the total income (excluding net
agricultural income) exceeds ₹ 2,50,000.
Choose the correct answer:
(a) (i) and (iii) (c) (i) and (iv)
(b) (ii) and (iii) (d) (i), (ii) and (iv)

MCQ’s Answers
1. (d) 2. (b) 3. (d) 4. (b) 5. (b) 6. (a) 7. (c) 8. (c) 9. (b) 10. (d)
11. (c) 12. (a) 13. (b) 14. (c) 15. (c) 16. (b) 17. (a) 18. (d) 19. (a) 20. (d)
21. (c)

CASE BASED MCQ


1. Mr. Kishan is engaged in the following activities on agricultural land situated in India, total area of land is 5 acres.
Activity A: He grows saplings or seedlings in a nursery spreading over on one acre land, the sale proceeds of which is ₹
5,00,000. Cost of plantation is ₹ 1,40,000. Basic operations are not performed for growing saplings or seedlings.
Activity B: He grows cotton on 3 acres land. 40% of cotton produce is sold for ₹ 4,00,000, the cost of cultivation of
which is ₹ 2,25,000. The cost of cultivation of balance 60% cotton is ₹ 3,37,500 and the market value of the same is ₹
6,00,000, which is used for the purpose of manufacturing yarn. After incurring manufacturing expenses of ₹ 1,00,000,
yearn is sold for ₹ 8,50,000
Activity C: Land measuring 1 acre is let out to Mr. Ramesh on monthly rental of ₹ 15,000 which is used by Mr. Ramesh
as follows:
- 50% of land is used for agricultural purpose
- 50% of land is used for non-agricultural purpose.
Based on the facts of the case scenario given above, choose the most appropriate answer to the following questions:
[ICAI MCQ Booklet]

1.1 What amount of income arising from activity A would constitute agricultural income in the hands of Mr. Kishan?
(a) ₹ 5,00,000 (c) ₹ 3,60,000
(b) Nil (d) ₹ 1,40,000
CA SHREY RATHI AGRICULTUAL INCOME 12.6

1.2 What amount of income from activity B with respect to sale of cotton would constitute agricultural income or/and
business income in the hands of Mr. Kishan?
(a) ₹ 1,75,000 as agricultural income
(b) ₹ 1,75,000 as business income
(c) ₹ 1,75,000 as agricultural income and ₹ 2,62,500 as business income
(d) ₹ 4,00,000 as agricultural income

1.3 What amount of the income from activity B with respect to sale of Yarn constitute agricultural income or/and
business income in the hands of Mr. Kishan?
(a) ₹ 1,50,000 as agricultural income
(b) ₹ 2,62,500 as agricultural income and ₹ 1,50,000 as business income.
(c) ₹ 3,37,500 as agricultural income and ₹ 1,50,000 as business income
(d) ₹ 4,12,500 as business income

1.4 What amount of income arising from activity C constitute agricultural income or otherwise in the hands of Mr.
Kishan?
(a) Whole amount of 1,80,000 would be agricultural income
(b) Whole amount of 1,80,000 would be business income
(c) ₹ 90,000 would be agricultural income and ₹ 63,000 is chargeable to tax as income from house property
(d) ₹ 90,000 would be agricultural income and ₹ 90,000 is chargeable to tax under the head “Income from Other
Sources”

1.5 Compute the gross total income of Mr. Kishan for the P.Y. 2022-23, assuming he has no other source of income.
(a) ₹ 2.40,000 (c) ₹ 5,02,500
(b) ₹ 3,30,000 (d) ₹ 2,13,000

MCQ’s Answers
1.1 (c) 1.2 (a) 1.3 (b) 1.4 (d) 1.5. (a)
CA SHREY RATHI TDS & TCS 13.1

BILL GATES

CHAPTER 13
TAX DEDUCTION AT SOURCE &
TAX COLLECTION AT SOURCE

“Don’t compare yourself with anyone


in this world…if you are doing so, you
are insulting yourself”

PRACTICAL QUESTIONS
Q 1: DLF Ltd. has let out a commercial building to Vodafone Ltd. for ₹ 5,00,000 p.m. plus GST @ 12%. Compute the net
amount payable & tax to be deducted at source by Vodafone Ltd. for the P/Y 2022-23.

Sol 1: Computation of net amount payable & tax to be deducted at source by Vodafone Ltd. for the P/Y 2022-23:
Particulars (₹)
Rent payable by Vodafone Ltd. (₹ 5,00,000 x 12) 60,00,000
(+) GST @ 12% 7,20,000
Total amount payable 67,20,000
(-) TDS @ 10% u/s 194I (TDS shall not be deducted on the GST portion assuming it has been
separately shown in the invoice) (₹ 60,00,000 x 10%) 6,00,000
Net amount payable 61,20,000

Q 2: Mr. Bhaggu is doing clothing business. He submits you the following information:
Turnover for the financial year:
2021-22 - ₹ 1,00,10,000
2022-23 - ₹ 98,99,000
State whether the provisions of TDS are attracted for the following expenses incurred during the financial year 2022-23:
(Assuming all payments were made in the month of April 2022)
(i) Interest paid to Indian Bank on term loan – ₹ 71,900.
(ii) Advertisement expenses of two individual expenses to Mr. Ali of ₹ 28,000 & ₹ 39,000.
(iii) Factory rent paid – ₹ 3,00,000.
(iv) Brokerage paid to a sub-broker – ₹ 8,000.

Sol 2: As the turnover of Mr. Bhaggu for the previous F/Y 2022-22, i.e. ₹ 1,00,10,000, has exceeded the monetary limit of
100 lakhs, he has to comply with the tax deduction provisions during the F/Y 2022-23, subject to the exemptions provided
under the relevant sections for applicability of TDS provisions.
(i) Interest paid to Indian Bank on term loan – ₹ 71,900: TDS u/s 194A is not attracted in respect of interest paid to a
banking company.
CA SHREY RATHI TDS & TCS 13.2

(ii) Advertisement expenses of two individual expenses to Mr. Ali of ₹ 28,000 & ₹ 39,000: TDS provisions u/s 194C gets
attracted if the amount paid to a contractor exceeds ₹ 30,000 in a single payment or ₹ 1,00,000 in aggregate during
the financial year. TDS @ 1% is applicable in case of individual/HUF contractors & 2% in case of other contractors.
Therefore TDS @ 1% on ₹ 39,000 shall be levied. TDS = ₹ 390.
(iii) Factory rent paid – ₹ 3,00,000: TDS u/s 194I @ 10% shall apply as the rental payment exceeds ₹ 2,40,000. TDS = ₹
30,000.
(iv) Brokerage paid to a sub-broker – ₹ 8,000: No TDS u/s 194H shall be levied as the commission does not exceed ₹
15,000.

Q 3: Mr. Bhide is employed in a company in Jaipur. His income from salary is ₹ 8,00,000 annually. He made a payment of ₹
1,00,000 eligible for deduction u/s 80C. He also informed his employer that he has incurred a loss of ₹ 60,000 u/h house
property. Compute the amount of tax deductible at source per month.

Sol 3: Computation of tax deductible at source per month for Mr. Bhide:
Particulars (₹)
Annual salary 8,00,000
(-) Loss u/h house property 60,000
Gross Total Income 7,40,000
(-) Deductions u/c VI-A: 80C 1,00,000
Total Income
6,40,000
Tax liability:
Tax on ₹ 6,40,000 (assuming the age of Mr. Bhide is less than 60 years) 40,500
(+) Health & Education Cess @ 4% 1,620
Total tax liability
42,120
Average Rate of Tax (42,120 / 8,00,000 x 100) 5.265%
TDS to be deducted per month (₹ 8,00,000/12 x 5.265%) 3,510
Notes: Loss u/h house property can be deducted from the salary income provided the name, address & PAN of the lender
is obtained. Similarly, deductions u/c VI-A can be made provided proper evidences regarding the investments are
provided.

Q 4: Compute amount of tax to be deducted at source on the following payments made by XY Ltd. during the P/Y 2022-23
as per the provisions of Income Tax Act, 1961.
(a) Payment of ₹ 1,00,000 to Mr. Rajbeer, a transporter who furnishes his PAN on 28th July 2022.
(b) Payment of fee for professional services of ₹ 54,000 to Mr. Gagan who is having PAN on 13th May 2022.
(c) Payment of ₹ 22,000 to MN Ltd. for repair of building on 29th September 2022.
(d) Payment of ₹ 3,50,000 to Mr. Ajay for purchase of chemicals for business on 15th January 2023.
(e) Payment of ₹ 1,90,000 made to Mr. Lalu for compulsory acquisition of his house as per law of the State
Government on 19th October 2022.
(f) The company held a lucky draw. A car worth ₹ 2,80,000 was given to the prize winner on 22nd July 2022.
(g) The company is engaged in the business of insurance. It paid ₹ 60,000 as insurance commission to its agent on 27th
December 2022.
CA SHREY RATHI TDS & TCS 13.3

Sol 4: Computation of tax to be deducted by XY Ltd. in the following payments:


S. No. Particulars (₹)
(a) Payment of ₹ 1,00,000 to Mr. Rajbeer, a transporter who furnishes his PAN: No TDS is required to ---
be deducted u/s 194C on payment to Mr. Rajbeer since he does not own 10 or less goods
carriages at any time during the previous year (assumed) & has furnished his PAN.

(b) Payment of fee for professional services of ₹ 54,000 to Mr. Gagan who is having PAN on 13th May 5,400
2022: As per section 194J, liability to deduct TDS @ 10% gets attracted where the payment made
as fees for professional services and royalty, individually, exceeds ₹ 30,000 during the financial
year. Therefore, TDS shall be deducted on ₹ 54,000.

(c) Payment of ₹ 22,000 to MN Ltd. for repair of building: Provisions u/s 194C are not attracted, since ---
the payment for repair of building is less than the threshold limit of ₹ 30,000.

(d) Payment of ₹ 3,50,000 to Mr. Ajay for purchase of chemicals for business: No TDS provisions are ---
applicable on purchase of goods for the purpose of business.

(e) Payment of ₹ 1,90,000 made to Mr. Lalu for compulsory acquisition of his house as per law of the ---
State Government: As per section 194LA, any person responsible for payment to a resident any
compensation on compulsory acquisition shall deduct tax @ 10% where such compensation
exceeds ₹ 2,50,000. In the given case, no TDS as the payment made does not exceed ₹ 2,50,000.

(f) The company held a lucky draw. A car worth ₹ 2,80,000 was given to the prize winner: As per 84,000
section 194B, TDS shall be deducted @ 30% on lottery prize where it exceeds ₹ 10,000. Where the
lottery is in kind, the organiser shall ensure deduction of tax before releasing the winnings. It has
been assumed that the organiser has taken reasonable steps for deduction of tax.

(g) Company paid ₹ 60,000 as insurance commission to its agent: As per section 194D, TDS is 2,250
deductible @ 5% if the commission or brokerage exceeds ₹ 15,000. Therefore, TDS shall be
deductible in this case.
Total TDS deductible by XY Ltd. 91,650

Q 5: An amount of ₹ 40,000 was paid to Mr. X on 01.07.2022 towards fees for professional services without deduction of
tax at source. Subsequently another payment of ₹ 50,000 was due to Mr. X on 27.02.2023, from which tax @ 10%
(amounting to ₹ 9,000) on the entire amount of ₹ 90,000 was deducted. However, this tax of ₹ 9,000 was deposited only
on 23.06.2023. Compute the interest chargeable u/s 201. [ICAI Study Module]

Sol 5: Computation of interest u/s 201 for Mr. X:


Where a person liable to deduct tax fails to do so, he shall be charged @ 1% p.m. or part of the month from the date on
which tax was deductible to the date tax is actually deducted. But where a person deducts tax and fails to pay such tax, he
shall be liable to interest @ 1.5% p.m. or part of the month from the date on which tax was deducted to the date it is
actually paid. Interest shall be calculated on the amount of TDS.

Interest from 01.07.2022 – 27.02.2023: ₹ 4,000 x 1% x 8 = ₹ 320


Interest from 27.02.2022 – 23.06.2023: ₹ 9,000 x 1.5% x 4 = ₹ 540
₹ 860

Q 6: Discuss whether the person making payment is liable to deduct tax at source in the following circumstances. Also
compute the amount of tax to be deducted.
(i) Interest of ₹ 2,900 paid by ZYX Ltd. to Luv and ₹ 5,500 to his HUF on 27.04.2022, by way of account payee cheque
on account of debenture of the company held by them separately. Debentures are listed in Bombay Stock
Exchange.
(ii) Sitting fees of ₹ 29,900 paid to director of the company on 12.10.2022.
CA SHREY RATHI TDS & TCS 13.4

(iii) Sports World Magazine paid ₹ 3,80,000 to Brian Lara, a non-resident cricketer, for writing an article for the August
issue. The payment for the same was made on 25.08.2022.
(iv) Interest of ₹ 43,300 on recurring deposit credited to Ms. Zubia on 31st March 2023.
(v) Punjab National Bank pays ₹ 90,000 p.m. as rent on the last day of each month to the State Government for a
building.
(vi) An engineer was reimbursed ₹ 40,000 on 15th April 2022 towards expenses incurred by him for a company. Such
amount is separate from his contract amount.
(vii) By virtue of an agreement with Nationalised Bank, a Catering organisation (Pvt. Co.) receives ₹ 50,000 p.m. towards
supply of food, water, snacks etc. during office hours to the employees of the bank. The contract came into force
from 1st July 2022.
(viii) Fees paid to Dr. Lahoti by Sunder (HUF) ₹ 42,000 for surgery performed on a member of his family.
[ICAI Study Module (modified)]

Sol 6: Determining the applicability of TDS provisions:


(i) Section 193 provides deduction of tax at source @ 10% on interest where such interest exceeds ₹ 5,000. In this
case, interest to Luv i.e. ₹ 2,900 shall not be subject to TDS provisions whereas interest to Kush i.e. ₹ 5,500 shall be
subject to TDS @ 10% as interest exceeds ₹ 5,000. TDS = ₹ 550.
(ii) Section 194J provides for deduction of tax at source @ 10% from any sum paid by way of remuneration or fees or
commission to a resident director, which is not in the nature of salary. The threshold limit of ₹ 30,000 is not
applicable in respect of any sum paid to the assessee. Therefore, tax @ 10% has to be deducted at source u/s 194J
on ₹ 29,900. TDS = ₹ 2,990.
(iii) Section 194E provides that the person responsible for payment of any amount to a non-resident sportsman for
contribution towards any game or sports in India has to deduct tax at source @ 20% + Health & Education Cess @
4%. Therefore, payment of ₹ 3,80,000 made by Sports World Magazine to Brian Lara for writing an article shall be
subject to TDS provisions. TDS = ₹ 79,040.
(iv) Section 194A shall be levied @ 10% where interest other than interest on securities exceeds ₹ 5,000 (₹ 40,000 in
case of banking company or co-operative societies or a post office). Therefore, interest of ₹ 43,300 on recurring
deposit shall be liable to TDS @ 10%. TDS = ₹ 4,330.
(v) Section 194I provides for deduction of tax at source @ 10% where rent paid exceeds ₹ 2,40,000. However, where
the payee is Central Government or State Government, no TDS is deductible. Therefore, payment by Punjab
National Bank to State Government shall not be liable to TDS.
(vi) Reimbursement of expenses is not an income in the hands of the payee. Therefore, reimbursement of ₹ 40,000 by
the company to the engineer shall not fall under the ambit of TDS provisions.
(vii) Section 194C provides for deduction of tax at source where the amount paid to a contractor exceeds ₹ 30,000 in a
single payment or ₹ 1,00,000 in aggregate during the financial year. TDS @ 1% is applicable in case of
individual/HUF contractors & 2% in case of other contractors. In this case, as the contractor is a private company,
TDS shall be deductible @ 2% of ₹ 4,50,000 (₹ 50,000 x 9). TDS = ₹ 9,000.
(viii) As per the provisions of section 194J, a Hindu Undivided Family is required to deduct tax at source on fees paid for
professional services only if it is subject to tax audit in the preceding financial year.
However, if such payment made for professional services is exclusively for personal purpose of any member of HUF,
then the liability to deduct tax is not attracted. Therefore, in the given case, even if Sunder (HUF) is liable to tax
audit in the immediately preceding financial year, TDS shall not be deducted.

MULTIPLE CHOICE QUESTIONS


1. The deduction of tax at source from the salary shall be made at the time of:
(a) accrual of salary (c) credit or payment of the salary, whichever is earlier
(b) payment of salary (d) Any of the above at the choice of employer
CA SHREY RATHI TDS & TCS 13.5

2. The liability to deduct tax at source in case of income from interest on securities arises at the time of:
(a) payment of interest
(b) accrual of interest
(c) Credit of interest to the account of the payee or payment thereof, whichever is earlier
(d) any of the above at the choice of the payer of the interest

3. No deduction of tax at source on interest on listed debentures is to be done by the widely held company:
(a) if the interest is paid by account payee cheque
(b) the amount of interest paid or payable during the financial year does not exceed ₹ 5,000
(c) interest is paid or payable to an individual who is resident in India.
(d) All of the above

4. Mr. K an individual has borrowed a sum of ₹ 10,00,000 on 1/4/2022 @ 18% p.a. from SBI. Mr. K in this case should
deduct tax on such interest paid amounting to:
(a) ₹ 1,80,000 (c) ₹ 18,000
(b) ₹ 1,87,200 (d) Nil

5. Mr. A, a salaried individual, pays rent of ₹ 51,000 per month to Mr. B from June, 2022. Which of the statement is
true?
(a) No tax is deductible at source since Mr. A is a salaried individual.
(b) Tax is deductible at source every month @ 5% on rent paid to Mr. B.
(c) Tax is deductible at source every month @ 10% on rent paid to Mr. B.
(d) Tax is deductible at source @ 5% on annual rent from the rent paid for March 2023.

6. Mr. X, a resident Indian, wins ₹ 1,80,000 in a Game Show. Which of the statement is true?
(a) Tax is deductible u/s 194B @ 30%
(b) Tax is deductible u/s 194B @ 31.2%
(c) Tax is deductible u/s 194B @ to be decided by the winner
(d) Tax is deductible u/s 194B @ to be decided by the organizer of the game show

7. Mr. J has deposited a sum of ₹ 2,00,000 on 1/4/2022 with a scheduled bank for one year at the interest rate of 10%
pa. The bank should deduct tax at source amounting to:
(a) ₹ 2,080 (c) ₹ 4,000
(b) Nil (d) ₹ 2,000

8. Mr. X paid fees for professional services of ₹ 40,000 on 10th May 2022 to Mr. A. Mr. A is engaged only in the business
of operation of call centre. Tax is to be deducted by Mr. X at the rate of (ICAI MCQ Booklet)
(a) 1% (c) 10%
(b) 2% (d) 5%

9. Mr. Janardan an individual, whose turnover of the business for the preceding year exceeded ₹ 100 lakhs, has
engaged a contractor Mr. Akash for building his residential premises. On 30/11/2022, he has made a payment of ₹
60,00,000 to an individual contractor. Mr. Janardan should deduct the tax at source amounting to:
(a) ₹ 60,000 (c) ₹ 2,25,000
(b) ₹ 1,20,000 (d) ₹ 3,00,000

10. No tax is to be deducted at source if the amount credited / paid to the contractor during the relevant previous year
does not exceed:
(a) ₹ 30,000
(b) ₹ 1,00,000
(c) ₹ 30,000 at one time or ₹ 1,00,000 in aggregate in the financial year
(d) ₹ 1,30,000
CA SHREY RATHI TDS & TCS 13.6

11. A company has taken a house on rent @ ₹ 20,000 pm. The company should deduct tax on account of such rent
paid/credited on monthly basis amounting to:
(a) Nil (c) ₹ 2,000
(b) ₹ 120 (d) ₹ 24,960

12. Who shall not be liable to deduct TDS under section 194C?
(a) Any Individual
(b) Any HUF
(c) Any Individual and Any HUF whose accounts were not liable to audit in preceding financial year
(d) Any Individual and Any HUF whose accounts were liable to audit in preceding financial year

13. What shall be the rate at which TDS on insurance commission shall be deducted under section 194D when the payee
is a domestic company?
(a) 20% (c) 5%
(b) 10% (d) 15%

14. What is maximum amount upto which TDS on commission on sale of lottery tickets shall not be deducted?
(a) ₹ 5,000 (c) ₹ 1,000
(b) ₹ 20,000 (d) ₹ 15,000

15. What shall be the rate of TDS under section 194I on payment of rent of plant and machinery where rent is paid to
any person other than individual or HUF?
(a) 1% (c) 10%
(b) 2% (d) 20%

16. A person makes payment of ₹ 35,000 to a contractor who is a transport operator is not subject to TDS if
(a) The recipient owns 10 or less than 10 goods carriages at any time during the previous year.
(b) The recipient is engaged in the business of plying, hiring or leasing of goods carriage
(c) the recipient has furnished declaration to this effect along with his PAN to the payer.
(d) All of the above

17. Submission of PAN is mandatory for


(a) Payer to Payee (c) Company only
(b) Payee to Payer (d) Individual only

18. If PAN is not submitted by assessee than TDS will deducted at


(a) the higher of rate specified in Act or 20% (c) 30%
(b) 20% (d) Maximum Marginal Rate of Tax

19. TDS shall not be deducted if payment is made to:


(a) Government (c) A mutual fund specified under section 10(23C)
(b) Reserve bank of India (d) All the above

20. Certificate for the deduction of TDS as lower rate is issued by ....... after receiving an application from …...
(a) Assessing Officer, Assessee (c) CIT, Assessing Officer
(b) Assessee, Assessing Officer (d) Assessing Officer, CIT

21. After deduction of TDS, deductor must deposit such TDS to ..................... within specified time
(a) Central Government (c) Both (a) and (b)
(b) State-Government (d) None of the above

22. When the payer is other than Government, TDS of March month shall be deposit upto:
(a) 30th April (c) Same day
(b) 7th April (d) None of the above
CA SHREY RATHI TDS & TCS 13.7

23. Which of the following are correct due dates for deductor other than government for filling TDS returns:
(a) 31st July, 31st October, 31st January, 31st May
(b) 15th July, 15th October, 15th January, 15th May
(c) 15th July, 31st October, 31st January, 15th May
(d) None of the above

24. Due date of furnishing of TDS certificate in case of salary:


(a) 31st May of the following relevant financial year
(b) 15th May of the following relevant financial year
(c) 15th June of the following relevant financial year
(d) 30th June of the following relevant financial year

25. Assessee shall be deemed to assessee in default in which of following cases:


(a) If deductor has not deduct the whole or any part of the tax
(b) If after deduction, the deductor fails to pay whole or any part of tax
(c) If after deduction, the deductor makes late payment
(d) In all of the cases mentioned above

26. When prize is given partly in cash & party in kind, Income tax is to be deducted only from cash?
(a) True (c) Partly true
(b) False (d) None of the above

27. Mr. P, a resident of age of 70 years, is due to receive ₹ 2.20 lakhs on 31/3/2023 on LIC policy taken on 1/4/2010, for
which the sum assured is ₹ 2 lakhs and the annual premium is ₹ 45,000. What will be the applicability of provisions
for tax deduction at source u/s 194DA?
(a) Tax is deductible on ₹ 2,20,000 since annual premium exceeds 10% of sum assured
(b) Tax is deductible on ₹ 2,20,000 since annual premium exceed 20% of sum assured
(c) Tax is deductible on ₹ 2,20,000 since annual premium exceeds 15% of sum assured
(d) None of the above

28. Prakash is employed with XYZ Ltd. from 05-11-2018. He resigned on 31-03-2023 and wants to withdraw the
accumulated balance of employer’s contribution in his EPF Account i.e., ₹ 55,000. The tax deducted on such
withdrawal would be: (ICAI MCQ Booklet)
(a) ₹ 500 u/s 192 (c) ₹ 500 u/s 192A
(b) ₹ 5,500 u/s 192 (d) ₹ 5,500 u/s 192A

29. Any person responsible for paying to a resident any sum exceeding ₹ 2.5 lakh towards compensation for compulsory
acquisition of his urban industrial land under any law has to deduct income-tax at the rate of
(a) 10% (c) 20%
(b) 15% (d) 2%

30. TDS from Interest on securities have been discussed under


(a) Section 192 (c) Section 192A
(b) Section 193 (d) Section 194A

31. TDS u/s 194M shall be deductible @ ………


(a) 1% (c) 5%
(b) 2% (d) 10%

32. Mr. Chaudhary has a car showroom. He purchased 10 cars worth ₹ 18,00,000 each from the manufacturer on 5th April
2022. The manufacturer in this case shall collect TCS amounting to:
(a) Nil (c) ₹ 18,000
(b) ₹ 1,80,000 (d) ₹ 13,500
CA SHREY RATHI TDS & TCS 13.8

33. Rate of TCS on timber obtained under a forest lease shall be ………
(c) 1% (c) 5%
(d) 2% (d) 2.5%

34. What is the exemption limit for TDS withdrawl u/s 194N wherein the assessee has filed return in the previous 3 years?
(a) ₹ 20,00,000 (c) ₹ 1,00,00,000
(b) ₹ 50,00,000 (d) ₹ 2,00,00,000

35. Mr. Gaurav, a businessman having turnover of 182 lakhs opted for presumptive income basis took a residential
property on rent for ₹ 60,000 p.m. from Mr. Sultan on 1st February 2023. The amount of TDS to be deductible under
the provisions of Income Tax Act shall be:
(a) Nil (c) ₹ 12,000
(b) ₹ 6,000 (d) ₹ 9,000

36. Mr. Pankaj won a lottery prize of ₹ 35,000. The winnings shall be distributed in 3 instalments of ₹ 5,000, ₹ 10,000 & ₹
20,000. The TDS to be deducted by the lottery organiser is such case shall be:
(a) ₹ 9,000 (c) ₹ 6,000
(b) ₹ 10,500 (d) ₹ 4,500

37. SS Ltd. company withdrew ₹ 1.35 crores from bank on 22nd July 2022 & ₹ 70 lakhs on 28th March 2023 in cash. SS Ltd.
has filed return of income for the preceding 3 years. TDS to be deducted by the bank u/s 194N shall be:
(a) ₹ 2,10,000 (c) ₹ 1,40,000
(b) ₹ 4,10,000 (d) ₹ 3,70,000

38. Interest on FD when granted to resident senior citizen is liable to tax @ 10% if the interest amount exceeds:
(a) ₹ 5,000 (c) ₹ 40,000
(b) ₹ 10,000 (d) ₹ 50,000

39. Mr. Farhan filed his TDS return for the 2nd quarter on 19th November 2022. TDS deposited by him during this quarter
was ₹ 4,650. Penalty for late filing of return shall be:
(a) ₹ 3,800 (c) ₹ 10,000
(b) ₹ 4,650 (d) ₹ 1,900

40. TDS certificate is issued in Form ………….. where TDS is deducted u/s 194M.
(a) 16 (c) 16B
(b) 16A (d) 16C

41. Mr. X, a resident, is due to received ₹ 4.50 lakhs on 31-03-2023, towards maturity proceeds of LIC policy taken on 1-4-
2020, for which the sum assured is ₹ 4 lakhs and the annual premium is ₹ 1,25,000. Mr. Z, a resident, is due to receive
₹ 95,000 on 1-10-2022 towards maturity proceeds of LIC policy taken on 1-10-2014 for which the sum assured is ₹
90,000 and the annual premium is ₹ 10,000. (ICAI MCQ Booklet)
(a) Tax is required to be deducted on income comprised in maturity proceeds payable to Mr. X and Mr. Z
(b) Tax is required to be deducted on income comprised in maturity proceeds payable to Mr. X
(c) Tax is required to be deducted on income comprised in maturity proceeds payable to Mr. Z
(d) No tax is required to be deducted on income comprised in maturity proceeds payable to either Mr. X or Mr. Z

42. Mr. Ramesh, Mr. Mahesh and Mr. Suresh, jointly owned a flat in Mathura, which was let out to Dr. Rajesh from 01-04-
2022. The annual rent paid by Dr. Rajesh for the flat was ₹ 5,40,000, credited equally to each of their account. Mr.
Rajesh approached his tax consultant to seek clarity in relation to deduction of tax on payment of the rent. He
informed his consultant that he occupied such flat for his personal accommodation and his receipts from his
profession during the previous year 2021-22 was ₹ 58 lakhs. As tax consultant, choose the correct answer – (ICAI MCQ
Booklet)
(a) No tax at source is required to be deducted since the rental payments are towards flat occupied for personal
purpose
(b) Tax is required to be deducted at source since the rent payment exceeds ₹ 2,40,000 and Dr. Rajesh is an individual
having gross receipts from profession exceeding ₹ 50 lakh in the preceding financial year.
CA SHREY RATHI TDS & TCS 13.9

(c) No tax is required to be deducted at source since the rent credited to each co-owner is less than ₹ 2,40,000
(d) No tax is required to be deducted at source since Dr. Rajesh’s gross receipts during the preceding financial year
were less than ₹ 1 crore

43. Mr. Nihar maintains a Savings A/c and a Current A/c in Mera Bank Ltd. The details of withdrawals on various dates
during the previous year 2022-23 are as follows:
Date of Cash withdrawal Saving Accounts Current Accounts
05-04-2022 15,00,000 -
10-05-2022 - 22,00,000
25-06-2022 20,00,000 -
17-07-2022 - 5,00,000
28-10-2022 35,00,000 -
10-11-2022 - 38,00,000
12-12-2022 25,00,000 -
Mr. Nihar regularly files his return of income. Is Mera Bank Limited required to deduct tax at source on the
withdrawals made by Mr. Nihar during the previous year 2022-23? If yes, what would the amount of tax deducted at
source? (ICAI MCQ Booklet)
(a) TDS of ₹ 4,60,000 is required to be deducted
(b) No, TDS is not required to be deducted as the cash withdrawal does not exceed ₹ 1 crore neither in saving
account nor in current account
(c) TDS of ₹ 3,00,000 is required to be deducted.
(d) TDS of ₹ 1,20,000 is required to be deducted.

44. Mr. P is a professional who is responsible for paying a sum of ₹ 2,00,000 as rent for use of building to Mr. Harshit, a
resident, for the month of February, 2022. The gross receipts of Mr. P are as under:
From 01-04-2021 to 31-03-2022: ₹ 55,00,000
From 01-04-2022 to 28-02-2023: ₹ 45,00,000
Whether Mr. P is responsible for deducting any tax at source from the rent of ₹ 2,00,000 payable to Mr. Harshit? (ICAI
MCQ Booklet)
(a) Tax at source is required to be deducted u/s 194-I at the rate of 10%
(b) Tax at source is required to be deducted u/s 194-IB at the rate of 5%
(c) Tax at source is required to be deducted u/s 194-IB at the rate of 10%
(d) No tax is required to be deducted at source.

45. Mr. A has two bank accounts maintained with ICICI Bank and HDFC Bank. From 01-04-2022 till 31-03-2023, Mr. A
withdrew the following amounts as cash from both the said accounts;
HDFC Bank - ₹ 50 Lakh
ICICI Bank - ₹ 120 Lakh
What shall be the amount of tax to be deducted at source u/s 194N by HDFC Bank and ICICI Bank, respectively, while
making payment in cash to Mr. A assuming Mr. A has filed his return of income for P.Y. 2019-20, P.Y. 2020-21 and P.Y.
2021-22 respectively? (ICAI MCQ Booklet)
(a) ₹ 1,00,000 and ₹ 2,40,000 (c) ₹ 60,000 and ₹ 1,00,000
(b) Nil and ₹ 40,000 (d) ₹ 50,000 and ₹ 1,20,000

46. Mr. Ram acquired a house property at Chennai from Mr. Satyam, a resident, for a consideration of ₹ 85 lakhs, on 23-
08-2022. On the same day, Mr. Ram made two separate transactions, thereby acquiring an urban plot in Gwalior from
Mr. Vipun, a resident, for a sum of ₹ 50 lakhs and rural agricultural land from Mr. Danish, a resident, for a
consideration of ₹ 75 lakhs. Which of the following statements are correct assuming that in the consideration
amounts as aforementioned all the charges incidental to transfer of the immovable property are included? (ICAI MCQ
Booklet)
(a) No tax deduction at source is required in respect of any of the three payments.
(b) TDS @ 1% is attracted on all the three payments.
(c) TDS @ 1% on ₹ 85 lakhs and ₹ 50 lakhs are attracted. No TDS on payment of ₹ 75 lakhs for acquisition of rural
agricultural land
(d) TDS @ 1% on ₹ 85 lakhs is attracted. No TDS on payments of ₹ 50 lakhs and ₹ 75 lakhs.
CA SHREY RATHI TDS & TCS 13.10

47. Mr. X, a resident Indian, wins ₹ 10,000 in a lottery. Which of the statement is true? (ICAI MCQ Booklet)
(a) Tax is deductible u/s 194B @ 30% (c) No tax is deductible at source
(b) Tax is deductible u/s 194B @ 30.9% (d) None of the above

48. Which of the following details/evidences are required to be furnished by an employee to his/her employer in respect
of deduction of interest under the head “Income from house property”, when the employer is estimating the total
income of the employee for the purpose of tax deduction at source u/s 192?
(i) Amount of Interest payable or paid
(ii) Rate of interest payable or paid
(iii) Name of the lender
(iv) Address of the lender
(v) PAN or Aadhar number as the case may be, of the lender
(vi) TAN of the lender
Choose the correct answer (ICAI MCQ Booklet)
(a) (i), (iii), (v) (c) (ii), (iv), (v) (vi)
(b) (i), (iii), (iv), (v) (d) (i), (ii)

49. A firm pays salary and interest on capital to its resident partners. The salary and interest paid fall within the limits
specified in section 40(b). Which of the following statements is true? (ICAI MCQ Booklet)
(a) tax has to be deducted u/s 192 on salary and u/s 194A on interest
(b) tax has to be deducted u/s 192 on salary but no tax needs to be deducted on interest
(c) no tax has to be deducted on salary but tax has to be deducted u/s 194A on interest
(d) no tax has to be deducted at source on either salary or interest

50. Dr. Sargun maintained two bank A/cs, one current A/c with Canara Bank for her profession and a saving bank A/c with
State Bank of India. The following are the details of her withdrawals from these A/cs during the previous year 2022-
23:
Date of withdrawals Canara Bank State Bank of India
25-04-2022 25,00,000
27-04-2022 15,50,000
31-08-2022 29,00,000
01-09-2022 14,20,000
05-09-2022 14,00,000
07-10-2022 18,21,000
11-12-2022 26,23,000
12-02-2023 7,56,000
25-03-2023 16,13,000
She furnished her return of income for the A.Y. 2022-23 and A.Y. 2021-22 on or before the limit prescribed u/s 139(1).
However, for the A.Y. 2020-21 and A.Y. 2019-20, she has furnished her return of income belatedly.
Is any tax deductible at source u/s 194N on the withdrawals made by Dr. Sargun from Canara Bank and SBI Bank? If
yes, at what rate and what amount? (ICAI MCQ Booklet)
(a) TDS is deductible at source on ₹ 1,00,20,000 @ 5% by Canara Bank and tax is deductible at source @ 2% on ₹
25,63,000 by SBI.
(b) TDS is deductible at source on ₹ 20,20,000 @ 5% by Canara Bank and no tax is deductible by SBI.
(c) TDS is deductible at source on ₹ 20,20,000 @ 2% by Canara Bank and no tax is deductible by SBI.
(d) TDS is deductible at source on ₹ 1,00,20,000 @ 5% by Canara Bank and tax is deductible at source @ 5% on ₹
25,63,000 by SBI.

51. Mr. T an Indian Citizen and resident of India, earned dividend income of ₹ 4,500 from an Indian company, which was
declared on 1-10-2022 and paid in cash to Mr. T. what are the tax implications with respect to the dividend in the
hands of Mr. T and Indian Company? (ICAI MCQ Booklet)
(a) Such dividend is taxable in the hands of Mr. T and Indian company is required to deduct tax at source @ 5%.
(b) Such dividend is taxable in the hands of Mr. T and Indian company is required to deduct tax at source @ 10%.
(c) Such dividend is taxable in the hands of Mr. T. However, Indian company is not required to deduct tax at source
since it does not exceed ₹ 5,000.
(d) Such dividend is exempt in the hands of Mr. T. Hence, Indian company is not required to deduct tax at source.
CA SHREY RATHI TDS & TCS 13.11

52. TPR & Co., a partnership firm selling its product X through the digital facility provided by MKY Limited (an E-commerce
operator). MKY Limited has credited in its books of account, the account of TPR & Co. on 31st January, 2022 by sum of
₹ 4,80,000 for the sale of product X made during the month of January 2022. Out of ₹ 4,80,000, it made payment for ₹
4,00,300 on 3rd February, 2022. Further, Mr. Pawan, who purchased the product X through the facility provided by
MKY Limited, has made the payment of sum of ₹ 40,000 directly to TPR & Co. on 15th January, 2022. Which statement
is correct regarding requirement of deduction of tax at source by MKY Limited? (ICAI MCQ Booklet)
(a) No tax is required to be deducted at source.
(b) MKY Limited is required to deduct tax at source ₹ 4,800 under section 194C.
(c) MKY Limited is required to deduct tax at source ₹ 3,900 under section 194O.
(d) MKY Limited is required to deduct tax at source ₹ 5,200 under section 194O.

53. Mr. Vyas, aged 80, is a retired government employee. On 1st April 2022, he received the maturity amount of his LIC
policy amounting to Rs. 3,50,000. This policy was taken by Mr. Vyas on 1st April 2014 on which the sum assured was ₹
3,00,000 and the annual premium was ₹ 40,000. His other income comprised of pension amounting to ₹ 85,000. Mr.
Vyas Furnishes a declaration in Form 15H for non-deduction of tax at source to the insurance company stating that his
net tax liability for the year is NIL. Choose the correct statement from below: (ICAI MCQ Booklet)
(a) The declaration made by Mr. Vyas is wrong and the insurance company has to deduct tax of ₹ 3,500 under
section 194DA.
(b) The claim by Mr. Vyas is right and insurance company is not required to deduct tax at source.
(c) The insurance company has to deduct tax under section 194DA since declaration in Form 15H cannot be made
for tax deduction under section 194DA.
(d) The declaration made by Mr. Vyas is wrong and the insurance company has to deduct tax of ₹ 1,000 under
section 194DA.

MCQ’s Answers
1. (b) 2. (c) 3. (d) 4. (d) 5. (d) 6. (a) 7. (b) 8. (b) 9. (d) 10. (c)
11. (a) 12. (c) 13. (c) 14. (d) 15. (b) 16. (d) 17. (b) 18. (a) 19. (d) 20. (a)
21. (a) 22. (a) 23. (a) 24. (c) 25. (d) 26. (b) 27. (b) 28. (d) 29. (a) 30. (b)
31. (c) 32. (a) 33. (d) 34. (c) 35. (c) 36. (b) 37. (a) 38. (d) 39. (a) 40. (b)
41. (b) 42. (c) 43. (d) 44. (b) 45. (b) 46. (d) 47. (c) 48. (b) 49. (d) 50. (c)
51. (b) 52. (d) 53. (b)
CA SHREY RATHI ADVANCE PAYMENT OF TAX 14.1

BARACK OBAMA

CHAPTER 14
ADVANCE TAX

"We did not come to fear the


future, we came here to shape it”

PRACTICAL QUESTIONS
Q 1: The estimated total income of KLM Ltd. is ₹ 2,00,000. The company earned ₹ 1,00,000 as LTCG on sale of property
earned on 19-10-2022. Compute the advance tax payable in instalments by the company assuming ₹ 11,000 has been
deducted at source during the P/Y 2022-23. Assume rate of tax to be 30%.

Sol 1: Computation of different instalments of advance tax payable:


KLM Ltd. estimated ₹ 2,00,000 as its total income in the beginning of the year as estimation of LTCG of ₹ 1,00,000 on 19-
10-2022 is practically not feasible in the beginning of the year. It can only be dealt when such capital gain actually arises.
Tax on ₹ 2,00,000 @ 30% = ₹ 60,000
Add: Health & Educational Cess @ 4% = ₹ 2,400
= ₹ 62,400
Less: TDS = ₹ 11,000
Tax liability = ₹ 51,400
1st instalment on 15th June, 2022 @ 15% of tax liability (₹ 51,400 x 15%) = ₹ 7,710
2nd instalment on 15th September, 2022 @ 45% of tax liability (₹ 51,400 x 45%) = ₹ 23,130
Revised tax liability ability after happening of LTCG of ₹ 1,00,000 on 19-10-2022:
Tax on ₹ 2,00,000 @ 30% = ₹ 60,000
Tax on ₹ 1,00,000 @ 20% = ₹ 20,000
= ₹ 80,000
Add: Health & Education cess @ 4% = ₹ 3,200
= ₹ 83,200
Less: TDS = ₹ 11,000
Tax liability = ₹ 72,200
3rd instalment on 15th December 2022 @ 75% of revised tax liability (₹ 72,200 x 75%) = ₹ 54,150.
4th instalment on 15 March 2023 @ 100% tax liability (₹ 72,200 x 100%) = ₹ 72,200.

Q 2: DLF Ltd. estimated its tax payable to be ₹ 5,00,000 for the P/Y 2022-23 and has paid advance tax accordingly but
actual tax liability of the company was found to be ₹ 5,50,000 and the difference of tax amount was paid on 17-12-2023.
Compute interest u/s 234B & 234C.
CA SHREY RATHI ADVANCE PAYMENT OF TAX 14.2

Sol 2: Computation of interest u/s 234B & 234C:


Particulars 15th June 15thA Sept. 15th Dec. 15th March
(15%) (₹) (45%) (₹) (75%) (₹) (100%) (₹)
Actual tax liability – ₹ 5,50,000 82,500 2,47,500 4,12,500 5,50,000
Tax paid – ₹ 5,00,000 75,000 2,25,000 3,75,000 5,00,000
Deficiency 7,500 22,500 37,500 50,000
Interest rate 3% 3% 3% 1%
Interest u/s 234C Nil Nil 1,125 500
Total interest u/s 234C 1,625
Note: Interest for 1st
Installment (15%) due on 15th
June & 2nd
Installment (45%) due on 15% September shall not be
charged as DF Ltd. has paid more than 12% of actual tax liability in case of 1st Installment and more than 36% in case of 2nd
Installment.
Interest u/s 234B becomes applicable when the assessee has paid less than 90% of the assessed tax. Since DLF Ltd. has
paid more than 90% of the assessed tax, interest u/s 234B shall not apply.

Q 3: Ms. Ankita provides you the following information:


1. Income under the head house property - ₹ 15,00,000
2. Gift of painting from a friend with market value - ₹ 2,00,000
3. Gift of shares and securities from her husband - ₹ 4,00,000
4. Agricultural income - ₹ 3,00,000
She has paid advance tax as given below:
Upto 15th September, 2022 - ₹ 30,000
Upto 15th December, 2022 - ₹ 50,000
Upto 15th March, 2023 - ₹ 60,000
Balance amount of tax was paid on 10th October 2023.
Compute her advance tax liability for the A/Y 2023-24 and also interest u/s 234B and 234C.

Sol 3: Computation of advance tax liability for the A/Y 2023-24 & interest u/s 234B & 234C
Particulars (₹) (₹)
Income from house property 15,00,000
Income from other sources:
o Gift from a friend 2,00,000
o Gift of shares and securities from her husband (gift from relative is exempt) - 2,00,000
Total Income 17,00,000
Though agricultural income of ₹ 3,00,000 is exempt from tax by virtue of Section 10(1); but it shall be considered for rate
purposes.
Particulars (₹)
Step 1: Compute tax on total income (i.e. non-agricultural income + agricultural income) = [₹ 17,00,000 + 4,12,500
₹ 3,00,000 = ₹ 20,00,000]
Step 2: Compute tax on agricultural income + exemption limit as if it is the total income = [₹ 3,00,000 + ₹
2,50,000 = ₹ 5,50,000] 22,500
Step 3: Step 1 – Step 2 is taxable 3,90,000
Step 4: Health & Education Cess @ 4% 15,600
Total tax liability 4,05,600

Computation of interest u/s 234B & 234C:


Interest u/s 234B: (₹ 4,05,600 – ₹ 60,000) x 1% x 7 = ₹ 24,192.
CA SHREY RATHI ADVANCE PAYMENT OF TAX 14.3

Interest u/s 234C:


Particulars 15th June 15th Sept. 15th Dec. 15th March
(15%) (₹) (45%) (₹) (75%) (₹) (100%) (₹)
Actual tax liability - ₹ 4,05,600 60,840 1,82,520 3,04,200 4,05,600
Tax Paid - 30,000 50,000 60,000
Deficiency 60,840 1,52,520 2,54,200 3,45,600
Deficiency rounded off to multiples of ₹ 100 for 60,800 1,52,500 2,54,200 3,45,600
interest calculations
Interest rate 3% 3% 3% 1%
Interest u/s 234C 1,824 4,575 7,626 3,456
Total Interest u/s 234C ₹ 17,481

Q 4: Compute the advance tax payable by Mr. Raju from the following estimated income for the P/Y 2022-23:
(₹)
1) Income from salary 6,60,000
2) Rent from house property (per month) 30,000
3) Interest on Government securities 18,000
4) Agricultural income 70,000
5) Contribution towards PPF 60,000
6) Tax deducted at source by the employer 37,910

Sol 4: Computation of advance tax payable by Mr. Raju for the P/Y 2022-23:
Particulars (₹) (₹)
Income from salary 6,60,000
Rent from house property (₹ 30,000 x 12) = Net Annual Value 3,60,000
Less: Statuary deduction @ 30% 1,08,000 2,52,000
Income from other sources: Interest on Govt. securities 18,000
Gross Total Income 9,30,000
Less: Deduction u/s 80C: Contribution towards PPF 60,000
Total Income 8,70,000
o Though agricultural income of ₹ 70,000 is exempt from tax by virtue of section 10(1); but it shall be considered for
rate purposes.
Particulars (₹)
Step 1: Compute tax on total income (i.e. non-agricultural income + agricultural income) = [₹ 8,70,00 + ₹ 1,00,500
70,000 = ₹ 9,40,000]
Step 2: Compute tax on agricultural income + exemption limit as if it is the total income = [₹ 70,000 + ₹ 3,500
2,50,000 = ₹ 3,20,000]
Step 3: Step 1 – Step 2 is tax payable 97,000
Step 4: Health & Education Cess @ 4% 3,880
Total Tax Liability 1,00,880
Less: Tax deducted at source 37,910
Final tax liability (Advance tax payable) 62,970

Q 5: A partnership firm made the following payments of advance tax during the financial year 2022-23:
1. Upto 15th September, 2022 - ₹ 8,25,000
2. Upto 15 December, 2022 - ₹ 16,64,000
3. Upto 15th March, 2023 - ₹ 26,23,000
Return of income filed by the firm is ₹ 88,00,000 u/h PGBP and ₹ 9,50,000 by way of LTCG on sale of a property effected
on 18-12-2022. Final payment was made on 18-12-2023.
Calculate the interest payable by the assessee u/s 234B and 234C.
CA SHREY RATHI ADVANCE PAYMENT OF TAX 14.4

Sol 5: Computation of interest u/s 234B & 234C:


The partnership firm estimated ₹ 88,00,000 as its total income in the beginning of the year as estimation of LTCG of ₹
9,50,000 on 10-10-2022 is practically not feasible in the beginning of the year. It can only be dealt when such capital gain
actually arises.
Tax on ₹ 88,00,000 @ 30% = ₹ 26,40,000
Add: Health & Education cess @ 4% = ₹ 1,05,600
Tax liability = ₹ 27,45,600
1st Installment on 15th June 2022 @ 15% of tax liability (₹ 27,45,600 x 15%) = ₹ 4,11,840
2nd Installment on 15th September 2022 @ 45 of tax liability (₹ 27,45,600 x 45%) = ₹ 12,35,520
3rd Installment on 15th December 2022 @ 75% of tax liability (₹ 27,45,600 x 75%) = ₹ 20,59,200
Revised tax liability after happening of LTCG of ₹ 9,50,000 on 18-12-2022:
Tax on ₹ 88,00,000 @ 30% = ₹ 26,40,000
Tax on ₹ 9,50,000 @ 20% = ₹ 1,90,000
= ₹ 28,30,000
Add: Health & Education cess @ 4% = ₹ 1,13,200
Tax liability = ₹ 29,43,200
4th Installment on 15th March 2023 @ 100% of tax liability (₹ 29,43,200 x 100%) = ₹ 29,43,200
Computation of interest u/s 234B & 234C:
Interest u/s 234B: (₹ 29,43,200 – ₹ 26,23,000) x 1% x 9 = ₹ 28,818.
Interest u/s 234C:
Particulars 15th June 15th Sept. 15th Dec. 15th March
(15%) (₹) (45%) (₹) (75%) (₹) (100%) (₹)
Actual tax liability 4,11,840 12,35,520 20,59,200 29,43,200
Tax paid - 8,25,000 16,64,000 26,23,000
Deficiency 4,11,840 4,10,520 3,95,200 3,20,200
Deficiency rounded off to multiples of ₹ 100 for 4,11,800 4,10,500 3,95,200 3,20,200
interest calculations
Interest rate 3% 3% 3% 1%
Interest u/s 234C 12,354 12,315 11,856 3,202
Total Interest u/s 234C ₹ 39,637

MULTIPLE CHOICE QUESTIONS


1. Advance tax is payable in ..................... installments by a non-corporate assessee
(a) 3 (c) 4
(b) 2 (d) 1

2. Obligation to pay advance tax arises when tax payable is .....................


(a) ₹ 10,000 or more (c) ₹ 10,000
(b) ₹ 10,000 or less (d) ₹ 5,000 or more

3. Advance tax shall not be payable by an individual resident having income like interest, rent, etc. in India, who
(a) Does not have any income chargeable under the head of Profits and gains of business or profession
(b) Is of the age of sixty years or more at any time during the previous year
(c) Both of (a) and (b)
(d) None of (a) or (b)

4. How shall advance tax be computed?


(a) Tax on total income + Tax deducted at source (c) Tax on total income x Tax deducted at source
(b) Tax on total income - Tax deducted at source (d) Tax on total income / Tax deducted at source
CA SHREY RATHI ADVANCE PAYMENT OF TAX 14.5

5. Where no instalment is due and any income is earned after 15th March of the P/Y, the entire tax should be paid by
.....................
(a) 15th March (c) 31st July
st
(b) 31 March (d) 15th September

6. What shall be payable by assessee for deferment of advance tax beyond due dates given under section 211.
(a) Interest under section 234A (c) Interest under section 234C
(b) Interest under section 234B (d) None of the above

7. For the last instalment of 15th March PY, the interest liability u/s 234C would be .............. for ………….
(a) 1%, 3 months (c) 1.5%, 3 months
(b) 1%, 1 month (d) 2%, 3 months

8. In case of assessee being company, if advance tax payable in second instalment is ....................., no interest under
section 234C shall be payable
(a) 36% or less (c) 12% or more
(b) 36% or more (d) 12% or less

9. Mr. Sohan estimated his tax payable for PY 2022-23 at ₹ 1,00,000. What is the amount of advance tax payable upto
15th September 2022?
(a) ₹ 45,000 (c) ₹ 15,000
(b) ₹ 75,000 (d) ₹ 1,00,000

10. For the period of PY 2022-23, XYZ Ltd. has estimated its tax payable to be ₹ 2,00,000. What is the amount of advance
tax payable upto 15th December 2022?
(a) ₹ 90,000 (c) ₹ 1,50,000
(b) ₹ 30,000 (d) ₹ 2,00,000

11. For the PY 2022-23, GC Ltd has not paid any advance tax till 10/2/2023 and in the earlier years the company was
assessed in the manner given below:
2019-2020 Scrutiny Assessment done for ₹ 7,00,000
2020-2021 Best Judgment Assessment done for ₹ 10,00,000
2021-2022 Income Tax Return submitted for ₹ 8,00,000
Assessing Officer shall use which of the income as base for computation of advance tax:
(a) ₹ 8,00,000 (c) ₹ 7,00,000
(b) ₹ 10,00,000 (d) None of the above

12. Interest u/s 234B is calculate on the amount of difference between the ..................... and .....................
(a) Advance tax paid and assessed tax (c) Assessed tax and advance tax paid
(b) Advance tax paid and 90% of the assessed tax (d) None of the above

13. The scheme of advance tax is commonly known as


(a) A step towards national building (c) Earn more pay more
(b) Pay as much as you can (d) Pay as you earn

14. The tax liability of assessee is ₹ 11,000. This sum is deposited on 16th March in lump sum. This will be treated as
(a) Self assessment tax (c) Advance tax with no interest
(b) Advance tax but subject to interest (d) None of the above

15. It has been provided that if casual income or capital gain arises ..................... of any installment, then, the entire
amount of tax payable on such income should be paid in the ..................... of advance tax which are due
(a) After due date, last instalment (c) After due date, remaining installment
(b) Before due date, last installment (d) before due date, remaining installment
CA SHREY RATHI ADVANCE PAYMENT OF TAX 14.6

16. A transport operator has opted for presumptive taxation scheme under section 44AE for PY 2022-2023. He is liable to
pay advance tax -
(a) In one installment (c) In three installments
(b) In two installments (d) In four installments

17. Mr. A whose total sales is ₹ 201 lakhs, declares profit of ₹ 10 lakhs for the F.Y. 2022-23. He is liable to pay advance tax
in: (ICAI MCQ Booklet)
(a) 1 instalment (c) 3 instalments
(b) 2 instalments (d) 4 instalments

18. Compute the advance tax liability of Mr. Lahoti from the following information:
1. Income from salary – ₹ 4,50,000
2. Estimated income from FD – ₹ 30,000
3. Long-term capital gains – ₹ 20,000
4. Deposit in PPF – ₹ 10,000
5. TDS – ₹ 3,000
(a) Nil (c) ₹ 14,160
(b) ₹ 2,200 (d) ₹ 13,500

19. Where Mr. Raj (a non-resident and aged 65 years) is a retired person, earning rental income of ₹ 40,000 per month
from a property located in Delhi. He is residing in Canada. Apart from rental income, he does not have any other
source of income. Is he liable to pay advance tax in India? (ICAI MCQ Booklet)
(a) yes, he is liable to pay advance tax in India as he is a non-resident and his tax liability in India exceeds ₹ 10,000.
(b) No, he is not liable to pay advance tax in India as his tax liability in India is less than ₹ 10,000.
(c) No, he is not liable to pay advance tax in India as he has no income chargeable under the head “Profit and gains of
business or profession” and he is of the age of 65 years.
(d) Both (b) and (c)

20. For the purpose of computation of interest on advance tax, the principal amount shall be rounded off:
(a) To the nearest multiple of ₹ 100 (c) To the nearest multiples of ₹ 10
(b) To the multiples of ₹ 100 (d) No rounding off is required.

21. Mr. Jha, an employee of FX Ltd, attained 60 years of age on 15-05-2022. He is resident in India during F.Y. 2022-23 and
earned salary income of ₹ 5 lakhs (computed). During the year, he earned ₹ 7 lakhs from winning of lotteries. What
shall be his advance tax liability for A.Y. 2023-24? Assume he does not opt to pay tax under section 115BAC. (ICAI
MCQ Booklet)
(a) ₹ 2,20,000 + cess ₹ 8,800 = ₹ 2,28,800, being the tax payable on total income of ₹ 12 lakhs
(b) ₹ 2,10,000 + cess ₹ 8,400 = ₹ 2,18,800, being the tax payable on lottery income of ₹ 7 lakhs
(c) ₹ 10,000 + cess ₹ 400 = ₹ 10,400, being the net tax payable on salary income since tax would have been deducted
at source from lottery income.
(d) Nil

22. An interior decorator has opted for presumptive taxation scheme under section 44ADA for A.Y. 2023-24: (ICAI MCQ
Booklet)
(a) He is liable to pay advance tax on or before 15-03-2023
(b) He is not liable to advance tax
(c) He is liable to pay advance tax in three instalments i.e., on or before 15-09-2022, 15-12-2022 and 15-03-2023
(d) He is liable to pay advance tax in four instalments i.e., on or before 15-06-2022, 15-09-2022, 15-12-2022 and 15-
03-2023

MCQ ANSWERS
1. (c) 2. (a) 3. (c) 4. (b) 5. (b) 6. (c) 7. (b) 8. (b) 9. (a) 10. (c)
11. (b) 12. (c) 13. (d) 14. (b) 15. (c) 16. (d) 17. (d) 18. (a) 19. (b) 20. (b)
21. (d) 22. (a)
CA SHREY RATHI RETURN OF INCOME 15.1

NELSON MANDELA

CHAPTER 15
RETURN OF INCOME

"Do not judge me by my successes,


judge me by how many times I fell
down and got back up again”

THEORITICAL QUESTIONS
Q 1: Vineet submits his return of income on 12-09-2023 for A.Y. 2023-24 consisting of income under the head salaries,
“income from house property” and bank interest. On 21-12-2023, he realized that he had not claimed deduction under
section 80TTA in respect of his interest income on the savings Bank Account. He wants to revise his return on income. Can
he do so? Examine.
Would your answer be different if he discovered this omission on 21-03-2024? [ICAI Study Module]

Sol 1: Since Mr. Vineet has income only under the heads “Salaries”, “Income from house property” and “Income from
other sources”, he does not fall under the category of a person whose accounts are required to be audited under the
Income-tax Act, 1961 or any other law in force. Therefore, the due date of filing return for A/Y 2023-24 under section
139(1), in his case, is 31st July, 2023. Since Mr. Vineet had submitted his return only on 12.9.2022, the said return is a
belated return under section 139(4).
As per section 139(5), a return furnished under section 139(1) or a belated return u/s 139(4) can be revised. Thus, a
belated return under section 139(4) can also be revised. Therefore, Mr. Vineet can revise the return of income filed by him
under section 139(4) in December 2023, to claim deduction under section 80TTA, since the time limit for filing a revised
return is upto 31st December of the relevant assessment year, which is 31.12.2023.
However, he cannot revise return had he discovered this omission only on 21-03-2024, since it is beyond 31.12.2023,
being the last date of revising the return.

Q 2: State with reasons whether you agree or disagree with the following statements:
(a) Return of income of Limited Liability Partnership (LLP) could be verified by any partner.
(b) Time limit for filing return u/s 139(1) in the case of Mr. A having total turnover of ₹ 160 lakhs for the year ended
31.03.2023, whether or not opting to offer presumptive income u/s 44AD, is 31st October 2023.
(c) Where the Karta of HUF is absent from India, the return of income can be verified by any male member of the family.
[ICAI Study Module]
Sol 2:
(a) Disagree: The return of income of LLP should be verified by a designate partner. Any other partner can verify the
Return of Income of LLP only in the following cases:
(i) Where for any unavoidable reason such designated partner is not able to verify the return, or,
(ii) Where there is no designated partner.
CA SHREY RATHI RETURN OF INCOME 15.2

(b) Disagree: In case Mr. A opts to offer his income as per the presumptive taxation provisions of section 44AD, then, the
due date u/s 139(1) for filing of return of income for the year ended 31.03.2023, shall be 31st July, 2023.
In case Mr. A does not opt for presumptive taxation provisions under section 44AD and, has to get his accounts
audited under section 44AB, since his turnover exceeds ₹ 1 crore, the due date for filing return would be 31st October,
2023.

(c) Disagree: Section 140(b) provides that where the Karta of a HUF is absent from India, the return of income can be
verified by any other adult member of the family; such member can be a male or female member.

MULTIPLE CHOICE QUESTIONS


1. Where the Karta of Hindu undivided family is absent from India, the return of income can be verified by any male
member of the family.
(a) True
(b) False

2. Belated return can be filed by an assessee before the completion of assessment or before completion of
(a) Previous year (c) 31st December of the relevant assessment year
(b) Assessment year (d) 1 year from the end of the previous year

3. In case of a company, its income tax return can be signed by


(a) Managing director (c) Any manager
(b) any member (d) Chief executive officer

4. Due date of furnishing return of income for a working partner of a firm whose accounts are required to be audited is:
(a) 31st July of the assessment year (c) 30th November of the assessment year
th
(b) 30 September of the assessment year (d) 31st October of the assessment year

5. Mr. JP dies on 15/11/2022 and his total income till this date was ₹ 2,55,000. Thereafter the business of Mr. JP was
inherited by his son Mr. SP & his total income from such business was ₹ 2,88,000. The son does not have any other
income. In this case the son:
(a) has to file a consolidated income tax return for the amount of ₹ 5,43,000
(b) has to file separate income tax returns, one on behalf of his father for ₹ 2,55,000 & other in his own capacity for
₹ 2,88,000
(c) has to file only one income tax return on behalf of his father for ₹ 2,55,000
(d) has to file only one income tax return on behalf of his father for ₹ 5,43,000

6. As per section 139(1), a company shall have to file income tax return:
(a) When its total income does not exceed the maximum amount which is not chargeable to income-tax
(b) When its total income exceeds the maximum amount which is not chargeable to income-tax
(c) When it has suffered losses
(d) In all cases irrespective of any income or loss earned by it.

7. The last date of filing income tax return u/s 139(1) for PY 2022-2023 in case of non-corporate assesses whose
accounts are not liable to be audited shall be:
(a) 30th September of the previous year (c) 31st July of the assessment year
st
(b) 31 October of the assessment year (d) 31st July of the previous year

8. The filling of return of loss in case of a person other than a company or firm is:
(a) Mandatory
(b) Mandatory if the assessee has to carry forward the loss
(c) Not mandatory
(d) None of the above
CA SHREY RATHI RETURN OF INCOME 15.3

9. If there is a loss under the house property, it will be allowed to be carried forward. In this case, however the assessee:
(a) has to submit the return of loss before the due date mentioned under section 139(1)
(b) need not submit the return of income
(c) must submit the return of income but it can be a belated return
(d) none of the above

10. Belated return u/s 139(4) can be filed at any time:


(a) before the expiry of the relevant assessment year
(b) before the expiry of one year from the end of the relevant assessment year
(c) before the expiry of one year from the end of the relevant assessment year or before the assessment is complete,
whichever happens to be earlier
(d) before the expiry of relevant assessment year or before the assessment is complete, whichever happens to be
earlier

11. For the PY 2022-2023, assessee has suffered a business loss of ₹ 2,50,000. His income from house property is ₹
1,80,000. His due date of return was 31/7/2023 but he submitted ITR on 9/9/2023, the assessee in this case:
(a) Shall be allowed to carry forward the loss of ₹ 70,000
(b) Shall not allowed to carry forward any loss
(c) Shall be allowed to set off current year business loss to the extent of ₹ 1,80,000 but shall not be allowed to carry
forward the balance loss of ₹ 70,000
(d) None of the above

12. For the PY 2022-2023, the business loss of the assessee was ₹ 1,00,000 and the current year depreciation was ₹
1,40,000. The assessee furnished the income tax return on 15/12/2023 although the due date was 31/10/2022. In this
the assessee shall:
(a) be allowed to carry forward business loss of ₹ 1,00,000 and unabsorbed depreciation of ₹ 1,40,000.
(b) neither be allowed to carry forward business loss nor the unabsorbed depreciation
(c) not be allowed to carry forward business loss but shall be allowed to carry forward unabsorbed depreciation
(d) be allowed to carry forward business loss but shall not be allowed to carry forward unabsorbed depreciation

13. LP Ltd. submitted the income tax return for the PY 2022-23 on 5/12/2023 finds some mistake in the return submitted
by it. In this case LP Ltd. can revise the income tax return till
(a) 31/3/2023 (c) 31/12/2023
(b) 31/3/2024 (d) cannot revise the ITR

14. The assesses filed his return of income in requisite form without making the payment of tax. The ITR so filed is
(a) valid (c) defective
(b) not valid (d) None of the above

15. For AY 2020-2021, Mr. KJ incurred a loss of ₹ 40,000 under the head PGBP and filed return of loss within due date. He
again incurred loss of ₹ 50,000 during AY 2021-2022 but for this year he did not file return. In AY 2023-2024 i.e. PY
2022-2023 he earned income of ₹ 5,00,000. How much loss can Mr. KJ carry forward and set off in this year?
(a) ₹ 90,000 (c) ₹ 40,000
(b) ₹ 50,000 (d) Nil

16. Mr. PJ has total income of ₹ 14,90,000 for the P/Y 2022-2023. If a return is filed after the due date u/s 139(1) but
before 31/12/2023 then the late fees of ..................... shall be imposed under section 234F
(a) ₹ 2,000 (c) ₹ 5,000
(b) ₹ 10,000 (d) ₹ 1,000

17. Revised return substitutes ..................... and shall be deemed to be have been filed on the date on which original
return was filed
(a) Original return (c) Original return or revised return filed earlier
(b) Revised return filed earlier (d) None of the above
CA SHREY RATHI RETURN OF INCOME 15.4

18. If the AO considers that return is defective, he may intimate the defect to the assessee and give him .....................
(a) Opportunity to rectify the defect within 20 days
(b) Opportunity to rectify the defect within 15 days
(c) No opportunity shall be given
(d) Opportunity to rectify the defect within 30 days

19. During the PY 2022-2023, Mr. DJ has income under the head house property ₹ 107 Lakhs. In this case, his last date of
filing of return shall be .....................
(a) 31st July, 2023 (c) 30th September, 2023
st
(b) 31 July 2023 (d) 31st October, 2023

20. For filing returns of income in respect of various entities, Income-tax Act, 1961 has prescribed
(a) Two due dates (c) Four due dates
(b) Three due dates (d) Only one due date

21. One is required to obtain PAN if total sales turnover or gross receipts are or is likely to exceed ..................... in any
previous year?
(a) ₹ 5,00,000 (c) ₹ 10,00,000
(b) ₹ 1,00,000 (d) None of the above

22. Mr. Z, a salaried individual has a total income of ₹ 8 lakhs for AY 2023-24. He files his return of income for PY 2022-23
on 28th August, 2023. He is liable to pay late fee of (ICAI MCQ Booklet)
(a) ₹ 1,000 under section 234F (c) ₹ 10,000 under section 234F
(b) ₹ 5,000 under section 234F (d) Not liable to pay any fee

23. Quoting of Aadhar number is mandatorily required in filing of application for allotment of PAN from
(a) 1st April 2018 (c) 1st October 2017
st
(b) 1 December 2017 (d) 1st July 2017

24. The provisions relating to Tax Return Preparer have been discusses under
(a) Section 139A (c) Section 139B
(b) Section 139C (d) Section 139D

25. Mr. Kumar having income from house property only filed his return of income on 22nd November 2023. His tax
liability of ₹ 35,000 was paid a week before filing of return. Compute interest, if any u/s 234A.
(a) ₹ 1,400 (c) ₹ 700
(b) Nil (d) ₹ 1,225

26. Which of the following persons have been exempted to quote their Aadhar number in filing of application form for
allotment of PAN?
(a) Non-resident of India (c) Foreign citizen
(b) Person of the age of 80 years or more (d) All of the above

27. Permanent Account Number is a ………………. alphanumeric number.


(a) 8 digits (c) 9 digits
(b) 10 digits (d) 11 digits

28. Quotation of PAN is mandatory in case of payment in cash in connection with foreign travel or purchase of any foreign
currency where such payment exceeds
(a) ₹ 50,000 (c) ₹ 1,00,000
(b) ₹ 2,00,000 (d) ₹ 2,50,000
CA SHREY RATHI RETURN OF INCOME 15.5

29. Which of the following persons are not eligible to act as Tax Return Preparer?
(a) Any officer of a scheduled bank with which the assessee maintain a current account
(b) Any legal practitioner who is entitled to practice in any civil court in India
(c) A Chartered Accountant
(d) All of the above

30. Where the amount paid by the assessee is less than the aggregate of tax, interest & fees, then the amount so paid
shall first be adjusted against
(a) Penalty/Fees (c) Interest
(b) Income Tax (d) None of the above

31. Mr. X is a working partner and Mr. Y is a non-working partner of XYZ partnership firm. XYZ Partnership firm subjected
to tax audit under section 44AB for the P.Y. 2022-23. What is the due date for filing return of income for Mr. X and Mr.
Y for the A.Y. 2023-24? (ICAI MCQ Booklet)
(a) 31st July, 2023 for both Mr. X and Mr. Y
(b) 31st October, 2023 for both Mr. X and Mr. Y
(c) 31st July, 2023 for Mr. X and 31st October, 2023 for Mr. Y
(d) 31st July, 2023 for Mr. Y and 31st October, 2023 for Mr. X

32. An individual client has consulted you on the matter of PAN. He is carrying on the business of sale & purchase of
electronic appliances. His turnover is ₹ 3,00,000 and profit is ₹ 75,000 for the P.Y. 2022-23. He has asked you to
provide him threshold of turnover, if any, exceeding which he has to apply for PAN. (ICAI MCQ Booklet)
(a) More than ₹ 2,00,000 (c) More than ₹ 3,00,000
(b) More than ₹ 2,50,000 (d) More than ₹ 5,00,000

33. Arun’s gross total income of P.Y. 2022-23 is ₹ 45,000. He deposits ₹ 45,000 in PPF. He pays electricity bills aggregating
to ₹ 1.20 lakhs in the P.Y. 2022-23. Which of the statement is correct? (ICAI MCQ Booklet)
(a) Arun is not required to file his return of income u/s 139(1) for P.Y. 2022-23, since his total income before giving
effect to deduction under section 80C does not exceed the basic exemption limit.
(b) Arun is not required to file his return of income u/s 139(1) for P.Y. 2022-23, since his electricity bills do not exceed
₹ 2,00,000 for the P.Y. 2022-23.
(c) Arun is not required to file his return of income u/s 139(1) for P.Y. 2022-23, since neither his total income before
giving effect to deduction under section 80C exceeds the basic exemption limit nor his electricity bills exceed ₹ 2
lakh for the P.Y. 2022-23.
(d) Arun is required to file his return of income u/s 139(1) for P.Y. 2022-23, since his electricity bills exceed ₹ 1 lakh
for the P.Y. 2022-23.

34. Iskon Inc., a foreign company and non-resident in India for A.Y. 2023-24, engaged in the business of trading of tube-
lights outside India. The principal officer of the company has approached you to enlighten him regarding the
provisions of the Income-tax Act, 1961 pertaining to the person who is required to verify the return of income in case
of Iskon Inc. Advicehim as to which of the following statements are correct, assuming that the company has a
managing director- (ICAI MCQ Booklet)
I. The return of income in case of Iskon Inc. can be verified by the managing director.
II. The return of income in case of Iskon Inc. can be verified by any director, irrespective of the availability or
otherwise of the managing director.
III. The return of income in case of Iskon Inc. can be verified by a person who holds a valid power of attorney from
such company to do so, irrespective of the availability or otherwise of the managing director.
Choose the correct answer:
(a) I or II or III (c) I or III
(b) Only I (d) Only III
CA SHREY RATHI RETURN OF INCOME 15.6

35. Which of the following returns can be revised under section 139(5)? (ICAI MCQ Booklet)
(i) A return of income filed u/s 139(1)
(ii) A belated return of income filed u/s 139(4)
(iii) A return of loss filed u/s 139(3)
Choose the correct answer:
(a) Only (i) (c) Only (i) and (iii)
(b) Only (i) and (ii) (d) (i), (ii) and (iii)

36. Mr. Dinesh, a resident in India, has gross total income of ₹ 2,30,000 comprising of interest on saving A/c and rental
income during the previous year 2022-23. He incurred expenditure of ₹ 2,00,000 for his son for study tour to Europe.
Whether he is required to file return of income for the assessment year 2023-24? If yes, what is the due date? (ICAI
MCQ Booklet)
(a) Yes, 31st July of A.Y. (c) Yes, 31st October of A.Y.
th
(b) Yes, 30 September of A.Y. (d) No, he is not required to file return of income

37. During the A. Y. 2023-24, Mr. Kabir has a loss of ₹ 6 lakhs under the head “Income from house property”, loss of ₹ 5
lakhs from business of profession and income of ₹ 3 lakhs from long term capital gains. He filed his return of income
for the A.Y. 2023-24 on 31-12-2023. Determine the total income of Mr. Kabir for A.Y. 2023-24 and the amount of loss
which can be carried forward in a manner most beneficial to him? (ICAI MCQ Booklet)
(a) Total income Nil; loss of ₹ 4,00,000 from house property and loss of ₹ 4,00,000 from business or profession
(b) Total income ₹ 1,00,000; loss of ₹ 4,00,000 from house property
(c) Total income Nil; No loss is allowed to be carried forward.
(d) Total income Nil; loss of ₹ 6,00,000 from house property.

38. Mr. Kumar, aged 62 years resident ad ordinarily resident, is a retired employee with a monthly pension of ₹ 15,000.
He has no other source of income. He has house property in Bhatinda and his only son is living in London and has a
house over there. He met with an accident and died and thereby leaving the house at London in the name of his
father, Mr. Kumar. Mr. Kumar seeks your advice, as to whether he is required to file his income-tax return u/s 139?
(ICAI MCQ Booklet)
(a) Yes, he is mandatorily required to file his income-tax return as he is a resident and ordinarily resident in India
and has asset located outside India
(b) No, he is not required to file return of his income as his income is below basic exemption limit.
(c) Yes, he is required to file his return of income as his income exceeds basic exemption limit
(d) No, he is not required to file his return of income as he is a senior citizen and retired employee

39. Mr. Alexa is a resident but not ordinarily resident in India for P.Y. 2022-23. He is doing job in M/s Kothari Chemicals as
Accountant & earns ₹ 25,000 per month. He had no other income in India but having a vacant land in Canada which he
had got from his father after his demise. He had no income from Canada also. Mr. Alexa come to you for consulting
whether he is required to file his return of income for A.Y. 2023-24? (ICAI MCQ Booklet)
(a) Alex is not required to file his return of income as his total income does not exceed the basic exemption limit.
(b) Alex is required to file his return of income as his total income as he is beneficiary of the assets located outside
India.
(c) Alex is not required to file his return of income as his total income does not exceed the basic exemption limit
and is resident but not ordinarily resident during the P.Y. 2022-23.
(d) Alex is not required to file his return of income as his total income exceeds the basic exemption limit.

MCQ Answers
1. (b) 2. (b) 3. (a) 4. (d) 5. (b) 6. (d) 7. (c) 8. (c) 9. (c) 10. (d)
11. (c) 12. (c) 13. (c) 14. (b) 15. (c) 16. (c) 17. (c) 18. (b) 19. (a) 20. (b)
21. (a) 22. (b) 23. (d) 24. (c) 25. (a) 26. (d) 27. (b) 28. (a) 29. (d) 30. (a)
31. (b) 32. (d) 33. (d) 34. (c) 35. (d) 36. (d) 37. (d) 38. (a) 39. (c)
CA SHREY RATHI COMPUTATION OF TOTAL INCOME & TAX LIABILITY 16.1

SACHIN TENDULKAR

CHAPTER 16
COMPUTATION OF TOTAL INCOME
& TAX LIABILITY

"Never let your failures get to your


heart and never let your success get
to your head.”

Q 1: Balamurugan furnishes the following information for the year ended 31-3-2023:
Particulars (₹)
Income from business (1,35,000)
Income from house property (15,000)
Lottery winnings (Gross) 5,00,000
Speculation business income 1,00,000
Income by way of salary 60,000
Long term capital gain 70,000
Compute his total income, tax liability and advance tax obligation. Assume he does not opt for 115BAC.
[ICAI Study Module]

Sol 1: Computation of total income of Balamurugan for the year ended 31.3.2022:
Particulars (₹) (₹)
Salaries 60,000
Less: Loss from house property (15,000) 45,000
Profits and Gains of Business or Profession
Speculation business income 1,00,000
Less: Business loss set off (1,35,000) -
Balance business loss of ₹ 35,000 to be set off against long-term capital gain
Capital Gains
Long term capital gain 70,000
Less: Business loss set-off (35,000)
Long-term capital gain after set off of business loss 35,000
Income from other sources: Lottery winning (gross) 5,00,000
Total income 5,80,000

Computation of tax liability:


Particulars (₹)
On lottery winning of ₹ 5,00,000 @ 30% 1,50,000
On total income of ₹ 80,000 (Excluding lottery winnings) Nil
1,50,000
Add: Health & Education Cess @ 4% 6,000
Total tax liability 1,56,000
CA SHREY RATHI COMPUTATION OF TOTAL INCOME & TAX LIABILITY 16.2

Advance tax liability: The assessee need not pay advance tax since the total income (excluding lottery income) liable to tax
is below the basis exemption limit. Further, in respect of lottery income, tax would have been deducted at source @ 30%
under section 194B. Since the remaining tax liability of ₹ 6,000 (₹ 1,56,000 – ₹ 1,50,000) is less than ₹ 10,000, advance tax
liability is not attracted.
Notes:
1. The basic exemption limit of ₹ 2,50,000 has to be first exhausted against salary income of ₹ 45,000. The unexhausted
basic exemption limit of ₹ 2,05,000 can be adjusted long term capital of ₹ 35,000 as per section 112, but not against
lottery winnings which are taxable at a flat rate of 30% u/s 115BB.
2. The first proviso to section 234C(1) provides that since it is not possible for the assessee to estimated his income from
lotteries, the entire amount of tax payable (after considering TDS) on such income should be paid in the remaining
instalments of advance tax which are due. Where no such instalment is due, the entire tax should be paid by 31st
March, 2022. The first proviso to section 234C(1) would be attracted only in case of non-deduction or short-deduction
of tax at source u/s 194B. In this case it has been assumed that tax has been deducted at source from lottery income.

Q 2: Mr. Purvi, aged 55 years, is a Chartered Accountant in practice. She maintains her accounts on cash basis. Her Income
and Expenditure Account for the year ended March 31, 2023 reads as follows:
Expenditure (₹) Income (₹)
Salary to staff 15,50,000 Fess earned:
Stipend to articled assistant 1,37,000 Audit ₹ 27,88,000
Incentive to articled assistants 13,000 Taxation services ₹ 15,40,300
Office rent 12,24,000 Consultancy ₹ 12,70,000 55,98,300
Printing and stationery 12,22,000 Dividend on shares of Indian Companies
Meeting, seminar and conference 31,600 (Gross) 10,524
Purchase of car (for official use) 80,000 Income from UTI (Gross) 7,600
Repair, maintenance and petrol of car 4,000 Honorarium received from various 15,800
Travelling expenses 5,25,000 institution for valuation of answer papers
Municipal tax paid in respect of house 3,000 Rent received from residential flat let out 85,600
property
Net profit 9,28,224
57,17,824 57,17,824
Other information:
a) Allowable rate of depreciation on motor car is 15%.
b) Value of benefits received from client during the course of profession is ₹ 10,500.
c) Incentives to articled assistants represent amount paid to two articled assistants for passing IPCC Examination of first
attempt.
d) Repairs and maintenance of car include ₹ 2,000 for the period from 01-10-2022 to 30-9-2023.
e) Salary include ₹ 30,000 to a computer specialist in cash for assisting Ms. Purvi in one professional assignment.
f) The travelling expenses include expenditure incurred on foreign tour of ₹ 32,000 which was within the RBI norms.
g) Medical insurance premium on the health of dependent brother and major son dependent on her amounts to ₹ 5,000
and ₹ 10,000 respectively, paid in cash.
h) She invented an amount of ₹ 10,000 in National saving certificate.
i) She has paid ₹ 70,000 towards advance tax during the P.Y. 2022-23.
Computation of total income and tax payable of Ms. Purvi for the A/Y 2023-24 as per regular provisions & 115BAC and
advice. [ICAI Study Module]
CA SHREY RATHI COMPUTATION OF TOTAL INCOME & TAX LIABILITY 16.3

Sol 2: Computation of total income and tax liability of Ms. Purvi as per the regular provisions for the A/Y 2023-24:
Particulars (₹)
Income from house property (WN 2) 57,820
Profit and gains of business or profession (WN 1) 9,20,200
Income from other sources (WN 3) 33,924
Gross Total Income 10,11,944
Less: Deduction u/c VI-A:
80C: Investment in NSC 10,000
80D: Medical insurance premium (not allowed as premium is paid in cash) -
Total income 10,01,944
Total Income (rounded off) 10,01,940
Computation of tax liability for Ms. Purvi:
Particulars (₹) (₹)
Tax on total income
Up to ₹ 2,50,000 Nil
Next ₹ 2,50,000 @ 5% 12,500
Next ₹ 5,00,000 @ 20% 1,00,000
Remaining ₹ 1,940 @ 30% 582 1,13,082
Add: Health & education cess @ 4% 4,523
Tax liability 1,17,605
Less: Advance Tax 70,000
Less: Tax deducted at source on dividend from Indian company u/s 194 1052
Less: Tax deducted at source on income from UTI u/s 194K 760
Net Tax Liability 45,793
Tax payable (rounded off) 45,790

Computation of total income and tax liability of Ms. Purvi as per 115BAC for the A/Y 2023-24:
Particulars (₹)
Income from house property (WN 2) 57,820
Profit and gains of business or profession (WN 1) 9,20,200
Income from other sources (WN 3) 33,924
Gross Total Income 10,11,944
Less: Deduction u/c VI-A:
80C: Investment in NSC [not allowed as per 115BAC(2)] ---
80D: Medical insurance premium [not allowed as per 115BAC(2)] ---
Total income 10,11,944
Total Income (rounded off) 10,11,940
Computation of tax liability for Ms. Purvi:
Particulars (₹) (₹)
Tax on total income
Up to ₹ 2,50,000 Nil
Next ₹ 2,50,000 @ 5% 12,500
Next ₹ 2,50,000 @ 10% 25,000
Next ₹ 2,50,000 @ 15% 37,500
Remaining ₹ 11,940 @ 20% 2,388 77,388
Add: Health & education cess @ 4% 3,096
Tax liability 80,484
Less: Advance Tax 70,000
Less: Tax deducted at source on dividend from Indian company u/s 194 1052
Less: Tax deducted at source on income from UTI u/s 194K 760
Net Tax Liability 8,672
Tax payable (rounded off) 8,670
CA SHREY RATHI COMPUTATION OF TOTAL INCOME & TAX LIABILITY 16.4

Advice: Since tax payable as per the provisions of Section 115BAC is lower than the tax payable under the regular
provisions of the Income Tax Act, 1961, it would be beneficial for Ms. Purvi to opt for 115BAC. She has to exercise this
option on or before the due date of furnishing the return of income i.e. 31st October 2023, in her case since she is liable to
get her books of account audited.

Working Notes:
1. Income under the head “Profits & Gains of Business or Profession”
Particular (₹) (₹)
Net profits as per Income and Expenditure Account 9,28,224
Add: Expenses debited but not allowable under the Income Tax Act, 1961:
o Salary paid to computer specialist in cash disallowed u/s 40A(3) since such cash 30,000
payment exceeds ₹ 10,000.
o Amount paid for purchase of car is not allowable u/s 37(1) since it is a capital 80,000
expenditure
o Municipal taxes paid in respect of residential flat let out 3,000 1,13,000
Add: Income taxable under this head but not credited to P/L A/c
o Value of benefit received from clients during the course of profession (tax income 10,500
u/s 28)
Less: Income credited but not taxable under the head PGBP:
o Dividend on shares of Indian companies 10,524
o Income from UTI 7,600
o Honorarium for valuation of answer paper 15,800
o Rent received from letting out of residential flat 85,600 1,19,524
Less: Expenses allowed but not debited to P/L A/c
Depreciation on motor car @ 15% (see note (a) below) 12,000
Profits and gains from Business & Profession 9,20,200
Notes:
a) It has been assumed that the motor car was put to use for more than 180 days during the previous year and hence,
full depreciation @ 15% has been provided for under section 32(1)(ii).
b) Incentive to article assistants for passing IPCC examination in their first attempt is deductible under section 37(1).
c) ₹ 32,000 expended on foreign tour is allowable as deduction assuming that it was incurred in connection with her
professional work. Since it has already been debited to income and expenditure account, no further adjustment is
required.
d) Repairs and maintenance paid in advance for the period 01.04.2023 to 30.09.2023 i.e. for 6 months amounting to ₹
1,000 is allowable since Ms. Purvi is following the cash system of accounting.
2. Income from House Property
Particulars (₹)
Gross annual value u/s 23(1) 85,600
Less: Municipal taxes paid 3,000
Net annual value (NAV) 82,600
Less: Deduction u/s 24 @ 30% of NAV 24,780
Income u/h house property 57,820
3. Income from Other Sources:
Particular (₹)
Dividend on shares of Indian companies 10,524
Income from UTI 7,600
Honorarium for valuation of answer papers 15,800
Income from Other Sources 33,924
CA SHREY RATHI COMPUTATION OF TOTAL INCOME & TAX LIABILITY 16.5

Q 3: Mr. Rajiv, aged 50 years, a resident individual and a practicing Chartered Accountant, furnishes you the Receipts and
Payments Account for the F/Y 2022-23: (Mr. Rajiv follows cash basis of accounting)
Receipts & Payment Account
Receipts (₹) Payments (₹)
Opening balance (1.4.2022) Cash in hand 12,000 Staff salary, bonus and stipend to article 21,50,000
and cash at bank clerks
Fee from professional services 59,38,000 Other administrative expenses 11,48,000
Rent 50,000 Office rent 30,000
Motor car loan from Canara Bank @ 9% Housing loan repaid to SBI (includes 1,88,000
p.a.) 2,50,000 interest of ₹ 88,000)
Life insurance premium 24,000
Motor car (acquired in Jan. 2023 by cheque) 4,25,000
Medical insurance premium (for self and 18,000
wife)
Books bought on 01.07.2022 (annual 20,000
publication by cheque)
Computer acquired on 1.11.2022 (for 30,000
profession use)
Domestic drawings 2,72,000
Public Provident Fund subscription 20,000
Motor car maintenance 10,000
Closing balance (31.3.2023): Cash on hand
and at bank 19,15,000
Total 62,50,000 Total 62,50,000
Following further information is given to you:
1. He occupies 50% of building for one evidence and let out the balance for residential use at monthly rent of ₹ 5,000.
The building was constructed during the year 1997-98 when loan was taken.
2. Motor car put to use both for official and personal purpose. One fifty of the motor car use is for personal purpose. No
car loan interest was paid during the year.
3. The written down value of assets as on 1-4-2022 are given below.
Furniture & Fitting ₹ 60,000
Plant & machinery (Air- conditioner, photocopiers, etc ₹ 80,000
Computers ₹ 50,000
Compute the total income of Mr. Rajiv for the Assessment Year 2023-24 assuming that he has not opted to pay tax under
section 115BAC. [ICAI Study Module]
CA SHREY RATHI COMPUTATION OF TOTAL INCOME & TAX LIABILITY 16.6

Sol 3: Computation of total income of Mr. Rajiv for the A/Y 2023-24:
Particulars (₹) (₹) (₹)
Income from house property
Self-occupied property
Net annual value Nil
Less: Deduction u/s 24(b): Interest on housing loan 30,000 (30,000)
50% of ₹ 88,000 = ₹ 44,000 but limited to loss from self-occupied property
Let out property
Annual value (rent receivable has been taken as the annual value in the
absence of other information) 60,000
Less: Deduction u/s 24
30% of net annual value ₹ 18,000
Interest on housing loan (50% of ₹ 88,000) ₹ 44,000 62,000 (2000) (32,000)
Profits and gains of business or profession
Fees from Professional Services 59,38,000
Less: Expenses allowable as deduction:
• Staff salary, bonus and stipend 21,50,000
• Other administrative expenses 11,48,000
• Office Rent 30,000
• Motor car maintenance (₹ 10,000 x 4/5) 8,000
• Car loan interest: not allowable (since the same has not been paid and
the assessee follows cash system of accounting) Nil
• Less: Depreciation
o Motor car: ₹ 4,25,000 x 7.5% x 4/5 25,500
o Books being annual publication @ 40% 8,000
o Furniture and fitting @ 10% of ₹ 60,000 6,000
o Plant and machinery @ 15% of ₹ 80,000 12,000
o Computer @ 40% of ₹ 50,000 20,000
o Computer (New) ₹ 30,000 @ 40% x ½ thereon 6,000 34,13,500 25,24,500
Gross Total Income 24,92,500
Less: Deduction u/c VI-A
Deduction u/s 80C:
Housing loan principal repayment 1,00,000
PPF subscription 20,000
Life insurance premium 24,000 1,44,000
Deduction under section 80D: Medical insurance premium 18,000 1,62,000
Total Income 23,30,500

Q 4: From the following particulars furnished by Mr. X for the year ended 31.3.2023, you are requested to compute his
total income and tax payable for the assessment year 2023-24, assuming that he does not opt for paying tax under section
115BAC.
a) Mr. X retired on 31.12.2022 at the age of 58, after putting in 26 years and 1 month of service, from a private company
at Mumbai.
b) He was paid a salary of ₹ 25,000 p.m. and house rent allowance of ₹ 6,000 p.m. he paid rent of ₹ 6,500 p.m. during his
tenure of service.
c) On retirement, he was paid a gratuity of ₹ 3,50,000. He was covered by the payment of Gratuity Act. Mr. X had not
received any other gratuity at any point of time earlier, other than this gratuity.
d) He had accumulated leave of 15 days per annum during the period of his service, this was encased by Mr. X at the
time of his retirement. A sum of ₹ 3,15,000 was received by him in this regard. His average salary may be taken as ₹
24,500. Employer allowed 30 days leave per annum.
e) After retirement, he ventured into textile business and incurred a loss of ₹ 80,000 for the period upto 31.3.2023.
f) Mr. X has invested ₹ 1,00,000 in public provident fund. [ICAI Study Module]
CA SHREY RATHI COMPUTATION OF TOTAL INCOME & TAX LIABILITY 16.7

Sol 4: Computation of total income of Mr. X for A/Y 2023-24:


Particulars ₹ ₹
Income from salary
Basic salary (₹ 25,000 x 9) 2,25,000
House rent allowance (₹ 6000 x 9) 54,000
Less: Exemption u/s 10(13A) (Note 1) 36,000 18,000
Gratuity received 3,50,000
Less: Exemption u/s 10(10)(iii) (Note 2) 3,50,000 ---
Leaves encashment received 3,15,000
Less: Exemption u/s 10(10AA) (Note 3) 2,45,000 70,000
Gross salary 3,13,000
Less: Standard deduction u/s 16(ia) 50,000
Income u/h Salary 2,63,000
Profits and Gains of Business or Profession
Business loss of ₹ 80,000 to be carried forward as the same cannot be set off against salary Nil
income
Gross Total Income 2,63,000
Less: Deduction u/s 80C
Investment in Public Provident Fund 1,00,000
Total income 2,56,750
Tax on Total Income Nil
Notes:
1. As per section 10(13A), house rent allowance will be exempt to the extent of least of the following three amounts
a) HRA actually received (₹ 6,000 x 9) ₹ 54,000
b) Rent paid in excess of 10% of salary [(₹ 6,500 x 9) - 10% of ₹ 2,25,000] ₹ 36,000
c) 50% of salary ₹ 1,12,500
2. Gratuity of ₹ 3,75,000 is exempt u/s 10(10)(iii), being the minimum of the following amounts:
a) Actual amount received ₹ 3,50,000
b) Half month average salary for each year of completed (15/26 x ₹ 25,000 x 26) ₹ 3,75,000
c) Statutory limit ₹ 20,00,000
3. Leave encashment is exempt upto the least of the following:
a) Actual amount received ₹ 3,15,000
b) 10 months average salary (₹ 24,500 x 10) ₹ 2,45,000
c) Statutory limit ₹ 3,00,000
d) Cash equivalent of unavailable leave calculated on the basis of maximum 30 days for every year of actual service
rendered to the employer (refer note 4 below) ₹ 3,18,500
Since the leave entitlement of Mr. X as per his employer’s rules is 30 days credit for each for year of service and he had
accumulated 15 days per annum during the period of his service, he would have availed the balance 15 days leave every
year.
Leave entitlement of Mr. X on the basis of 30 days for every year of actual service rendered by him to the employer
= 30 days/year x 26 = 780 days
Leave availed by Mr. X during the period of his service = 15 days/year x 26 = 390 days
Earned leave to the credit of Mr. X at the time of his retirement = 390 days
Cash equivalent of earned leave to the credit of Mr. X at the time of his retirement
= 390 x ₹ 24500/30 = ₹ 3,18,500.
CA SHREY RATHI COMPUTATION OF TOTAL INCOME & TAX LIABILITY 16.8

Q 5: From the following details, compute the total income and tax liability of Mr. Siddhant, aged 31 years of Delhi both as
per the regular provisions of the Income Tax Act, 1961 and as per 115BAC for the A/Y 2023-24. Advice Mr. Siddhant
whether he should opt for 115BAC or not.
Particulars (₹)
Salary including dearness allowance 3,35,000
Bonus 11,000
Salary of servant provided by the employer 12,000
Rent paid by Siddhant for his accommodation 49,600
Bills paid by the employer for gas, electricity and water provided free of cost at the above flat 11,000

1. Siddhant purchased a flat in a co-operative housing society in Delhi for ₹ 4,75,000 in April, 2016 which was financed
by a loan from life insurance corporation of India of ₹ 1,60,000 @ 15% interest, his own saving of ₹ 65,000 and a
deposit from a nationalized bank for ₹ 2,50,000 to whom this flat was given on lease for ten years. The rent payable
by the bank was ₹ 3,500 per month. The following particular are relevant.
Municipal taxes paid by Mr. Siddhant ₹ 4,300
House Insurance ₹ 860
2. He earned ₹ 2,700 in share speculation business and lost ₹ 4,200 in cotton speculation business.
3. In the year 2017-18, he gifted ₹ 30,000 to his wife and ₹ 20,000 to his son who was aged 11. The gifted amounts were
advanced to Mr. Rajesh, who was paying interest @ 19% per annum.
4. Siddhant received a gift of ₹ 30,000 each from four friends.
5. He contributed ₹ 50,000 to public provident fund. [ICAI Study Module]

Sol 5: Computation of total income and tax liability of Siddhant according to regular provisions for the A/Y 2023-24:
Particulars (₹) (₹)
Salary income
Salary including dearness allowance 3,35,000
Bonus 11,000
Value of perquisites:
Salary of servant 12,000
Free gas & electricity 11,000
Gross salary 3,69,000
Less: Standard deduction u/s 16(ia) 50,000
Income u/h Salary 3,19,000
Income from House Property
Gross annual value (GAV) (rent receivable is taken as gross annual value in the absence
of other information) (₹ 3,500 x 12) 42,000
Less: Municipal taxes paid 4,300
Net Annual value (NAV) 37,700
Less: Deduction u/s 24
(a) 30% of NAV 11,310
(b) Interest on loan from LIC @ 15% of ₹ 1,60,000 [see note 2] 24,000
Income u/h House Property 2,390
Profits & Gains from Business & Profession
Income from share speculation business 2,700
(-) loss from cotton speculation business 4,200
Net loss from speculative business has to be carried forward as it cannot be set off
against any other head of income. 1,500
Income from other sources
Income on account of interest from money gifted to his minor son is includible in the 3,800
hands of Siddhant as per section 64(1A)
Less: Exempt under section 10(32) 1,500
2,300
Interest income earned from money gifted to wife has to be clubbed with the income of
the assessee as per section 64(1) 5,700
CA SHREY RATHI COMPUTATION OF TOTAL INCOME & TAX LIABILITY 16.9

Gift received from four friends (taxable u/s 56(2)(x) as the aggregate amount received
during the year exceeds ₹ 50,000) 1,20,000 1,28,000
Gross Total Income 4,49,390
Less: Deductions u/c VI-A: 80C (Public Provident Fund) 50,000
Total Income 3,99,390
Computation of tax liability:
Particulars (₹)
Tax on total income 7,470
Less: Rebate u/s 87A [Tax liability or ₹ 12,500 whichever is lower] 7,470
Tax liability Nil

Computation of total income and tax liability of Siddhant according to 115BAC for the A/Y 2023-24:
Particulars (₹) (₹)
Salary income
Salary including dearness allowance 3,35,000
Bonus 11,000
Value of perquisites:
Salary of servant 12,000
Free gas & electricity 11,000
Gross salary 3,69,000
Less: Standard deduction u/s 16(ia) [not allowed as per 115BAC(2)] Nil
Income u/h Salary 3,69,000
Income from House Property
Gross annual value (GAV) (rent receivable is taken as gross annual value in the absence
of other information) (₹ 3,500 x 12) 42,000
Less: Municipal taxes paid 4,300
Net Annual value (NAV) 37,700
Less: Deduction u/s 24
(c) 30% of NAV 11,310
(d) Interest on loan from LIC @ 15% of ₹ 1,60,000 [see note 2] 24,000
Income u/h House Property 2,390
Profits & Gains from Business & Profession
Income from share speculation business 2,700
(-) loss from cotton speculation business 4,200
Net loss from speculative business has to be carried forward as it cannot be set off
against any other head of income. 1,500
Income from other sources
Income on account of interest from money gifted to his minor son is includible in the 3,800
hands of Siddhant as per section 64(1A)
Less: Exempt under section 10(32) [not allowed as per 115BAC(2)] Nil
3,800
Interest income earned from money gifted to wife has to be clubbed with the income of
the assessee as per section 64(1) 5,700
Gift received from four friends (taxable u/s 56(2)(x) as the aggregate amount received
during the year exceeds ₹ 50,000) 1,20,000 1,29,500
Gross Total Income 5,00,890
Less: Deductions u/c VI-A: 80C (Public Provident Fund) [not allowed as per 115BAC(2)] Nil
Total Income 5,00,890
Computation of tax liability:
Particulars (₹)
Tax on total income 12,589
Add: Health & Education Cess @ 4% 504
Tax liability 13,093
Tax Liability (rounded off) 13,090
CA SHREY RATHI COMPUTATION OF TOTAL INCOME & TAX LIABILITY 16.10

Notes:
1. It is assumed that the entire loan of ₹ 1,60,000 is outstanding as on 31.3.2023.
2. Since Siddhant’s own flat in co-operative housing society, which he has rented out to a nationalized bank, is also in
Delhi, he is not eligible for deduction u/s 80GG in respect of rent paid by him for his accommodation in Delhi, since
one of the condition to be satisfied for claiming deduction u/s 80GG is that assessee should not one any residential
accommodation in the same place.

Q 6: Mr. Y carries on his business. An analysis of his trading and profit & loss for the year ended 31-3-2023 revealed the
following information:
a. The net profit was ₹ 11,20,000.
b. The following incomes were credited in the profit and loss account:
Dividend from UTI (gross) ₹ 22,000
Interest on debenture (gross) ₹ 17,500
Winning from races (gross) ₹ 15,000
c. It was found that same stocks were omitted to be included in both the opening and closing stocks, the values of which
were:
Opening stock ₹ 8,000
Closing stock ₹ 12,000
d. ₹ 1,00,000 was debited in the profit and loss account, being contribution to a University approved and notified under
section 35(1)(ii).
e. Salary includes ₹ 20,000 paid to his brother which is unreasonable to the extent of ₹ 2,500.
f. Advertisement expenses include 15 gift packets of dry fruits costing ₹ 1,000 per packet presented to important
customers.
g. Total expenses on car was ₹ 78,000. The car was used both for business and personal purposes. 3/4th is for business
purposes.
h. Miscellaneous expenses included ₹ 30,000 paid to A &Co, a goods transport operator in cash on 31-01-2023 for
distribution of the company’s product to the warehouses.
i. Depreciation debited in the books was ₹ 55,000. Depreciation allowed as per income tax rules. 1962 was ₹ 50,000.
j. Drawings ₹ 10,000
k. Investment in NSC ₹ 15,000.
Compute the total income & tax liability of Mr. Y for the A/Y 2023-24, assuming that he has not opted to pay tax u/s
115BAC. [ICAI Study Module]

Sol 6: Computation of income of Mr. Y for the A/Y 2023-24:


Particulars (₹)
Profits and Gains of Business or Profession (WN 1) 11,21,500
Income from other sources:
Dividend from UTI ₹ 22,000
Interest on debentures ₹ 17,500
Winning from races ₹ 15,000 54,500
Gross Total Income 11,76,000
GTI excluding winning from races 11,61,000
Less: Deduction u/s 80C: Investment in NSC 15,000
Total income (excluding winning from races) 10,46,000
Add: Winning from races 15,000
Total income 10,61,000
CA SHREY RATHI COMPUTATION OF TOTAL INCOME & TAX LIABILITY 16.11

Working Notes:
Computation of Profits and Gains of Business or Profession:
Particulars (₹) (₹)
Net profit as per Profit and Loss Account 11,20,000
Add: Expenses debited to profit and loss account but not allowable as deduction
❖ Salary paid to brother disallowed to the extent considered (Section 40A(2)) 2,500
❖ Motor car expenses attributable to personal use not allowable (₹ 78,000 x ¼) 19,500
❖ Excess depreciation debited in the books of account (₹ 55,000 - ₹ 50,000) 5,000
❖ Drawings (not allowable since it is personal in nature) [See note (iii)] 10,000
❖ Investment in NSC [See note (iii)] 15,000 52,000
Less: Expenses allowed but not debited to P/L A/c
❖ Contribution to a University approved and notified u/s 35(1)(ii) is eligible for weighted
deduction @ 100%. Since the actual contribution (100%) has been debited to profit and loss ---
account, no further deduction has to be deducted.
❖ Under–statement of opening stock 8,000 8,000
Less: Income credited to P/L A/C but not taxable as business income
❖ Income from UTI (taxable u/h income from other sources) 22,000
❖ Interest on debentures (taxable u/h income from other sources) 17,500
❖ Winnings from races (taxable u/h income from other sources) 15,000 54,500
Add: Income taxable u/h PGBP but not credited to P/L A/c
❖ Understatement of closing stock 12,000
Total income 10,21,500
Notes:
1. Advertisement expenses of revenue nature, namely, gift of dry fruits to important customers, is incurred wholly and
exclusively for business purposes. Hence, the same is allowable as deduction under section 37.
2. Disallowance u/s 40A(3) is not attracted in respect of cash payment exceeding ₹ 10,000 to A &Co. a goods transport
operator, since in case of payment made for playing hiring or leasing goods carriages limit of ₹ 35,000 is applicable
(i.e. payment of up to ₹ 35,000 can be made in cash without attracting disallowance under section 40A(3)).
3. Since drawing and investment in NSC have been given effect to in the profit and loss account, the same have to be
added back to arrive at the business income.

Q 7: Rosy and Mary are sisters, born and brought up at Mumbai. Rosy got married in 1982 and settled at Canada since
1982. Mary got married and settled in Mumbai. Both of them are below 60 years. The following are the details of their
income for the previous year ended 31.03.2023:
Particulars Rosy (₹) Mary (₹)
Pension received from State Government - 60,000
Pension received from Canadian Government 20,000 -
Long-term capital gain on sale of land at Mumbai 1,00,000 1,00,000
Short-term capital gain on sale of shares of Indian listed companies in respect of which STT 20,000 2,50,000
was paid
LIC premium paid - 10,000
Premium paid to Canadian life Insurance corporation at Canada 40,000 -
Mediclaim policy premium paid - 25,000
Investment in PPF - 20,000
Rent received in respect of house property at Mumbai 60,000 30,000

Compute the taxable income and tax liability of Mrs. Rosy and Mrs. Mary for the assessment year 2023-24 and tax
thereon. Ignore the provisions of 115BAC. [ICAI Study Module]
CA SHREY RATHI COMPUTATION OF TOTAL INCOME & TAX LIABILITY 16.12

Sol 7: Computation of taxable income of Mrs. Rosy and Mrs. Mary for the A/Y 2023-24:
Particulars Rosy (₹) Mary (₹)
Salary
Pension received from State Government 60,000
Less: Standard deduction u/s 16(ia) 50,000
Pension received from Canadian Government is not taxable in the case of a non-resident -
since it is earned and received outside India
Salary income (a) - 10,000
Income from House Property
Rent received from house property at Mumbai 60,000 30,000
(assumed to be the net annual value in the absence of other information)
Less: Deduction u/s 24(a) @ 30% 18,000 9,000
House property income (b) 42,000 21,000
Capital gains
Long-term capital gain on sale of land at Mumbai 1,00,000 1,00,000
Short term capital gain on sale of shares of Indian listed companies in respect of which STT
was paid 20,000 2,50,000
Capital Gains income (c) 1,20,000 3,50,000
Gross Total Income (D) [a + b + c] 1,62,000 3,81,000
Less: Deductions u/c VI-A
Deduction u/s 80C:
• LIC premium paid - 10,000
• Premium paid to Canadian Life Insurance Corporation 40,000 -
• Investment in PPF - 20,000
Deduction u/s 80D: Medicilam premium paid (assuming that the same is paid by cheque) - 25,000
40,000 55,000
Total deduction u/c VI-A is restricted to income other than capital gains taxable u/s 111A &
112 40,000 31,000
Total income (D - E) 1,22,000 3,50,000
Computation of Tax Liability:
Particulars Rosy (₹) Mary (₹)
Tax liability of Mrs. Rosy for A/Y 2023-24:
Tax on long- term capital gains @ 20% 20,000
Tax on short-term capital gains @ 15% 3,000
23,000
Tax liability of Mrs. Mary for A/Y 2023-24:
Tax on short–term capital gains @ 15% of ₹ 1,00,000 [i.e. ₹ 2,50,000 less ₹ 1,50,000, being 15,000
the unexhausted basic exemption limit as per proviso to section 111A]
Less: Rebate under section 87A [Tax liability or 12,500, whichever is lower] 12,500
2,500
Add: Health & Education Cess @ 4% 920 100
Final Tax Liability 23,920 2,600
Notes:
1. In case of resident individuals, if the basic exemption limit is not fully exhausted against other income, then the long-
term capital gains/short-term capital gains will be reduced by the unexhausted basic exemption limit and only the
balance will be taxed at taxed at 20% / 15% respectively. However, this benefit is not available to non-resident.
Therefore, while Mrs. Mary can adjust her unexhausted basic exemption limit against long term capital gains and
short-term capital gains taxable u/s 111A, Mrs. Rosy cannot do so.
2. Long term capital gains is taxable at the rate of 20% and short term capital gains is taxable at the rate of 15%, it is
more beneficial for Mrs. Mary to first exhaust her basic exemption limit of ₹ 2,50,000 against loan term capital gains
of ₹ 1,00,000 and the balance limit of ₹ 1,50,000 (i.e. ₹ 2,50,000 - ₹ 1,00,000) against short- term capital gains.
CA SHREY RATHI COMPUTATION OF TOTAL INCOME & TAX LIABILITY 16.13

Q 8: Miss Charlie, an American national, got married to Mr. Radhey of India in USA on 02.03.2022 and came to India for
the first time on 16.03.2022. She left for USA on 20.9.2022. She returned to India again on 27.03.2023 while in India, she
has purchased a show room in Mumbai on 30.04.2022, which was leased out to a company on a rent of ₹ 25,000 p.m.
from 01.05.2022. She had taken loan from a bank for purchase of this show on which bank had charged interest of ₹
97,500 upto 31.03.2023. She had received the following gifts during the year.
From parents of husband ₹ 51,000
From married sister of husband ₹ 11,000
From two very close friend of her husband ₹ 1,51,000 and ₹ 21,000 ₹ 1,72,000
a) Determine her residential status and compute the total income chargeable to tax along with the amount of tax
payable on such income for the A/Y 2023-24.
b) Would her residential status undergo any change assuming that she is a person of Indian origin and her total income
from Indian sources is ₹ 18,00,000 and she is not liable to tax in USA? [ICAI Study Module]

Sol 8: a) Computation of residential status total income & tax liability for the A/Y 2023-24:
Under section 6(1), an individual is to be resident in India in any previous year, if he satisfies any one of the following
conditions:
(i) She has been in India during the previous year for a total period of 182 day or more or
(ii) She has been in India during the 4 years immediately preceding the previous year for a total period of 365 days or has
been in India for at least 60 days in the previous year.
If an individual satisfies anyone of the conditions mentioned above, the individual is a resident. If both the conditions are
not satisfied, the individual is a non-resident.
Therefore, the resident status of Miss Charlie, an American National, for A/Y 2023-24 has to be determined on the basis of
her stay in India during the P/Y 2022-23 and in the preceding four financial years.
Her stay in India during the previous year 2022-23 and in the preceding four years are as under:
P/Y 2022-23
01.04.2022 to 19.09.2022 - 172 days
27.03.2023 to 31.03.2023 - 5 days
Total - 177 days
Four preceding previous years
P/Y 2021-22 (16.03.2022- 31.03.2022) - 16 days
P/Y 2020-21 - Nil
P/Y 2019-20 - Nil
P/Y 2018-19 - Nil
Total - 16 days
The total stay of the assessee during the previous year in India was less than 182 days and during the four years preceding
this year was for 16 days. Therefore, due to non-fulfilment of any of the two conditions for a resident, she would be
treated as non- resident for the A/Y 2023-24.

Computation of total income of Miss. Charlie for the A/Y 2023-24


Particulars (₹) (₹)
Income from House Property:
Show room located in Mumbai remained on rent from 01.05.2022 to 31.3.2023 @ ₹ 25000
p.m.
Gross Annual value [₹ 25,000 x 11] 2,75,000
Less: Municipal taxes Nil
Net Annual Value (NAV) 2,75,000
Less: Deduction under section 24
(a) Standard deduction @ 30% of NAV ₹ 82,500
(b) Interest on loan ₹ 97,500 1,80,000 95,000
CA SHREY RATHI COMPUTATION OF TOTAL INCOME & TAX LIABILITY 16.14

Income from Other Sources:


Gifts received from non-relatives is chargeable to tax as per section 56(2)(x) if the
aggregate value of such gifts exceeds ₹ 50,000.
o ₹ 50,000 received from parents of husband would be exempt, since parents of
husband fall within the definition of relatives and gifts from a relative are not ---
chargeable to tax
o ₹ 11000 received from married sister of husband is exempt, since sister-in –law falls
within the definition of relative and from a relative are not chargeable to tax. ---
o Gifts received from two friends of husband ₹ 1,51,000 and ₹ 21,000 aggregation to ₹
1,72,000 is taxable under section 56(2)(x) since the aggregate of ₹ 1,72,000 exceeds ₹
50,000. 1,72,000 1,72,000
Total Income 2,67,000
Computation of tax payable by Miss Charlie for the A/Y 2023-24:
Particulars (₹)
Tax on total income of ₹ 2,67,000 850
Add: Health & Education Cess @ 4% 34
Tax liability 884
Tax liability (rounding off) 880
Note: Since Ms. Charlie is a non-resident for the A/Y 2023-24, rebate u/s 87A would not be available to her, even though
her total income is less than ₹ 5,00,000.

b) Residential status of Miss Charlie in case she is a person of Indian origin and her total income from Indian sources
exceeds ₹ 18,00,000
If she is a person of Indian origin and her total income from Indian sources exceeds ₹ 15,00,000 (₹ 18,00,000, in her case),
the condition of stay in India for a period exceeding 120 days during the previous year and 365 days during the four
immediately preceding previous years would be applicable for being treated as a resident. Since her stay in India exceeds
120 days in the P.Y. 2022-23 but the period of her stay in India during the four immediately preceding previous years is
less than 365 days (only 16 days) her residential status as per section 6(1) would continue to be same i.e., non-residential
in India.
Further, since she is not a citizen of India, the provisions of section 6(1A) deeming an individual to be a citizen of India
would not get attracted in her case, even though she is a person of Indian origin and her total income from Indian sources
exceeds ₹ 15,00,000 and she is not liable to pay tax in USA.
Therefore, her residential status would be non-resident in India for the previous year 2022-23.

Q 9: Dr. Niranjana, a resident individual, aged 60 years is running a clinic. Her income and expenditure account for the
year ending March 31st, 2023 is as under:
Expenditure (₹) Income (₹)
To Medicine consumed 35,38,400 By Consultation and medical charge 58,85,850
To Staff salary 13,80,000 By Income-tax refund (principal ₹ 5,000 5,450
To Clinic consumable 1,10,000 interest ₹ 450)
To Rent paid 90,000 By Dividend from units of UTI 10,500
To Administrative expenses 2,55,000 By winning from game show no T.V (net of 35,000
To Amount paid to scientific research TDS of ₹ 15,000)
association approved u/s 35 1,50,000 By Rent 27,000
To Net Profit 4,40,400
Total 59,63,800 Total 59,63,800
Other information:
1. Rent paid includes 30,000 paid by cheque towards rent for residential house in Surat.
2. Clinic equipments are:
01.04.2022 Opening W.D.V ₹ 5,00,000
07.12.2022 Acquired (cost) by cheque ₹ 2,00,000
CA SHREY RATHI COMPUTATION OF TOTAL INCOME & TAX LIABILITY 16.15

3. Rent received relates to property situated at Surat. The municipal tax of ₹ 2000, paid in December, 2022 has been
included in “administrative expenses”.
4. She received salary of ₹ 7,500 p.m. from “Full Cure Hospital” which has not been included in the “consultation and
medical charges”.
5. Dr. Niranjana availed a loan of ₹ 5,50,000 from a bank for higher education of her daughter. She repaid principal of ₹
1,00,000 and interest thereon ₹ 55,000 during the year 2022-23.
6. She paid ₹ 1,00,000 as tuition fee (not in the nature of development fees/donation) to the university for full time
education of her daughter.
7. An amount of ₹ 28,000 has also been paid by cheque on 28th March, 2023 for her medical insurance premium.
From the above, compute the total income of Dr. Smt. Niranjana for the A/Y 2023-24 under the regular provisions of the
Income Tax Act, 1961, assuming that she has not opted to pay tax u/s 115BAC. [ICAI Study Module]

Sol 9: Computation of total income and tax liability of Dr. Niranjana for A/Y 2023-24:
Particulars (₹) (₹) (₹)
Income from salary:
Basic salary (₹ 7500 x 12) (Gross) 90,000
Less: Standard deduction u/s 16(ia) 50,000 40,000
Income from house property:
Gross Annual Value (GAV) 27,000
Less: Municipal taxes paid by the owner 2,000
Net Annual value (NAV) 25,000
Less: Deduction u/s 24
(a) Standard deduction @ 30% 7,500 17,500
Income from Profession:
Net profit as per Income and Expenditure account 4,40,400
Less: Income credited but not taxable under this head
o Rent received 27,000
o Dividend from units of UTI 10,500
o Winning from game show on T.V (Net of TDS) 35,000
o Income tax refund 5,450 77,950
Less: Expenses allowable but not debited to P/L A/c:
o Depreciation on clinic equipments:
On ₹ 5,00,000 @ 15% 75,000
On ₹ 2,00,000 @ 7.5% 15,000 90,000
(on equipments acquired during the year in December, 2022 she is
entitled to depreciation @ 50% of normal deprecation, since the same
are put to use for less than 180 days during the year)
Add: Inadmissible expenses:
o Rent for her residential accommodation included in Income and
Expenditure A/C 30,000
o Municipal tax paid relating to residential house at Surat including in
administrative expenses 2,000 32,000 3,04,450
Income from Other Sources:
Interest on income-tax refund 450
Dividend from UTI (taxable in the hands of the unit holder) 10,500
Winning from the game show on T.V (₹ 35,000 + ₹ 15,000) 50,000 60,950
Gross Total Income 4,22,900
Less: Deduction u/c VI-A:
o 80C: Tuition fee paid for full time education of her daughter. 1,00,000
o 80D: Medical insurance premium (fully allowed since she is a senior
citizen) 28,000
o 80E: Interest on loan taken for higher education is deductible
without any limit. 55,000 1,83,000
Total Income 2,39,900
CA SHREY RATHI COMPUTATION OF TOTAL INCOME & TAX LIABILITY 16.16

Notes:
1. Income-tax refund will be excluded from computation of total income as it not allowed as an expense at the time of
payment. Interest received will be taxed under the head “Income from Other Sources”.
2. Since Dr. Niranjana is staying in a rented premise in Surat itself, she would not be eligible for deduction u/s 80GG as
she owns a house in Surat which she has let out.
3. Winning form game show on T.V. should be grossed up for the chargeability under the head “Income from other
source” (₹ 35,000 + ₹ 15,000). Thereafter, while computing tax liability, of ₹ 15,000 should be deducted to arrive at
the tax payable winning from game are subject to tax @ 30% as per section 115BB.

Q 10: Mr. Venus Provides the Following details for the previous year ending 31.3.2023:
Salary from HNL Ltd ₹ 50,000 p.m.
Interest on FD with SBI for the financial year 2022-23 ₹ 72,000 (net of TDS)
Brought forward long-term capital loss of A/Y 2018-19 ₹ 96,000
Long term capital gain ₹ 75,000
Loss of minor son ₹ 90,000 computed in accordance with the provision of Income Tax Act, 1961. Mr. Venus transferred his
own house to his minor son without adequate consideration few years back minor son let it out and suffered loss. Further
there is a loss of ₹ 2,00,000 in his wife’s business. She carried business with funds which Mr. Venus gifted to her in
previous years)
You are required to compute taxable income of Mr. Venus for the A/Y 2023-24 assuming that he does not wishes to opt
for 115BAC.

Sol 10: Computation of taxable income of Mr. Venus for the A.Y. 2023-24:
Particulars (₹) (₹)
Salaries
Gross salary (₹ 50,000 x 12) 6,00,000
Less: Standard deduction u/s 16(ia) 50,000
Income u/h Salary 5,50,000
Less: Loss from house property in respect of which Mr. Venus is the deemed owner to
owner to be set off against his salary income as per section 71(1) [See note 1] 90,000 4,60,000
Capital gains
Long term capital gain 75,000
Less: Brought forward long-term capital loss of A/Y 2018-19 set off against current year
long term capital gain as per section 74(1) & (2) [See note 2] 75,000 Nil
Balance long–term capital loss of ₹ 21,000 (₹ 96,000 – ₹ 75,000) of A.Y. 2018-19 to be
carried forward to A/Y 2024-25 [See note 2]
Income from Other Sources
Interest on fixed deposit with SBI (₹ 72,000 X 100/90) 80,000
Less: Business loss incurred by wife includible in Mr. Venus’s set off against interest
income as per section 71(1) [See note 3 & 4] 80,000 Nil
Balance business loss of ₹ 1,20,000 (₹ 2,00,000 - ₹ 80,000) to be carried forward to A/Y
2024-25.
Taxable Income 4,60,000
Notes:
1. As per section 27(i) Mr. Venus is the deemed owner of the house transferred to his minor son without consideration.
Hence, the income from house property transferred to minor son without adequate consideration, Mr. Venus can set-
off the same against salary income, since he is the deemed owner of such property.
2. As per section 74(1) and 74(2), brought forward long-term capital loss can be set-off only against long-term capital
gains. Unabsorbed long–term capital loss can be carried forward for a maximum of eight assessment years (upto A.Y.
2026-27, in the case) for set off against long term capital gains.
CA SHREY RATHI COMPUTATION OF TOTAL INCOME & TAX LIABILITY 16.17

3. As per section 64(1)(iv), income from funds gifted to spouse by an individual and invested in business by the spouse is
includible in the hands of the individual. As per explanation 2 to section 64, income includes “loss”. Hence, in the
given case, loss arising out of the business carried on by Mrs. Venus is to be included in the income of Mr. Venus, as
she has carried on business with the funds gifted to her by Mr. Venus.
4. As per section 71(2A), business loss cannot be set off against salary income. However, the same can be set off against
income from other sources (consisting of fixed deposit).

Q 11: Mr. Rajesh is serving in a public limited company as General Manager (finance). His total emoluments for the year
ended for the year ended 31st March, 2023 are as follows:
Basic salary ₹ 5,90,000
House Rent Allowance ₹ 1,80,000
Entertainment allowance ₹ 2,800
Apart from the above, his employer has sold the following assets to him on 1st January, 2023.
Laptop computer for ₹ 20,000 (acquired in September, 2021 - ₹ 1,20,000)
Car 1800 CC for ₹ 3,20,000 (purchased in April, 2020 for ₹ 8,50,000).
He also owns a residential house, let out for a monthly rent of ₹ 15,000. The fair rental value of the property for let out
period is ₹ 1,50,000 the house was self–occupied by him from 1st January, 2023 to 31st March 2024. He has taken a loan
from bank of ₹ 20 lacs for the construction of the property, and has repaid ₹ 1,05,000 (including interest ₹ 40,000) during
the year.
Mr. Rajesh sold shares of different Indian companies on 14th April, 2022:
Name Sales value (per share) (₹) Purchase price (per share) (₹) Acquired on No. of shares
A Ltd. 150 120 (STT paid at Acquisition) 2nd May, 2014 200

B Ltd. 82 65 (STT paid at Acquisition) 16th April, 2021 125


Sale proceeds were subjects to brokerage of 0.1% and securities transaction tax of 0.125% on the gross consideration. He
received income tax refund of ₹ 5,750 (including interest ₹ 750) relating to the A/Y 2021-22. Assume FMV of A Ltd. shares
to be ₹ 144 as on 31st January 2018.
Compute the total income of Mr. Rajesh for the Assessment year 2023-24 assuming Mr. Rajesh does not wish to opt for
115BAC.

Sol 11: Computation of total income of Mr. Rajesh for the A/Y 2023-24:
Particulars (₹)
Income from Salaries (WN 1) 9,86,800
Income from House Property (WN 2) 1,00,000
Capital Gains (WN 3) [₹ 1,170 + ₹ 2,115] 3,285
Income from Other Sources: Interest on income tax refund 750
Gross Total Income 10,90,835
Less: Deduction u/c VI-A:
80C: Repayment of housing loan (principal) 65,000
Total income 10,25,835
Total income (rounded off) 10,25,840
CA SHREY RATHI COMPUTATION OF TOTAL INCOME & TAX LIABILITY 16.18

Working Notes:
1. Income from Salaries:
S. No. Particulars (₹) (₹)
1. Basic salary 5,90,000
2. House Rent Allowance (as no rent is paid by Mr. Rajesh, the whole HRA shall 1,80,000
be taxable)
3. Entertainment allowance 2,800
4. Perquisites (relating to sale of movable assets by employer)
Laptop computer: Cost [September, 2021] 1,20,000
Less: Depreciation at 50% for one completed year 60,000
WDV [September, 2022] 60,000
Less: Amount paid to the employer 20,000
Perquisite value of laptop (A) 40,000

Car: Cost [April, 2020] 8,50,000


Less: Deprecation for the 1st year (April, 2020 to March, 2021) @ 20% on 1,70,000
the basis of WDV
WDV [April, 2021] 6,80,000
Less: Deprecation for the 2nd Year (April, 2021 to March, 2022) @ 20% of 1,36,000
WDV 5,44,000
WDV [April, 2022] 3,20,000
Less: Amount paid to the employer 2,24,000
Perquisite value of car (B) 2,64,000
Perquisite value (A) + (B) 10,36,800
Gross Salary 50,000
Less: Standard deduction u/s 16(ia) 9,86,800
Income chargeable under the head “Salaries”

2. Income from House Property


Section 23(2) provides that net annual value of self-occupied house shall be taken as nil. However, section 23(3) provides
that the benefit of self-occupation would not be available if the house is actually let during the whole or part of the
previous year. This implies that benefit of taken the annual value as nil would be available only if the house is self-
occupied for the whole year.
In this case, therefore, the benefit of taking annual value as nil is not available since the house is self-occupied only for 3
months. In such a case, the gross annual value has to be computed as per section 23(1). Accordingly, the fair rent for the
whole year should be compared with the actual rent for the let–out period and whichever is higher shall be adopted as
the gross annual value.

Particulars (₹) (₹)


Gross Annual value (higher of fair rent for the whole year and actual rent for the let-out 2,00,000
period)
Fair rent for the whole year = ₹ 1,50,000 x 12/9 2,00,000
Actual rent received = ₹ 15,000 x 9 1,35,000
Less: Municipal taxes paid by the owner Nil
Net annual value (NAV) 2,00,000
Less: Deduction u/s 24
(a) 30% of NAV 60,000
(b) Interest on loan [see note below] 40,000 1,00,000
Income from house property 1,00,000
Note: It is presumed that the interest of ₹ 40,000 paid on housing loan represents the interest actually due for the year.
CA SHREY RATHI COMPUTATION OF TOTAL INCOME & TAX LIABILITY 16.19

3. Income u/h “Capital Gains”


Shares in A Ltd. will fall within the purview of Section 112A as the shares are LTCA being equity shares on which STT is paid
and have been acquired prior to 1st February 2018.
Particulars (₹)
Sale consideration (200 x ₹ 150) 30,000
Less: Brokerage @ 0.1% 30
29,970
Less: Cost of acquisition (refer below) 28,800
1,170
Cost of Acquisition of shares of A Ltd.
COA in relation to LTCA being equity shares on which STT is paid and acquired before 1st February 2018 shall be higher of:
(i) cost of acquisition of such asset; and ₹ 24,000
(ii) lower of
(a) the fair market value of such asset as on 31st January 2018; and ₹ 28,800
(b) sale value received or receivable as a result of transfer of the capital asset ₹ 30,000
Therefore, COA shall be ₹ 28,800. However, benefits of indexation shall not be available.

Shares in B Ltd. are held for less than 12 months and hence the capital gains arising on sale of shares is a short-term
capital gain chargeable to tax @ 15% as per section 111A, since the transaction is subject to subject to securities
transaction tax. It may be noted, however, that securities transaction tax is not a deductible expenditure.
Particulars (₹)
Sales consideration (82 x ₹ 125) 10,250
Less: Brokerage @ 0.1% 10
Net sale consideration 10,240
Cost of acquisition (65 x ₹ 125) 8,125
Short-term capital gains 2,115

Q 12: Mr. X, an individual set up a unit in Special Economic Zone (SEZ) in the financial year 2018-19 for production of
washing machines. The unit fulfills all the conditions of Section 10AA of the Income-tax Act, 1961. During the financial year
2021-22, he has also set up a warehousing facility in a district of Tamil Nadu for storage of agricultural produce. It fulfills
all the conditions of Section 35AD. Capital expenditure in respect of warehouse amounted to ₹ 75 lakhs (including cost of
land ₹ 10 lakhs). The warehouse became operational with effect from 1st April, 2022 and the expenditure of ₹ 75 lakhs was
capitalized in the books on that date.
Relevant details for the financial year 2022-23 are as follows:
Particulars (₹)
Profit of unit located in SEZ 40,00,000
Export sales of above unit 80,00,000
Domestic sales of above unit 20,00,000
Profit from operation of warehousing facility (before considering deduction u/s 35AD) 1,05,00,000
Compute income-tax (including AMT under Section 115JC) liability of Mr. X for A.Y. 2023-24 both as per regular provisions
of the Income Tax Act, 1961, and as per Section 115BAC for A.Y. 2023-24.
Advise Mr. X whether he should opt for Section 115BAC. [ICAI Study Module]
CA SHREY RATHI COMPUTATION OF TOTAL INCOME & TAX LIABILITY 16.20

Sol 12: Computation of total income and tax liability of Mr. X for A.Y. 2023-24 (under the regular provisions of the Income
Tax Act, 1961
Particulars (₹) (₹)
Profit and gains of business or profession
Profit from unit in SEZ 40,00,000
Less: Deduction u/s 10AA (See Note - 1 below) 32,00,000
Business income of SEZ unit chargeable to tax 8,00,000
Profit from operation of warehousing facility 1,05,00,000
Less: Deduction u/s 35AD (See Note - 2 below) 65,00,000
Business income of warehousing facility chargeable to tax 40,00,000
Total income 48,00,000
Computation of liability (under the normal/ regular provisions)
Tax on ₹ 48,00,000 12,52,500
Add: Health and Education cess @ 4% 50,100
Total tax liability 13,02,600
Computation of adjusted total income of Mr. X for levy of Alternate Minimum Tax
Particulars (₹) (₹)
Total Income (computed above as per regular provisions of income tax) 48,00,000
Add: Deduction under Section 10AA 32,00,000
80,00,000
Add: Deduction under Section 35AD 65,00,000
Less: Depreciation under Section 32: On building @ 10% of ₹ 65 lakhs 6,50,000 58,50,000
Adjusted Total Income 1,38,50,000
Alternate Minimum Tax @ 18.5% 25,62,250
Add: Surcharge @ 15% (since adjusted total income > ₹ 1 crore) 3,84,338
29,46,588
Add: Health and Education cess @ 4% 1,17,863
Tax Liability 30,64,451
Tax liability u/s 115JC (rounded off) 30,64,450
Since the regular income-tax payable is less than the alternate minimum tax payable, the adjusted total income shall be
deemed to be the total income and tax is leviable @ 18.5% thereof plus surcharge @ 15% and HEC @ 4%. Therefore, tax
liability as per Section 115JC is ₹ 30,64,450.
Computation of total income and tax liability of Mr. X for A.Y. 2023-24 (under the provisions of Section 115BAC of the
Income-tax Act, 1961)
Particulars (₹) (₹)
Total Income (computed above as per regular provisions of income tax) 48,00,000
Add: Deduction under Section 10AA 32,00,000
80,00,000
Add: Deduction under Section 35AD 65,00,000
Less: Depreciation under Section 32: On building @ 10% of ₹ 65 lakhs 6,50,000 58,50,000
Total Income 1,38,50,000
Tax on ₹ 1,38,50,000 as per 115BAC 38,92,500
Add: Surcharge @ 15% (since total income > Rs.1 crore) 5,83,875
44,76,375
Add: Health and Education cess @ 4% 1,79,055
Tax Liability 46,55,430
Notes:
(1) Deductions u/s 10AA and 35AD are not allowable as per Section 115BAC(2). However, normal depreciation u/s 32 is
allowable.
(2) Individuals or HUFs exercising option u/s 115BAC are not liable to alternate minimum tax u/s 115JC. Since the tax
liability of Mr. X under Section 115JC is lower than the tax liability as computed u/s 115BAC, it would be beneficial for
CA SHREY RATHI COMPUTATION OF TOTAL INCOME & TAX LIABILITY 16.21

him not to opt for Section 115BAC for A.Y. 2023-24. Moreover, benefit of alternate minimum tax credit is also
available to the extent of tax paid in excess over regular tax.
AMT Credit to be carried forward under Section 115JEE
Particulars (₹)
Tax liability under Section 115JC 30,64,450
Less: Tax liability under the regular provisions of the Income-tax Act, 1961 13,02,600
17,61,850
Notes:
(1) Deduction under Section 10AA in respect of Unit in SEZ
= Profits of Unit in SEZ x Export turnover of Unit in SEZ
Total turnover of Unit in SEZ
= ₹ 40,00,000 X ₹ 80,00,000 = ₹ 32,00,000
₹ 1,00,00,000
(2) Deduction @ 100% of the capital expenditure is available under Section 35AD for A.Y. 2023-24 in respect of specified
business of setting up and operating a warehousing facility for storage of agricultural produce which commences
operation on or after 01-04-2009.
Further, the expenditure incurred, wholly and exclusively, for the purposes of such specified business, shall be allowed
as deduction during the previous year in which he commences operations of his specified business if the expenditure
is incurred prior to the commencement of its operations and the amount is capitalized in the books of account of the
assessee on the date of commencement of its operations.
Deduction under Section 35AD would, however, not be available on expenditure incurred on acquisition of land. In
this case, since the capital expenditure of ₹ 65 lakhs (i.e. ₹ 75 lakhs - ₹ 10 lakhs, being expenditure on acquisition of
land) has been incurred in the F.Y. 2021-22 and capitalized in the books of account on 01-04-2022, being the date
when the warehouse became operational, ₹ 65,00,000, being 100% of Rs.65 lakhs would qualify for deduction under
Section 35AD.

13. Ramdin, aged 33 years, working as Manager (Sales) with Frozen foods Ltd., provides the following information for the
year ended 31.3.2023:
- Basic Salary ₹ 15,000 p.m.
- DA (50% of it is meant for retirement benefits) ₹ 12,000 p.m.
- Commission as a percentage of turnover of the company 0.5%
- Turnover of the company ₹ 50 lakhs
- Bonus ₹ 50,000
- Gratuity ₹ 30,000
- Own contribution to R.P.F ₹ 30,000
- Employer’s contribution to R.P.F 20% of basic salary
- Interest credited in the R.P.F. account @ 15% p.a. ₹ 15,000
- Gold ring worth ₹ 10,000 was given by employer on his 25th wedding anniversary.
- Music system purchased on 1.04.2022 by the company for ₹ 85,000 and was given to him for personal use.
- Two old light goods vehicles owned by him were leased to a transport company against the fixed charges of ₹
6,500 p.m. Books of account are not maintained.
- Received interest of ₹ 5,860 on Bank FDRs on 24-04-2022 and interest of ₹ 6,786 (Net) from the debentures of
Indian Companies on 5-5-2022.
- Made payment by cheques of ₹ 15,370 towards premium of Life Insurance policies and ₹ 22,500 for Mediclaim
Insurance policy for self and spouse
- Invested in NSC ₹ 30,000 and in FDR of SBI for 5 years ₹ 50,000.
- Donations of ₹ 11,000 to an institution approved u/s 80G and of ₹ 5,100 to Prime Minister’s National Relief Fund
were given during the year by way of cheque.
Compute the total income and tax payable thereon for the A/Y 2023-24. Assume Ramdin does not opt for section
115BAC. [ICAI Study Module]
CA SHREY RATHI COMPUTATION OF TOTAL INCOME & TAX LIABILITY 16.22

Ans 13: Computation of total income of Mr. Ramdin for the A/Y 2023-24:
Particulars (₹) (₹)
Income from salaries
Basic salary (₹ 15,000 x 12) 1,80,000
Dearness Allowance (₹ 12,000 x12) 1,44,000
Commission on turnover (0.5% of ₹ 50 lakhs) 25,000
Bonus 50,000
Gratuity (See Note 1) 30,000
Employer’s contribution to recognized provident fund
Actual contribution [20% of ₹ 1,80,000] 36,000
Less: Exempt (See Note 2) 33,240 2,760
Interest credited in recognized provident fund account @ 15% p.a. 15,000
Less: Exempt upto 9.5% p.a. 9,500 5,500
Gift of gold ring worth ₹ 10,000 on 25th wedding anniversary by employer 10,000
Perquisite value of music system given for personal use (Being 10% of actual
cost) i.e., 10% of ₹ 85,000. 8,500
Gross Salary 4,55,760
Less: Standard deduction under section 16(ia) 50,000
Income under the head Salary 4,05,760
Profit and Gains of Business of Profession
Lease of 2 light goods vehicles on contract basis against fixed charges of ₹ 6,500 1,80,000
p.m. In this case, presumptive tax provisions of section 44AE will apply i.e. ₹
7,500 p.m. for each of the light goods vehicle (₹ 7,500 x 2 x 12). He cannot claim
lower profits and gains since he has not maintained books of account.
Income from Other Sources
Interest on bank FDRs 5,860
Interest from debentures (₹ 6,786 x 100/90) 7,540 13,400
Gross Total Income 5,99,160
Less: Deduction u/c VI-A:
Section 80C
Premium on life insurance policy 15,370
Investment in NSC 30,000
FDR of SBI for 5 years 50,000
Employee’s contribution to recognized provident fund 30,000 1,25,370
Section 80D – Mediclaim insurance 22,500
Section 80G (See Note 4) 10,600
Total Income 4,40,690
Tax on Total Income
Income Tax [5% of ₹ 1,90,690 (i.e. ₹ 4,40,690 - ₹ 2,50,000) 9,535
Add: Rebate u/s 87A, since total income does not exceed ₹ 5,00,000 9,535
Tax Liability Nil
Less: Tax deducted at source (₹ 7,540 - ₹ 6,786) 754
Net tax refundable 754
Tax Refundable (rounded off) 750
Notes:
1. Gratuity received during service is full taxable.
2. Employer’s contribution in the recognized provident fund is exempt up to 12% of the salary i.e. 12% (Basic salary
+ DA for retirement benefits + commission based of turnover)
= 12% [₹ 1,80,000 + (50% of ₹ 1,44,000) + ₹ 25,000]
= 12% of ₹ 2,77,000
= ₹ 33,240
CA SHREY RATHI COMPUTATION OF TOTAL INCOME & TAX LIABILITY 16.23

3. An alternate view possible is that only the sum in excess of ₹ 5,000 is taxable in view of the language of Circular
No. 15/2001 dated 12-12-2001 that such gifts upto ₹ 5,000 in the aggregate per annum would be exempt,
beyond which it would be taxed as a perquisite. As per this view, the value of perquisite would be ₹ 5,000. In
such a case the income from Salaries would be ₹ 4,00,760.
4. Deduction under section 80G is computed as under:
Particulars (₹)
Donation to PM National Relief Fund (100%) 5,100
Donation to institution approved under section 80G (50% of Rs.11,000) (amount contributed ₹
11,000 or 10% of Adjusted Gross total income i.e. ₹ 45,129, whichever is lower) 5,500
Total Deduction 10,600
Adjusted gross total income = Gross total income – Deduction u/s 80C and 80D = ₹ 5,99,160 - ₹ 1,47,870 = ₹
4,51,290.

MULTIPLE CHOICE QUESTIONS


1. Which of the following states is not true with respect to A.Y. 2023-24? (ICAI MCQ Booklet)
(a) No exemption under section 80TTA would be available to resident senior citizens
(b) Share of profit will not be exempt in the hands of partner, if firm claims exemption of income under section 10AA
(c) Long term capital gains of ₹ 90,000 on STT paid listed equity shares would not be subject to income-tax under
section 112A
(d) Exemption under section 10(32) on income of minor child is allowed for more than two children also

2. Gross total income of Arpita for P.Y. 2022-23 is ₹ 6,00,000. She had taken a loan of ₹ 7,20,000 in the financial year
2019-20 from a bank for her husband who is pursuing MBA course from IIM, Kolkata. On 02-04-2022, she paid the first
instalment of loan of ₹ 45,000 and interest of ₹ 65,000. Computed her total income for A.Y. 2023-24. (ICAI MCQ
Booklet)
(a) ₹ 6,00,000 (c) ₹ 4,90,000
(b) ₹ 5,35,000 (d) ₹ 5,55,000

3. Mr. Uttam presents you following data related to his tax liability for A.Y. 2023-24:
Particulars (₹ in lakhs)
Tax Liability as per regular provisions of Income-tax Act, 1961 15
Tax Liability as per section 115JC 12
AMT credit brought forward from A.Y. 2021-22 5
What shall be the tax liability of Mr. Uttam for A.Y. 2023-24? (ICAI MCQ Booklet)
(a) ₹ 12 lakhs (c) ₹ 10 lakhs
(b) ₹ 15 lakhs (d) ₹ 7 lakhs

4. Mr. Nekinsaan, aged 43 years, provides following income details for P.Y. 2022-23 as follows:
Particulars (₹ in lakhs)
Capital Gains under section 112A 120
Capital Gains under section 111A 110
Other income 520
What shall be the tax liability of Mr. Nekinsaan as per regular provisions of Income-tax Act, 1961 for A.Y. 2023-24?
(ICAI MCQ Booklet)
(a) ₹ 260.06 lakhs (c) ₹ 256.52 lakhs
(b) ₹ 253.68 lakhs (d) ₹ 253.56 lakhs

5. Continuing Q 4, what shall be tax liability of Mr. Nekinsaan as per regular provisions of the Income Tax Act, 1961 for
A.Y. 2023-24 if the Other Income is ₹ 480 lakhs? (ICAI MCQ Booklet)
(a) ₹ 218.20 lakhs (c) ₹ 218.73 lakhs
(b) ₹ 221.03 lakhs (d) ₹ 242.25 lakhs
CA SHREY RATHI COMPUTATION OF TOTAL INCOME & TAX LIABILITY 16.24

6. Mr. Bandu, aged 37 years, provides following details for P.Y. 2022-23 as follows:
Particulars (₹ in lakhs)
Textile Business Income 22
Speculative Business Income (4)
Textile Business Loss b/f from P.Y. 2019-20 (5)
Business income of spouse included in the income of Mr. Bandu as per section 64(1)(iv) 2
Deductions available under Chapter VI-A 3
TDS 1
TCS 0.5
Advance tax paid 1.3
What shall be the net tax payable/(refundable) as per regular provisions of the Income-tax Act, 1961 for A.Y. 2023-24
for Mr. Bandu? (ICAI MCQ Booklet)
(a) ₹ 24,200 (c) ₹ 2,11,400
(b) (₹ 1,60,000) (d) ₹ 12,500

7. Mr. Raj, aged 32 years, presents you the following data for A.Y. 2023-24:
Particulars (₹ in lakhs)
Gross Receipts from Business conducted entirely through banking channels (opted for section 70
44AD)
Capital Gains under section 112A 5
Capital gains under section 111A 3
Winnings from horse races 1
What would be the tax liability as per the regular provisions of the Income-tax of Mr. Raj for the A.Y. 2023-24? (ICAI
MCQ Booklet)
(a) ₹ 1,28,440 (c) ₹ 1,38,840
(b) ₹ 1,05,560 (d) ₹ 1,45,080

MCQ ANSWERS
1. (b) 2. (b) 3. (a) 4. (d) 5. (c) 6. (a) 7. (a)

You might also like